You are on page 1of 522

KERTAS SO ALAN PEPERIKSAAN BERCETAK - SEM.

*1 2010/2011 (FKE)
SEMESTER U
BIL
KOD KURSUS
1.
SEE 2523
2.
SET 4523
3.
SEM 4333
4.
SEE 3223
5.
SEE 3533
6.
SEM 4153
7.
SEE 3243 / SEE 4243
8.
SEP 4003
9.
SEL 4283
10.
SEM 4173
11.
SEP 4043 / SWB 4043
12.
SEU 3053
13.
SEE 4453
14.
SEE 3263
15.
SEE 4443
16.
SEE 4153
17.
SEP 4263
18.
SEE 1023
19.
SEE 1123
20.
SEE 3113
21.
SEE 4113
22.
SEE 1003
23.
SEE 2063
24.
SEE 2053
25.
SEE 2253
26.
SEL 4363
27.
SEP 4243
28.
SET 4543
29.
SEI 4143
30.
SEL 4263
31.
SET 3583
32.
SEL 4533
33.
SEL 4223
34.
SEI 4123
35.
SEP 4253
36.
SEL 4233
37.
SEI 4153
38.
SEE 4133
39.
SEE 1223
40.
SEE 2113
41.
SET 3573
42.
SET 4593
43.
SEU 2033
44.
SEU 2052
45.
SEL 4373
46.
SEE 4463
47.
SEU 2012
48.
SEE 4433
49.
SEI3133
50.
SEE 3433
51.
SET 4533
52.
SEE 4012
53.
SEE 2043
54.
SEP 4123
55.
SEL 4743

CONFIDENTIAL

UTM

UNIVERSITI TEKNOLOGI MALAYSIA

FINAL EXAMINATION SEMESTER II


SESSION 2010/2011
COURSE CODE

SEE 2523

COURSE NAME

ELECTROMAGNETIC FIELD THEORY

LECTURERS

ASSOC. PROF. DR. ABU SAHMAH BIN MOHD


SUPAAT
ASSOC. PROF. DR. NORAZAN BIN MOHD
KASSIM
DR. MOHD HANIFF BIN IBRAHIM
DR. MUHAMMAD RAMLEE BIN KAMARUDIN
DR. NOOR ASNIZA BT MURAD
MRS. FAREHA BT ABDUL RAHMAN

PROGRAMME

SEC / SEE / SEI / SEL / SEM / SEP / SET / SEW /


SWB

SECTION

01-05/10

TIME

2 HOURS 30 MINUTES

DATE

04 MAY 2011

INSTRUCTION TO CANDIDATE : ANSWER FOUR (4) QUESTIONS ONLY.


THREE (3) QUESTIONS FROM PART A AND
ONE (1) QUESTION FROM PART B.

THIS EXAMINATION BOOKLET CONSISTS OF 10 PAGES INCLUDING THE FRONT COVER

SEE 2523
2

PART A
Ql(a) With the aid of a model of four point charges and the definition of electric field potential
difference, derive an equation to determine the energy stored in a system of charge.
[5 marks]
(b) A coaxial cable has an inner radius a and an outer radius c. Assume that both cylinders
are perfectly conducting. The inner and outer cylinders contain charges of +Q and -Q
respectively. When oriented along the z axis, the relative permittivity for first dielectric
material (a < r < b) is sri and the relative permittivity for second dielectric material {b < r
< c) is sr2 assuming that a<b <c. Determine:
the electric field intensity in regions r<a>a<r<b and b < r <c; [6 marks]

(i)
(ii)

the absolute electric potential in regions r<a, a<r<b and b < r < c; and
[6 marks]

(iii)

the electric energy per unit length in each of the dielectric materials. [3 marks]

State clearly any assumptions made.

(c) Referring to the same coaxial cable mentioned in question 1(b) above, let a = 1.0 cm, b =
1.75 cm and c = 2.8 cm. Assume that the first dielectric is Teflon and the second
dielectric is Polystyrene. Using information given in Table Ql(c) below, determine the
maximum voltage that can be applied to the coaxial cable.
Table Q 1(c)

Sr

Breakdown electric field


(V/m)

5.4
2.26
2.56
2.1

200 x 10b
47 x 106
20 x 10b
60 x 106

Relative Permittivity

Mica
Polyethyline
Polystryene
Teflon

Explain what will happen with regards to the maximum applied voltage to the coaxial
cable if Teflon is being replaced with Mica.
[5 marks]

SEE 2523
3

Q2 (a) Explain clearly what is the Uniqueness theorem.


[3 marks]
(b) Consider two potential solutions which in terms of cylindrical coordinate system are
given by:
V, = 90/b;
and
V2 = (100b-10r)/rb
where b is a constant.
(i)

Show that both solutions satisfy Laplaces equation.


[4 marks]

(ii)

Show that both solutions yield the same potential at r = b.


[2 marks]

(c) A parallel plate capacitor consists of two conductors are located at z = 0 and z = 0.01 m
respectively. Each conductor has surface area of 1.5 m2.The lower conductor is grounded,
meanwhile the other conductor is at a potential of 10 V. The relative permittivity in the
region between the conductors varies as a function ofz and is given by sr = 1 + z.
Assuming that the potential only depends on z and the fringing is ignored, find:
(i)

The electric potential in the region between the two conductors;

[6 marks]

(ii)

The surface charge densities on the two conductors;

[3 marks]

(iii)

The capacitance; and

[4 marks]

(iv)

Show that the total bound charge is zero for this capacitor.

[3 marks]

SEE 2523
4

Q3(a) A coaxial cable centered along the z axis is shown in Figure Q3(a). The inner conductor
at r < a is carrying current IA in +z direction while the outer conductor b < r < c is
carrying current IA in - z direction. The region a < r < b is a magnetic material with
relative permeability p.r of 2.
(i)

Obtain the magnetic field intensity in all the regions.

(ii)

Find total magnetic flux in the region a < r < b and 0 < z < 1 [4 marks]

(iii)

Determine the magnetized surface current on the surfaces of the magnetic


material.
[3 marks]

(iv)

Find the inductance per meter of the coaxial cable

[2 marks]

(v)

Find the magnetic energy per meter stored in the cable

[2 marks]

[6 marks]

(b) (i) Referring to Figure Q3(b), find the magnetic force per unit length along filamen
current I2
[5 marks]
(ii)

How would the magnetic force behave if both currents are in the same direction.
[3 marks]

Current Ij

current I-

distance d
Figure Q3(b)

SEE 2523
5

Q4(a) With the aid of a suitable diagram and assumption, derive the mathematical expression of
boundary condition of two magnetic materials if the magnetic field propagates from one
magnetic material with permeability ju2to another magnetic material with
permeability//,.

[8 marks]

(b) A magnetic field intensity in a region defined by y -x - 2 < 0 is Hx = 2x-6y + 4z A/m.


The permeability of the magnetic material of the region is given by //, = 5jua. Determine:
(i)

Magnetization vector M, and magnetic flux density 5,.

(ii)

Magnetic field intensity H2, magnetic flux density B2, magnetization vector M2 in

[2 marks]

region y-x-2 > 0 with relative permeability 2.


[10 marks]
(iii)

Magnetized surface current density Jsm at the boundary.


[5 marks]

SEE 2523
6

PARTS
Q5(a) About 20 KM from Madinah Munawwarah, there is a mountain called Jabal of
Magnetism. Normally, the tourist guide will bring the tourists to this place either by
buses, vans or cars. The miracle happens at the valley of the mountain is that the bus, car
or van will be pulled by some force (maybe Magnetic Force) in a certain direction where
the speed can reach up to 120 Kmph. As a researcher, you have been given the task to
investigate the emf induced by the force. You may use galvanometer, loop or any
relevant measurement setup. Please state clearly any assumptions made in order to obtain
the induced emf.
[5 marks]
(b)The loop shown in Figure Q5(b) is inside a uniform magnetic field B = 0.1 ay Tesla. If
side DC of the loop cuts the flux lines at the frequency of 100 Hz and the loop lies in the
yz-plane at time t = 0, find
(i)

The induced emf at t = 2 ms;

[7 marks]

(ii)

The induced current at t = 4 ms ;

[3 marks]

State clearly any assumptions made.


(c) Repeat Q5(b) (i) and (ii) above for B = 0.02/ ax T and R = 1 ohm.
(You may 'use-. a6 = sin 0 ax 4- cos 0 ay)
z

Figure Q5(b)

[10 marks]

SEE 2523
7

Q6(a) By applying suitable assumptions and equations, show that the wave attenuation and
phase constants are depicting equal formulation for the case of propagation in good
conductors.
[5 marks]
(b) An electric field of E(x,t) = 5 sin (2.5 x 109t +24) az V/m, is excited into a copper sheet
as shown in Figure Q6(b). If the crC0pper = 5.8 x 107 S/m, find the phase and amplitude at a
depth of 0.1 mm, 2 mm, 5 mm and 10 mm. Sketch the electric field magnitude versus
propagation distance inside the copper sheet.
[8 marks]

Figure Q6(b)
(c) Light propagation in rectangular optical waveguide structure (Figure 6(c)) can be
regarded as a uniform plane wave propagation. The waveguide material is assumed to
have the following parameters: er = 80, fj.r = 1, o = 0.4 S/m. Laser pulse is then coupled
into the waveguide structure at the position z = 0, in which +z direction is considered to
be the direction of propagation. As a result, the following magnetic field value has been
measured at z = 10 m.
H(10,t) = 20 cos (2u x 103t + 15) ay mA/m
(i)

Determine the general formulations of E(z,t) and H(z,t)


(ii)

[10 marks]

Find the time-average power of the propagated wave at z = 20 m. [2 marks]

Figure 6(c)

SEE 2523
8

ELECTROSTATIC FIELD
Coulombs Law E= faR
Gausss Law ($D ds=Oen
Force on a point charge F=EC
Electric field for finite line charge
E =-^(sina, + sina)+-(coso:3 -cosaOf
47Z0lrv 'rK lJ
Electric field for infinite line charge
= A r
2 7te0r
Electric flux density D = sE
El ectric flux y/E =Q = (^D- ds
Divergence theorem cj>D ds= J (V D^dv
sv

MAGNETOSTATIC FIELD
Biot-Savart Law H= {* ffiJ 4nR
Ampere Circuital law (^H-d =Jen
Force on a moving charge F = Q(u x 5)
Force on a current^Ptemeftt F = /5T x ~B
Magnetic field for finite current
H =------ (sin a2 + sin a,
4mMagnetic field for infinite current
H=l
2 7vr
Magnetic flux density B=/JH
' Magnetic flux <ym = B ds

Stokes theorem c}>H-d = (V x H) ds


i ,t

Potential difference VAB = - \ E d\


B
Absolute potential V = I , ~
K J 4 tus R
0
Gradient of potential E = -V V
Energy stored in an electric field
wc~\ 1(5-1)*
^V
Total current in a conductor'
/= \j- ds where J = crE
Polarization vector P =De^E
Bound surface charge density
Psb=P-h
Volume surface charge density
A*=-V-?
Electrical boundary conditions
and -E^-E^
Resistance J? =
as **#-

Magnetic potential, (A) B = V x A


Energy stored in a magnetic field
Wm=l\(BW)h
^V

Magnetization vector M = %mH


where Zm =MMagnetized surface current density
Magnetized volume current density
Jm=VxM
Magnetic boundary conditions
B\n = An an<3 tJs
Inductance L = y where A=

Capacitance C-
^ab
Poissons equation V2V =

Laplace equation VJF = 0


Maxwell equation V )=/? ^ Vx = 0

Maxwell equation V B = 0, V xH = J

SEE 2523
9

TIME VARYING FIELD


Maxwell equation V-D=pv

Gausss Law for electric field

Vxi=-~

Faradays Law

V- B =0

Gausss Law for magnetic field


Ampere Circuital Law

VxH = J +

dt

Characteristics of wave propagation in lossy medium (cr^O)ju=/^jl.is=s^sr)


Electric field, E (z, t) = cos (cot - ftz)x

E
Magnetic field, H(z,t) = jje~a:cos(o}t-/3z + 9)y
m
1 (V
IIS
Attenuation constant a = co
Jl+ i] -1
\cosJ

1 U

Phase constant

f\2
<y

(5=0)

1+
2 j

+1

\COSJ

Intrinsic impedance 77 = 7 w h e r e
[ l + / cosf J

M-i

(E Hy

Poynting vector @=ExH


Average power

=
cos

8=\la

Skin depth
Poynting theorem

t a n 2d

ds =

-E2+-+lH2
12 2 .

iv- foE2dv

SEE 2523
10

Kecerunan Gradient
' df *df df
Vf = x + y + z
dx dy dz
.df 0 df \df
Vf = r +~ + z
dr r d<f> dz
vf = ?&+!L'L+JL-K.
dr r d$ r sin 6 d<f>

Kecapahan Divergence
_ -j dA dA 5A.
V-A =-x +-v +dx dy dz

1 cL4, dA.
r d<j> dz

d(rAr)
dr

V-A =V-A =

+---------- + -

1 \d(r'A,y

r1

dr.

d(Ae sin 0)
1
H--------rsin#

L de J

1 dA,
+r sin (9 dfi

Ikal Curl

' dA. dAv} JdAr dA.\-jfdAr dA: N

VxA=x

dy dz
f\

Vx A=r

dAs dA^ f**

dz dx

\ dx dy
J
A
N
z^*'-d(rA,) dAr

dAr dA:
+
+ </>.
dz dr

dr
d</>
r d(/> dz
A / d(A sin 6) 8A N
'd{rAe) dA,)
1 dA, d(rA4)'
&
4
e
+
V xA =
r
dr
dd )
r sin# d<j> dr
rsin#
d6 d<j>

Laplacian

dx2 dy2 dz2

V2f =

l_d_
r
r dr k dr;

df) + dV + 5V
r1 df dz2

f
i
1 d(,df~\ .1
&d + sin# I. +
V2 f =r
2
2
J r2 dr
r sin2 9
de
dr
r sin# d&

CONFIDENTIAL

UTM
UNIVERSITI TEKNOLOGI MALAYSIA

FINAL EXAMINATION SEMESTER II


SESSION 2010/2011
COURSE CODE

SET 4523

COURSE NAME

OPTICAL COMMUNICATION SYSTEM

LECTURERS

PROF. DR. ABU BAKAR BIN MOHAMMAD


ASSOC. PROF. DR. SEVIA MAHDALIZA BT
IDRUS

PROGRAMME

SET

SECTION

01-02

TIME

2 HOURS 30 MINUTES

DATE

05 MAY 2011

INSTRUCTION TO CANDIDATE :

ANSWER FOUR (4) QUESTIONS ONLY.

THIS EXAMINATION BOOKLET CONSISTS OF 7 PAGES INCLUDING THE FRONT COVER

-2-

SET 4523
Ql. (25 Marks)

(a)

Discuss the two main reasons why optical fiber is a favored communication channel for longhaul communications compared to copper channels.

(b)

(4 marks)

Consider a step index fiber;


(i)

Determine the cutoff wavelength to exhibit single mode operation when the core
refractive index and radius are 1.46 and 4.5 |im, respectively, with the relative index
difference being 0.25%.

(2 marks)

(ii) Calculate the fiber core size if using different source at 1320 nm and 1550 nm. (4 marks)
(iii) Discuss the result obtain in (b) (i).
(c)

(2 marks)

As a design engineer, you have been assigned to analyze a proposed optical link as illustrated
in Figure Ql. However the task is limited by the information given below.
- An optical source in the transmitter module has an rms spectral width of AX nm.
The optical fiber which is a step index fiber with a core refractive index 1.35 and relative
refractive index difference of 1%.

Length: 15 km
Figure Ql
By making reasonable assumption(s), estimate the value of AX, if the maximum achievable
bit rate of the link is 1 Mbps.

(13 marks)

-3SET 4523
Q2. (25 Marks)

a)

Describe how dispersion limits the bandwidth of a silica optical fiber link.

(5 marks)

b)

Suggest three methods to increase the transmission bit rate of a single mode silica optical fiber
link.
(5 marks)

c)

An optical link is required between two cities of 100-km apart. The link is intended for the
distribution of broadband digital signal to 10 premises connected to the receiving node of the
link.

i)

Draw a possible layout of the system indicating components chosen.

(5 marks)

ii)

Explain the reasons in deciding on the choice of the components.

(5 marks)

iii) Estimate the minimum bit rate of the digital signal delivered to each premise. (5 marks)

-4SET 4523

Q3. (25 Marks)

(a)

Atoms and molecules in the atmosphere scatter light in the same way that atoms in glass scatter
light in an optical fiber. The shorter the wavelength, the stronger will be the scattering. Where
do you think the sky gets its blue colour and justify your answer?

(b)

(4 marks)

You need to design a digital link to connect two points of 50 km apart. The bit rate needed is
40 Mbps with BER = 1012 indicated in Figure Q3.
Determine whether the components listed are suitable for the link in terms of;
(i) Power budget

(7 marks)

(ii) Rise time budget, assuming NRZ is used.

(7 marks)

Source and detector connector:


2dB per connector

Dispersion 1 ns/km
Figure Q3

(c) Estimate the link signal to noise ratio (SNR) if the load resistor, Rl given is 7kfL (7 marks)

-5SET 4523
Q4. (25 Marks)

(a)

Explain how light is generated in a light emitting diode (LED).

(5 marks)

(b)

Discuss in detail the reasons why an LED is not suitable for a broadband optical link. (5 marks)

(c)

You are required to analyze the capability of a 25-km high bit rate optical link. It uses a
suitable optical fiber as the channel. The available components are as given in Table Q4.

(i) Estimate the bit rate of the link.

(8 marks)

(ii) Calculate the maximum link length if the data rate is not critical.

(7 marks)

Table Q4 Photodiode and Laser Specification


Parameter
Photodiode
Wavelength
APD Responsivity
Sensitivity
Rise time
Dark current
Laser diode
Output power
Wavelength range
Spectral width
Risetime

Symbol

Test Condition

Min

Typical

Max

Unit

1250

1620

R-apd

X =1550 nm
X=1310 nm

P rmin

tdet

Idark

10
8.5
-34
0.1
20

nm
A/W

Pout

AX
tsource

0.5
15201580
3
0.5

dBm
ns
nA

mW
nm
nm
ns

-6-

SET 4523
Q5. (25 Marks)

(a)

Provide and explain the source of attenuation in a silica optical fiber with regard to the
wavelength used.

(b)

(5 marks)

Consider a 300km telephone system as shown Figure Q5 below, with splice every 10km with
loss of 0.1 dB. The cable has an attenuation of 0.25dB/km at 1550nm. It links two east coast
cities, Kuantan to Kota Baru, in a step index single mode fiber carrying a single wavelength
at 2.5Gbps. The links uses two optical amplifiers placed 100km apart with each having gain
of 30dB. Both switching offices and optical amplifiers are connected with 1 dB connector
termination at each end. Assuming that the chromatic dispersion of the fiber is specified at
3ps/nmkm at 1550nm. Both transmitter and receiver have similar rise time of lOOps and the
PIN photodetector sensitivity is at 40dBm.

300 km single mode fiber with 2.5 Gbps one wavelength


4----------------------------------------------------------------------------------

Kota Bharu
Switching
Office

30dB

Kuantan
Switching
Office

Figure Q5

(i) Estimate the system power margin if coupled 0 dBm light source.

(10 marks)

(ii) Estimate the system rise time budget in considering Fabri Perot laser with line width of 1 nm
and Distributed Feedback laser of 0.1 nm.

(10 marks)

-7SET 4523
Appendix
General
NA = na sin 6a - w, (2AY2; A =

\r <a
U;

n, (l - 2 A)05 =n2;r>a
f
/\2
r
NA2=n,(2 &f'
1a
V K y y
Attenuation

V = NA\VC =2.405
AG
Mg =

ni

1-2A

(r) =

a
or + 2 a"2 ;

*7c-gw = 0.5 NA2;tjc_S! = NA:

r=l. 7(0.85/2)'
=4.4x10_3^2A2^5
P* =5.9x10-2d2Xam

A Vc
Dispersion
Du =122(l-AmM)

Noise

,a2z,

v^y
,. = R.P=mip

Tr

2 , 2 l>2
= ( cr + cr

R,

Rl
B=

CT = C , + C
SNRpm =

2 nRLCT

4 KTBFn
2 sB{l p +1 d ) +
*L

e
ex.
n = (r^/vXi/e)=rJiEt
i = (p./p)=>i(e,/v)

AKTBFn

. 2. ..
vr I Vamp

^C/

J, /

= ^7ini ~ = 7in, 7

he

1.24^73

Optical Sources/Detectors
A = (l.24/s)
Pin,

i10.5

ff! = M/P+/D)i

(',-'.)/i = AM72ni|
5r = 0.2/<x

SNR APD
2eB{lP+Id)M:

r, = PMh'x7

4 KTBFn
RlM

B = f/sl2
7-w=i.i(rs2+7-F!+r0!+r/)
f-W/^

=i/Vi+W2
1

p, =p+cl + m + dl
BER = 0.5 erfc

/SNR05^

2V3
r=

Ll = T*l
P0
he

E = hf = he/ /I
tr=2A9R,Cd
C J=^-

Bm =

1
drift

5TS =

Zn,A
C

Constant
Boltzman Constant, /C=l .38x1 O23 J/K
Electron Charge, e=1.6 xlO'19C
Planck's Constant, h=6.66xl034 Js
T=27C =300K

CONFIDENTIAL

UTM

UNIVERSITI TEKNOLOGI MALAYSIA

FINAL EXAMINATION SEMESTER II


SESSION 2010/2011
COURSE CODE

SEM 4333

COURSE NAME

MECHATRONICS SYSTEMS DESIGN

LECTURERS

PROF. MADYA DR. ROSBI BIN MAMAT

PROGRAMME

SEM

SECTION

01

TIME

2 HOURS 30 MINUTES

DATE

28 APRIL 2011

INSTRUCTION TO CANDIDATE

ANSWER ANY FOUR (4) QUESTIONS ONLY.

THIS EXAMINATION BOOKLET CONSISTS OF 11 PAGES INCLUDING THE FRONT COVER

2-

SEM4333

SI.
a)

Jelaskan dengan ringkas satu (1) faedah dan satu (1) keburukan pendekatan
mekatronik di dalam membangunkan sesuatu produk.
Briefly describe one (1) advantage and one (1) disadvantage of mechatronic
approach for developing a product.
[6 MARKAH/marAs]

b)

Satu motor arus terns dengan kawalan angker boleh diwakili oleh litar
elektromekanik yang ditunjukkan di dalam Rajah S 1.1. Voltan masukan va(t) akan
menghasilkan daya kilas T(t) yang berkadar terns dengan arus angker i (t):
A DC motor with armature control can be represented with an electromechanic
circuit as shown in Figure Sl.l. The input voltage va(t)will produce a torque
T(t) which is proportional to the armature current i (t):
m=Ktia{t)
Daya kilas ini seterusnya menghasilkan sudut putaran aci 9(t) manakala pusingan aci
pula menghasilkan voltan balik di dalam litar angker:
The torque will produce a rotational angle 8{t) of the shaft while the shaft rotation
will produce reverse voltage in the armature circuit:
vb(t) = Kb9(t)
i)

Dengan menggunakan analogi galangan, dapatkan hubungan va,vb,ia di dalam


litar angker. Unsur pasif sistem putaran mekanik diberikan di dalam Rajah S1.2.
Using the impedance analogy, determine the relations between va,vb,ia in the
armature circuit. The passive elements for mechanical rotational system are
given in Figure SI. 2.
[5 MARKAH/marfcs]

ii)

Jika nilai aruhan (La) di dalam litar elektrik dan geseran (B) boleh diabaikan,
tunjukkan bahawa rangkap pindah bagi motor arus terns dengan kawalan
angker diberikan oleh:
[6 MARKAH/mar&s]
If the inductance (La) in the electric cicuit and the friction (B) can be ignored,
show that the transfer function for the armature control DC motor is given by:
0{s)

Kt

Va(s) s(RaJs + K(Kb)


iii)

Tuliskan rangkap pindah bagi motor arus terus ini jika motor ini mempunyai
parameter-parameter berikut:
Give the transfer function for the DC motor if the motor has the following
parameters:
[2 MARKAH/marta]
Ra = 20Q, K, = IN.mA"1, Kb = SV.s.rad-1, J = 2N.m.s2.rad~1.

3-

SEM4333

iv)

Lukiskan gambarajah blok penyelakuan bagi motor arus terns dengan parameter
yang diberikan di dalam bahagian iii) di atas jika blok-blok penyelakuan yang
ada hanyalah blok-blok yang ditunjukkan di dalam Rajah SI .3.
Draw the simulation block diagram for the DC motor with given parameters as
in part iii) above if the only available blocks are as shown in Figure SI. 3.
[6 MARKAH/marfo]

Ra

La

Elektrik

Mekanik

Rajah Sl.l: Motor arus terus dengan kawalan angker


Figure Sl.l: Armature control DC motor

Peredam/
> Damper
Inertia
T(t) = J 6(t)
T daya \d\asl torque
J inersia/inertia

01

02

T{t) = B(ei(t)-62(t))
T daya kilas/torque
B pemalar likalJviscousity constant

Rajah S1.2: Unsur pasif sistem putaran mekanik


Figure SI. 2: The passive elements for mechanical rotational system

Penjumlah/Summer Pengamirllntegral Gandaan/Gain


Rajah S1.3: Blok-blok penyelakuan yang ada
Figure S1.3: Available simulation blocks

4-

SEM4333

S2.
a)

Jelaskan perbezaan cara kerja di antara pengekod-pengekod digital berikut yang


digunakan sebagai penderia pengesan kedudukan:
Explain the difference in the working of the following digital encoders used for
position sensors.
[8 MARKAH/marfa]
i) Pengekod mutlak (Absolute encoder)
ii) Pengekod menokok (Incremental encoder)

b)

Satu pengekod menokok dengan cakera optik yang ditunjukkan di dalam Rajah S2.1
digunakan untuk mengesan halaju putaran satu motor arus terns. Isyarat yang
diperolehi dari pengekod tersebut juga ditunjukkan di dalam Rajah S2.1. Kirakan
halaju putaran motor tersebut di dalam unit putaran per minit (rpm).
An incremental encoder with optical disc shown in Figure S2.1 is used to measure
the speed of rotation of a DC motor. The signal obtained from the encoder is also
shown in Figure S2.1. Calculate the speed of rotation of the DC motor in revolutiion
per minutes (rpm).
[7 MARKAH/marfcs]

0.2 saat

Rajah S2.1: Cakera optik dan isyarat keluaran pengekod menokok


Figure S2.1: Optical disc and output signal from an incremental encoder
c)

Rajah S2.2 menunjukkan sebahagian dari pemacu motor arus terns jenis L298 di
dalam konfigurasi tetimbang-H yang digunakan untuk kawalan dwihala motor
menggunakan isyarat bermodulat lebar denyut (PWM).
Figure S2.2 shows part of DC motor driver type L298 in the H-bridge configuration
which is used for bidirectional motor control using pulse-width modulated (PWM)
signal.
i)

Pin manakah pada pemacu L298 ini yang perlu disambungkan kepada isyarat
PWM ?
[4 M A R KA HI marks \
Which pin/pins on the L298 driver are required to be connected to the PWM
signal ?

ii)

Terangkan hubungan di antara halaju motor dengan lebar denyut isyarat PWM
tersebut.
[6 MARKAH/marto]
Explain the relation between the speed of motor with the width of the PWM
signal.

5-

SEM4333

Inputs

Rajah S2.2: Pemacu motor L298


Figure S2.2: L298 Motor Driver

Forward
Reverse
Fast Motor Stop

o
11
X

o o
II II
X o

r~

3
II
. = Low

Function

C = H;D = L
X
II
Q
_J
II
O

Ven H

H = High X

Free Running
Motor Stop
= Don't care

6-

SEM4333

S3.
a)

Terangkan satu (1) faedah dan satu (1) keburukan menggunakan PC terbenam
seperti PC 104 sebagai pengawal terbenam di dalam satu pencetak wama harga
rendah.
[4 MARKAH/marfcs]
Explain one advantage and one disadvantage of using embedded PC such as PCI 04
as the embedded controller in a low cost color printer.

b)

Satu penderia tekanan yang menghasilkan voltan keluaran berbanding dengan


tekanan yang diukur mempunyai ciri seperti yang ditunjukkan di dalam Rajah S3.1.
A pressure sensor which produce output voltage with respect to the pressure
measured has a characteristic as shown in Figure S3.1.
i) Cari julat dinamik bagi penderia tekanan ini.
Find the dynamic range for this pressure sensor.

[4 MARKAH/marfcs]

ii) Penderia ini akan digunakan untuk mengukur tekanan bagi julat 20 - 75 kPa.
Dapatkan rangkap pindah penderia ini yang menghubung voltan keluaran (Volt)
dengan tekanan (kPa) yang diukur menggunakan ciri biasa (TYP).
This sensor will be use to measure pressure in the range of 20 - 75 kPa.
Determine the transfer function for the sensor which relates the output voltage
(Volts) and the measured pressure (kPa) using the typical characteristic (TYP).
[6 MARKAH/wifl^s]

ID

ui a vu a o o w o uj o o w
T-CMCM(OCOTt^ii3in0(DsNoOOOO)O>OO

Pressure (ref. to sealed vacuum) in kPa


Rajah S3.1: Plot Ciri bagi satu penderia tekanan
Figure S3.1: Characteristic plot for a pressure sensor

CM

7-

SEM4333

c)

Satu penukar analog ke digital jenis 8 bit dengan julat masukan skala penuh 0-5
volt digunakan untuk membaca keluaran penderia tekanan yang dinyatakan di dalam
bahagian b) di atas.
An 8-bit analog-to-digital converter with full-scale input range of 0-5 volts is used
to read the output of the pressure sensor described in part b) above.
i)

Kirakan perubahan voltan minimum yang dapat dibaca oleh penukar analog ke
digital ini.
Calculate the minimum voltage change that can be read by the analog-to-digital
converter.
[3 M A RKA HI marks]

ii)

Kirakan perubahan tekanan (kPa) minimum yang dapat dibaca oleh penukar
analog ke digital ini jika keluaran penderia tekanan itu disambungkan secara
langsung kepada penukar analog ke digital.
Calculate also the minimum pressure change that can be read by the analog-todigital converter if the sensor output is directly connected to the analog-to-digital
converter.
[3 MAR KA HI marks \

iii)

Rekakan satu litar penyesuai isyarat yang sesuai untuk membolehkan penukar
analog ke digital ini dapat membaca tekanan bagi julat 20 - 65 kPa dengan skala
penuh 0 - 5 V.
Design a suitable signal conditioning circuit such that the analog-to-digital
converter can read the pressure for the range of 20 - 65 kPa with full-scale
input ofO-5V.
[5 MARKAH/ma/Tu]

SEM4333

S4.
Sudut pergerakan satu lengan robot boleh dimodelkan oleh rangkap pindah berikut, di
mana 6 ialah sudut pergerakan lengan dan V ialah voltan masukan kepada motor yang
mengawal lengan tersebut:
The angle of movement for an arm robot can be modelled by the following transfer
functiion, where 6 is the angle of arm movement and V is the input voltage to the motor
that controller the arm.
pKS)

V(s) s(s2 +13s + 40)

a)

Rekakan satu pengawal terbitan berkadaran bagi mengawal sudut pergerakan robot
tersebut yang memenuhi prestasi berikut: peratus lajakan PO < 5%, dan tiada ralat
keadaan mantap. Gunakan kaedah londar punca. Tunjukkan langkah rekabentuk anda
dengan jelas. Rangkap pindah bagi pengawal terbitan berkadaran adalah seperti
berikut:
Design a proportional derivative controller for controlling the angle of movement for
the arm robot which satisfies the following performance: percentage overshoot
PO < 5%, and no steady-state error. Use the root locus method. Show your design
steps clearly. The transfer function for the proportional derivative controller is:
[14 MARKAH/warfa]
Gc(s) = Kc(l + Tds)

b)

Jika pengawal terbitan berkadaran di dalam bahagian a) di atas hendak dijadikan


pengawal digital secara perisian dengan tempoh persampelan T = 10 milisaat,
dapatkan persamaan pembeza yang memberikan algoritma pengawal digital tersebut.
If the proportional derivative controller in part a) above is to be converted into a
digital control software with a sampling period of T = 10 miliseconds, determine the
diiference equation that gives the algorithm for the digital controller.
[11 M A RK AH/mflrAv]

9-

SEM4333

S5.
Satu sistem elektromekanik menghasilkan keluaran seperti yang ditunjukkan oleh Rajah
S5 apabila diberikan satu masukan langkah (Am = 0.8volt) di dalam keadaan gelung
terbuka. Satu pengawal kamiran berkadaran (PI) perlu direka bagi sistem elektromekanik
ini. Rangkap pindah bagi pengawal PI diberikan oleh:
An open-loop electromechanical system produced output as shown in Figure S5 when
given a step input (Au = 0.6 volts). A proportional integral controller (PI) is to be
designed for this electromechanical system. The Transfer function for PI controller is
given by:
Gc(s) = Kc( 1+-L)
Tts
Di

mana

Kc,Ti

adalah

parameter

pengawal

PI

yang

perlu

dicari

bagi

sistem

elektromekanik ini.
Where Kc,Ti are PI controller parameters to be determined for this electromechanical
system.

Rajah S5: Sambutan masa sistem elektromekanik dengan masukan langkah


Figure S5: Time response of the electromechanical system with step input

11-

SEM4333

Lampiran 1: Ukuran Prestasi Sistem Terkawal


Attachment 1: Controlled System Performance Measures
Bagi sistem terkawal dengan sambutan masa sistem tertib kedua:
For a controlled system with second order time-response:
C ( s ) _ ______________
G)
R ( s ) s2 + 2ga>ns + co2
Masa Menaik (Rise Time): Tr

1.8

bagi 0.5 < < 0.7


n

Masa Puncak (Peak Time)'. T =


CD

Masa Mengenap (Settling Time): Ts

bagi ralat keadaan mantap (for

steady state error) 2%

0 0.1 0.2 0.3 0.4 0.5 0.6 0.7 0.8 0.9 1.0
c

Hubungan peratus lajakan (PO) dengan nisbah redaman g


The relation between percentage overshoot (PO) with damping ratio C

Lampiran 2: Rumus Penalaan Pengawal PI


Attachment 2: PI Controller Tuning Formula
K.
Model Process: Gn (s) =--------
P v +1

Pengawal: Gc(s) = Kc (1 + ~)
TlS

2.5r
K

II

PI: Kc=----------------------- p-

CONFIDENTIAL

UTM
UNIVERSITI TEKNOLOGI MALAYSIA

FINAL EXAMINATION SEMESTER II


SESSION 2010/2011
COURSE CODE

SEE 3223

COURSE NAME

MICROPROCESSOR

LECTURERS

ASSOC. PROF. HARUN BIN ISMAIL


DR. IZZELDIN IBRAHIM MOHAMED
ABDELAZIZ
MR. KAMAL BIN KHALIL

PROGRAMME

SEC / SEE / SEL / SEM / SET / SWB

SECTION

01-02/10

TIME

2 HOURS 30 MINUTES

DATE

08 MAY 2011

INSTRUCTION TO CANDIDATE

THERE ARE 2 SECTIONS IN THIS SCRIPT:


SECTION A (COMPRISING OF QUESTION 1,2
AND 3) AND SECTION B (COMPRISING OF
QUESTION 4, 5, AND 6).
ANSWER ONLY TWO (2) QUESTIONS FROM
EACH SECTION (A TOTAL OF FOUR
QUESTIONS).
CANDIDATES ARE ALLOWED TO BRING INTO
THE EXAM HALL THE FOLLOWING
HARDCOPY:
THE MC68000 16-/32 BIT MICROPROCESSOR
PROGRAMMING REFERENCE CARD.

THIS EXAMINATION BOOKLET CONSISTS OF 16 PAGES INCLUDING THE FRONT COVER

-2SEE 3223
QUESTION 1 (SECTION A)
Would you agree the terms CPU, Processor and Microprocessor in modem terms
refer to the same entity?
Yes

u.

No

(1 marks)
Please indicate below, would you use a microprocessor or a microntroller in each of
the application below?
PDA
Home Appliance
(2 marks)

in.

In 68000, what does programmers model refer to? Please tick in appropriate box.
1. The complete instruction set
2. The 68k internal resources available
for programming
3. The external resources
(3 marks)

IV.

The important thing that makes programming powerful and interesting is the ability to
provide decision making and branching. Name three instructions that provide the in
the 68K instruction set that provide the capabilities?
(2 marks)

v. Name the programming language for 68000 that is taught in class?


(2 marks)
VI.

What is required in order to convert the source code into the object code or machine
code?
(2 marks)

Vll.

Label the signals that are used in memory control logic shown in Figure Q1 (a)
R/W*

UDSh
LOWR*

>
(3 marks)
Figure Q1 (a)

-3SEE 3223

Vlll.

Indicate in the Figure Q1 (b) below how you would divide the 68000 addressable
memory spaces into 8 equally sized blocks.
74138

(4 marks)
Figure Q1 (b)
IX.

State the type of address decoding technique used in question (viii) above.
(2 marks)

x.

What is the size of each block of memory defined in question (viii) above.
(2 marks)

xi.

Whenever a subroutine is used, an area in memory is allocated. Name the area


allocated and in what device it is implemented?
(2 marks)

-4SEE 3223

QUESTION 2 (SECTION A)
The content of some registers and memory locations are shown in Figure Q2. Preceding
execution of instruction has effect on subsequent execution of instruction. Write down your
answers in the spaces provided. Remove these pages and insert in your Answer Book.
Register
D5
0000 3000
D6
0000 0004
D7
9999 7777
AO
0000 3000
A1
0000 3002
A2
0000 3004
A7(SP) 00FF FFFE
Memory
Address Contents
$003000 1234
$003002 5678
$003004 9101
$003006 1213
$003008 1415
$00300A ABCD
$00300C 1B1C
$00300E EF10
$003010 AABB

Memory
Address
Contents
00FF FFF0
1122
00FF FFF2
5566
00FF FFF4
3245
00FF FFF6
6798
00FF FFF8
ABCD
OOFF FFFA
DEED
OOFF FFFC
7532
OOFF FFFE
9367
0100 0000
4465

Figure Q2
a. Determine the content of the registers indicated below.
(4 marks)
MOVE.L ( S P ) + , D 7

D7
A7
b. Determine the content of memory location affected following the execution of the
instruction below.
(6 marks)
MOVEM.W D5-D7/A0-A2, -(SP)
Address

Contents

-5SEE 3223

c. What is the content of registers after the execution of the instruction below
(6 marks)
MOVE .L (SP)+, D0-D1/A3
Registers
DO
DI
A3
A7

Contents

d. The following instructions are executed in sequence, determine the corresponding


affected registers?
___________
SWAP DO
DO
EXG D1,D5
D5
EOR D0,D7
D7
NEG D7
D7
NOT D5
D5

(5 marks)

e. Determine the content of the destination after the execution of each of the following
instruction and also the addressing mode of the source operand.
(4 marks)
Content of
Destination
MOVE.W (A2),D0
MOVE.B 1(A1),D5
MOVE.W 2(A2,D6.W),D7
MOVE.W -(SP),D6

Addressing Mode
of Source

-6SEE 3223
QUESTION 3 (SECTION A)
a. Trace the execution of each of the instruction in the program below, and answer the
questions that follow.
(10 marks)

START

ULANG

FIRST
LEN
DEST

ORG

$400

LEA

FIRST,A0

LEA

DEST+(LEN*2),A1

MOVE.W

#LEN,DO

MOVE.W

(A0) +, -(A1)

SUB. W

#1, DO

BNE
BRA

ULANG

ORG

$1000

DC. W
EQU
DS.W
END

100,200,300,400,500,600
(*FRIST)/2
LEN
START

i)

What is the address of the label START?

ii)

State the address of the label FIRST

iii)

What is the value of LEN transferred to DO?

iv)

What is the content of DO at the end of the program execution?

v)

Determine the address and the content of the location DEST


when the program terminates.
0000100C

b. Given the assembly listing below, trace the following program by filling the
accompanying Table Q3. Write your answer in the boxes provided so remove these
pages and insert in your Answer Book. Assume initial content of all registers are
zero, the SP = $01000000 and [SP] = FF FF FF FF.
(10 marks)
00001000
00001000 43F9 00003000
00001006 45F9 00003008
0000100C 3229 0002
00001010 3F01
00001012 4EB9 00001020
00001018 3480
0000101A 103C0009
0000101E 4E4F
00001020
00001020 302F0004
00001024 COCO
00001026 4E75
00001028
00003000
00003000= 000A 0014 001E 0028
00003008
0000300C

1 START
2
3
4
5
6
7
8
9
10
11 FIRST
12
13
14
15
16 DATA
17 ANS
19

ORG $1000
LEA DATA.A1
LEA ANS,A2
MOVE.W 2(A1),D1
MOVE.W D1,-(SP)
JSR FIRST
MOVE.W D1,(A2)
MOVE.B #9,DO
TRAP #15 I
MOVE.W 4(SP),D1
MULU D1.D1
RTS
ORG $3000
DC.W 10,20,30,40
DS.L 1
END START

7SEE 3223

Trace
No.

PC before
Execution

00001000

Next Instruction

PC after
Execution

Initial condition
lea data,al

00001006

A1

A2

DI

SP

[SP]

00000000

00000000

00000000

01000000

FFFFFFFF

00003000

00000000

00000000

01000000

FFFFFFFF

2
3
4
5
6
7
8
9
10
11

Table Q3
(Remove this page and insert in your Answer book).

-8SEE 3223
c.

A 64 bit value can be created by concatenating two 32 bit register pairS such
as D1:D0. Show by writing a short program code, how this 64 bit value can
be divided by the value 4. (Note: Unsigned number is assumed)
(5 marks)

-9SEE 3223
QUESTION 4 (SECTION B)
a. i) To initiate system initialization (system reset) for a 68000 system, two lines
must be held low for specified period of time. What are they?
(1 mark)
ii) What is the minimum period of time specified in question i) ?
(1 mark)

iii) What is the signal used to notify the 68000 when there is a problem with the
current bus cycle?
(1 mark)

iv) The HALT* line is bidirectional. If the line is driven by an external device to
its active state, what will happen to the 68000?
(1 mark)
v) If the HALT* line is activated prior to the BERR* becoming activated, what
will be the action taken by 68000?
(1 mark)
vi) If the HALT* line is not activated prior to the BERR* becoming activated,
what will be the action taken by 68000?
(1 mark)
vii) If, during a normal bus cycle, the VPA* input is activated, what does this
indicate to the 68000?
(1 mark)
viii) Determine the state of the asynchronous bus signal when the following
instruction is executed:
(1 mark)
ADD.B DO,$2003
b. Given the memory map of a certain 68000 microprocessor system (figure Q4),
determine the boundary addresses and the size of device space as required below.
(7 marks)
000000
Device A
0FFFFF
Device B

13FFFF
AFFFFF

Figure Q4
1 x 8 bit switch
2 x 8 bit leds

-10SEE 3223
i)

State the type of device used for device A.

ii) State the chip number and the number of chips required to implement device
A space.

iii) What is the starting address for device B?

iv) State the type of device used to implement device B.

v) State the chip number and the number of chips required to implement device B
space.

vi) What is the starting address of the I/O devices space?

vii)If the I/O devices are placed at odd addresses and each I/O device only
requires 1 address, what is the end address of the I/O devices space?

c. Given the following program code, trace the code and show the bus activity for
instructions at line 2 to line 7, by filling Table Q4 given below. Bus activity for
bus cycles 1 and 2 are given.
(10 marks)
00001000
00001000
00001006
0000100C
00001010
00001012
00001014
00001016
0000101A
00002000
00002000=
00002008
00002008
00002008

41F9
43F9
3328
3011
D159
30E1
103C
4E4F

00002000
0000200C
0002

0009

000A 0014 0028 0032


=00000004
=0000200C

1
2
3
4
5
6
7
8
9
10
11
12
13
14

START

DATA
LEN
TARGET

ORG
LEA
LEA
MOVE.W
MOVE.W
ADD. W
MOVE.W
MOVE.B
TRAP
org
DC.W
EQU
EQU
END

$1000
DATA,A0
TARGET,Al
2 (A0 ) , -(A1)
(Al),DO
DO, (Al) +
-(Al), (A0) +
#9,DO
#15
$2000
10,20,40,50
(*-DATA)/2
* + LEN
START

-11SEE 3223

a[23:0]

0000
1000

0000
1002

d[15:8]

41

00

d[7:0]

F9

00

LDS*

'L'

'L'

UDS*

'L'

'L'

R/W*

'H'

'H'

Bus cycle

11

12

Bus cycle

10

13

14

15

16

17

18

19

20

a[23:0]
d[15:8]
d[7:0]
LDS*
UDS*
R/W*

Table Q4

-12SEE 3223

QUESTION 5 (SECTION B)
Suppose an MC68000 program contains the instruction TRAP#3. And suppose that
after assembly this instruction is located at memory address $044000. Just prior to
execution of this instruction
[SSP] = $00016030
[SR] = $0504
a.

Execution of the TRAP #3 instruction causes the processor to jump to an


exception service routine. At what address does the processor find the address
of the exception service routine?
(3 marks)

b.

Was the processor in supervisor mode or user mode before the exception
occurred?
(2 marks)

c.

Execution of the TRAP instruction causes a context save that is the same as the
context save for hardware interrupts. Complete the following table of the
supervisor stack following the context save, but before the beginning of the
exception service routine. Indicate which address SSP points to. Note that each
location corresponds to a word whose value you should indicates in
hexadecimal notation. If the content of a certain location cannot be determined
from the given information, indicate this using "XXXX"
Address
$16028
$1602A
$1602C
$1602E
$16030
$16032
$16034
$16036
$16038

Contents

(10 marks)
d.

At what address will the processor resume operation after RTE instruction of
the exception service routine?
(2 marks)

e.

Suppose the next instruction to be executed after the exception service routine
complete is BLT instruction. Will the branch be taken?
(2 marks)

-13SEE 3223
What levels of hardware interrupt could cause the TRAP #3 exception service
routine to be interrupted, given the above conditions? Note that the execution
of the TRAP instruction has no effect on the interrupt mask.
(3 marks)
Assume that a vectored device supplies vector number $44 to the 68000
processor during an interrupt acknowledge cycle. At what address does the
processor find the address of the ISR?
(3 marks)

-14SEE 3223
QUESTION 6 (SECTION B)
A dot matrix display can be constructed using conventional LEDs driven by
74LS374 latches to a 5x7 character pixel. A column of the pixel consists of 7
LEDs and can be driven by one 74LA374 latch. A column pattern can be
enabled by writing to the address of the latch location. Thus a 5x7 led display
will be driven by 5 latches at different address locations (consecutive). A
panel of 3 5x7 display will be driven by 15 latches. Taking the base address
of the panel display as $B00000 and using partial address decoding, suggest
an address decoding circuit for the intended display.
(14 marks)
<

oooo ooo
OOOO 90009

oooo ooo
oooo
#ooo
oooo o

Ll

.L15

Figure Q6.

b. In order to display a certain text or message, the appropriate column pattern


for the intended text or message has to be defined prior to the actual display.
For the word in Figure Q6 above, create the definition for the column patterns
for each of the letter in the word FUN, using the DC.B directive.
(4 marks)

For the definition created in part (b). above, write the appropriate program
code to display the word to the respective dot matrix display using the address
decoder designed in part (a), above.
(7 marks)

-15SEE 3223

Appendix

Oetal Transparent Latch


with 3-State Outputs;
Octal D-iype Flip-Flop
with 3-State Output
The 5N74LS373 cctBlJts of flight laitfces wllh 3-stats outputs Esr
tus oigsntad system application!. Th? ll^s-Ekps appear transpircnl
1d the da? (data dangss asj^dtnnouily) whan Latch Enabtii |LET| Is
HIGH. Wbsn LE Is LOW, tha data Chit: mat! tho srtip times Is
latdisd. Data appears ca tte bus wtno tbs Output Enable (OE) Is
LOW. WhonQElsHrCKthtbus Duljiut tsln tha high Impedirce state.
Tha SK7JLS74 Is ab(gtv5pri,low-pnvtiirOctalD-tjpe Fllp-Fiip
fm Hiring xaparate D-typs Iqiils fbc tath flip-flop and 3-stain outputs
for bus ottanlsd application. A buffered Oocfc (CP) ail Output:
Enable (OE) Js common In all flJp-Baps. Tha SN71LS&74 Is
manu&dutBd using advancad l^wFcmwSitoUkjrlichmbj}- arid Is
cornpattbfc' wtlh all QN'JwmtixaduclwTTL Eimliias.

SN74LS373 SN74LS374
CONNECTION DIAGRAM S P (TOP VIEW)
SN74L5174

SN?LS373

% % Di ot o* c* Q| 0* cp

*H 5i D; Ct 0| % ft fc 04 LE

nsiniriiinrimran7iri3inrirni

LijljjljjljjljjljjljjljjljjLitj
TiiEijiiiii^mEnrarLiCiis
VE 0, D| D, 0, O j D j D j C >TtB
3 C

Ftoipat wrWi H E O | D | O i D ) % ^ I b l l j W
hBtftasnafftwfe
Mansion Btacpsiltt
NOTE:

teMkvltaRacbcp.

LQADWS [Haft 3g
PIN MAKES

HIGH

LOW

D|-D[< CttblrfJfi

05IU.
OSUL.
05 ILL

tUSUL
D-&UL
R35UL

ESUi.

1SU.L

UE

LakhEiafete$teteHISH! bfil

& cut(^stKn^ox^m\Bi
Oi-O) OUpJs
H0TC5:

asm.

m\\L

TTL IHtlxti {U.L|*4|iAHU nftlOK.


TRUTH TA B LE

LS3T4

LS373
<5E

On

Dn

er

Gh

_r

_i-

Qa

2*

LE

LE

H-MWH V-akaaprLirflri
L*LCW VWa^LwwJ

Xi iiMlwUI
Zl-t^ih*F*4inDi
"IMk Ctfiwterfl^tefi fftKtodbyttmKiiiaalttOyut:CMfcii>yut03^l.

-16SEE 3223

TlWf HfiMSlX
HHSf-iS!it SiM dspsfKfer'S <Sii
the eonditktfii t tha rtria binary Mtea 4npwti and tfw shrw

NW (npMfc Tw* KttaMow *nd era

Rift ARHANGEMEMT

*tNs in*

Mi ntitm ft* Mt *wr xtr*1 3#1s os Inwrtwl wJiW*


topwling. A 24MIn <teaotf*r e*n bt imgtenMttci without
MWff** feftrtsrti tn$ * 32-i<* dMOdftr ttQuittt ftty **

fevtntr. Aft WHfef* Input N laws at a*?* ins;;? for

<S*n'mpl*Kii^ spalioniftm,
.BLOCK DIAGRAM

^FUNCTION table

H;M> le*l, t i tow i*ri. X; hwtews*

; (?. * a C s n

CONFIDENTIAL

UTM
UNIVERSITI TEKNOLOGI MALAYSIA

FINAL EXAMINATION SEMESTER II


SESSION 2010/2011

COURSE CODE
COURSE NAME

SEE 3533
COMMUNICATION PRINCIPLES

LECTURERS

DR. DAVID IAN FORSYTH


DR. MOHD HAIZAL BIN JAMALUDIN
DR. NADIATULHUDA BT ZULKIFLI
DR. NOORASMAWATI BT SAMSURI
MR. ALIAS BT MOHD

PROGRAMME

SEC / SEE / SEI / SEL / SEM / SEP / SET / SEW

SECTION

01-05

TIME

2 HOURS 30 MINUTES

DATE

25 APRIL 2011

INSTRUCTION TO CANDIDATE

ANSWER FOUR (4) QUESTIONS ONLY.

THIS EXAMINATION BOOKLET CONSISTS OF 8 PAGES INCLUDING THE FRONT COVER

2
SEE3533

(a) (i) With the aid of a diagram, briefly discuss how a communication system is
organized.

(5 marks)

(ii) Define what noise is and state two effects of noise.

(3 marks)

(b) Noise power can be modelled using Thevenin equivalent circuit. For an electronic
device operating at a temperature of 27C with a bandwidth of 10 MHz and a 100 Q
load resistance, determine:
(i) the thermal noise power in dBm.

(2 marks)

(ii) tht signal-to-noise ratio (SNRde) if the signal voltage is 10 V.

(3 marks)

(iii)the noise voltage if there are two additional resistors connected in series to the
existing one. Assume Ri = 50 Q and R2 = 75 Q.

(2 marks)

(c) A three cascaded amplifier (A, B and C) for a radio receiver system is indicated in the
Figure 1.

ABC
Figure 1
(i) Determine the minimum power at the input of the receiver, S, operating at a T, =
150K, so that the SNR0 is not less than 30 dB. Assume the bandwidth is 10 MHz
and the environmental temperature is 290K.
(7 marks)
(ii) In order to minimize the effect of noise, a suitable configuration of the cascaded
system needs to be identified. In your opinion which one of these amplifiers
should be placed at the 1st stage and why?
(3 marks)

3
SEE3533
(a) A modulating signal, Vm(t) of Figure 2 (a) with time period of Tm is transmitted from
Faculty of Electrical Engineering to Kolej Perdana using AM Double Side Band Full
Carrier (AM-DSBFC) technique. The signal carriers frequency is 1000 times greater
than the modulating signals frequency. If the carriers amplitude, Ec is equal to 20 V,
determine the followings:
(i) Equations of the modulating, carrier and modulated signals.

(6 Marks)

(ii) Modulation index, m.

(2 Marks)

(iii) Sketch the frequency spectrum of the modulated signal.

(4 Marks)

Vm(t)

Figure 2 (a) Modulating signal

(b)

A Single Sideband Suppressed Carrier (SSBSC) is an alternative to send the


information signal in Figure 2 (a).

Figure 2 (b)
(i) State the advantages of SSBSC compared to other AM techniques. (4 Marks)
(ii) From the generation circuit in Figure 2 (b), prove that only lower side band signal
will be generated.
(9 Marks)

4
SEE3533
(a) A carrier voltage given by Vc(t) = 10 sin (2nl06t + n/2) is frequency modulated by a
signal with voltage given by Vm(t) = 2 cos (2nfmt) where fm is in the range between 50
to 200 Hz and sensitivity factor, Kf =0.05 KHz/V.
(i) Determine the modulation index and sketch the spectrum when fm is at the
minimum. Repeat for the situation when fm is at the maximum.

(6 Marks)

(ii) Discuss the influence offm on FM bandwidth and spectrum. What is the minimum
required bandwidth?

(5 Marks)

(b) Figure 3 shows an end-to-end frequency modulated communication system from


transmitter to the receiver.
(i) Draw a suitable block diagram to represent A that utilizes a crystal oscillator.
(5 Marks)
(ii) Discuss the impact if the mixer stage is removed from the system

(3 Marks)

(iii) Sketch waveforms at points 1, 2 and 3 if input information is a monotone


sinusoidal signal.

(6 Marks)

Local
Oscillator

Vjt) A

~ 1 Ji
1 ..........
Frequency |/v
Multiplier, nj

pYJ Frequency
y S Multiplier, n2

J Filler
; (bpf)

Transmitter

!d
dt

31

-e-

Envelc

Detector

Receiver
Figure 3 FM Transmission System

I1

5
SEE3533
(a) Describe all three main processes in an analog to digital (A/D) conversion. (3 Marks)
(b) An analog signal contains an information signal, Vm (t) is a perfect sinusoidal signal
with a time period, T- 30s and peak-to-peak voltage of 40 V. Within this time period,
the signal is then multiplied with a pulse signal Vd(t) in order to get the sampled signal
of Pulse Amplitude Modulation(VpAM)(i) Assuming the pulse width, of Is, sampling time period, ^equals to 5s, amplitude
of 20 V and the first pulse starts at t = 2s, sketch the Vm(t), Vd(t) and VPAM(t) signals
in time domain.
(6 Marks)
(ii) Using the Vpam(0 graph, predict the amplitude for the first six samples.
(5 Marks)
(iii) Assuming the A/D converter uses 3 bits-PCM. Calculate:
a. Quantization level, L

(1 Mark)

b. Quantization interval, AV

(1 Mark)

c. Based on the graph plotted in b(i), predict the quantization value for each
sample.

(4 Marks)

d. From the calculation, prove that the sampling process follows the Nyquist
sampling theorem

(2 Marks)

(c) Sketch the output signal of the code 110101 using


(i) NRZ-L

(1 Mark)

(ii) RZ unipolar

(1 Mark)

(iii) RZ-AMI

(1 Mark)

6
SEE3533
(a) A binary data stream of bit period of 1 |is is to be transmitted on a radio link at fixed
frequency of 5 MHz using PSK modulation method.
(i) Sketch waveforms of the data stream and modulated signal at the original carrier
frequency for a bit stream of 101010.

(4 Marks)

(ii) Discuss the suitability of ASK and FSK to replace the PSK in the above system if
the requirement of carriers frequency does not change.

(4 Marks)

(iii) Briefly compare noise and bandwidth performances of ASK, FSK and PSK.
(4 Marks)

(b) Figure 5 shows a communication system that deploys multiplexing techniques. The
central office accepts traffic from four edge nodes where each node is assigned a
unique frequency and connected to 100 Basic Units (BU).
(i) State the multiplexing techniques involved in the system.

(2 Marks)

(ii) Determine the transmission rate of links 1, 2 and 3.

(9 Marks)

(iii) Label the time slots of frames originating from link 1.

(2 Marks)

Vm,,fma>=4 KHz
Vm2Jmm=4 KHz
Vm3,fmax=8

KHz

Information 1

Information 31
Information 1 to 31: 16 Kb/s each
/\
J^

>

Link 3

Central
Exchange
V/

Figure 5 Communication System utilizing Multiplexing

7
SEE3533
APPENDICES
A) Boltzmans Constant = 1.38 x 10'23 J!K
B) Trigonometry Identities

ou

cos(/?sin<un/) =

Jo{P)

'251Ji>>(P)cos2n(0t
/7=1

sin(^ sin coj) = 2]T J2n+] (/?)sin(2 +1 )coj


sin2 x + cos2 x = 1
cos2 x - s i n 2 x = cos2x
cos2 x = ~(l + cos2x)
sin2 x = -^-(l -cos2x)
sin(x y) = sin x cos y cos x sin y
cos(x y) = cos x cos y + sin x sin y
sin x sin y = [cos(x - y) - cos(x + y)\
cos x cos y

~ [cos(x - y) + cos(x + _y)]

sin x cos y = ~ [sin(x - y) + sin(x + _y)]

8
SEE3533
C) BESSEL TABLE

Modulation index

Coefficient

Sidebands(Jalursisi) - J

Jo

r1

P @ mf

2nd

-> id
J

4 th

5"

6l"

y 1 Ii

8 lh

0.25

0.98

0.12

0.5

0.94

0.24

0.03

1.0

0.77

0.44

0.11

0.02

1.5

0.51

0.56

0.23

0.06

0.01

2.0

0.22

0.58

0.35

0.13

0.03

2.4

0.52

0.43

0.20

0.06

3.0

-0.26

0.34

0.49

0.31

0.13

0.04

0.01

4.0

-0.40

-0.07

0.36

0.43

0.28

0.13

0.05

0.02

5.0

-0.18

-0.33

0.05

0.36

0.39

0.26

0.13

0.05

0.02

6.0

0.15

-0.28

-0.24

0.11

0.36

0.36

0.25

0.13

0.06

gill

0.02

CONFIDENTIAL

UTM
UNIVERSITI TEKNOLOGI MALAYSIA

FINAL EXAMINATION SEMESTER II


SESSION 2010/2011
COURSE CODE

SEM 4153

COURSE NAME

ROBOT TECHNOLOGY FOR AUTOMATION

LECTURERS

PROF. DR. SHAMSUDIN BIN MOHD AMIN

PROGRAMME

SEM

SECTION

01

TIME

2 HOURS 30 MINUTES

DATE

25 APRIL 2011

INSTRUCTION TO CANDIDATE : ANSWER ANY FOUR (4) QUESTIONS.

THIS EXAMINATION BOOKLET CONSISTS OF 14 PAGES INCLUDING THE FRONT COVER

2
SEM 4153
QUESTION 1

Figure Ql(a) shows a two-robot work cell for assembling fire extinguishers. The work cell
consists of 4 welding machines, input and output gravity actuated part feeders, complete with a
press and marriage station (as shown in Figure Ql(b)).

The fire extinguisher is assembled comprising of 4 parts namely neck, upper tank, lower tank
and skirt as illustrated in Figure Ql(c).

The robots used in the assembly task are 6 degree-of-freedom revolute robots as shown in
Figure Ql(d). Taking the robot base as the reference point (0, 0, 0), the necks of 1 pound
weight each, are located at (0.5, 2, 1), the upper tanks weighing 4.5 pounds each, at (2.5, 2, 1)
and Sub Assembly 1 at (2, 0, 2). All dimensions in feet. The robot speeds are as follow:

(a)

Straightline motion without load

= 4 feet/sec

Straightline motion with load

= 2 feet/sec

Jogging motion without load

= 10 feet/sec

Jogging motion with load

= 7 feet/sec

Sub Assembly 1 is for assembling of the upper part of the fire extinguisher, carried
out at Station 1. Describe the detailed step by step activities of the robot (for RTM
computation).

(b)

By using the RTM method, compute the cycle time to assembly the upper part of the
fire extinguish. Table Ql shows selected RTM symbols and value.

(c)

(6 marks)

(14 marks)

What would you recommend to reduce the cycle time by 30 percent ?


(5 marks)

3
SEM 4153

Table Ql: Selected RTM Symbols and Values

Element Symbol Element time, s


1

R1

S / V + 0.40 forS>K/2.5
0.40 for S'<F/2.5

Ml

Parameters
S = distance moved (ft)
V = velocity (ft/sec)
This is used for short moves

For payloads of less than 1.0 lb


S/F+0.40 for 5 >K/2.5 S = distance moved (ft)
V = velocity (ft/sec)
0.40 for S < V ! 2.5 This is used for short moves
For payloads of between 1 and 5.0 lb
S I V + 0.60 f o r S > V / 2.5
S = distance moved (ft)
V = velocity (ft/sec)
0.60 for S < V ! 2.5
This is used for short moves
For payloads of between 5 and 15 lb
S / V + 0.90 for5>F/2.5
S = distance moved (ft)
V = velocity (ft/sec)
0.90 for 5 < K / 2 . 5
This is used for short moves

4.1

SE1

0.1K

V = previous velocity (fit/sec)

7.1

GR1

0.1

Assumed to be independent of
any parameters

RE

0.1

Assumed to be independent of
any parameters

T = robot delay time

10

D = process delay time

4
SEM 4153

QUESTION 2

(a)

What are the five main benefits of using robotic spray painting as compared to a manual
process?
(5 marks)

(b)

Explain the advantages of using the inverter based welding power supply as compared to
the one using silicon-controlled-rectifier, in a welding process?
(5 marks)

(c)

A two-storey robotic arc welding work cell comprising of eight industrial robots is used
to weld car bodies as illustrated in Figure Q2. You are required to describe the
appropriate robotic arc welding system for this work cell in terms of:

i.

Suitable features of the arc welding robots

ii.

The technology of appropriate arc welding process (please provide the schematic
diagram of the arc welding set-up)

iii.

The factors that affect the control of the welding quality.


(9 marks)

(d)

You are required to set-up the implementation of synchronized motion between robot arc
welding gun and a two-axis robot-positioner unit for a robotic arc welding application.
Detail out the controller unit used, and the way synchronization will be carried out. You
may additionally illustrate it with a flow chart with appropriate explanation.
(6 marks)

5
SEM 4153

QUESTION 3

(a)

Discuss the comparative advantages and disadvantages of the different technologies for
vehicle guidance in automated guided vehicles (AGV).
(9 marks)

(b)

The Automated Guided Vehicle System (AGVS) includes load station 1 where raw parts
enter the system for delivery to any of three production stations 2, 3 and 4. Unload station
5 receives finished parts from the production stations. Load and unload times at stations 1
and 5 are each 1.5 min. Productions rates for each workstation are indicated by the
delivery requirements in Table Q3. A complicating factor is that some parts must be
transshipped between stations 3 and 4. Vehicles move in the direction indicated by the
arrows in Figure Q3.

(i)

Determine the average delivery distance, Lj.


(4 marks)

(ii)

If the vehicles operate according to the following scheduling rule to minimise the
distances the vehicles travel empty: Vehicles delivering raw work parts from
station 1 to stations 2, 3 and 4 must pick up finished parts at these respective
stations for delivery to station 5. Determine the empty travel distances associated
with each delivery and develop a From-to Chart.
(8 marks)

(iii)

Suppose the AGVs travel at a speed of 40m/min, and the traffic factor = 0.90,
delivery distance = 103.8m, (a) determine the value of Le for the layout based on
your table, (b) how many automated guided vehicles will be required to operate
the system? Assume availability A = 100% and efficiency E = 1.0.
(4 marks)

6
SEM 4153

Table Q3: From-To Chart showing flow rates,


load/hr (value before the slash mark) and travel distances, m
(value after the slash mark) between stations in a layout
From

To
1
2
3
4
5

1
0
0
0
0
0

2
10/X
0
0
0
0

3
8/Y
0
0
0
0

4
5/Z
0
0
0
0

5
0
9/A
4/B
7/C
0

7
SEM 4153

QUESTION 4
A robotized work cell used to assemble name initial cards is shown in Figure Q4. An AGV
equipped with two-level conveyors is used to retrieve parts to be assembled, from an
Automated Storage and Retrieval System (ASRS) and transports to the robotic assembly
station.

(a)

With the aid of sketches, compare the relative advantages of the different lighting
techniques in a robotic vision system.
(6 marks)

(b) For the set-up in Figure Q4, propose an appropriate vision set-up for inspection of the ID
cards.
(5 marks)
(c)

Describe the controller architecture for the various cells consisting of SCARA robot for
assembly, Cartesian robot for automated screwing, an automated product inspection by
machine vision, and the multiple-flow conveyor.
(8 marks)

(d) Design the overall hierarchical levels of management of information flow for the set-up,
specifying the specific functions of each level. Please provide details of the type of
equipment involved if you were to implement the design.
(6 marks)

8
SEM 4153

QUESTION 5

(a) An interlock gate is used to provide safety guard for a robot system with a turn-off time of
4.5 seconds. The gate, with a height of six-foot-high requires an approach speed of 2500 mm
per second and an additional distance of 0.6 meters is needed. Determine the minimum
distance from the interlocked gate to the closest part of the robot work envelope.
[3 marks]

(b) A robot in a work cell has to be replaced. The work cell has the added distance for the safety
system is set at 1.5 feet, and a standard mechanical interlocked gate that is 3.5 feet from the
closest edge of the robot work envelope. Determine the minimum stopping time for the new
robot for the current guard conditions (assume K=8.2 ft/s for worst case situation).
[5 marks]

(c)

A robot work cell deploys a guard device in the form of a proximity laser scanner (PLS).
The articulated robot arm has a maximum work envelope, including the gripper, of 8.5 feet,
and the response time of the robot system is 2.5 seconds. Determine the safety zone radius
required for the system with a 50 percent safety factor. Use the PLS sensitivity in Figure Q5
(below) to determine the value for C, the additional distance. Also calculate the early
warning zone radius so that a minimum warning of 2.5 seconds would be provided before
the safety zone is entered.
[11 marks].

(d) The robot selected as the replacement in the previous problem cannot meet the maximum
stopping-time specifications. Describe two other design options, including hardware
specification, which would not require relocating the guard fence and gate.
[6 marks]

9
SEM 4153

Figure Ql(a): Two Robot Fire Extinguisher Assembly Work Cell

Safety fence
Upper tank
conveyor

Sub Assembly
12

Neck

Skirt

1STATION 11

Circumferential weld
12

TT

Preweld assembly
-r-Tl
f

f STATION 21

Lower tank
conveyor

Robot
Controller 1

Outpu
rSTATION 31 conveyi >r

Robot -----Controller 2

10
SEM 4153

Figure Ql(b): Layout from the rear entry perspective

Neck

Upper
part

Lower
pan

Upper tank

Lower tank

Skirt

Figure Ql(c): Part of Fire Extinguisher

Figure Ql(d): Yaskawa Motoman K30 Revolute Robot

11
SEM 4153

Figure Q2: An arc welding work cell for car body assembly, showing a four robot arrangement

12
SEM 4153

Proc
A lit.

AGV Guide Path

-+HAGV

<d-J>

40

40
30

Proc
Aut
Proc
Aut

10

t"
25

Direction
of
Vehicle
movement

I
1
<------ pU-

Unld
Man

35
40

10

f
1

Load
Man

Figure Q3: AGVS Layout for production system


Key: Proc = processing operation
Aut = automated
Unld = unload
Man = manual operation
Dimensions in meters (m)

Figure Q4: A Computer Integrated Manufacturing Robotic Cell

14
SEM 4153

Scanning range

Safety zone radius


Early warning zone radius
Surveyed area radius

Object sensitivity

70 mm (2.75 in.) at 4 m scanning range


(variable at closer range)

Response time

Less or equal to 80 mm safety zone.


Less or equal to 40 mm safety zone

Safety category

Single component failure detectability; EN


954, Category 3

Figure Q5: PLS specifications

4 m (13 ft)
15 m (48 ft)
50 m (164 ft)

CONFIDENTIAL

UTM
UNIVERSITI TEKNOLOGI MALAYSIA

FINAL EXAMINATION SEMESTER II


SESSION 2010/2011

COURSE CODE

SEE 3243 / SEE 4243

COURSE NAME

DIGITAL SYSTEMS

LECTURERS

DR. MUHAMMAD NASIR BIN IBRAHIM

PROGRAMME

SEC / SEM / SEW

SECTION

01

TIME

2 HOURS 30 MINUTES

DATE

11 MAY 2011

INSTRUCTION TO CANDIDATE

ANSWER ALL QUESTIONS IN PART A.


ANSWER TWO (2) QUESTIONS IN PART B.

THIS EXAMINATION BOOKLET CONSISTS OF 6 PAGES INCLUDING THE FRONT COVER

-2SEE 3243/SEE 4243


PART A
Ql. (25 MARKS)
(a)
A K-map for a function F is shown in Figure 1(a)

Figure 1(a)
(i)

(b)

Obtain a minimum SOP expression for the function F [5 marks]


(ii)
Realize the equation using a 16:1 MUX with E as entered variable
[5 marks]
A K-map for a function G is shown in Figure 1(b)

Figure 1(b)
(i)
(ii)

(c)

Obtain a minimum POS expression for the function G [5 marks]


Realize the equation using a 4 to 16 decoder and other basic gate
[5 marks]

Transform the circuit in Figure 1(c) into an equivalent circuit using NAND gates only.
[5 marks]

-3SEE 3243 / SEE 4243


Q2. (25 MARKS)
(a)
A and B are two 4-bit 2's complement numbers. Design a comparator circuit which has
two outputs, G and L, where the output G = 1. L = 0 if A > B, G = 0, L = 1 if A < B, and G =
0, L =0 if A = B. The output G =1, L = 1 should never occur. Use bit slice technique.

(i)

Draw in detail a block diagram which shows how the bit slices are connected to
produce the output as specified. Label the inputs and outputs of the bit-slice. All
bit slices must be the same.
[10 marks]

(ii)

Produce a complete truth table for the bit-slice

[6 marks]

(b)
The 74163 1C, shown in Figure Q2 is a standard binary counter. Design a circuit based on
74163 1C to produce a counter which counts according to this sequence:
3, 4, 5, 6,7, 8, 9, 10, 3, 4, 5,.... etc.
[9 marks]
74x163

>ctx
0 OR
--- 0 LD
EMP
EOT
---A Qa
---8 Qfi
___
C QC
D
RCO -----

Figure Q2

-4SEE 3243 / SEE 4243

PART B
Q3. (25 MARKS)
(a)
Answer the following questions:
(i)
Draw the block diagram of a Moore machine.
[2 marks]
(ii)
Draw the block diagram of a Mealy machine.
[2 marks]
(iii)
Draw the excitation table for J-K Flip flop, D-Flip flop and T flip-flop.
[6 marks]
(b)

Given the following state diagram (Figure Q3):

Figure Q3
(i)
(ii)
(iii)

Determine whether it is a Moore or Mealy machine.


[2 marks]
Determine the state table.
[5 marks]
Determine Boolean Expression of the Next State logic circuit using D Flip-flop.
Also, determine Boolean expression of the output.
[8 marks]


-5SEE 3243/SEE 4243

Q4. (25 MARKS)


Design the FSM of a Mealy machine for a clocked synchronous sequential circuit having a single
input x and a single output z such z=l if and only if the current input and the previous three
inputs correspond to either of the sequences 0110 or 1001. The 1 output is to occur at the time
of the fourth input of the recognized sequence. Outputs of 0 are to be produced at all other
times. The sequences are allowed to overlap, thus the circuit is not required to reset upon the
occurrence of the fourth input.
The circuit is to be implemented using JK flip-flops with the following state assignments: 8=111,
b=110, c=010, d=101, e=001, f=011 and g=100. The state of the input x, the output z and the
circuit state should be indicated using LEDs and a master reset R should be used to reset the
flip=flops to the initial state 'a'.
(a)

Draw the state machine.

[12 marks]

b)

Draw the state table.

[13 marks]

-6SEE 3243/SEE 4243

Q5. (25 MARKS)


Figure Q5 shows a sequential circuit based on JK flip-flop.
clock

J1

Ql

>
Ki

Ql

>2
>

q2

"D30

k2

Figure Q5
The excitation equation for a JK flip-flop is:
Q+= JQ + KQ
(a)

You are supposed to use formal method to get the state transition diagram of the
circuit.
(i) Draw K-maps for each bit of next states, and the output
[6 marks]
(ii) Draw the state transition diagram
[7 marks]

(b) The circuit is to be re-designed by replacing the JK flip-flops with D flip-flops.


(i) Get the equation for the D input of each flip-flop, as well as the output.
[6 marks]
(ii) Draw the complete circuit diagram.
[6 marks]

CONFIDENTIAL

UTM

UNIVERSITI TEKNOLOGI MALAYSIA

FINAL EXAMINATION SEMESTER II


SESSION 2010/2011
COURSE CODE

SEP 4003

COURSE NAME

PHYSIOLOGY & INTRODUCTION TO


MEDICINE

LECTURERS

DR. ADEELA AROOJ

PROGRAMME

SEP

SECTION

01

TIME

3 HOURS

DATE

30 APRIL 2011

INSTRUCTION TO CANDIDATE

SECTION A : ANSWER ALL QUESTIONS.


SECTION B : CHOOSE & LABEL TWO (2)
DIAGRAMS ONLY.
SECTION C : ANSWER FOUR (4) QUESTIONS
ONLY.

THIS EXAMINATION BOOKLET CONSISTS OF 13 PAGES INCLUDING THE FRONT COVER

SEP 4003

SECTION A (40 MARKS) (1 hour)


1. Structure that can be observed with the naked eye is called
A. gross anatomy
B. microscopic anatomy
C. macroscopic anatomy
D. cytology
2. Which type of feedback loop will result in a self-amplification response in the same
direction, rather than leading to a corrective effect?
A. Negative feedback loop
B. Positive feedback loop
C. Homeostatic control mechanisms
D. Equilibrium balancing loop
3. In eukaryotic cell, two structures that are surrounded by a double unit membrane are
A. nucleus and ribosome
B. nucleus and mitochondria
C. nucleus and cytosol
D. nucleus and smooth ER
4. Transitional epithelium can be found in the
A. urinary system
B. respiratory system
C. digestive system
D. reproductive system
5. The integrative functions of the nervous system are performed mainly by
A. afferent neurons
B. efferent neurons
C. neuroglia
D. intemeurons
6. A myelinated nerve fiber can produce action potentials only in specialized regions called
A. nodes of Ranvier
B. intemodes
C. synaptic knobs
D. axon hillock
7. Some neurotransmitters can have either excitatory or inhibitory effects depending on the
type of
A. receptors on the postsynaptic neuron
B. synaptic vesicles in the axon
C. synaptic potentiation that occurs
D. postsynaptic potentials on the synaptic knob

3
SEP 4003
8. The parasympathetic nervous system affects all of these organs EXCEPT
A. heart
B. pupillary smooth muscles
C. salivary glands
D. adrenal glands
9. The point at which an impulse is transmitted from one neuron to another neuron is called
A. dendrite
B. glial cell
C. synapse
D. terminal plate
10. Which of the structure below is caudal to the hypothalamus?
A. Thalamus
B. Corpus callosum
C. Cerebral aqueduct
D. Pituitary gland
11. The primary motor area of the cerebrum is the_________gyrus of the frontal lobe
A. precentral
B. central
C. postcentral
D. acentral
12. Cerebrum can be divided into these lobes EXCEPT
A. frontal
B. distal
C. occipital
D. insula
13. Serum is a blood plasma without
A. sodium ions
B. calcium ions
C. clotting proteins
D. globulins
14. Production process of all blood formed elements is called
A. hemodialysis
B. hemopoiesis
C. hemoglobin
D. erythropoiesis

4
SEP 4003
15. Which of these is a granulocyte?
A. Monocyte
B. Lymphocyte
C. Macrophage
D. Eosinophil
16. Which structure is related to the blood flow from the right atrium to the right ventricle?
A. Pulmonary valve
B. Tricuspid valve
C. Bicuspid valve
D. Aortic valve
17. The cardiac conduction system includes all of the following EXCEPT
A. SA node
B. AV node
C. chordae tendineae
D. purkinje fibers
18. All statements are TRUE about muscle tissue EXCEPT
A. I-band is more darker than A-band
B. Z-disc anchor the thin filaments and elastic filaments
C. sarcomere is a functional contractile unit of a muscle fiber
D. tropomyosin blocks the active sites of G actins
19. When you are lifting a box, your muscles are in_______________contractions
A. isometric and concentric
B. isometric and eccentric
C. isotonic and concentric
D. isotonic and eccentric

20. Choose the ONLY lymphatic organ which has both afferent and efferent lymphatic
vessels.
A. Spleen
B. Lymph node
C. Tonsil
D. Thymus
21. Accessory structures of the skin DOES NOT include
A. epidermis
B. nail
C. hair
D. exocrine glands

5
SEP 4003

22. The appendicular skeleton includes


A. sternum
B. skull bones
C. vertebrae
D. femur
23. Angular movements are
A. limb rotation
B. head rotation
C. including supination and pronation
D. including flexion and extension
24. Given these parts of the small intestine
I.
Ileum
II.
Duodenum
III.
Jejunum
Choose the arrangement that lists the parts in the order food encounters them, as it passes
from the stomach through the small intestine
A. I, II, III
B. Ill, II, I
C. II, I, III
D. II, III, I

25. Which of the following statement is INCORRECT about liver?


A. It is the largest blood reservoir in body
B. It has left lobe larger than the right
C. It regulates metabolism of carbohydrate, protein and lipid
D. It contains Kupffer cells
26. Which one is NOT a part of gastric juice composition?
A. Mucus
B. Hydrocloric acid
C. Intrinsic factor
D. Trypsinogen
27. Which of the photoreceptor cells is NOT correctly matched with its function?
A. Rods - light sensitive
B. Cones - color vision
C. Rods - clear images
D. Cones - fast and sharp images

6
SEP 4003

28. The fibrous tunic of the eye includes the


A. sclera
B. conjunctiva
C. iris
D. choroid
29. Which statement is FALSE regarding the ear physiology?
A. Auditory ossicles act as levers to amplify movement and sound
B. Membranous labyrinth contains endolymph
C. Inner ear function is to maintain equilibrium
D. Optic nerve is involved in hearing
30. A TRUE statement about taste buds is
A. stimulate by tastants
B. can be found on larynx
C. have highest threshold for bitter substances
D. can adapt in taste within 10 minutes
31. Based on the chemical structure, which one is NOT a protein hormone?
A. Hormones of anterior pituitary
B. Hormones of pancreas
C. Sex hormones
D. Hormones of hypothalamus
32. The plasma volume is approximately
A. two third (2/3rd) of the total body water
B. one third (l/3rd) of the total body water
C. three fourth (3/4th) of total body water
D. one fourth (l/4th) of total body water
33. Which hormone is NOT correctly matched to its function?
A. Oxytocin - milk ejection
B. Thyroid hormones - increase metabolic rate
C. Aldosterone - maintains homeostasis by regulating Na+ & K+
D. Insulin - increases blood sugar level
34. A TRUE statement about pituitary is
A. lies above hypothalamus
B. anterior lobe (adenohypophysis) is attached to hypothalamus through nerve fibres
C. secretes prolactin hormone
D. does not lie in hypophyseal fossa of sphenoid bone

7
SEP 4003

35. Smaller cartilage of the larynx includes


A. epiglottis
B. arytenoid
C. thyroid cartilage
D. cricoid cartilage
36. Principal muscle for normal quiet breathing DOES NOT include
A. intercostal muscles.
B. diaphragm
C. muscles of abdomen
D. elasticity recoil of the ribs and lungs
37. Which one is incorrectly matched regarding lung volumes and capacities?
A. Ttidal volume - volume of air in one breath
B. Residual volume - amount of volume left after expiratory reserve is expelled
C. Vital capacity - inspiratory capacity+ tidal volume
D. Total lung capacity - vital capacity + residual volume
38. Which of following is NOT a component of urinary system?
A. Ureters
B. Urinary bladder
C. Urethera
D. Adrenal gland
39. TRUE statements about kidney are
I.
removal of waste products
II.
osmosis regulation
III.
acid base balance regulation
IV.
renin-angiotensin
A.
B.
C.
D.

I
I, II & III
I & II
All of the above

40. Which is a FALSE statement regarding female reproductive system?


A. Ovary is an internal genital organ
B. Fallopian tube is an external genital organ
C. Ovary secretes estrogen and progestogens
D. Union of sperm and ova usually takes place in fallopian tube

SEP 4003

SECTION B (20 Marks) - Choose and label only 2 out of 4 diagrams


Question B1 (10 Marks) (30 minutes)

Figure B1

Question B2 (10 Marks)

10

SEP 4003

Question B3 (10 Marks)

Figure B3
1.----------------------------------------------------------------------------------------------------------------------------------------- 6.----------------------------------

7.----------------

3 .

8 .----------------

4 .

9.----------------

5 .

10.--------------

11

SEP 4003

Figure B4
1.------------------------------------------------------- -- 6.2 .

-- 7. -

3 .

-- 8. -

4 .

---9. -

5 .

---10.

12

SEP 4003

SECTION C (40 Marks) - Answer only 4 out of 6 questions (1 hour 30 minutes)


Question Cl
a. The Human autonomic nervous system (ANS) can be divided into sympathetic and
parasympathetic division. List 5 important points that differentiate both divisions.

[5 Marks]
b. Briefly explain the physiological mechanism of the visceral reflex arc in maintaining the
blood pressure homeostasis.
[5 Marks]

Question C2

Figure Cl
a. Figure Cl shows the cardiac muscle action potentials process. Discuss and elaborate what
happens during :
i. A
ii.
B
iii.
C
iv.
D
v.
E
[10 Marks]

13
SEP 4003

Question C3
a. Some of the functions of muscular tissue are related to movement, stability, communication,
control of body openings and passages, and also heat production. Discuss 5 important
characteristics of muscle tissue which are related to the activities above.
[5 Marks]
b.

Human muscle tissues can be divided into skeletal, smooth and cardiac muscle. List 5
parameters to distinguish between skeletal muscle tissues and smooth muscle tissues.

[5 Marks]

Question C4
a.

Oogenesis and spermatogenesis are process of gamete formation in females and males
respectively. List down the differences between these two processes?
[5 Marks]

b. Menstrual cycle comprises a series of changes occurring in the endometrium of the uterus.
Briefly outline the phases of female menstrual cycle.
[5 Marks]

Question C5
a. Swallowing of food consists of three different phases. Briefly discuss these different phases
of swallowing mechanism.
[10 Marks]

Question C6
a.

Inner ear is responsible for sense of hearing and equilibrium. Describe briefly 2 main
divisions of the inner ear (labyrinth).
[5 Marks]

b.

Sound amplification is the function of the middle air. Enumerate different factors and
structures involved in sound amplification.

[5 Marks]

CONFIDENTIAL

UTM

UNIVERSITI TEKNOLOGI MALAYSIA

FINAL EXAMINATION SEMESTER II


SESSION 2010/2011
COURSE CODE

: SEL 4283

COURSE NAME

: ANALOG CMOS DESIGN

LECTURERS

: PROF. DR. ABU KHARI BIN AAIN

PROGRAMME

: SEW

SECTION

: 01

TIME

: 3 HOURS

DATE

: 25 APRIL 2011

INSTRUCTION TO CANDIDATE

: PART A: ANSWER ALL QUESTIONS.


PART B: ANSWER TWO (2) QUESTIONS ONLY.
PLEASE REFER TO PAGE 7 FOR POWER
SUPPLY
VOLTAGE
(Vdd)
AND
PROCESS
PARAMETERS.

THIS EXAMINATION BOOKLET CONSISTS OF 8 PAGES INCLUDING THE FRONT COVER

SEL 4283
PART A
Ql.[18 marks]. Please refer to Figure 1 which shows an amplifier system. Assume all
transistors are in saturation region. Do not ig tore body effect but the body effect
transconductance, gmb can be ignored. Use Table 2 and solve the followings:

b)
c)
d)
e)
f)
g)

a) Calculate the size (W/L) of transistor Mi and transistor M3. [3 marks]


Calculate the minimum size (W/L) of transistor M2.
[5 marks]
Calculate the mid band gain, Avoi (Vouti/Vjn) in dB.
[2 marks]
Calculate the pole location at Vouti[2 marks]
Calculate the mid band gain, Avo2 (V0ut2A^in) in dB.
[2 marks]
Calculate the pole location at Vout2[2 marks]
From part c) to f) sketch the total frequency response of the system Avo in
dB vs. frequency. Clearly show the locations of the poles, and the fsdB
frequency.
[2 marks]

Vdd

Figure 1

3
SEL 4283

Vdd

Q2.[12]. Please refer to Figure 2. All transistors are in saturation region.


Ignore body effect. Given that Ci = 35 fF, C2 = 10 fF and C3=5 fF. Calculate
the followings:
a)

The output resistance at node Vi.

[2 marks]

b)

The output resistance at node V02..

[2 marks]

c)

The total capacitance at

node

Voi, taking

consideration
d)

The total capacitance at


consideration.

Miller effect into


[3 marks]

node

V02, taking

Miller effect into


[3

marks]
e)

Estimate the f3dB frequency.

[2 marks]

4
SEL 4283
PART B

Table 1
Sml
3 mmho

m2
400 (o.mho

Rl
200 Q

Tdsl
250 kQ

Figure 3a

Q3. [10] .Please refer to Figure 3 a and Table 1. Given that the effective
transconductance of the circuit is Gm. Gm is eqm l to:

Gm =

gm\

(1)

+ gmlRL

As a result of equation (1), the mid band voltage gain of circuit in Figure 3a is given
as:
Vout/V jn Gmrout

(2)
V DD

Vjn (7\j\

V BP

Figure 3 c

r<is2
50 kQ

5
SEL 4283
Apply the above concept and answer the following questions. In all cases, do not
approximate your calculations.
Q3.
a)
b)
c)
d)

Please refer to Figure 3b). Calculate the output resistance,


From part 3a) calculate the mid band voltage gain, Ay.
Please refer to Figure 3c). Calculate the output resistance,
From part 3c) calculate the mid band voltage gain, Ay.

rout- [3 marks]
[2 marks]
rout- [3 marks]
[2 marks]

Q4.[10 marks]. Please refer to Figure 4 and Table 2.


a)

b)
c)
d)

Figure 4a) shows a simple common gate amplifier. Determine the


operating region of the transistor and calculate its drain current.
[3 marks]
From part a) calculate the mid band gain, Vout/Vm.
[2 marks]
Figure 4b) shows a simple common drain amplifier. Transistor Mi
is operating in linear region. Calculate its bulk voltage. [3 marks]
From part c), choose the correct location of Vout and calculate the
mid band gain, Vout/Vjn.
[2 marks]

Vdd

Figure 4a)

Vdd

Figure 4b)

SEL 4283

Q5.[10 marks]. Please refer to Figure 5a), 5b) ard Table 2.

a) From Figure 5a, calculate the range of Vin which keeps transistor Mi in
saturation region. Ignore body effect.
[3 marks]
b) From part a), determine the range of drain current, Id.
[2 marks]
c) Repeat part a) for Figure 5b. Include body effect.
[2 marks]
d) From part c), determine the range of drain current, Id
[2 marks]

'DD

Vp

Rd
2

kQ

_youT
-1 V

^ IN

Ml? Rs
Vin

5/i

2ka

VV
Figure 5b

7
SEL 4283

FORMULAE

All Vdd has 3 V voltage supply

Table 2

NMOS
PMOS

KP
150 n
50 |i

VTO
0.5
-0.5

2 <p

0.4
0.4

0.6

0.01

0.6

0.02

I d = f^(j V g s | - | V t [)2 (l + X \ V d s |)

(1) -Drain current saturation region

I d = V g s \ - \ V t |)(j V d s |)

(2 ) - drain current linear region

Vt = Vro+y(j(2cp+\VBs\) -J2<p)
Vt = Vto-y(j(2(p+\VB.^) -

(3) - NMOS threshold voltage


(4) -PMOS threshold voltage

(5) -transconductance saturation region

(6 ) -drain source small signal resistance


saturation region

fCJV T/

gm --------------------------- Vds

L
K W n s t/ \
gds ----------(Vgs - Vf)

(7) --transconductance linear region

(8 ) -conductance linear region

You can simplify the equations whenever necessary

SEL 4283

Simplified method to calculate output resistance

rout

I"ds 1

1 rdsRL

(1

ILM|

Vin

M]
-c

rds\ + Rl
Tout
1 + gmWdsl

Rt

Mi

Tout
{gml + gds l)

CONFIDENTIAL

gJUTM

UNIVERSITI TEKNOLOGI MALAYSIA

FINAL EXAMINATION SEMESTER II


SESSION 2010/2011
COURSE CODE

SEM 4173

COURSE NAME

ARTIFICIAL INTELLIGENCE

LECTURERS

DR. MOHD FAUZI BIN OTHMAN

PROGRAMME

SEC / SEM / SEP

SECTION

01

TIME

2 HOURS 30 MINUTES

DATE
INSTRUCTION TO CANDIDATE :

26 APRIL 2011
THIS
EXAM
PAPER
CONSISTS
OF
4
QUESTIONS.
YOU
ARE
REQUIRED
TO
ANSWER ANY 3 QUESTIONS (25 MARKS
EACH).
TOTAL
MARKS
FOR
THIS
EXAMINATION
ARE
75%.
YOU
ARE
REQUIRED
TO
ATTACH
THIS
EXAM
QUESTION TO YOUR ANSWER BOOKLET.
ALL
EXAM
REGULATIONS
MUST
BE
STRICTLY ADHERED TO DURING THIS
EXAMINATION.

THIS EXAMINATION BOOKLET CONSISTS OF 7 PAGES INCLUDING THE FRONT COVER

-2SEM4173

Question 1

a.

Draw a biological neuron and label all its main components. Discuss the
activity that takes place in the neuron at each of the components by
emphasizing on the flow of the telectrochemical signals from input to
output.

(4 marks)
b. Explain the two major classes of learning paradigms: supervised learning and
unsupervised (self-organised) learning. Give three examples for each
network.

(8 marks)
c.

What are the problems with using a perceptron as a biological model? How
does the perceptron learn?

(4 marks)

d. i. Give two main problems related to the back-propagation learning algorithm.


ii. How can learning be accelerated in multilayer neural networks?
iii. Define the generalised delta rule.

(5 marks)

e. Three-neuron Kohonen network is given as follows:


Input vector X: X= [0.52; 0.12]
Initial weights: wi= [ 0.27; 0.81],
w2= [0.42; 0.70],
w3= [0.43;0.21],
Use a=0.2. Find the winning neuron and its updated weight.

(4 marks)

-3SEM 4173

Question 2
a. i. What is a folly connected multilayer perceptrons?
ii. Describe in details on how multilayer perceptrons learn.
(10 marks)

b.

What is a self-organising feature map? Describe with a diagram the featuremapping Kohonen model.

c.

(5 marks)

Demonstrate multilayer network learning of the binary logic function


Exclusive_OR with the following initial weights and threshold levels:
wi3=0.5, wi4=0.9,

w23=0.4,

w 35 =-1.2,

w 45 =1.1,

03=O.8, 04=-O.l and 65=0.3.

Assume a=0.1.

(10 marks)

-4SEM 4173
Question 3
a) Define the term Artificial Intelligence. Why fuzzy logic can lead to more
human intelligent machines?
(5 marks)
b) Give one difference between a crisp set and a fuzzy set?
(2 marks)
c) Define fuzzy inference. What are the main steps in the fuzzy inference
process?
(5 marks)
d) For the given membership function as shown in Fig. Q3(d) determine the
defiizzified output value by these methods.
i) weighted average method
ii) mean-max method

Fig Q3(d)
(5 marks)
e) Discuss how you would design any two of the following products to have fuzzy
inference in a real world environment
(i). Autofocus Camera
(ii). Rice cooker
(iii). Washing machine
(iv). An Inverted Pendulum
In your discussion, include the following aspects/factors:
- -A practical choice of the fuzzy input and output variables
- A practical number of quantizations and choice of linguistic labels
- Draw and indicate a reasonable number of membership functions for each
fuzzy variable
-Indicate practical values of the universes of discourse
-Three examples of rules
-The most practical choice of defuzzification technique
-Some advantages you would expect from the fuzzy logic product compared
with a conventional product
(8 marks)

-5SEM 4173
Question 4
Examine the fuzzy sets as illustrated in Fig. Q4 and solve the following questions:

B
1.

*j

0.
/
0.

0.

X'
/

0.
_z_

0.
o1

89

10

Figure Q4
a. Write down the mathematical equations of the membership functions of fuzzy
sets A, B and C.

(7 marks)
b. Based on the fuzzy sets of Fig. Q4, calculate the following:
(i).

DIL(A)

(ii).

(a)0 3 n CON(B) (subscript 0.3 denotes alpha cut set)

For questions (a) and (b),use only discrete elements in the universe, u (e.g., 0,
1, 2, 3, etc.) and membership values, fj. that to be obtained from equations in
question 3a.

(7 marks)
c. Using triangular waveforms, proof the following distributivity property of
fuzzy sets is true (show your answers step by step using several graphs).
A u ( B n C ) = ( A k j B ) c \ ( A uC)

(6 marks)

-6SEM 4173
d. Supposing: X = {xi ,x }, Y ={yi y2 y3} and Z ={z3 ,z5 }
2

Given two "fuzzy relations" P(X, Y) and Q(Y, Z) as

P(X, Y) = 0.3 0 0.7


0.8 1 0.4

Q(Y, Z ] = 0.5 1
0 0.9
0.2 0.6

Will the fuzzy relation holds? Prove by using the following fuzzy
composition:
(i). "max-min
(ii). "min-max

(5 marks)

-7SEM4173

Attachment

Fuzzy set operations

Table of Summary of Fuzzy Set Operators


Operator expressions

Equality

ugU

Union

/orallueU

Intersection

^rUw)=mH^(4Mw)}

Complement

V A (u) = \-n A (u)

Normalization

V-

Concentration

ueU

W = ^ (u))
M' con[a) ( ) (i1 a (w))
norMa)

w =

Dilation
Intensification

/orallusU

ugU
ugU

ueU
U

L\J^ua( u )) 2 f r0 ^ Ma( ) ^ 0.5


!N7i'A^ 12(l-u {ujf for0.5 < u (u) < 1
A
A
u

Algebraic product

UAe&)

foraWw gU

~ UA (M)'6'b(^)}

Bounded sum

uA(BB(u) = min(l, uA (u) + uB(u)}

Bounded product

uAeB(u) = max[0 ,uA(u)+uB(u) -1}

/orallueU

uA(u) for i4u) = 1


Drastic product

(u) = \uB(u) foruA(u) = \


0 for uA(u), uB(u) < 1

Algebraic Sum

M A is (M)

= M a ()+M b

(u)

* M a (u )M b () V u e U

Fuzzy Logic Hedges


Very F
CON(F)
More or Less F
DIL(F)
NotF
1-F

ybrallu gU

CONFIDENTIAL

UTM
UNIVERSITI TEKNOLOGI MALAYSIA

FINAL EXAMINATION SEMESTER II


SESSION 2010/2011
COURSE CODE

SEP 4043 / SWB 4043

COURSE NAME

CLINICAL ENGINEERING

LECTURERS

MR. LUKMAN HAKIM BIN ISMAIL


MR. SYED MOHD NOOH BIN SYED OMAR

PROGRAMME

SEP

SECTION

01

TIME

2 HOURS 30 MINUTES

DATE

09 MAY 2011

INSTRUCTION TO CANDIDATE

ANSWERS FIVE (5) QUESTIONS ONLY.

THIS EXAMINATION BOOKLET CONSISTS OF 4 PAGES INCLUDING THE FRONT COVER

-2SEP 4043

Question 1 [10 Marks]


a.

Explain the 3 main application of an infusion device.


[3 Marks]

b.

An ICU department in the hospital that you are working with is preparing to
purchase volumetric infusion pumps. As the Head of Biomedical Engineering
Department, you are required to develop a short report on the purchasing
considerations for the new pumps. Elaborate the features of a smart infusion pump
that need to be considered by the ICU department.
[7 Marks]

Question 2 [10 Marks]


a. Define what is nosocomial infection and list 2 various forms of it.
[3 Marks]
b. In MS 2058, user is required to decontaminate medical device before it can be sent
for maintenance or repair. Describe the 3 important steps in decontamination process
for surgical instrument set.
[7 Marks]

Question 3 [10 Marks]


a. Before performing any electrical safety test on a medical device, a few general checks
and tests need to be done first. Using MS IEC 60601-1 standard as a basis, describe in
detail these checks and tests.
[6 Marks]
b. What are the consequences that may occur if the general checks and tests are not
performed?
[4 Marks]

-3 SEP 4043

Question 4 [10 Marks]


a. Electromagnetic Interference (EMI) can cause disturbance that interrupts the function
of medical devices by disruptive electromagnetic energy that is transmitted from one
device to another. Discuss on how this energy can be transmitted and give examples
for each answer.
[3 Marks]
b.

A company has developed a magneto-cardiography and short wave diathermy


equipment for use at therapy centres in residential areas. Referring to CISPR
classification, determine the class and group for BOTH equipments. Provide
justifications for your answer.

[3 Marks]
c. Explain what is meant by RF immunity value for medical devices in critical care unit
and discuss on how to resolve electromagnetic compatibility (EMC) problems in such
area.

[4 Marks]
Question 5 [10 Marks]
a. Differentiate between electrical microshock and macroshock.

[2 Marks]
b. Referring to electrical conduction in the human body, majority of the body resistance
is in the skin. Describe the advantages of dry skin, in the event of electrical shock and
how can these advantages be diminished?

[3 Marks]
c. A build up of static electricity in an operating theatre may be hazardous. Give two
reasons why and explain each reason. To avoid above hazard, describe precautions
that are used in an operating theatre to prevent accumulation of static discharge.

[5 Marks]

-4SEP 4043

Question 6 [10 Marks]


a. Explain the following terms with examples:
i.
ii.

Scheduled maintenance program.


Unscheduled maintenance program
[4 Marks]

b. What is preventive maintenance and corrective maintenance? How would good


maintenance practice impact patient care?
[6 Marks]

CONFIDENTIAL

UTM
UNIVERSITI TEKNOLOGI MALAYSIA

FINAL EXAMINATION SEMESTER II


SESSION 2010/2011
COURSE CODE

SEU 3053

COURSE NAME

ELECTRICAL TECHNOLOGY

LECTURERS

MS FARID AH BT HUSSIN

PROGRAMME

SPE

SECTION

01

TIME

2 HOURS 30 MINUTES

DATE

25 APRIL 2011

INSTRUCTION TO CANDIDATE

ALL STUDENT ARE REQUIRED TO ANSWER:


PART A: COMPULSORY.
PART B: ANSWER THREE (3) QUESTIONS
ONLY.

THIS EXAMINATION BOOKLET CONSISTS OF 7 PAGES INCLUDING THE FRONT COVER

2
SEU 3053

PART A

Ql. (i) Sketch, label and state clearly each of the following electrical quantities for star,
(Y) and delta, (A) configurations:
a.

Phase voltage , Vp

b.

Line voltage, Vl

c.

Phase current, Ip

d.

Line current, II

[4 marks]
(ii)

Verify for both star and delta connected systems, an expression for the total
average power, P for a balanced load is given by:
P

= V3 V l I l c o s 6

Where, Vl, II and cos 0 are line voltage, line current and power factor
respectively.

[4 marks]
Q2. Explain briefly any 2 (two) of the following items:
(i)

Permeability

(ii)

Flux density

(iii)

Magnetic field

(iv)

Magnetic field strength

[4 marks]
Q3. Briefly explain the types of losses that occur in a single phase transformer.

[5 marks]
Q4. (i) What are the significant differences in terms of operation between motor and
generator?

[4 marks]
(ii)

Answer one (1) from the following items:


(a)

Describe the basic structure of electric machines.

(b)

Explain the construction of induction machines.

[4 marks]

3
SEU 3053

PART B

Ql. (a) A balanced three phase system with a line voltage of 400 V is supplying a
balanced star connected 15 kW load at lagging power factor of 0.85.
i)

Find the magnitude of phase and line current.

[3 marks]
ii)

Calculate the per-phase load impedance.

[2 marks]

(b) A balanced three-phase, delta connected load is connected to a three phase


supply. The line voltage is 400 V. The load impedance per phase is (10.8 + J5.23)
Q. Phase sequence is positive and Vry is taken as a reference,

i.

Determine the phase currents (magnitude and phase angle)

(6 marks)
ii.

Sketch phasor diagram for voltage and phase current.

[3 marks]
iii.

Determine the line currents (magnitude and phase angle)

(3 Marks)
iv.

Calculate the apparent power (S), average power (P) and reactive power (Q)
absorbed by the 3-phase load

(6 Marks)
v.

Determine complex power in the rectangular and polar form

(2 marks)

4
SEU 3053

Q2. (a) State four methods that can be used to increase the electromagnetic flux in the
Figure Q2(a).
[4 marks]

(b)

A core made of cast steel is shown in Figure Q2(b-i). The cores magnetization
curve is given in Figure Q2 (b-ii). The flux in the center limb is 3.6 x 1CT4
Weber, the cross sectional area is 2cm x 2 cm and the left limb is wound with a
wire of 300 turns.
(i)

Draw the magnetic equivalent circuit for the core.

[5 marks]
(ii)

Determine total flux produced by the magnetizing coil

[9 marks]
(iii)

Calculate the current, / needed to produce the flux in part (ii)

[7 marks]
L------- 8 cm ------------------------6 cm

Figure Q2(b-i)

5
SEU 3053

B-H Curve

H (AT/m)
Figure Q2(b-ii)

6
SEU 3053

Q3. (a) Briefly describe the transformer on no load condition with the aid of suitable
diagram.

[10 marks]
(b)

A single phase transformer rated at 15 kVA, 480/230 V, 50 Hz has resistances and


leakage reactances as follow:
Primary winding: resistance 1 Q; reactance 5 Q
Secondary winding : resistance 0.025 Q; reactance 0.05 Q
The iron core loss resistance and magnetizing reactance referred to primary,
winding are 400 Q and 360 Q respectively.

(i)

Determine all the parameters referred to the high voltage side

[4 marks]
(ii)

Draw the approximate transformer circuit

[2 marks]
(iii)

If the transformer is supplying full load at power factor of 0.85 lagging at


secondary winding with the secondary terminal voltage of 240 V, determine the
supply voltage and efficiency of the transformer in this condition.

[9 marks]

7
SEU 3053

Q4. (a) (i) Explain briefly two purposes of using transformer in power transmission
system

[3 marks]
(ii)

Describe basic structure of a transformer.

[3 marks]
(b)

Open circuit and short circuit tests are done to a single phase 20 kVA, 230/2300

V, 50 Hz transformer. The results are as follows:

Open circuit test

Short circuit test

(reading taken on low voltage side)

(reading taken on high voltage side)

V = 230 Volt

V= 230 Volt

1 = 4.5 A

I = 8.7 A

P = 350 W

P = 500 W

Based on the results, determine:


(i)

All the transformer parameters referred to the low voltage side. Then draw
the approximate equivalent circuit.

[11 marks]
(ii)

The voltage regulation at full load condition with unity load power factor,
and the secondary terminal voltage is fixed at 2300 V.

[8 marks]

CONFIDENTIAL

UTM
UNIVERSITI TEKNOLOGI MALAYSIA

FINAL EXAMINATION SEMESTER II


SESSION 2010/2011

COURSE CODE

SEE 4453

COURSE NAME

POWER SYSTEM CONTROL

LECTURERS

ASSOC. PROF. DR. MOHAMMAD YUSRI BIN


HASSAN

PROGRAMME

SEE

SECTION

01

TIME

2 HOURS 30 MINUTES

DATE

25 APRIL 2011

INSTRUCTION TO CANDIDATE

ANSWER FOUR (4) QUESTIONS ONLY.

THIS EXAMINATION BOOKLET CONSISTS OF 7 PAGES INCLUDING THE FRONT COVER

-2SEE 4453

Ql.(a)

You have been assigned as a project engineer to build a new power


plant. What are the factors that you should consider in order to generate
the power at minimum cost? Discuss briefly.
(5 marks)

(b)

A system consists of two plants, Plant 1 and Plant 2 are connected by a


transmission line. A load is located at Plant 2. Data for the loss equation
indicate that 100 MW transmitted from Plant 1 to the load results into a
loss of 10MW. Find the required economic generation for each plant and
the power received by the load when incremental cost for the system is

RM/MWh. Assume that incremental fuel costs for Plant 1 and Plant 2
can be approximated by the following equations:

Xi=0.007P,+4.1 RM/MWh
A,2=0.014P2 + 4.6 RM/MWh
(12

(c)

marks)

Find the monthly saving for economic dispatch to serve a load of 200
MW between the plants of Ql(b) compared .with equal share. Neglect
line loss.
(8

marks)

-3SEE 4453

Q2. (a) Discuss briefly any one of these trading arrangements.


i) Single Buyer
ii) Pool Trading
iii) Bilateral contract

(5 marks)

(b)

Consider 4 generators, G], G2 , G3 and G4 , operating in the single buyer


trading model with loads, Li and L2 as shown in Figure Q2(b). Each
generator has a declared capacity with capacity and energy prices as
indicated in Table Q2(b). The transmission elements are assumed to be
lossless. Each of the line has a transmission limit of 300 MW.

L| =500 MW

L2=1000 MW

Figure Q2(b) Single Buyer Operation

Table Q2(b) Generator Capacity and Prices


Generator

Declared

Capacity price

Energy Price

Capacity (MW)

(RM/MWmonth)

(RM/MWh)

G,

650

36000

120

g2

2070

36000

140

g3

2100

36000

160

g4

440

36000

180

-4SEE 4453

(i)

Design

competitive

market

based

on

pool

trading

for

unconstrained dispatch and determine the hourly income of each


generator. LOLP is assumed to be 0. State any assumptions.

(6

(ii)

marks)

Discuss the pros and cons of both single buyer and pool trading
model in term of generator income adequacy.
(6

(iii)

Determine

total

generator

incomes

in

Q2b(i)

marks)

for

(n-1)

constrained dispatch.

(8

marks)

-5SEE 4453

Q3. (a) State four (4) basic requirements for successful operation of power
system.
(5 marks)

(b)

A 210 MVA synchronous generator operates initially at 3000 rpm, 50


Hz. A 75 MW load is suddenly applied to the machine and the steam
valve to the turbine commences to open after

sec due to the time lag in

the governor system.


(i)

Calculate the frequency to which the generated voltage drops


before the steam flow commences to increase to meet the new
load. The value of the stored energy for the machine is 5 kW-s
per kVA.

(ii)

What will happen if the turbine commences to open after 2 sec


instead of 1 sec?
(6

(c)

marks)

Two synchronous machines operate in parallel and supply a total load of


300 MW. The capacities of the machines are 150 MW and 300 MW and
have generator droop characteristics of 4% and 3% from no load to full
load respectively.

(i)

Calculate the portion of the load taken by each machine,


assuming free governor action.
(10

(ii)

marks)

What is the new load portion taken by 300 MW machine if the


droop characteristic of this machine is changed to 4%?

(4 marks)

-6SEE 4453

Q4. (a) Show how the scalar voltage difference between two nodes in a network
shown in Figure Q4 (a) is given approximately by:

AV =

RP + XQ
V

(10 marks)

(b)

A 60 MVA, 69.3 kV, three phase synchronous generator has


synchronous reactance of 15 Q/phase and negligible armature resistance.

(i)

The generator is delivering rated power at 0.8 power


factor leading at the rated terminal voltage to an infinite
bus. Determine the magnitude of the generated emf per
phase and the power angle 5.
(4 marks)

(ii)

If the generated emf is 36 kV perphase, what is the


maximum three phase power that the generator can
deliver before losing its synchronism?
(2 marks)

(iii)

The generator is delivering 48 MW to the bus at the rated


voltage with its field current adjusted for a generated emf
of 46 kV per phase. Determine the armature current,
reactive power and the power factor. State whether the
power factor is lagging or leading?
(9 marks)

-7SEE 4453

Q5. (a) Discuss briefly the role of converters and thyristor valves in the HVDC
system.

(6 marks)

(b)

Smoothing reactor is connected in series with HVDC converter for two


reasons. State these reasons.

(4 marks)

(c)

A HVDC link with 6 valve bridge connected converters comprises a line


of loop 5 2 and is connected to transformers giving secondary voltage
of HOkV at each end. Each transformer has a per phase leakage
reactance of 15 Q. The rectifier is operated at a = 10. The inverter
operates on constant 8 control with a 8 = 15 and y = 20.
(i)

How much direct current and DC power is being transmitted


from the rectifier end?

(ii)

What is the maximum direct current that can be transmitted from


the rectifier if the inverter operates on constant P control?

(iii)

Draw the complete equivalent circuit of this HVDC link.

(15 marks)

V.

CONFIDENTIAL

UTM
UNIVERSITI TEKNOLOGI MALAYSIA

FINAL EXAMINATION SEMESTER II


SESSION 2010/2011

COURSE CODE

SEE 3263

COURSE NAME

ELECTRONICS SYSTEMS

LECTURERS

MR. ISMAIL BIN ARIFFIN

PROGRAMME

SEE / SEE / SEI / SEM

SECTION

01

TIME

2 HOURS 30 MINUTES

DATE

28 APRIL 2011

INSTRUCTION TO CANDIDATE

ANSWER 4 (FOUR) QUESTIONS ONLY.

THIS EXAMINATION BOOKLET CONSISTS OF 12 PAGES INCLUDING THE FRONT COVER

' SEE 3263

QUESTION 1
(a)

Define the linear regulator and the switching regulator.

[4 marks]

(b)

A regulator has a no-load output voltage of 18 V and a full-load output of 17.8 V


at a load current of 50 mA. Determine the voltage regulation as:

(c)

(i)

a percentage change from no-load to full-load

[3 marks]

(ii)

a percentage change from each mA change in load current. [3 marks]

An IC voltage regulator shown in Figure Ql(b) is able to operate with much more
higher of output current, IL. If VEBi(on) = VEB2(on) = 0.7 V, Io(max) = 1A and Pi=15,
(i)

state the function of transistors Qi and Q2,

[2 marks]

(ii)

determine Ici and IQ when RL = 100 Q.

[3 marks]

Figure Ql(a)
(d)

Figure Ql (b) shows a serial voltage regulator with Q2 as the feedback path.
Given (3j= P3 = 100 for transistor Qi and Q3 p2 = 50 for transistor Q2 and VBe(on)
= 0.7V for all transistors.
(i)

Determine the value of resistors R3, R2, Rsc.

[6 marks]

(ii)

Calculate the maximum power dissipation by Qi and Q2. [4 marks]

3
SEE 3263

Figure Ql(b)

4
SEE 3263

QUESTION 2
(a)

Figure Q2 shows the transformer-coupled class A power amplifier circuit.


(i)

State two advantages of using transformer-coupled compared to RC


coupling class A power amplifier.

[2 marks]

(ii)

Sketch and label the DC and ac load lines.

[9 marks]

(iii)

Calculate the maximum power efficiency.

[5 marks]

'' Vcr - ' 22 V

Figure Q2

(b)

A power transistor with thermal resistance, 0JA = 12 C/W dissipates a power of


25 W at ambient temperature, TA = 25 C. If the maximum junction temperature,
Tjmax =

200 C,

(i)

Show that the transistor will be superheated (over heat).

(ii)

If the cooling system is used, calculate the ambient temperature, TA


needed to prevent overheating.

(iii)

[3 marks]

[3 marks]

If the heat sink is used to prevent overheating at ambient temperature of


25 C, calculate the required junction to ambient thermal resistance, 0JA,
[3 marks]

SEE 3263

QUESTION 3
(a)

Define an oscillator.

[2 marks]

(b)

Specify the conditions for oscillation to occur.

[2 marks]

(c)

Figure Q3 (a) is a Wein-bridge oscillator with two loop feedbacks; the positive
loop feedback and the negative loop feedback.
(i)

What are the purposes of both feedback loops?

[2 marks]

(ii)

Explain the function of resistor, R3.

[2 marks]

(iii)

Determine the value of R2 for oscillation to occur.

[2 marks]

(iv)

Determine the value of the close-loop gain, Acl at the beginning of

(v)

oscillation.

[2 marks]

Find the oscillation frequency, fQ, for the circuit.

[2 marks]

D,

D,

Figure Q3(a)

(d)

Figure Q3(b) is a multivibrator circuit using a 555 timer.


(i)

What are the reference voltages at both comparator circuits in the 555
timer internal circuitry if the supply voltage is Vcc = +10 V? [2 marks]

(ii)

Determine the oscillation frequency, fo, for the circuit.

(iii)

Find the value of capacitor, Ci to produce the oscillation frequency,


fo = 25 kHz.

(iv)

[2 marks]

[2 marks]

If the resistor, Rj is changed to 3.3 kfl determine the value of R2 to


produce a 75% duty cycle.

[3 marks]

SEE 3263
(v)

Give one suggestion on how the circuit could be re-designed to produce a


50% duty cycle.

[2 marks]

Figure Q3(b)

7
SEE 3263
QUESTION 4
(a) (i) State the general definition of a filter.
(ii)

State ONE advantage of active filter over passive filter.

[1 mark]
[1 mark]

(b) Figure Q4 shows an active filter circuit. Please refer to Table 1, Table 2 and Table 3
in Appendix for Question 4 on page 10 if necessary.
(i)

Determine the damping coefficient (a), the response type and the filter type for
each stage.

(ii)

[3 marks]

Calculate the cut-off frequency, the centre frequency, the gain and the quality
factor Q for the filter circuit in Figure Q4.

[5 marks]

(iii) Draw and label the frequency response for the filter circuit shown in Figure Q4.
[3 marks]
C,

(c) Design a Bessel response, Sallen-Key equal component, low pass active filter to fulfill
the following specifications:
The 3dB cut-off frequency is 1.2 kHz.
At frequency of 10 kHz, the gain drops by 54 dB from the pass band gain.
Use capacitor value, C = 0.033 pF in designing this active filter and ignore the effect
of biasing current. Please refer to Table 1, Table 2 and Table 3 in Appendix for
Question 4 on page 10 if necessary.

[12 marks]

SEE 3263

QUESTION 5
(a)

Briefly explain the following SCR characteristics:


(i)

Forward Break-over voltage, Vbr(f)

(ii)

Gate triggering current, Igt

(iii) Holding current, IH


[6 marks]

(b)

Refer to Figure Q5. Given VBr for Schockley diode is 20 V while VGt of the
SCR is 2 V.
Obtain,
(i)

the required value of Vr2 for SCR to trigger

[2 marks]

(ii)

the conduction angle, 0C

[4 marks]

(iii) the average voltage (Vave) at the load, RL.

[3 marks]

(iv) the ac power at the load, Rl

[4 marks]

(v)

[6 marks]

the waveforms of Vr2, VL and VAk

9
SEE 3263

QUESTION 6
(a) For each of the following statement, state the type of analogue to digital converter
whether it is of type digital ramp, SAC or Flash:
(i)

Faster conversion technique.

(ii)

Require a very complex circuit.

(iii)

Produce a step ladder signal.

(iv)

Has a constant conversion time and does not depending on analogue input
(Va) time.

(v)

Does not require a digital to analogue converter (DAC).

(vi)

Use analogue comparators.


[6 marks]

Figure Q6(b)

(b) Refer to Figure Q6(b).


(i) Explain the function of Vin(+) and Vin(-)
(ii)

Explain the function of INTR

(iii)

Explain the function of pin Vref/2


[6 marks]

10

SEE 3263
(c)

The LM 35 in Figure Q6(b) is a linear temperature sensor IC that is designed to


produce a 10 mV output for every degree Celcius. For example, at 25 C, the
sensor produces an output of 250 mV and at 50 C it produces an output of
500 mV. The analogue input voltage, Vm, at full scale is fixed at 2.56 V
V
(i) What is the voltage value required to be set at pin ? [3 marks]
(ii) Calculate the required resistor value, RP2 to be varied.

[3 marks]

(iii) Obtain the resolution for this ADC.

[3 marks]

(iv) Determine the output value of D7-D0 when temperature is 100 0 C


[4 marks]

11

SEE 3263

APPENDIX FOR QUESTION 4


Table 1
Filter Type____ ;_________________Transfer Function
A

First order, low pas

H(s)

First order, high pass

A. s
H(s) = -

s + coc

S + CGc

Ao(c)

, ---------_
2 order, low
pass

H(s) =

>nd
u:l_________
2"
order, high
pass

u/\ H(s)
=

2nd order, band pass

H(s) = :^o^o^Q)----------s +(<d0/Q)s + ((d0)

2nd order, band stop

H(s) = ^o(s + (tDo) )

s +(roc/Q)s + (fflc)
A c2
s +(<bc /Q)s + (goc)

s2 + (o / Q)s + (0)
H(s) is in terms of Ao, coo, fflc, and Q.

Table 2
NORMALIZED POLYNOMIALS DEFINING
THE BUTTERWORTH POLES
Order________ Normalized Denominator Polynomials
1

(s + 1)

(s2 + 1.414s + 1)

(s + l)(s2+s +1)

(s2 + 0.765s + l)(s2 + 1.848s +1)

(s + l)(s2 + 0.618s+ l)(s2 + 1.618s + l)

(s2 + 0.518s + l)(s2 + 1.414s + l)(s2 + 1.932s+ 1)

12

SEE 3263
Table 3
Filter
Order
2

Stage
1
1

3
2
2

4
2
1

2
2
2

2
2

a
kip
a
kip
a
kip
a
kip
a
kip
a
kip
a
kip
a
kip
a
kip
a
kip
a
kip

Bessel

Butterworth

1.732
0.785

1.414
1

0.753
1.447
0.687
1.916
0.696
1.241
0.621
0.642
1.775
0.619
1.091
0.549
1.959
0.621
1.636
0.590
0.977
0.523

a - damping coefficient

kip - low frequency correction factor

1
1
1

1.848
1

0.765
1
1

1.618
1

0.618
1

1.932
1

1.414
1

0.518
1

1 dB
Chebyshev
1.054
1.238

2 dB
Chebyshev

2.212

3.105
0.402
1.095
1.088
2.146
0.224
1.057
4.484
0.578
1.602
0.142
1.037

0.496
1.098
1.275
1.992
0.281
1.060
3.571
0.714
1.577
0.180
1.040
1.314
2.881
0.455
1.364
0.125
1.023

0.886

1.333

1.121

3.115
0.363
1.375
0.0989
1.024

3 dB
Chebyshev
0.766
1.390
3.344
0.326
1.091
0.929
2.257
0.179
1.052
5.617
0.468
1.628
0.113
1.034
0.958
3.355
0.289
1.385
0.0782
1.025

CONFIDENTIAL

UTM
UNIVERSITI TEKNOLOGI MALAYSIA

FINAL EXAMINATION SEMESTER II


SESSION 2010/2011
COURSE CODE

SEE 4443

COURSE NAME

POWER SYSTEM ANALYSIS

LECTURERS

ASSOC. PROF. DR. MOHD WAZIR BIN


MUSTAFA
ASSOC. PROF. DR. AZHAR BIN KHAIRUDDIN

PROGRAMME

SEE

SECTION

01-02

TIME

2 HOURS 30 MINUTES

DATE

26 APRIL 2011

INSTRUCTION TO CANDIDATE

THIS
PAPER
CONSISTS
OF FOUR
(4)
QUESTIONS.
ATTEMPT THREE (3)
QUESTIONS INCLUDING QUESTION FOUR (4).
ALL ANSWERS TO NEW QUESTION SHOULD
START ON A NEW PAGE.
ALL CALCULATIONS AND
MUST BE CLEARLY STATED.

ASSUMPTIONS

THIS EXAMINATION BOOKLET CONSISTS OF 7 PAGES INCLUDING THE FRONT COVER

-2-

SEE4443

Question 1
(a)

(i) Explain briefly how outcomes from solution of load flow study can aid engineers
in two aspects of power system operation.

[4 marks]
(ii)

Using suitable mathematical representations, describe load flow analysis using


Gauss Siedel technique.

[10 marks]
(b)

In a five-bus system, the elements of admittance matrix are given as follow:


Y2,=Y23 = 0
Y22 = 7.146Z-84.60 p.u.
Y24 = 2.490Z95.10 p.u.
Y25 = 4.980Z95.10 p.u.
Knowing that only Bus 2 is a load bus with P2 - jQ2 = -2 + j0.7 p.u., and using flat start,
determine voltage V2 after second iterations using Gauss Siedel method.

[10 marks]

(c)

Given data for a three-bus system as in Table Q1 (c):


Table Ql(c)

Bus no.

Bus type

Voltage

Generated

Generated

Real

Reactive

(p.u.)

Real Power

Reactive

Power

Power

(p.u)

Power

Demand

Demand

(p.u.)

(p.u.)

(p.u.)

Slack

1.05Z00

Unknown

Unknown

Voltage -

1.02ZS

1.0

Unknown

Unknown

1.5

0.9

controlled
3

Load

The bus admittance matrix (YbUS) for the system arranged in sequential manner similar to
number assignment of the buses is given as follows:

-3-

SEE 4443

54Z - 68 22/AW 31Z1080


Ybus

22Z116 58Z - 63 33Z1070 p.u.


l31Z108

(i)

33Z1070 67Z-67

Derive the functions for all elements of Jacobian matrix in terms of


relevant variables of the system.

(ii)

Calculate the Jacobian elements that correspond to reactive power at Bus 3


(Q3) for initial iteration in the load flow analysis. Use flat start values for
unknown parameters.

[11 marks]

Question 2
(a)

Develop the expressions for analysing symmetrical fault in a large power system in
systematic manner using Z-Bus matrix.

[10 marks]
(b)

The busbars of a power system shown in Figure Q2(b) are in two sections, P and
Q, separated by a reactor X. Each per unit value of components in the system is
based on its components ratings. Using bases of 10 MVA and nominal voltage,
determine the maximum short-circuit MVA with which circuit breaker at point F
has to deal if symmetrical fault occurs at that point.
[10 marks]

-4SEE 4443
Ga
15MVA
12% 12% 10%

Gb
15MVA

Gc
8 MVA

Figure Q2(b)

(c)

The per unit bus impedance matrix for a 4-bus power system is given in sequential
order of bus numbers as follow:
0.150

0.075

0.140

0.135

0.075

0.1875

0.090

0.0975

0.140

0.090

0.2533

0.210

0.135

0.0975

0.210

0.2475

By assuming that the system is operating at nominal voltage, calculate the bus
voltages during fault if the following cases occur:
(i)

A bolted symmetrical fault at Bus 2.

(ii)

A three-phase fault through a fault impedance of jO.0025 per unit at Bus 4.

In both cases, can you calculate the fault currents in the lines of the system? Give
reason for your answer.
[15 marks]

-5SEE 4443

Question 3
(a)

Explain what is symmetrical components of an unbalanced three-phase system.

[8 marks]
(b)

What is the magnitude of the neutral-to-ground impedance ZN for a Y connected


generator in a zero-sequence diagram. Explain why?

[7 marks]
(c)

A four bus power system is shown in Figure Q3(c). The equipment ratings are given
in Table Q3.

Table Q3
System
MVA

kV

X(P)

Generator G|

500

13.8

Generator G2

750

Generator G3

X2(pu)

Xo(pu)

0.2

0.2

0.10

18

0.18

0.18

0.09

1000

20

0.17

0.17

0.09

Transformer T,

500

13.8 A/500 Y

0.12

0.12

0.12

Transformer T2

750

18 A/500 Y

0.10

0.10

0.10

Transformer T3

1000

20 A/500 Y

0.10

0.10

0.10

Component

Xi(pu)

Each line

50 a

50 Q

G,

T3

T2

G,

T, 1
Line 1

Y<SH<>

AY

0.028 Q

150 Q

Lin e.

Gj

Line 3

Figure Q3(c)

Assume that the prefault voltage is 1.0 per unit and prefault current neglected.

Aground

-6-

SEE 4443
(i)

Find all the positive, negative and zero sequence parameters value in per unit on a
1000 MVA, 20 kV base in the zone of generator G3.

(ii)

Draw the positive, negative and zero sequence network diagram. Write all the
parameters values on the sequence network.

(iii) If a single line to ground fault occurs at Bus 3, find the subtransient fault current in
ampere.
(iv) If the Y connection of transformer Ti is grounded, repeat part (iii).

[20 marks]

-7-

SEE 4443

Question 4
(a) Write the swing equation and define all abbreviations used.
[6 Marks]

(b) A synchronous generator is supplying a real power of 1.0 pu to an infinite bus as


shown in Figure Q4. A temporary three phase fault occurs in Line 2, at one-tenth the
distance from the infinite bus end.
(i)

What is the rotor angle when the generator is operating synchronously?

[6 Marks]

(ii) What are the generator output, accelerating power, and acceleration when the
fault occurs?

[6 Marks]
(iii) If the fault is cleared after 10 cycles, by opening of the faulted line, compute
the rotor angle, decelerating power, and deceleration immediately after the
faulted line is opened. Assume the power frequency to be 50 Hz and the
inertia constant of the generator to be 3.5 MJ/MVA. All values in the circuit
diagram are in per unit on a common base.

[6 Marks]
(iv) Plot rotor angle versus time for duration of 0.5 seconds. Take the
180 f
constants = ^ (At) 2 and time step At=0.05 seconds.

[6 Marks]

Figure Q4

CONFIDENTIAL

UTM

UNIVERSITI TEKNOLOGI MALAYSIA

FINAL EXAMINATION SEMESTER II


SESSION 2010/2011
COURSE CODE

SEE 4153

COURSE NAME

DIGITAL CONTROL SYSTEMS

LECTURERS

PROF. DR. JOHARI HALIM SHAH BIN OSMAN

PROGRAMME

SEE / SEI / SEM

SECTION

01

TIME

2 HOURS 30 MINUTES

DATE

29 APRIL 2011

INSTRUCTION TO CANDIDATE

ANSWER FOUR (4) QUESTIONS ONLY.

THIS EXAMINATION BOOKLET CONSISTS OF 8 PAGES INCLUDING THE FRONT COVER

if

SEE 4153

QUESTION 1

(a) An automobile wheel is 60.96 cm in diameter and the rim has four (4) equally
spaced spokes. The car is in motion and is being filmed with a camera that takes
24 frames per second. At what speed do the spokes first appear to rotate
backward ?
[5 marks]
(b) A circular potentiometer is used to measure shaft position. As the shaft rotates,
the potentiometer produces a saw tooth output voltage. In a computer controlled
system, the potentiometer output is sampled every T seconds. Calculate the
longest value of T that can be used without introducing ambiguity of position
measurement if the maximum shaft velocity is 100 rad/s'.
[5 marks]
(c) The signal F(s) = --------- --- -------- is being sampled by an ideal sampler with a
0 + l)Cs + 5)2
sampling period T = 0.5 s . Evaluate its pulse transform function F*{s).
[75 marks)

QUESTION 2
(a)

Given the following difference equation, find the solution y ( k ) if the initial
conditions are _y(0) = l, >>(-l) = 0, and the input u ( k ) is the discrete impulse
function.
[10 marks ]

y ( k +1) - 6 y ( k ) + 6 y ( k - 1) = u ( k )
(b) Figure Q2 illustrates a discrete-time control system in some supervisory control
configuration with sampling period T = Is. Determine the closed-loop pulse
transfer function for the system.
[15 marks ]

Figure Q2 : A Discrete-Time Control System

Dr/2011

-3SEE 4153
QUESTION 3
A discrete-time control system with a sampling period T = 0.2 s is shown in Figure
Q3 where a zero-order-hold (ZOH) is used to convert the discrete-time signal into a
continuous-time signal. Answer the following question :

(a) Determine the closed-loop pulse transfer function of the system; [10 marks]
(b) Obtain the steady-state error as a function of the gain K if the input to the
system is a unit step function. Comment on the size of the error as K varies from
0 to oo.
[7 marks]
(c) Determine the range of the gain K for which the closed-loop discrete-time
system is stable.
[8 marks]

Figure Q3 : Discrete-time Control System


Note : If needed, the transfer function for ZOH is given as : G 70H (s) = -------------------- =-------

QUESTION 4
Consider the system as shown in Figure Q4. If the sampling period for the system is
0.1 second (T = 0.1 s),

(a) Obtain either the root locus plot OR the Bode plot for the system. [20 marks]
(b) Determine the closed-loop system stability.

[5 marks]

Figure Q4 : A Discrete-time Control System.

DrJ2011

-4SEE 4153
QUESTION 5
Consider the position servomechanism in Figure Q5.

(a) Obtain the Bode plot for the unconpensated system.

[14 marks]

(b) Design a phase-lead controller D(z ) such that the compensated systems phase
margin is increased to 40.
[11 marks]

Digital Phase-lead
Controller

Servomotor

Figure Q5 : The Position Control Servomechanism.

DrJ2011

SEE 4153
TABLE OF COMMONLY USED Z-TRANSFORMS
Laplace Transform,

F(s)

Time Function, f ( t )

z-Transform, F(z)

Number Sequence, { f ( k ) }

{1}

{*}

z / ( z - l)
z/(z- \f
z ( z + l)

~
~

{kak}

(z-lf
z
z - a
az
(z - a)2
zsina

{sin a k }

z2 -2zcosa + l

z(z - cosa)

{cos a k )

z 2 -2zcosa + l
z2 - a z cos b

[ a k cosM:}

z 1 - 2 a z cos b + a 2
5 { t ) = impulse function

1
1

u(t)

z -1

Tz

(2-i y

s2

T 2 z(z + l)

t2

s3

(z-iy

e~al

5+a

z e~ aT

a
s(s + a)

z(l-e~T)

l-e~al

Tze~ aT

(s + af
a
s 2 (s

+ a)

a
22

te-at

(z-e-Tf

\-e-a
t --------------

sin (at)

s +a
s
s 2 +a 2

1
(s + a) 2 + b 2

s+ a

(y + tf)2 +b 2

cos (at)
e ' sin(bt)
bV

e~ m cos(bt)

Tz [ l ~ e ' o T ) z
(z-l) 2 a(z-\)[z - e ~ a T )
z sin(<3r )

z2 -2zcos(ar)+l
z(z - cos(aT ))

z2 -2zcos(ar)+l
1 ze~ aT sin(6r)

b z 2 -2ze~ aT cos(bT) + e~ 2aT


z 2 - ze~ aT cos(&!T)

z2 - 2ze~ aT cos(bl)+ e~ 2aT

DrJ20U

-6SEE 4153
Z-TRANSFORMS & INVERSE Z-TRANSFORMS THEOREMS
Theorem

Time Function, /(t ),

z-Transform, F(z)

t = kT
f(z)=

/(*?>-* or
*=0

residue of
at the poles of F(^)

i)

F{s) Has k Simple Poles at s =


k

Sni

N(s) 1

where

=i
D(s)
Definition of ztransform

ds

m
ii) F(s) With Multiple-order Poles ,, s2, s k , with
multiplicity, m,, m2, 7^, respectively :
d-'

a?"-'

\-e

s=s-sn ,z=e

K ^-^^ (s - sS ~ F(s)

or for a pole at s = s n with multiplicity of mn (>1) :


1 dm-

m,

-1

{m~ l)!^m"-

( s ~s) m " F{s)

Ts

\-e s z

-1

F(z)/-'dz
2ttj Jr

= ' residue of F{z)zk~1 at poles of F(z)

i) simple pole at z = a :
Definition of the
Inverse ztransform

{residue) ,=a = (z -<3)F(z)z*_1


F{z)
ii) a pole at z

= a with multiplicity m :

{residue)
jm-1

z=a

(m - 1)!

[(z-a)"F(z) 2

4-i

dz

DrJ2011

-7SEE 4153

PROPERTIES OF THE Z-TRANSFORMS

Property

Time Function, f ( t ), t = k T

Multiplication by a
Constant

+i

Addition and
Subtraction

z-Transform, F(z)
= F i (z) + F 2 (z)
= aF(z)

of it)

Real Translation
(Shifting - right)

Rt-nT)

Real Translation
(Shifting - left)

Rt + nT)

Complex
Translation

1+

= z-F(z)+ f;/(rn7>-("+m)
m^-n

= z f ( z ) - f ( k T ) z -
L

4=0

<

= F(ze al )

Initial-Value

/(0) = lim/(l

- lim F(z)

Final-Value

Y\mf(kT)
k-><x>

= lim(l -z~ l )F(z)


z^1

k->0

PartialDifferentiation

Real Convolution

Z A(nT)f 2 (kT-nT)

^\Rt,aj\

2>00

=1^(2, a)
da

- F1(z)F2(z)

=o

Complex
Convolution

MOfiiO

2?g Jr 4

DrJ2011

-8-

SEE 4153

MATHEMATICAL MODELLING OF SAMPLING USING IDEAL SAMPLER

Domain of Expression

Input-Output Transfer Relations of the Ideal Sampler

Representation as an Impulse Train


in the Time Domain :
k=0

Fourier Transform

= 7 Z F(
(Frequency Domain):

Jna>s)

* =SQO

Laplace Transform (as an impulse


train) :

my
k=o

Laplace Transform [F(s) has k simple


poles]:

tT D(^) l-e- ns ^
m=

D(Z)
dD(

D'() =

=,

where ,n is the th simple pole of F(<%), n = 1 , 2 , k.


Laplace Transform [F(s) has k poles
with multiplicity m n > 1]

**(*) = I
n= 1 /=1

( m n - i ) \ <%m"~

K.. =

~Ts

(z -1)! dsl
OR

1 dm

K-!)!"

m-\

m- 1

l-e

-T(s-,f)

at the poles of Z 7 ^)

residue of F(Z)
(l_e-*w>)

DrJ20U

CONFIDENTIAL

UTM
UNIVERSITI TEKNOLOGI MALAYSIA

FINAL EXAMINATION SEMESTER II


SESSION 2010/2011
COURSE CODE

SEP 4263

COURSE NAME

BIOMEDICAL IMAGE PROCESSING

LECTURERS

DR. NASRUL HUMAIMI BIN MAHMOOD

PROGRAMME

SEP

SECTION

01

TIME

2 HOURS 30 MINUTES

DATE

30 APRIL 2011

INSTRUCTION TO CANDIDATE

THERE ARE FOUR (4) QUESTIONS, ATTEMPT


ALL. READ QUESTIONS CAREFULLY.
ORGANIZE YOUR WORK AND WRITE
LEGIBLY.

THIS EXAMINATION BOOKLET CONSISTS OF 6 PAGES INCLUDING THE FRONT COVER

2
SEP4263

QUESTION 1
a)

In medical image processing, an example of quantization involves representing


a grayscale image with 8 bits per pixel. What is the purpose of the quantization
process?

b)

[3 marks]

Explain why arithmetic mean filter (sometimes commonly known as an averaging


filter), will not perform well on an image distorted by salt-and-paper noise.
[5 marks]

c)

Figure Ql is a 4 bits/pixel image of size 5x5.

i.

What is the difference between global and local enhancement? [3 marks]

ii.

Sketch and label the histogram of the whole image.

[4 marks]

iii.

Write the negative image of the whole image.

[4 marks]

iv.

Find the output of 3x3 mean, median, min and max filters computed
at the center pixel denoted with a circle only.

[6 marks]

12

11

13

14

10

15

12

12

11

13

14

13

14

12

13

10

Figure Ql

3
SEP4263

QUESTION 2
a)

The identification of objects within an image can be a very difficult task. One way to
simplify the problem is to change the grayscale image into a binaiy image, in which
each pixel is restricted to a value of either 0 or 1. One of the techniques used on these
binaiy images is morphological image processing.

i.

With the aid of mathematical operation, describe the dilation and erosion in
terms of its morphological operation.

ii.

[6 marks]

Erode the image in Figure Q2a with the following structuring element.
[4 marks]

Structuring element

Figure Q2a

4
SEP4263

The aim of Otsus thresholding method is to find the threshold value where the sum of
foreground (brighter pixels) and background (darker pixels) spreads is at its
minimum. Figure Q2b is a 6x6 image with the histogram for the image is shown next
to it. If the threshold value is 3,

i.

Calculate the weight (Wb), mean (nb) and variance (cs\) for the
background.

ii.

iii.

[6 marks]

Calculate the weight (Wf), mean (|if) and variance (o2f) for the
foreground.

[6 marks]

Find the value of within class variance (a2w)-

[3 marks]

Figure Q2b

5
SEP4263

QUESTION 3
a)

State four types of physical signals arising from the patient in medical imaging
modalities.

b)

[4 marks]

The beauty of medical imaging is that we can see inside the human body in ways that
are less invasive than surgery or endoscopy. Describe briefly the basic principles of
the following medical imaging modalities.

c)

i.

Magnetic Resonance Imaging (MRI)

[5 marks]

ii.

Computed Tomography (CT-Scan)

[5 marks]

iii.

Nuclear Medicine

[5 marks]

Ultrasound imaging systems are comparatively inexpensive and completely


noninvasive at the low imaging intensities typically used. Therefore, these systems
are widespread and in common usage and offer several imaging modalities. Explain
the following modalities of ultrasound.
i.

A-mode and B-mode imaging

[3 marks]

ii.

Doppler Imaging

[3 marks]

6
SEP4263

QUESTION 4

a)

Medical image registration is one of the important tasks in medical imaging.


i.

Give a brief description of medical image registration and why it is


needed in medical imaging.

[5 marks]

ii.

State and explain what factors make the registration is hard to do? [6 marks]

iii.

Assuming that we have two images of the same object, a structural image
and a functional image. Propose a strategy on how you can register
these two images.

iv.

What algorithm/method that minimize the difference between two


clouds of points?

b)

[6 marks]

[2 marks]

Briefly describe what the biological effects and safety procedures are for:
i.

Magnetic Resonance Imaging

[3 marks]

ii.

Computed Tomography

[3 marks]

END OF QUESTION SHEET

CONFIDENTIAL

UTM
UNIVERSITI TEKNOLOGI MALAYSIA

FINAL EXAMINATION SEMESTER II


SESSION 2010/2011

COURSE CODE

SEE 1023

COURSE NAME

CIRCUIT THEORY

LECTURERS

PROF. DR. ZAINAL BIN SALAM


ASSOC. PROF. DR. NIK RUMZI BIN NIK IDRIS
DR. SHAHRIN BIN MD AYOB
DR. NOR ASIAH BINTI MOHAMAD
DR. NORHAFIZAH BINTI NGAJIKIN
MR. ABD. JAAFAR BIN SHAFIE
MR. ALIAS BIN MOHD YUSOF
MS. ZANIAH BINTI MUDA

PROGRAMME

SEC / SEE / SEI / SEL / SEM / SET / SEW / SWB

SECTION

01 - 07,10

TIME

2 HOURS 30 MINUTES

DATE

30 APRIL 2011

INSTRUCTION TO CANDIDATE

ANSWER FOUR (4) QUESTIONS ONLY.


ALL WORKING NEED TO BE SHOWN
CLEARLY.
DRAW NEAT DIAGRAMS WHEREVER
NECESSARY.

THIS EXAMINATION BOOKLET CONSISTS OF 7 PAGES INCLUDING THE FRONT COVER

2
SEE 1023

Ql. (a) Referring to Figure Ql(a), use Nortons theorem to determine the resistance Rl
[ 8 Marks ]

so that current flow through it is 2 mA.


1 kQ
rWV

2kQ

12 V

Figure Ql(a)

For the circuit in Figure Ql(b-i), find the value of Rl for maximum

(b) (i)

[ 3 Marks ]

power transfer to the load.


20V 3Q

W v
3Q
---- W V

jz :

Rl
2O

AAAr

Load

Figure Ql(b-i)
(ii)

For the circuit in Figure Ql(b-ii), find the maximum power that can be
transferred to load Rl.

[ 14 Marks ]

Rl

3
SEE 1023

Q2. (a) For the circuit in Figure Q2(a), determine:


. (i) the impedance Zl for maximum average power transfer and [ 2 Marks ]
(ii)

(b)

the value of the maximum average power transferred to Zl- [ 6 Marks ]

Three loads are connected to a voltage source, 240Z00 V(rms) as shown in


Figure Q2(b). Load 1 absorbs 16 kW and 28 kVAR, Load 2 absorbs 10 kVA at
0.6 power factor lead and Load 3 is an impedance (4.6 + j3.5) Q.
(i)

Find the impedance that is equivalent to the three parallel loads.


[11 Marks]

(ii)

What value of capacitance placed in parallel with the terminals A-B will
raise the power factor to 0.95 lagging. The frequency of supply voltage is
50 Hz.
[ 6 Marks ]

240Z00 Vn

Figure Q2(b)

4
SEE 1023

For a series resonant circuit, show that the half-power angular frequencies, coi^

Q3. (a)

are given by:


1

>1'2

2L

'

Lj

LC

where R, C and L are resistor, capacitor and inductor respectively.

[ 7 Marks ]
(b)

A series resonant circuit has R = 10 Q, L = 100 mH, C = 10 |j.F and a voltage


source, Vs = 10 Z0 Vnns- Determine:
[ 2 Marks ]

(i)

Half-power angular frequencies.

(ii)

Magnitude and phase angle of current, I at half-power angular frequencies.


[ 4 Marks ]
Magnitude and phase angle for:

(iii)

a.

Vr at lower cut-off angular frequency and

b.

Vl at upper cut-off angular frequency.

[ 5 Marks ]
(c)

The networks shown in Figure Q3(c-i) and Q3(c-ii) have the same impedance as
seen from the terminals a-b.
(i)

Show that the equivalent parallel network (RP,XL) in terms of the series
network (Rl, Xl) are given by:
R 2 +0 2 L

l
R p =-------------and X p =
D

R, 2 + cd 2 L 2

R,

(iii)

coL

Hence, prove that Quality factor, Q for Figure Q3(c-i) equals to Q of

Q3(c-ii).
[ 7 Marks ]

Xp

Figure Q3(c-i)

Figure Q3(c-ii)

5
SEE 1023

Q4. (a) Give three types of response for a parallel RLC circuit and their relationship to the
[ 4 Marks ]

circuit parameters.
(b)

A source-free parallel RLC circuit is described by the voltage second-order


differential equation as follows:
d 2 v dv
- +c 2.5 + v = 0
dt dt

Find the following:


(i)

[ 1 Marks ]

The characteristic equation of the circuit.


(ii)

The type of response of the circuit and the value of C and L if R = 2 Q.

[ 5 Marks ]
(c)

Figure Q4(c-i) shows an RLC circuit containing a voltage source and a switch
which has been closed for a long time. The switch is then opened at t = 0 s. The
response of the voltage across the capacitor is shown in Figure Q4(c-ii).
Vc(V)

Figure Q4(c-i)

Figure Q4(c-ii)

(0

What is the type of response for this second-order circuit? [ 1 Mark ]

(ii)

What are the values of the voltage sources, Vsi and VS2- What is the initial
voltage of the capacitor, Vc and the inductor current, i]_ at t = 0?

[ 5 Marks ]
(iii)

Write down the expression for vc(t) for t > 0.

(iv)

What should be the value of resistor, R] in order to change the type of


response to critical damped.

[7 Marks ]

[ 2 Marks ]

6
SEE 1023

Q5. (a) Given that two two-port networks represented by network A consists of
transmission matrix, \T a ] and network B consists of admittance matrix, \Y b ].

[ 2 Marks

(i)

How these two networks can be connected in cascade.

(ii)

Sketch and label clearly the block diagram showing the cascade
[ 3 Marks ]

interconnection of these two networks.


(iii)

Show that the resultant matrix of the cascade interconnection of the


individual two-ports can be represented by [T t ] = [T a ] + \T b ].

[ 5 Marks ]

(b)

(i) Consider the circuit in Figure Q5(b-i). Find the admittance parameters.

[ 10 Marks ]
(ii)

If terminals A-B of Figure Q5(b-i) is connected with resistor 1 Q. in


series with vP and terminals C-D is terminated with a 0.5 Q load resistor

[ 5 Marks

as shown in Figure Q5(b-ii), find .

20

i a
VW-

c
o+
V0

Two-Port Network

Figure Q5(b-i)

Figure Q5(b-ii)

0.5 a

7
SEE 1023

Conversion of two-port parameters

Z22

y11 y12

'

hn

^21

" z 2i z n

22

22

zn

A A
y y

T 22 - A t
Tx T
12 12

hn

1T
___ J n
.^12 T 12

hn hn

a2 az

y -y,

1 -h 12

Z12

Az Az

y y
21

Ah

h 12

Tn A t

h 22

h 22

T T
*21 21

-h 21

z 12

-y2i yn

Z 21 Z22

Ay A y

Al

IlL

Z22

Z22

ZlL
yn yn

-y -1
y2i y 21
y -yn
y2i y2i

li

Al

Z21

Z21

h ]2

Z21 Z21

Nota: Parameter T = transmission Parameter

B'

~ T
Mi

21

T
12

22 _

^12 A t
T T

___ 21

Z22

A p Pi 1P22 P12P21

hn

Z22 Z22

_A

' A

^21 T 21

h 21 h 22

22

21

1T

22

h 22 h 22

~ y 12
yti yn
1

-A h

22 22

22

22

-1 T
T T

-h n
T T
Ml A12

h2i

b2]

- h 22 -1
h 2 ] h 21

T T
21 22

CONFIDENTIAL

UTM
UNIVERSITI TEKNOLOGI MALAYSIA

FINAL EXAMINATION SEMESTER II


SESSION 2010/2011
COURSE CODE

SEE 1123

COURSE NAME

INSTRUMENTATION AND ELECTRICAL


MEASUREMENT

LECTURERS

DR. TAN CHEE WEI


DR. MUHAMMAD ABU BAKAR SIDIK
DR. NOURUDDEEN BASHIR UMAR
DR. RASHID AH BT ARSAT
DR. NORHUDAH BT SEMAN
DR. LEOW PEI LING
MR. LIM CHENG SIONG

PROGRAMME

SEC / SEE / SEI /SEL/ SEM / SEP / SET / SEW

SECTION

01-07

TIME

2 HOURS 30 MINUTES

DATE

05 MAY 2011

INSTRUCTION TO CANDIDATE

PART A:
ANSWER ALL QUESTIONS (25%)
PART B:
ANSWER THREE (3) QUESTIONS ONLY (75%)

THIS EXAMINATION BOOKLET CONSISTS OF 13 PAGES INCLUDING THE FRONT COVER

SEE1123

PART A: MULTIPLE CHOICES AND TRUE FALSE. (25%)

(Indicate your answers for PART A by using the answer sheet provided in the appendix and
attached it together with your answer booklet for submission)

1. Several steps for getting the certificate of accreditation is included below except
a.
b.
c.
d.

Application
Adequacy audit
Compliance assessment
Assuring the quality of test and calibration results.

2. Which of the following quantities has different dimension from the others?
a. Depth
c. Area

b. Distance
d. Width

3. Loading effect of a Voltmeter to the measurement circuit can be minimized using a


a.
b.
c.
d.

Very low impedance shunt resistor to the load


High series resistor to the load
Low input impedance voltmeter
High input impedance voltmeter

4. Which one below indicates a focus provided by SAMM (Skim Akreditasi Makmal
Malaysia or Laboratory Accreditation Scheme of Malaysia)?
a. Promoting industry, governments, regulators and consumers to the
accreditation system.
b. Assist and develop laboratory
c. Assisting and supporting developing accreditation systems
d. Harmonizing inspection arrangement
5. The following are classifications of error except
a.
b.
c.
d.

Uniform
Human
Systematic
Random

3
SEE1123

6. Primary standards are standards maintained by:

a.
b.
c.
d.

Industrial measurement laboratories


Actual measurements in physics
International Bureau of Weights and Measures (BIMP)
Organization/national laboratories

7. What is the function of Linear Variable Differential Transformer (LVDT)?


a.
b.
c.
d.

To step up or step down primary voltage supply to secondary coil


For measuring unknown resistance in a circuit
To measure linear displacement
To convert mechanical movement into electrical output

8. If the Wheatstones bridge below is balanced, find Rx.

a. Rx -Ra + Rb + Rc
c. Rx= Rc Rb / Ra

b. Rx-Rc(Ra+Rb)
d. Rx= Ra/ RcRb

9. The distance between two peaks of a signal measured on the x-axis is 2 cm, at
1 ms/div. The frequency of the signal is
a. 50 Hz
c. 1 kHz

b. 5 Hz
d. 500 Hz

10. A thermistor is inherently suitable for temperature measurement involving


a.
b.
c.
d.

High accuracy but low sensitivity


High sensitivity but narrow span
High sensitivity but low accuracy
Wide span but low sensitivity

4
SEE 1123

11. The construction of the electron gun inside the oscilloscope consists of the below
except
a.
b.
c.
d.

Heater
Cathode
Control grid
Deflecting plate

12. Which one below is NOT the main purpose of using dimension analysis
a.
b.
c.
d.

To determine the unit of the equation


To understand the physical situations of the equations and formula
To check the correctness of the physical quantities in an equation
To examine the relation of the physical parameters of an equation

13. The food that carries halal certifications promises the following benefits except
a.
b.
c.
d.

Hygiene and clean


Fulfill the halal food process practices
Rich in nutrition
Is marketable in Muslim countries

14. The sensitivity of a voltmeter with an internal resistance 50 Q is given as 1 kQ/V at


full scale 20 V. What is the input resistance of this voltmeter?
a.
b.
c.
d.

19.95 kQ
20 kO
0.95 kQ
0.9975 Q

15. Which of the following is NOT the rules for presenting graphical data
a. Graph should not be drawn outside of the boundaries corresponding to the
maximum and minimum data values measured
b. Title or caption explaining the data should be presented
c. Both axes need to be label with the variable and unit clearly stated
d. The independent variables should be plotted as y-axis

SEE 1123
Indicate TRUE or FALSE for the statements below:
No.

Statement

TRUE or
FALSE

16.

Random errors are errors that cannot be explained as human or


instrument or environment errors.

17.

In inductance standard, one Henry is equivalent to one Ampere per


second divided by one Weber (Wb).

18.

Chauvenets criterion is a statistical rejection of bad data in a set of


experiment data.

19.

Given the specification of an electrical appliance as: 240 V 0.1 %


The above statement refers to the terminology bias.

20.

Thermistor is a linear thermo-resistive sensor use for temperature


measurement.

21.

Strain gauge is constructed from a fme-wire element that looped back


and forth on a mounting plate that cause change in wires resistance
when the plate deforms.
The ISO 17025 standards states that if testing and calibration

22. . laboratories comply with ISO 17025, it must also operates accordance

with ISO 9001 or ISO 9002


23.

Given the quantity force, [F] = [MLT2]; then the dimension of the
quantity acceleration can be expressed by [a] = [M'!LT'2].

24.

By applying 85% of the from farm to table process, the applicant


should not have any problem in their application of HALAL
certificate.

25.

Electron gun of a cathode ray tube is used for generating electrons


beam in oscilloscope.

6
SEE 1123
PART B: ANSWER ONLY THREE OUT OF FOUR QUESTIONS

Question 1

a. Define the terms error and uncertainty. Then, briefly describe the systematic errors.
[10 Marks]

b. Referring to the circuit shown in Figure Qlb, the connecting wires are made of Nickel.
Rwire = 10 Q

-------- vw-----------25 V

Rload 15 Q

Figure Qlb
i.

What is the resistance in each of the nickel wires, RWjre if the temperature of the circuit
rises from 20 C to 40 C? Given the temperature coefficient of nikel, anikei-20 is
0.005866 PC. (Material thermal resistance equation is given in the appendix)\3 Marks]
ii. Assuming R]oad is a consumer load in an electrical power system, comment the effect of
the temperature rise of these wires to the voltage across the load by using circuit analysis
to prove your answer.
[10 Marks]
iii. At what temperature will the total resistances of the nickel wires have the value of 25 Q?
[2 Marks]

7
SEE 1123

Question 2
a. In designing a dc to dc converter, a researcher has found that the switching frequency to
obtain the minimum inductance is as below
(1-2D + D 2 )R

where D is the duty-cycle, R is the output resistance and f is the switching frequency. If
the resistor, R is 500 Q with an accuracy of 25 Q, the switching frequency is unknown
with certain variation while the duty-cycle is 0.5 + 0.01. Given the minimum inductor
value is 2.5 mH with accuracy of 37.5 ^H; determine the frequency of switching with its
variation by using the given equation without expanding the variable terms.
[6 Marks]

b.

Figure Q2b shows the basic principle of gravity pendulum,


4 I.

/
where,
g is the acceleration due to gravity near the surface of
the earth
/ is the length of the pendulum in meters
do is the largest angle attained by the pendulum
0 or d(t) is the angle attained by the pendulum at time t.

Figure Q2b
Both Go and 0 are measured in radians. The value of 6 at time / is given as,

;(valid only for a very small 0 where 6o 1)

Find the maximum uncertainty for 0(t), given that


g = 9.81 0.01 ms'2
/ = 0.5 0.02 m
0 0 = 0.3 0.03 rad
t = 10.05 s

[11 Marks]

Describe modulating and self-generating transducers. Explain the main difference between
these two transducers.
[3 Marks]

Figure Q2d shows a schematic diagram of a Serial-Capacitor type bridge which is


operating at 0.5 kHz. Given that the value for the resistor Ra, Rb, Rc are 2000 Q, 2800 Q
and 52 Q, respectively. The capacitor Ce is 0.5 pF. Determine the value of the components
in Zx.
[5 Marks]

Figure Q2d

9
SEE1123

Question 3
a. Strain gauge is used for sensing the strain.
i. Explain the piezo-resistive effect property and name a type of strain gauges. [4 Marks]
ii. State two advantages and disadvantages of semiconductor strain gauge. [4 Marks]
iii. A round copper wire is loaded with 30000 kg tensile force in which the modulus is
equal to 1.7 x 1010 kg/m2. Determine the ratio of the gauge factor over the initial
resistance of the gauge when the change of resistance is 250 |iH. The characteristic of
wire is 15 mm in radius and 500 mm long.
[5 Marks]

b.

Figure Q3b shows the plot for a set of wind speed measurements logged by a metrology
laboratory. Based on these data, the metrologist in charge found that the readings are
largely deviated from each other. For that reason, you have been assigned to help the
metrologist to perform the data analysis with the aid of Table Q3.
11

r- -

1'

X: 7

10

9
/

8
/

"D
CD
a> 6
Q.

X: 3
Y: 5.9

X: 1
Y: 5.7
'1 1

"

X: 5
Y: 5

..... 1 1-.....

C/D

a
c

X: 4
Y: 5.4

1;;------------- v/

4
\

3
/

*,
2
\

X: 2
. Y: 0.8

0 -

ii

4
Reading, x

Figure Q3b

,*
X: 6
. Y: 4

10
SEE 1123

Table Q3: Chauvenets Criterion for rejection of a reading.


Ratio of Maximum Acceptable Deviation to
Number of Readings,
Standard Deviation, d max / cr
n
3
1.38
4
1.54
5
1.65
6
1.73
7
1.80
10
1.96
15
2.13
i.

For a given probability of 68.3%, sketch the normal distribution curve of a population
data. In the normal curve, label the area of the rejected data and the accepted
probability.
[2 Marks]

ii.

Using Chauvenets Criterion, determine, if any data should be discarded from the set
of wind speed readings shown in Figure Q3b.
[10 Marks]

11

SEE 1123

Question 4
a.

With the aid of circuit analysis, explain the impact of using low input impedance in
voltmeter design. What is this phenomenon known as?
[5 Marks]

b. Figure Q4b shows the connection of a voltmeter to measure the potential different across
resistor R3. Find the voltage reading and % error of each reading obtained with a voltmeter
on
(i)

Meter 1: S = 2kQ/V with 1OV range,

(ii)

Meter 2: S = 5kQ/V with 10V range, and

(iii)

Meter 3: S = lOkTl/V with 10V range

[10 Marks]

Ri = 25k^

(iv) What can you conclude based on the calculations above?

[2 Marks]

c. Define Electrical and Mechanical Transducers and give two examples of each.
[8 Marks]

12
SEE 1123
APPENDIX - LIST OF EQUATIONS___________

No.

Equations

V. =

f-n-h
2e

I =e
/z

C=

R T = i?ro (1 + aAT)

a =

df +d] ++dLn

dCosx
dx

uy =

dR

- - Sin x

u, +

\2

+ . .+

dR

\2

-W, +
-w.
9xj
\dx 2

r^
l5xJ
/\2
-w_

K 9X n

AL = FL /AE

10

11

;=1

12

/ =

V"Zx'-&)2 V"Z^2-(Z^)2

13

SEE 1123
ANSWER SHEET FOR PART A (25%)
NAME:_____________________________________________________
IC NO:__________________________ SECTION/ PROGRAM:_____________/

MULTIPLE CHOICES:

(1)

(6)

(11)

(2)

(7)

(12)

(3)

(8)

(13)

(4)

(9)

(14)

(5)

(10)

(15)

TRUE OR FALSE STATEMENTS:

(16)

(21)

(17)

(22)

(18)

(23)

(19)

(24)

(20)

(25)

CONFIDENTIAL

UTM

UNIVERSITI TEKNOLOGI MALAYSIA

FINAL EXAMINATION SEMESTER II


SESSION 2010/2011
COURSE CODE

SEE 3113

COURSE NAME

CONTROL : ANALYSIS & DESIGN

LECTURERS

DR. SHAHDAN BIN SUDIN

PROGRAMME

SEE / SEL / SEM

SECTION

01

TIME

2 HOURS 30 MINUTES

DATE

06 MAY 2011

INSTRUCTION TO CANDIDATE

ANSWER FOUR (4) QUESTIONS ONLY.

THIS EXAMINATION BOOKLET CONSISTS OF 8 PAGES INCLUDING THE FRONT COVER

-2- -

SEE 3113

Question 1
a)

i)

If the natural response in the output of a system creates instability to the


system, what can we tell about exponential term in it?

ii) Does the presence of entire row of zeros in a Routh Table always mean
that the system has j co-axis poles?
iii) If there is a zero in the first column of the Routh table, what does this
indicate?

(3 marks)
b)

For the system in Figure Qlb, how many poles are located on/at the left-half
plane, right-half plane and on

jco-axis.

Note that the system has a positive

feedback.

(9 marks)
R(s)

C(s)

+o
i 1+

s5 + 4s4 + 2s3 - 2s2 + 3

Figure Qlb
c) A model for an airplanes pitch loop is as shown in Figure Qlc.
Controller

Aircraft dynamics

Gyro

Figure Qlc
i)

Find the range of gain K that will keep the system stable.

(11 marks)
ii) Can the system ever be unstable for positive values of K1
(2 marks)

-3 SEE 3113

Question 2
A unity feedback system has a forward transfer function
rM=

Kfr + i)

(s + 5)(s2 - 2s + 2)

a) How can we determine whether a root locus plot crosses the imaginary axis?

(2 marks)
b) By using the Routh-Hurwitz criterion, find the range of K to make the system
stable. When the system is marginally stable, calculate the value of poles.

(6 marks)
c) Prove, mathematically, that the point s 1 = 0.59 + j 4.95 is located on the
root locus.

(3 marks)
d) Draw the root locus of the system on graph paper provided.

(10 marks)
e) If the system is to operate at the damping ratio of 0.08, find the value of K to
achieve this operation.

(4 marks)

-4SEE 3113
Question 3
a) What is the effect on the performance of a system if the following
compensators are used, respectively:
i)

PI Controller

ii) PD Controller
iii) PID Controller

(4 marks)

b) A vehicle speed control system is shown in Figure Q3b.

Figure Q3b

The system without controller has a settling time of 4.44 s and peak time of
3.49 s.
Design a suitable controller so that,

the systems settling time is reduced to half of the original value

the peak time is maintained, and

the steady state error is reduced to zero for a unit step input.

(16 marks)
c) Roughly sketch the root locus of the uncompensated and compensated systems
in part (b) above.
(5 marks)

-5SEE 3113
Question 4
a)

i) Name two ways to plot the frequency response.


ii) Define the Gain Margin and Phase Margin
iii) From the bode plot of a given system, how do we know that the system is
unstable?

(3 marks)
b)

A unity feedback system has an open-loop transfer function of


^1000(5 + 1)

s(s + 2.5)(s + 10)(s + 50)


i)

By using the straight line approximation methods, plot the Bode plot of the
open-loop system when K = 1.

(14 marks)
ii) Obtain the Gain Margin and Phase Margin of the system.

(2 marks)
iii) Determine the stability of the system. Give your reason.

(2 marks)
iv) Find the value of gain K needed for the system to give a phase margin of
45.

(2 marks)
v) With the value of K obtained in part b(iv) above, re-plot the magnitude
curve to show that the required phase margin is obtained.
(2 marks)

-6-

SEE 3113
Question 5
a)

The performance of a system can be improved by using a lag compensator,


without resulting in any instability problem. Describe two (2) main functions
of a lag compensator in terms of the static error constants and the transient
response, respectively.

(5 marks)
b)

The roll dynamic of a towed-vehicle roll control system can be represented as


a unity feedback system, with a forwards transfer function given as,
rM

_ ^0 + 4)

(s + 2)(s + 6)(s + 8)

The bode plot of the open-loop system, with K = 24, is as shown in Figure
Q5b.
i)

Design a lag compensator, on the given bode plot (submit together with

your answer), so that the system operates at a phase margin of 45 and a


static error constant of 100.

(16 marks)
ii) Find the value of gain K for the compensated system.
(4 marks)

-7SEE 3113
K=24
Frequency (rad/s)

0.01

1000

| * I

J f-

*i

[."y^ :r^l
fc? fj< A

jwjsr ?*-*. sjiSh-^.sA^.^1

-" f- !<

i*

i * . * I * >- (*1> *>


*r * *~ J -4-j * j
1

Figure Q5b(i)

-8SEE 3113
K=24
Frequency (rad/s)

Figure Q5b(ii)

CONFIDENTIAL

UTM

UNIVERSITI TEKNOLOGI MALAYSIA

FINAL EXAMINATION SEMESTER II


SESSION 2010/2011

COURSE CODE

SEE 4113

COURSE NAME

MODERN CONTROL SYSTEM

LECTURERS

ASSOC. PROF. DR. MOHD FUAAD BIN


RAHMAT
ASSOC. PROF. DR. ZAHARUDIN BIN
MOHAMED

PROGRAMME

SEE / SEI / SEM

SECTION

01-02

TIME

2 HOURS 30 MINUTES

DATE

05 MAY 2011

INSTRUCTION TO CANDIDATE

ANSWER FOUR (4) QUESTIONS ONLY.

THIS EXAMINATION BOOKLET CONSISTS OF 12 PAGES INCLUDING THE FRONT COVER

-2SEE 4113

Question 1
(a). Give two reasons for modeling system in state space.

[2 marks]
(b). For electrical networks, what is a convenient choice of state variables?

[2 marks]
(c). Consider the electrical network shown in Figure Ql(a) where the input variable is Vi(t)
and the output variable is V0(t).

v,

Figure Ql(a): RLC network


i) Determine the state-space representation of the electrical network.
[Hint: The state variables should be chosen from your answer in Q1 (b)]

[10 marks]
ii) If possible, find a new state-space representation with the following state variables:
x l (t) = v l (t)
x 2 (t) = i(t)
x 3 (t) = v 2 (t)

[3 marks]

-3SEE 4113

(d). Consider a mechanical system shown in Figure Ql(b) where u and u 2 are the inputs and
and y2 are the outputs.

Figure Ql(b): Mechanical system


(i) Find a state-space representation of the mechanical system.

[5 marks]
(ii) Draw a signal flow graph to represent the system.

[3 marks]

-4SEE 4113

Question 2
(a) Give two reasons to represent a system by alternative forms.

[2 marks]
(b) Investigations on a mobile robot show that the system has Eigen-values of -2, -2, -4
and -4. The state-space equation of the robot can also be described in the Controller
Canonical Form as follows:

x{t) = Ax(t) + Bu(t)


y{t) = [2 3

l]x(0

(i) Find the transfer function of the mobile robot.

[3 marks]
(ii) Obtain the state-space representation and signal flow graph in Diagonal or Jordan
Canonical Form.
[7 marks]
(c) A simple crane system can be represented by a second order differential equation as
du(t)
d y(t) , , dy(t)
+ 6 ---------1-4 y(t) = 4--------- b u(t)
dt A dt dt

where y(t) is the output and u(t) is the input.


(i) Find a state-space representation of the system in a phase variable form.

[4 marks]
(ii) To design a controller, the state equation in the phase-variable form has to be
transformed into a new state equation as follows:

z=

' 3.4
-5.8

6.2

-9.4

z+

- 0.2"
0.4

This can be achieved with similarity transformation with a transformation matrix,


P =

2 Pn
P 21 P 2 2

Find the output equation of the new state-space representation.


[9 marks]

-5SEE 4113

Question 3
(a). A linear time-invariant system is characterized by the homogeneous state equation,

dx j (t)
dt
dx 2 (t)

01

Xj(0

-6 -5

_x 2 (t)_

dt

where the initial condition of the system is x(0) = [ 1 0]T


(i). Find the resolvent matrix, (s)

[4 marks]
(ii). Find the state transition matrix <D(t)

[2 marks]
(iii). Find the inverse state transition matrix cD'(t)

[2 marks]
(iv). Find the solution of x(t)

[4 marks]

(b). A linear time-invariant system is characterized by the non-homogeneous state equation,

y ( t ) = [l 1}

'o'
XI (0 ' 0 1 '
u(t)
+

1
-6e>
-5_-:>j|_x_x2(f)J
2(0_|_i
_X2 (0_
x l (0

x2(t)
Determine the non-homogeneous solution, x(t) and the output soloution, y(t) if the initial
condition of the system is x(0) = [ 1 0]T and u(t) is a unit step function.
[13 marks]

-6SEE 4113

Question 4
A magnetic levitation system is described by state space representation model below and
Figure Q4(a).

dt
y{t) = [1 0}

*\

+
1

-5

< (N

dt dx 2

1XI X

dx j

'o'
_i_

u(t)

(0

x 2 (t)

Figure Q4(a): state space representation of a plant with state variable feedback

Figure Q4(b): Integral control for steady state error design

-7SEE 4113

(a). Determine the controllability of the system.

[ 2 marks]
(b). Design a controller with a configuration shown in Figure Q4(a), that will yield a 10%
overshoot with a settling time of 0.5s.

[10 marks]
(c). Evaluate the steady state error for a unit step input.

[2 marks]
(d). What is the disadvantage of using state variable feedback in controller design and
suggest how to overcome the problem.

[2 marks]
(e). Repeat the design of (b) using integral control as shown in Figure Q4(b) and evaluate the
steady state error for a unit step input. Choose the third pole as 5 times farther than the
dominant poles.

[9 marks]

-8SEE 4113

Question 5
A pneumatic actuator system with servo valve system is described by a transfer function as
=------- lOCs + 2)-------

(s + l)(s + 3)(s + 4)
(a). Determine the state space representation in Observer Canonical Form.

[2 marks]
(b). Determine the observability of the system.

[2 marks]
(c). Give three reasons of using observer in control system.

[3 marks]
(d). Why is the design of an observer is separated from the design of a controller?

[2 marks]
(e). Design an observer for the plant with 5% overshoot and 1 second peak time. The
observer must respond 10 time faster than the plant. Place the third pole 20 times farther than
the observed dominant pole.

[13 marks]
(f). Draw the signal flow graph for the plant and the observer.
[3 marks]

-9SEE 4113

Question 6
(a) Give two examples on how an optimal control is used in control engineering.

[2 marks]
(b) Consider a system with a state equation, x ( t ) = A x ( t ) + B u ( t ) and a closed-loop system with
control input, u ( t ) = Kx . By using the performance index
00

J = j*(x T Ix + Jiu T lljjt


0

Prove that Jmin can be achieved by minimizing J = xT (O)Px(O)


Where I is an identity matrix, A, is a scalar weighting factor, x(0) is an initial condition and
P is a symmetric matrix.
[7 marks]
(c) A linear DC motor system can be represented in a state-space form as
01

x(t) +

x(t) =

_ .

O'

u(t)
_1_

The performance of the system is quite unsatisfactory because an un-damped response


results for a step input. To improve the system performance a feedback control is designed
with

u ( f ) = - x l ( t ) - k x 2 (t )

i) With the initial condition, x T (0) = [l l] determine the feedback gain k that minimizes
the performance index

J = J {x T x + u T ujfit

[12 marks]
ii) Plot the performance index, J versus the gain k.

[4 marks]

-10SEE 4113

Formula
1. RLC

v(/) = - i ( T ) d T i { ! ) =

C d 4 v(t) =

Capacitor
v(/) = Ri(t)

-Wr
Resistor

_1_
Cs

di

Cs

rG

dt

Ls

v(r)dr v{t) = L~^r

Ls

Note: The following set of symbols and units is used throughout this book: v(f) = V (volts), i(t) = A (amps),
q{t) = Q (coulombs), C = F (farads), R = (1 (ohms), G = U (mhos), L = H (henries).

2. MSD
Forcevelocity

Component

ForceImpedance
displacement Z M (s) = F(s)/X(s)

Spring

fit)

Jo

' v(r) dr fit) = Kx(t)

^K
Viscous damper
x(/)

An

f i t ) = ./;..v(f)

f{t) = fv

dx({)
dt

fvS

Mass

.v(/)
fit) = M-

M An

dv(t)
dt

Ms2

Note: The following set of symbols and units is used throughout this book: f \ ( ) =
(newtons), x ( t ) m (meters), v ( t ) = m/s (meters/second), K = N/m (newtons/ meter),
N-s/m (newton-seconds/meter), M = kg (kilograms = newton-seconds2/meter).

3.

XI - A] = 0
(AI-A)x = 0

4.
T(s) = = C(sl - A) 1B + D
U(s)
X

= Pz

z = p-'x
z(t) = P~ x APz(t) + P~ l Bu(t)
y(t) = CPz{t) + Du(t)

N
=

-11SEE 4113

(t)=J~l[(si-Ay]=*d~l

adj(sl - A)
si-A

X ( s ) = { s i - A ) ' 1 [x(0) +
x(t) = L~lX(s)

0(0

( k ^ + k 2e ^ ) ( k 3e * + k A e * J
[k5e^ + k6e^) (k7e^+k,eht)

x ( t ) = 0(?).x(O) + j* 0(? - t ) . B . u ( t ) . c I t

0(0) = I

0(0) = A

-ln(%OS7100)

n 1 + ln2(%aS7100)
4

T< =

^ = eo n yl\- 2
K = \kx k2 k3 ]

cM

= [b

ab a2b

7
L =

I,
C
CA
CA2

-12SEE 4113

7.

=r-Cx

Xn

x = Ax + Bz/

= -Cx + r

Xn

y - Cx

Y
A
Xtv

A0

X
+

-C 0

"

w+

"

_1-

y = [ C 0]
N

u = -Kx + K e x , t = -[K - K e \
''N

"(A-BK)

Xn

-C

BKe
0

X
xN

0
1

y = [ C 0}
X,;

e(oo) = l + C(A-BK)~1B
8.

J = | x T ( t ) x ( t ) d t => H T P + P H = - I
J = xT(0)Px(0)
J - \ u T ( f ) u ( t ) d t => H T P + P H = - Q
JO

= i + ktk

J = f [xT (0*(0 + u T ( t ) u ( t ) ] d t => H T P + P H = -(/ + 0


JO

CONFIDENTIAL

UTM

UNIVERSITI TEKNOLOGI MALAYSIA

FINAL EXAMINATION SEMESTER II


SESSION 2010/2011
COURSE CODE

SEE 1003

COURSE NAME

BASIC ELECTRICAL ENGINEERING

LECTURERS

DATO PROF. DR. AHMAD BIN DARUS

PROGRAMME

SEE / SEI / SEL / SEM / SEP / SET / SWB

SECTION

01

TIME

2 HOURS 30 MINUTES

DATE

06 MAY 2011

INSTRUCTION TO CANDIDATE

ANSWER FOUR (4) QUESTIONS ONLY.

THIS EXAMINATION BOOKLET CONSISTS OF 7 PAGES INCLUDING THE FRONT COVER

2
SEE 1003

A voltage source Vs is connected to a resistive circuit consisting of three resistors

Q1 (a)

as shown in Figure Ql(a). Prove that the current flowing through R3 is given by:
R2

------ x V
I" ^2^3 ^3^1 S

[5 marks]
Ri

AA/V
R2

K
6

Figure Ql(a)
(b)

A resistive circuit is shown in Figure Q1 (b)

Figure Ql(b)
i.

Calculate the total resistance as seen by the source.

ii.

Determine the current flowing through the 910 Q resistor.

[5 marks]

[5 marks]
iii.

Determine the voltage between points A and B.


[5 marks]

3
SEE 1003

(c) An electric circuit is shown Figure Ql(c)

-1 A
Figure Ql(c)
i. Determine the power absorbed by the element B and F

[3 marks]
ii. Determine also whether the elements are active or passive.

[2 marks]

4
SEE 1003

Q2 (a) State Kirchoff s Laws.


(b)

[3 marks]

Discuss clearly the importance of the concepts of supemode and supermesh in the
analyses of electrical circuits.

[5 marks]
(c)

A resistive circuit energised by independent and dependent sources is shown in


Figure Q2(c). Using nodal analysis, determine the power available at all sources.

[17 marks]
10 Q

AA/V
15 Q

10 Q

Figure Q2(c)

5
SEE 1003

Q3. (a) (i). Define capacitance.

[2 marks]

(ii). Prove that the energy developed in a capacitor is given by


Ec

1
9
=-CV2

[6 marks]
(b)

The voltage across a 2pF capacitor is shown in Figure Q3(b). Determine the
expression for the current through the capacitor.

[10 marks]
v C (V)

c)

Consider the circuit shown in Figure Q3(c-ii). Determine the charge on each
capacitor.

[7 marks]
9 pF

= = 9 pF
24 V

10 |u.F

72 pF

Figure Q3(c-ii)

6
SEE 1003

Q4 (a) Define what is time constant of a circuit.

[2 marks]
(b)

Prove that the time constant of an RL circuit is given by


_
T~

[6 marks]
(c)

In the circuit shown in Figure Q4(c), the switch K has been at position A for a
long time. It is moved to position B at a t = 0. Determine for time t> 0,
i.

The current flowing through the switch K.

[15 marks]
ii.

The output voltage v 0 (t).

[ 2 marks]

50 Q

50 Q

25 Q

Figure Q4(b)

7
SEE 1003

Q5 (a) Explain what is meant by the effective values of an alternating signal.


[2 marks]

(b)

The periodic waveform of a signal over one period is shown in Figure Q5(b).
Determine the effective value of the signal.
[8 marks]

10 11 12 13

Figure Q5(b)
(c)

Figure Q5(c) shows a circuit consisting of resistor, inductors and capacitors


supplied by alternating voltage and current sources. By using phasor technique,
determine /'/ (t) if
v 2(t) = 0.7571 cos(21 + 66.7 ^ volt
v 3(t) = 06064 cos(2t - 69.8 volt

[15 marks]
5Q

Vi

Figure Q5(c)

CONFIDENTIAL

UTM

UNIVERSITI TEKNOLOGI MALAYSIA

FINAL EXAMINATION SEMESTER II


SESSION 2010/2011
COURSE CODE

SEE 2063

COURSE NAME

ELECTRONICS DEVICES

LECTURERS

MRS. NORHAFIZAH BT RAMLI

PROGRAMME

SEE / SEM / SET / SEW

SECTION

01

TIME

2 HOURS 30 MINUTES

DATE

13 MAY 2011

INSTRUCTION TO CANDIDATE

ANSWER ALL QUESTIONS IN PART A&B.


PLEASE USE ANSWER SHEET (PAGE 11) FOR
THE OBJECTIVE QUESTIONS IN PART A AND
SUBMIT
THE
PAPER
WITH
YOUR
ANSWERING BOOKLET.

THIS EXAMINATION BOOKLET CONSISTS OF 11 PAGES INCLUDING THE FRONT COVER

SEE 2063
PART A : [Total 20 marks]
Choose the best answer for each question. Each question carries 1 mark.
Answer ALL questions on the answer sheet provided on page 11.

1.

2.

3.

How does the conductivity in pure semiconductor vary with temperature?


A.

Conductivity increases as temperature goes down

B.

Conductivity increases as temperature goes up

C.

Conductivity does not change with temperature

D.

Conductivity decreases as temperature goes up

Doping is used to
A.

Decrease the conductivity of an intrinsic semiconductor.

B.

Increase the conductivity of an intrinsic semiconductor.

C.

Stabilize the conductivity of an intrinsic semiconductor.

D.

Increase the insulative quality of an instrinsic semiconductor.

When a trivalent elements are used in doping, the resulting material is called_____
material and has an excess of__________ .

4.

5.

A.

N-type: valence-band holes.

B.

N-type: conduction-band electrons.

C.

P-type: valence-band holes.

D.

P-type: conduction-band electrons.

The isolated atomic structure associated with electron orbital shelss is called a/an
A.

Conduction band

B.

Energy band

C.

Valence band

D.

Energy gap

A positive ion is formed when


A.

a valence electron breaks away from the atom

B.

there are more holes than electrons in the outer orbit

C.

two atoms bond together

D.

an atom gains an extra valence electron

3
SEE 2063
When a PN junction is reverse-biased, its junction resistance is
A.

High

B.

Low

C.

Determined by the components that are external to the device

D. ... Constantly

A PN junction is forward biased when


A.

The applied potential causes the N-type material to be more positive than the
P-type material.

B.

The applied potential causes the N-type material to be more negative than the
P-type material

C.

The applied potential causes the P-type material to be more negative than the
N-type material

D.

Both materials are at the same potential.

Given a series silicon diode circuit with resistor R = 2 kH and an applied voltage of 10
Volts, what is Idq?
A.

4.65 mA

B.

1.0 mA

C.

10 mA

D.

0.5 mA

Why are bridge rectifiers prefered over full-wave center-tapped rectifiers?


A.

They do not require the use of a center-tapped transformer.

B.

They provide higher dc output voltages.

C.

They require a lower PIV rating.

D.

They require less space.

A half-wave rectifier utilizing a silicon diode is connected to an ac source with a voltage


of 20 Vmax. The dc output voltage is
A.

19.3 Vdc

B.

13.65 Vdc

C.

6.14 Vdc

D.

12.29 Vdc

4
SEE 2063
11.

12.

Which of the following circuits is used to eliminate a portion of a signal?


A.

A clipper

B.

A clamper

C.

A voltage multiplier

D.

A voltage divider

Which of the following circuits is used to change the dc reference of a signal without
changing the shape of the signal?

13.

A.

A clipper

B.

A clamper

C.

A voltage multiplier

D.

A voltage divider

The output voltage of the circuit in Figure A.12 will be clamped to

if the diode

is silicon.
A.

10.7 V

B.

5.7 V

C.

4.3 V

D.

9.3 V
C = 1 nF

Figure A.12

14.

A Zener diode is designed to operate in the


A.

Forward operating

B.

Reverse bias

C.

Reverse breakdown

D.

Zero voltage

region of its characteristic curve.

5
SEE 2063
15.

In the cut-off region the base-emitter junction and the base-collector junctions of the
transistor are

16.

A.

Both forward biased

B.

Both reverse biased

C.

Base-emitter junction is forward biased, base-collector junction is reverse biased.

D.

Base-emitter junction is reverse biased, base-collector junction is forward biased.

A given BJT has an alpha of 0.9985 and a collector current of 15 mA. What is the value
of base current?

17.

18.

A.

151.5 pA

B.

15.15 mA

C.

22.53 mA

D.

22.53 nA

A/An_________ is added to the fixed-biased configuration to improve bias stability.


A.

Base voltage

B.

Emitter resistor

C.

Collector resistor

D.

All of the above

When a BJT has its base-emitter junction forward biased and its base-collector junction
reversed biased, it is biased in the____________ .

19.

A.

Saturation region

B.

Cut-off region

C.

Active region

D.

Passive region

The depletion type of MOSFET can operate in the


A.

Depletion mode only

B.

Enhancement mode only

C.

Depletion mode and enhancement mode

D.

None of the above.

6
SEE 2063
20.

Which of the following is true for an N-channel D-MOSFET that is being operated in the
depletion mode?
A.

Id > Idss and VGs is positive

B.

Id < Idss and VGs is negative

C.

Id > Idss and VGs is negative

D.

Id < Idss and Vgs is positive

End of Part A

7
SEE 2063
PARTB: Instruction: Answer ALL questions in Part B.
QUESTION 1
a)

Sketch V0 in relation to input signal Figure 1(a) for at least one complete cycle. Assume
Vdzi = 3.3 V and all diodes are ideal diode.
(6 marks)
1 kQ
+
'zi
V.
3V

b)

Design a clamper circuit that gives a steady state input and output as shown in Figure 1(b).
Draw the corresponding circuit with all the components labeled. Suggest a suitable load
resistance value if use a 1 (iF capacitor.
(6 marks)
V:_
Ii

9.3V

5V

ln
t

0-

V.

Clamper
circuit

out 0

0.7V

-5V

f= 100 Hz
f= 100 Hz

Figure 1(b)
c)

Figure 1(c) shows a bridge rectifier for a regulated DC power supply. Draw the input signal,
the rectified output, V0, and the filtered output Vrl. Clearly distinguish one signal from
another and label all the peak voltages. What is the regulated DC output voltage?
Assume all diodes are ideal.
(8 marks)

Vj,

+
240 V nns
50 Hz

'RL

Figure 1(c)

8
SEE 2063
QUESTION 2
a)

Given the device characteristic in Figure 2(a), determine Vcc, Rb, and Rc for the fixedbias configuration of Figure 2(b).
(6 marks)
rcc

Rr

Figure 2(b)

b) For the biasing network in Figure 2(c), determine:


(0 Base current, Ib
(ii) Collector current, Ic
(iii) Emitter voltage, Ve(iv) Collector-to-emitter voltage, Vce
(v) Draw the dc load line. Label Ic(sat), VcE(max), and the Q-points.

Figure 2 (c)

(4 marks)
(2 marks)
(2 marks)
(2 marks)
(4 marks)

9
SEE 2063
QUESTION 3
a)

A self-bias circuit configuration of a depletion n-channel MOSFET has the following


parameters: Vdd = 20 V Rd = 6.2 kQ, Rg = 1 MQ and Rs = 2.4 kQ
(i)

Sketch and label the circuit configuration.

(2 marks)

(ii) Using data in Table 1, sketch the transfer characteristic curve, Id vs.Vgs-(3 marks)
(iii) Identify the drain-source saturation current, loss, and the pinch-off voltage, Vp.
(2 marks)
(iv)

Analyse the circuit, and determine the Q-point, IDq and Vdsq-

(4 marks)

Table 1: Data for transfer characteristic

b)

Id (mA)

0.5

4.5

12.5

VGS (V)

-8

-6

-4

-2

The E-MOSFET network in Figure 3 has the following parameters:


Id(od) - 3 mA at Vgs = 4V and VGS(th) = 2 V. Determine the operating point, (VGs and Id)

and VDs.

(9 marks)

VDD=+10V

Roi ?
10 MQ T

^G2 $
4.7 MQ

Figure 3

10
SEE 2063
QUESTION 4
For the DTL gate of Figure 4 below, assume all conducting junctions to have a voltage drop of
0.7 V.
4V

a) Find the current through Di when VA = 0.2 V and Vb = 4 V. Also find the voltage at the base
of the transistor Qi.

(5 marks)

b) If VA = Vb = 4 V, find the transistor base current, Ib. If VcEsat= 0.2 V, find the value of p.
(5 marks)
c) Complete the truth table of Figure 4 below. Given VcEsat= 0.2 V.
VA(V)

VB(V)

1.2

1.2
1.2

4.0
1.2

4.0

Vx

V Base

(8 marks)

Vy

State of Qi

4.0
4.0

d) Name the logic gate represented by Figure 4

(2 marks)

11
SEE 2063

NAME:

ICNo:

Lecturer:
Norhaflzah Ramli

Objective Question Answer Sheet (Part A):


Shade your answer in the circle and return this paper with your answer script.
1.
2.
3.
4.
5.
6.
7.
8.
9.
10.
11.
12.
13.
14.
15.
16.
17.
18.
19.
20

CD
CD
CD
CD
CD
CD
CD
CD
CD
CD
CD
CD
CD
CD
CD
CD
CD
CD
CD
CD

CD
CD
CD
CD
CD
CD
CD
CD
CD
<D
CD
CD
CD
CD
CD
CD
CD
CD
CD
CD

CD
CD
CD
CD
CD
CD
CD
CD
CD
CD
CD
CD
CD
CD
CD
CD
CD
CD
CD
CD

CD
CD
CD
CD
CD
CD
CD
CD
CD
CD
CD
CD
CD
CD
CD
CD
CD
CD
CD

CONFIDENTIAL

UTM
UNIVERSITI TEKNOLOGI MALAYSIA

FINAL EXAMINATION SEMESTER II


SESSION 2010/2011
COURSE CODE

SEE 2053

COURSE NAME

ELECTRICAL TECHNOLOGY

LECTURERS

ASSOC. PROF. MD. SHAH BIN MAJID


DR. SAIFULNIZAM BIN ABD KHALID
DR. MOHD JUNAIDI BIN ABDUL AZIZ

PROGRAMME

SEC / SEP / SET / SEW / SWB

SECTION

01-02 /10

TIME

2 HOURS 30 MINUTES

DATE

II MAY 2011

INSTRUCTION TO CANDIDATE

ALL STUDENTS ARE REQUIRED TO ANSWER


FOUR (4) QUESTIONS ONLY.

THIS EXAMINATION BOOKLET CONSISTS OF 7 PAGES INCLUDING THE FRONT COVER

2
SEE 2053

Q 1. (a) Show that for a balanced star and delta loads, the total real power (Pt) is given by
P T = V3F; /, cos 9

where V l is line voltage


II is line current
cosd is a power factor

[6 Marks]

(b) A three phase balanced supply is connected to a star connected 4 wire unbalanced
load. The loads are given as:

Blue Phase
Current magnitude =5A
Reactive Power (Q) =0.75 kVAr
Power factor = 0.9 leading

Yellow phase
Real power (P) = 9kW
Power factor = 0.92 lagging

The phase sequence is positive and use Vrn as reference. The neutral current is
1 Z90A . Find

(i)

Phase Voltages

[3 Marks]

(ii)

Line Currents

[6 Marks]

(iii)

Real and

[4 Marks]

(iv)

Reactive Power at red phase

Total real and reactive power

16 Marks]

3
SEE 2053

Q2. (a) Explain why constructing core is made of magnetic materials.

[3 Marks]

(b) How to reduce core loss in magnetic system.


(c)

[3 Marks]

A core with three legs is shown in Figure Q2(a). Its depth is 10 cm, and there are
500 turns on the center leg. The remaining dimensions are shown in the figure. The
core is composed of a steel having the magnetization curve shown in Figure Q2(b).
By assuming the flux leakage is negligible, answer the following questions about this
core:
(i)

What current is required to produce a flux density of 0.75 T in the central

leg of the core?


(ii)

[5 Marks]

What current is required to produce a flux density of 1.5 T in the central


leg of the core? Is it twice the current in part (i)?
(iii)

[5 Marks]

What are the reluctances of the central and right legs of the core under the

conditions in part c(i)?


(iv)

[3 Marks]

What are the reluctances of the central and right legs of the core under the

conditions in part c(ii)?


(v)

(3 Marks]

What conclusion can you make about reluctances in real magnetic cores?
[3 Marks]
~T~
10 cm

20 cm

10 cm
i

-10 cm-

-20 crrr

-15 cm-

Figure Q2(a)

-20 cm-

-10 cm-

4
SEE 2053

1.8

Flux

1.7

density

B(T)

1.6
1.5
1.4
1.
1.2
1.1
1
0.9
0.8
0.7
0.6
0.4
0.3
0.2
0.1
0
0 200 400 600 800 100012001400160018002000220024002600280030003200

Magnetizing force H (At/m)


Figure Q 2 (b)

Q3. (a) With the help of diagrams, explain briefly how energy conversion process occurs in
an electromechanical energy conversion system.

(b)

[6 Marks]

In the electromagnetic system of Figure Q3(b), the exciting coil has N = 1000 turns.
The cross-sectional area of the core is A = 5 cm x 5cm. Reluctance of the magnetic
circuit may be assumed negligible. Also neglect leakage flux and fringing effect.

5
SEE 2053

Immoveable part

Figure Q3(b)

(i)

X Nd> , . .

Starting from the definition of inductance L h =--------------- , derive the coil

i i

[6 Marks]

inductance as a function of air gap x.


(ii)

What is the field energy when the coil current / = 10 A and air gap x = 2
mm. What is the force on the moveable part under these conditions.

[7 Marks]
(iii)

Find the mechanical energy output when the air gap, x is reduced from
2mm to 1,5mm, assuming that the coil current is maintained constant at

10 A.

[6 Marks]

Q4. (a) (i) Explain the leakage inductance in a transformer.


(ii)

|3 Marks]

Draw and label a complete transformer equivalent circuit. ]3 Marks)

SEE 2053

(b)

An impedance of 1.6+j 1.2 Q is connected to the secondary terminal of a 2400-240V


transformer. The primary is connected to a 2200V line. Assume the transformer is
ideal. Find:
(i)

The secondary and primary current

[2 Marks]

(ii)

The impedance as seen at the line terminal

[1 Marks]

(iii) Apparent power and real power at the output and input side [2 Marks]
(c)

A 5 kVA, 2200/220 V, single-phase transformer has the following parameters:


high voltage side : R| = 3.4 Q, X \ = 7 . 2 Q ,
low voltage side : R2 = 0.028 Q , X2 = 0.06 Q,

The transformer is made to deliver rated current at 0.8 lagging power factor, to a load
connected on the low voltage side. If the load voltage is 220 V, find:

[6 Marks]

(i)

The terminal voltage on the high voltage side.

(ii)

The efficiency if the core loss is 30 watts at rated voltage and frequency.

{3 Marks]
(iii) The readings of voltmeter, ammeter and wattmeter for open-circuit test at
rated voltage and short-circuit test at rated current. The instruments are
connected on low voltage side for open-circuit test and high voltage side
short-circuit test. The no load current of the transformer is 3% of
full-load current.

[5 Marks]

Q5. (a) Name four (4) main parts of a DC machine. Explain briefly the function of these parts?

[6 Marks]
(b)

Draw the speed-torque characteristics of a DC series motor and from the nature of the
curve explain the applications of the series motor.

[5 Marks]

(c)

Give three (3) methods of speed control for a dc motor.

[3 Marks]

(d)

Briefly describe power losses in a shunt-connected dc motor.

[5 Marks]

7
SEE 2053

(e)

Briefly explain at least four (4) differences between three phase induction machines
and three-phase synchronous machines.

[6 Marks]

CONFIDENTIAL

FINAL EXAMINATION SEMESTER II


SESSION 2010/2011

COURSE CODE

SEE 2253

COURSE NAME

ELECTRONIC CIRCUIT

LECTURERS

DR. RUBITA BTE SUDIRMAN


DR. NORLAILI BTE MAT SAFRI
DR. FAUZAN KHAIRI BIN CHE HARUN
MR. CAMALLIL BIN OMAR
MR. ABD HAMID BIN AHMAD
MRS. ISMAWATI BTE ABD. GHANI
MS. MITRA BTE MOHD ADDI

PROGRAMME

SEC / SEE / SEI / SEL / SEM / SEP / SET / SEW

SECTION

01-07

TIME

2 HOURS 30 MINUTES

DATE

15 MAY 2011

INSTRUCTION TO CANDIDATE : ANSWER FOUR (4) QUESTIONS ONLY.

THIS EXAMINATION BOOKLET CONSISTS OF 11 PAGES INCLUDING THE FRONT COVER

-2SEE 2253
Q.l
(a)

An ideal voltage amplifier has a zero power signal dissipation and hence a zero power signal
loss in it. Give TWO other specifications of an ideal voltage amplifier.
[2 marks]

Vcc = + 20 V

+
vn

Figure Ql(a)
(b)

Referring to Figure Ql(a), given that [3 = 100, VT = 26 mV, VA = o, Cbc = = 2 pF,


Cbe = C* = 12 pF and VBE= 0.7 V;
(i)

Show that ICq = 1.05 mA.

[2 marks]

(ii)

Calculate the value of gm and r n .

[2 marks]

(iii)

Draw the hybrid-7i ac equivalent circuit at middle frequency.

[2 marks]

(iv)

Determine the mid-band voltage gain, Av = v0/vj and current gain, Af = i\JU.

[4 marks]

(v)

If CE is ignored, find the dominant high cut-off frequency for the amplifier.

[4 marks]

-3SEE 2253
Vcc = + 12 V

Figure Ql(b)
The amplifier in Figure Ql(b) has the following parameters: P = 100, VBe= 0.7 V,
O ==

90.7 mS, r n = 1. 1 kQ, and VA = oo.

0)

Give one advantage of the amplifiers circuit configuration.

[1 mark]

(ii)

What is the function of coupling capacitor and bypass capacitor?

[2 marks]

(iii)

Draw the hybrid-7r ac equivalent circuit at middle frequency.

[2 marks]

(iv)
(v)

Determine the amplifiers input impedance, Zj and output impedance, Z0.


Calculate the mid-band voltage vain, Ay = v0/vj.

[2 marks]
[2 marks]

-4SEE 2253
Q.2
Figure Q2(a) shows a MOSFET amplifier circuit connected to a Sine to Square Wave Converter
with its input and output signals shown. Figure Q2(b) shows the characteristic curve of the
E-MOSFET. The parameters for the E-MOSFET are: k = 0.6 mA/V2, VG = 5 V and rds= oo.

VDD = +15 V

Figure Q2(a): Amplifier Circuit with vs and v0 signals

-5SEE 2253
l c <A)

Figure Q2(b): Characteristic Curve of E-MOSFET

(a)

State two differences between a D-MOSFET and E-MOSFET.

[2 marks]

(b)

Name the amplifier's circuit configuration.

(c)

Calculate the value of R2 and Rs, if the biasing values for VG = 5 V and Ri = 4 MQ. [4 marks]

(d)

Draw the hybrid-rc ac equivalent circuit at middle frequency.

[3 marks]

(e)

Derive the equation for AVs-

[3 marks]

(f)

Calculate the input impedance, Zj.

(g)

Calculate AVs, gm and RD if RSjg= 500 Q..

[7 marks]

(h)

Determine the dominant low cut-off frequency, ft.

[4 marks]

[1 mark]

[1 mark]

-6-

SEE 2253
Q.3
(a)

State the advantages of an amplifier connected in cascode?

(b)

In what situation will you use a Darlington pair configuration?

(c)

Figure Q3 shows a multi-stage amplifier with cascade configuration. The transistors Qi and

[2 marks]
[1 mark]

Q2 have parameters as follows:


For Qi: loss = 8 mA; Vp = Vos(0ff) = 5 V; Idq = 2.03 mA; Vgsq = - 2.5 V, rdS= o
For transistor Q2: (3 = 179; VA = 00; VT = 26 mV; VBe = - 0.73 V
(i)

Name the amplifiers configuration of each stage.


(ii)

[2 marks]

Obtain the Q-points (IBq, Icq and Vceq) for the 2nd stage of amplifier circuit.
[5 marks]

(iii)

Calculate the ac parameters gmi of transistor Qi and gm2, and r^ of


tran s i stor Q2.

(d)

[4 m arks]

At middle frequency, obtain:


(i)

The hybrid-n ac equivalent circuit.

[3 marks]

(ii)

Input impedance Zj, and output impedance Z0.

[2 marks]

(iii)

Voltage gain Avi = v0,/vj, AV2 - v0/v0i, and overall gain Avs = v0/vs.

[6 marks]

Figure Q3

-7SEE 2253
Q.4

(a)

(b)

(i)

States two specifications of an ideal op-amp.

[2 marks]

(ii)

What is the difference between a comparator and a Schmitt Trigger?

[2 marks]

For circuit of Figure Q4(a), assume output saturate is at Vsat. If Ra = 10 kQ,


Rb=

20 kQ, R, = 5 kQ, R 2 = 20 kQ, Vsa,= 10 V and Vref = 2 V:

(i)

Derive the expression for V UT p and V L tp and determine V s when V 0 = V.

[6 marks]

(ii)

Draw the transfer characteristic curve.

[4 marks]

(iii)

Draw the output waveform, V0 accurately if Vs = 5 sin 2007tt (V).

[3 marks]

ref

Figure Q4(a)

(c) For an op-amp circuit in Figure Q4(b), voltages Va=Vb= sin cot (mV) and V, = V2
(mV). Determine and sketch the output waveforms of, V q i and i 0 .

[8 marks]

10 kD

'01

Figure Q4(b)

0.5 + sin cot

-8SEE 2253
Q.5
A differential amplifier circuit is given in Figure Q5.
(a)

(i) What kind of coupling does the circuit employ?


(ii)

Basic differential amplifier does not have the constant current source
in the circuit. What is so special about the constant current source?

(b)

(i) Determine the equivalent resistance R0.


(ii)

[1 mark]

[2 marks]

[4 marks]

Derive the expression for Vo2 in the form of AdVd + ACVC where Ad is
the differential-mode gain, Ac is the common-mode gain, Vd = (V,-V2)
and Vc = (Vi+V2)/2. For Qi and Q2 transistor models, use pib instead of
gmVji for the dependent current sources.

(c)

Calculate Ad and Ac and then the CMRR in dB.

(d)

If MOSFETs are used for Qi and Q2 instead of BJTs, will the CMRR be higher
or lower? Explain your answer.

[9 marks]

[5 marks]

[4 marks]
Vcc

Figure Q5

-9SEE 2253
Q.6
(a) Figure Q6(a) shows a block diagram of a negative feedback system.

Figure Q6(a)

(i)

(ii)

The main problem of an amplifier circuit without a feedback is stability.


Explain how a negative feedback can help stabilize the gain of an amplifier.

[3 marks]

Derive the expression for gain with feedback, Af in terms of A and p.

[3 marks]

(iii) Write down the expressions for the input impedance, Rjf and the output
impedance, Rof, in terms of A and (3 for trans-resistance amplifier.

[2 marks]

(iv) Show that an expression in (ii) above will result in the gain with feedback
becoming smaller than the gain without feedback.

[2 marks]

-10SEE 2253
(b) A trans-resistance amplifier (with voltage-parallel feedback) is shown in Figure Q6(b).

Figure Q6(b)

Given that the open loop gain of the amplifier, (.1 = 106 V/A, the differential input resistance,
Rid =10 kQ and the output resistance, r0 = 5 kQ. Use the feedback method to:
(0

v
Calculate the voltage gain without feedback, A = .
i,

[5 marks]

Calculate the feedback ratio for the feedback network, p = .


Vo
v
(iii) Calculate the closed-loop gain, Af = .
is

[4 marks]

(iv) Calculate the input impedance R,f and output impedance Rof.

[4 marks]

00

[2 marks]

-11-

SEE 2253
SIGNIFICANT EQUATIONS

Av =-2sa- = A
v.
v
A.. =-SHL = A v(oc)

R,

A,

Z +R

+ Z,

L /V s R.
i J

Z, ^
V s +Z; J

V^O

A R

AV
SR = 2- V/lls
At

+ Z, .

- Ai

rr

V Re3

+ r it + R BB r o

+R

c = cMo be

= cMo= c
V AV J

r ds

fVA

l=k(Vos-V,J

= 2k{V G S -V J h ) = 2^

Jsin x dx = -cos x

='

] GS

V. vpy

= Advd +Acvc

CM, = Cbc(l-Av) = Cgd (l - A v )

1D=DSS
I

75m=&mo
Cr

1 R c +Z.
vo

PR E3

= r. 1+ -

Ap=-^a- = AvAi
Pin
BmV.RE

R-L j

Y Rs ^
Z +R,

Pfft

A v(oc) ' Z,

rF h= or +

2jt(Ck+CJ

R,

A;

A v(oc)

fT =

v(oc)

DSS

D(on)

Iv - VGS(on)
V Th /
V

21,
V
V

jcos x dx = sin

gd

CONFIDENTIAL

UTM
UNIVERSITI TEKNOLOGI MALAYSIA

FINAL EXAMINATION SEMESTER II


SESSION 2010/2011

COURSE CODE

SEL 4363

COURSE NAME

DIGITAL IMAGE PROCESSING

LECTURERS

DR. MUSA BIN MOHD MOKJI

PROGRAMME

SEC/SEL

SECTION

01

TIME

2 HOURS 30 MINUTES

DATE

25 APRIL 2011

INSTRUCTION TO CANDIDATE

ANSWER FOUR (4) QUESTIONS ONLY.


NOTES
AND
ORIGINAL
BOOKS
ALLOWED.
CALCULATORS ARE ALLOWED.

ARE

THIS EXAMINATION BOOKLET CONSISTS OF 7 PAGES INCLUDING THE FRONT COVER

-2SEL 4363

QUESTION 1

Figure Ql.l
Figure Ql.l is an 8 bit image and its histogram is shown in Figure Q1.2.
a)

State two differences between piecewise linear transformation and histogram


equalization.
(4 marks)

b)

Sketch the histogram of the output image for each technique given below when
applied to the image shown in Figure Ql.l
i.
ii.

Power-law transformation with y = 0.5 and c = 1


Negative image
(8 marks)

c)

Describe how the output image will look like if image in Figure Ql.l is:
i.
ii.

d)

First applied with histogram equalization and then transformed with linear
transformation where s = r 50.
First transformed with linear transformation where s = r 50 and then
applied with histogram equalization.
(8 marks)

Sketch the piecewise linear transformation function (the transformation line) that can
be used to improve the contrast of image in Figure Ql.l. Minimum gray value of the
output image should be at 0 while the maximum gray value must equals to 255.
(5 marks)

-3SEL 4363
QUESTION 2

Figure Q2.1

Figure Q2.2

Figure Q2.1 shows an 8bit image with the size of 100x100 with a 10x50 rectangular white
(gray value = 255) box as foreground and black (gray value = 0) as background. Figure Q2.2
is then the magnitude frequency spectrum of the image in Figure Q2.1.
a)

If Figure Q2.1 is denoted as f ( x , y ) and Figure Q2.2 is F(k, /), compute F(0,0).
(6 marks)

b)

Explain whether statements below are true or false and give your reason to support it.
i.

Vertical edges in the image of Figure Q2.1 contribute to a single frequency in the
magnitude spectrum shown in Figure Q2.2.
(4 marks)

ii. Frequency for area other than edges in Figure Q2.1 is located on the DC
component.
(4 marks)
c)

Given a filter in frequency domain as below where H h p is a highpass filter and /? is a


constant.
H = H h p (fi - 1) + 1

i.

Describe the output image when image in Figure Q2.1 is passed through with the
filter H for /? = 0.3.
(5 marks)

ii. Suggest the value for /? so that filter H will act as a sharpening filter and the value
for /? such that it will act as an allpass filter.
(6 marks)

-4SEL 4363

Two images have been captured using one camera at uniform illumination. The first image is
a uniform gray board where its histogram is shown in Figure Q3.1. The second image
captured is shown in Figure Q3.2.
a)

Determine the type of noise which degraded both images captured by the camera.
(2 marks)

b)

Estimate mean and variance of the noise that degraded both captured images.
(6 marks)

c)

In Adaptive Local Noise Reduction Filtering technique, given below are the two
possible outputs of the filter. Based on the two outputs, determine which one will be
the output for the areas shown by the three square regions (A, B and C) shown in
Figure Q3.2. Explain your answer.

/0,y) = g ( x , y )
or
av
f ( x , y ) = g ( x , y ) ----- j [ g ( x , y ) - mj
aL

(9 marks)
d)

In case the camera used to capture image in Figure Q3.2 is not available and Figure
Q3.1 is also not available, which of the three square region A, B and C that can be
used to estimate the parameter of noise corrupting the image. Explain your answer.
(4 marks)

e)

Suggest other two spatial filters that can be used to remove the noise in the image
shown in Figure Q3.2
(4 marks)

-5SEL 4363
QUESTION 4
Gray value

Figure Q4.1
In order to identify the number or code on the barcode image shown in Figure Q4.1, width for
each of its bar (vertical lines) needs to be measured. As a guide, Figure Q4.2 shows a gray
value plot taken across line a-b. (Note: Figure Q4.2 is not image histogram)
a)

To measure the width of the bar, image in Figure Q4.1 should be first threshold. Thus,
Figure Q4.1 will have only 2 values, one for the background and zero for the
foreground. Then the width of the bar can be measured by counting the number of
columns occupied by the value one and zero. The question is, between global
thresholding and adaptive thresholding, which one is better for this application.
Explain your answer.
(5 marks)

b)

If global thresholding is to be applied to the image in Figure Q4.1, suggest a suitable


threshold value.
(5 marks)

c)

Instead of thresholding, the width of the bar can also be measured by first finding
edges of the bar and then measuring the distance between the edges. Figure Q4.3 is an
example of a first order derivative edge detector operator.
-1 1

Figure Q4.3
i.

Sketch the gray value plot for the image at scan line a-b (such as in Figure
Q4.2) when operator in Figure Q4.3 is applied to the image in Figure Q4.1.
(5 marks)

ii.

Based on results in (i), explain how the width of the bar can be measured
(5 marks)

iii.

If the above operator is replaced with Sobel operator, explain the differences
that might occur in the output image.
(5 marks)

-6-

SEL 4363

QUESTION 5

Figure Q5.1

Figure Q5.2
Figure Q5.1 shows a 3 bit image with histogram as shown in Figure Q5.2. This image is to be
stored in a memory disk which has only 120 bit of free memory.

a)

List down the three types of redundancy


(3 marks)

b)

Which type of redundancy can be found in Figure Q5.1? Explain your answer
(6 marks)

c)

One such method of compressing image in Figure Q5.1 is based on Figure Q5.3
below. Compute the size in bit of the compressed image shown in Figure Q5.3.
(8 marks)
Input
Image

QUANTIZER:
Quantization
into 2 bit image

MAPPER:
Run Length
Coding

Compressed
Image

Figure Q5.3
d)

It is known that size of compressed image in (c) is not small enough to be stored in
the memory disk. Suggest other compression method (can be lossless or lossy) that
can result an output with size less than the 120 bit. Show all computation.
(8 marks)

-7SEL 4363

QUESTION 6
a)

Compute the size in bits of a 100x100 image which consists only gray values 0 and 1.
Bit depth of this image is 8 bit.
(3 marks)

b)

What is the difference between binary image and 1 bit image?


(4 marks)

c)

Explain whether quantization process can be used to binarize an image. If yes, give
one disadvantage of using quantization as a binarization process. If no, give an
alternative technique for the image binarization process.
(6 marks)

d)

What is gray value normalization and give one example of its usage.
(6 marks)

e)

Explain whether the processes below can be used for image compression. If yes, state
which type of redundancy that is removed from the process.
(6 marks)
i. Quantization
ii. Normalization
iii. Resize into smaller image.

CONFIDENTIAL

UTM
UNIVERSITI TEKNOLOGI MALAYSIA

FINAL EXAMINATION SEMESTER II


SESSION 2010/2011

COURSE CODE

SEP 4243

COURSE NAME

BIOSYSTEM MODELING

LECTURERS

DR. AKRAM GASMELSEED ABDALLA


MUKHTAR

PROGRAMME

SEP

SECTION

01

TIME

2 HOURS 30 MINUTES

DATE

25 APRIL 2011

INSTRUCTION TO CANDIDATE

THERE ARE FOUR (4) QUESTIONS, ANSWER


ALL.
READ THE QUESTIONS CAREFULLY.
ORGANIZE YOUR WORK AND WRITE
LEGIBLY.

THIS EXAMINATION BOOKLET CONSISTS OF 8 PAGES INCLUDING THE FRONT COVER

2
SEP4243

Question 1 [25 marks]


a)

Consider an enzymatic reaction in which an enzyme can bind two substrate molecules, so
it can exist in one of three states, namely as a free molecule E, and as a complexes C/ and
C2, as follows:

S + EZ=r C } E + P
^2

+ CiZ~r
k.

c9

c} +p

Write down the rate equations for the concentrations [S], [Cj], and [C2]?

[6 marks]

b)

Draw a diagram showing the architecture of n-compartment closed system. Give the first
order differential equations describing the rate of change of concentration for the traced
substance (Qj) with time.
[6 marks]

3
SEP4243

c)

Figure 1 shows the flow of insulin in the body. A patient is given an insulin injection in
the upper arm. The insulin then dissolves into the bloodstream. Once in the blood, insulin
flows to and from the kidneys, as well as to and from the pancreas.

Figure 1

i.

Determine the rate equations for this system.


[2 marks]

ii.

Write a MATLAB code to solve these equations for the concentration of insulin
in each compartment. (Assume Kin = 2, Ki2= 1, K2i = 1.5, K]3 = 2, K3j = 1.75)
[6 marks]

iii.

Write a MATLAB code to plot the concentrations of insulin in kidneys


compartment for up to 10 minutes. (The only initial condition is X4(t) = 25, other
initial conditions are 0)
[5 marks]

4
SEP4243

Question 2 [25 marks]


a) Describe with aid of compartmental analysis the entrohepatic circulation?
[5 marks]

b) Define hypertonic, hypotonic, and isotonic and their relationship with osmosis into and
out of cells.
[4 marks]
c) Determine the factors that affect Renal Clearance (CIr).
[3 marks]

d) Describe the two-compartment model of Renal Clearance (Cl#).


[5 marks]

e) Complete the following function code which can be used to solve the Renal Clearance
(Clfi) rate equations:
function clearance = renal(t,y)
c = y(l); % central compartment
p = y(2); % peripheral compartment
[8 marks]

5
SEP4243

Question 3 [25 marks]


a)

The Hodgkin-Huxley model can be understood with the help of Figure 2. The semi
permeable cell membrane separates the interior of the cell from the extracellular liquid
and acts as a capacitor. If an input current Ist(t) is injected into the cell, it may add further
charge on the capacitor, or leak through the channels in the cell membrane. Because of
active ion transport through the cell membrane, the ion concentration inside the cell is
different from that in the extracellular liquid.

jj-+

/ inside

*v

:.++++++

\ outside

^a+

Figure 2. Schematic diagram of neuron cell membrane

i.

Draw the electrical equivalent circuit of a neuron, including membrane


capacitance and Na+, C1-, and K+ ion channels.
[3 marks]

ii.

Write the current/voltage equations for the electrical circuit.


[3 marks]

iii.

Derive the Hodgkin-Huxley equation describing the rate of change of membrane


potential as a function of ion conductances.
[3 marks]

iv.

Describe the rate of change of opening of K+ channel.


[3 marks]

6
SEP4243

b) The arm in Figure 3 weighs 41.5 N. The force of the gravity acting on the arm acts
through point A. Assume that Li = 0.082 m, L2 = 0.273 m and a = 14.0.

Figure 3 Schematic diagram of human arm

i.

Determine the magnitude of the tension force Ft in the deltoid muscle.


[4 marks]

ii.

Determine the magnitude of the tension force Fs of the shoulder on the humerus
(upper-arm bone) to hold the arm in the position shown.
[4 marks]

iii.

Determine the angle of tension force Fs relative to the x-axis, 0.


[4 marks]

7
SEP4243

Question 4 [25 marks]


a) Poisson equation is a class of elliptic partial differential equations. A form of the Poisson
equation is:
d2<f> d2<f> _ P
dx2+ dy2~ T

i.

Solve Poisson equation using the central finite difference method?


[5 marks]

ii.

Construct a MATLAB script segment that models the above equation in 1 cm2
membrane that firmly fastened. Assume = 0.5
T

[5 marks]

b) The following reaction stoichiometry describes the aerobic growth of yeast on ethanol.
CH3CH2OH + aC>2 + bNHh ---------------------- cCH1704N0.149O0.408 dC02 + eH20

i.

Write the mass balance equations in the standard linear form A*x = b.
[5 marks]

ii.

Find the stoichiometry coefficients? Assume the respiratory quotient (RQ) is


equal 0.66
[5 marks]

8
SEP4243
c) In muscle fiber, Oxygen (O2) is bound to Myoglobin (Mb), a much larger molecule, and
is transported as Oxymyoglobin (Mb02) through the following chemical reaction:

Write down the set of diffusion equations to model the concentration of the different
chemicals? Assume: s = [O2], e = [Mb], c = [MbC^] and let f denote the rate of uptake of
Oxygen into Oxymyoglobin.
[5 marks]

CONFIDENTIAL

UTM
UNIVERSITI TEKNOLOGI MALAYSIA

FINAL EXAMINATION SEMESTER II


SESSION 2010/2011

COURSE CODE

SET 4543

COURSE NAME

COMMUNICATION ELECTRONICS

LECTURERS

ASSOC. PROF. DR. MOHAMAD KAMAL BIN


A. RAHIM

PROGRAMME

SET

SECTION

01

TIME

2 HOURS 30 MINUTES

DATE

26 APRIL 2011

INSTRUCTION TO CANDIDATE

ANSWER FOUR (4) QUESTIONS ONLY.

THIS EXAMINATION BOOKLET CONSISTS OF 8 PAGES INCLUDING THE FRONT COVER

-2-

SET 4543

ANSWER FOUR QUESTIONS ONLY

Q1 (a) Draw the block diagram of the RF front end transmitter and a receiver and
discuss the significant of each block. What is the effect of image frequency in
the receiver circuit.

(b)

(15 marks)

An amplifer with a bandwidth of 1 GHz has a gain of 15 dB and a noise


temperature of 250 K. If the 1 dB compression point occurs for an input power
level of-10 dBm, with a suitable diagram, show linear dynamic range, 1 dB
compression point, minimum detectable signal and calculate the linear dynamic
range

(10 marks)

(Total marks = 25 marks)

Q2 A band pass filter receiver with a specification below must reject an adjacent channel
signal that is 160 MHz away from center frequency:

Upper frequency: f3dBH = 2170 MHz

Lower Frequency f3dBL = 2110 MHz

Selectivity (out-of-band attenuation) minimum 30 dB at f2 = 2300 MHz

Response type: Chebyshev with 0.5 dB

Source and load impedance: 50Q.

(a)

Find the filter order number

(b)

Determine the inductance and capacitor of the bandpass filter using n network

(6 marks)

(6 marks)

(c)

Draw the band pass filter circuit diagram using L and C components
(4 marks)

(d)

If the response is change to maximally flat response, find the new number of
filter order and the advantages of using this response

(9 marks)

(Total marks = 25 marks)

-3SET 4543

Q3 An amplifier has to be designed with maximum Gtu using transistor with the following
S parameter (Z0 = 50 ohm) at 6 GHz:
5 = 0.61Z-170", S 2 1 = 2.24Z32", S12 = 0, S22 = 0.72Z-83
(i)

Determine the stability of this amplifier.

(4 marks)

(ii)

What are source impedance and load impedance.

(4 marks)

(iii)

Find the maximum gain.

(4 marks)

(iv)

Design matching network using L section matching network.

(13 marks)

(Total marks = 25 marks)

Q4 Design a transistor oscillator at 6 GHz using FET in common source configuration


driving a 50 ohm load on the drain side. The S parameters are

Su =
(i)

0.9Z-150 0 ,

S2] =

2.6Z50 0 ,

S]2 =

0.2Z-15 0 , S 22 = 0.5Z-105 0

Calculate and plot the output stability circle and choose a suitable Tx for |rj|l

(10 marks)
(ii)

(15 marks)

Design the load and terminating network

(Total marks = 25 marks)

Q5

(a)
(b)

Draw a balance mixer circuits using a 90 hybrid.


(6 marks)
If the RF input is given by v^^) = VRF cos coRFt and local oscillator input is
given by v L 0 { t ) = VL0 coscoLOt. Using this balance mixer circuit, analyze the
output after filtering process.

(c)

If v^r(t) = 2cos(2^500x

106)t

and v L 0 ( t ) = cos(2^ x 450 x

106)

(14 marks)
t , show that

after filtering process the output will be i I F (t ) = - K 2 sin(2;r x 50 x 106) t , where


(5 marks)

K is the constant value


'0 j 1 O'

The S parameter for 90 hybrid junction is [S'] = =


\2 1 o 0 j
01j 0

(Total marks = 25 marks)

-4SET 4543

Transistor Equations

ssr

rin=su+^^L
i-s22rL

_ B} - J - 5 , 2 - 4 | C ] | 2

c r

52J5/-4C2

r, =

2C,
5, =l

r
r - ^ 12 29i ^

out 2 2 i

2C,
52=r+|522|2-|SI1|2-|A|2

|sn| -|<y -|a|

C . ^ . - A S * * C 2 = S22 - A S , , *
A = SUS22 Sl2S2l

2 i- r,

l-l-S-.,!+|A|"

G; =

'1^211
rs| M22r,

S\ 2$21

c
L

i^r-iAi2

w-w2

Cc = _ (S'l 1 AS22 )
A|:

7' _

Rs =

_ {$22 )
,2 I |2 5
22 ""A.

p~

|i-rrr,n|2|i-s22rLj2

^12^21
l^n|2-|A|2
Sl2S2[
in

99r
r c , i22i1r
11

i-s22n,

Filters Transformation

Low-pass

High-pass

skR

St* J .
CO.

Band-pass

Band-stop
l

gk&R

>gk&R

<*>Mk

A
o>oSkR
R

Sk

coR

agkA

AR

cgk

0JoSk

01, AR

. SjA
- conR

-5SET 4543

Table 1 Element values of equal ripple Low pass filter prototype for 0.5 dB ripple
N

gi

g2

0.6986

1.0000

1.4029

0.7071

1.9841

1.5963

1.0967

1.5963

1.000

1.6703

1.1926

2.3661

0.8419

. 1.9841

1.7058

1.2296

2.5408

1.2296

1.7058

1.000

1.7254

1.2479

2.6064

1.3137

2.4758

0.8696

1.9841

1.7372

1.2583

2.6381

1.3444

2.6381

1.2583

1.7372

1.0000

1.7451

1.2647

2.6564

1.3590

2.6964

1.3389

2.5093

0.8796

1.9841

1.7501

1.2690

2.6678

1.3673

2.7239

1.3673

2.6678

1.2690

1.7054

1.0000

10

1.7543

1.2721

2.6754

1.3725

2.7392

1.3806

2.7231

1.3485

2.5239

0.8842

g8

g9

gio

g3

g4

g5

g6

g7

gs

g9

gio

Table 2 Element values of Maximally flat Low pass filter prototype

gi

2.000

1.0000

1.4142

1.4142

1.0000

1.0000

2.0000

1.0000

1.0000

0.7654

1.8478

1.8478

0.7654

1.0000

0.6180

1.6180

2.0000

1.6180

0.6180

1.0000

0.5176

1.4142

1.9318

1.9318

1.4142

0.5176

1.0000

0.4450

1.2470

1.8019

2.0000

1.8019

1.2470

0.4450

1.0000

0.3902

1.1111

1.6629

1.9615

1.9615

1.6629

1.1111

0.3902

1.0000

0.3473

1.0000

1.5321

1.8794

2.0000

1.8794

1.5321

1.0000

0.3473

1.0000

10

0.3129

0.9080

1.4142

1.7820

1.9754

1.9754

1.7820

1.4142

0.9080

0.3129

g3

g4

gs

g6

g?

- 6 -

SET 4543

iH
Figure 1 Attenuation versus normalized frequency for equal ripple

(dB)

filter prototypes 0.5 dB

Figure 2 Attenuation versus normalized frequency for maximally flat filter

-7SET 4543
NAMC

TITL ' ' '------------------------------------------------------------------------------------------------------------

SMTh CHART FORM 52-BSPW (9-86J

KAr ELECT r ic comi*ny, PINE BBOOR.N.O, O IMS. PWINTCD in USa'

OWt. NO
MTS

IMPEDANCE OR ADMITTANCE COORDINATES

A II WIT

SET 4543
NAME
S**TH CHART FOWd 82-B5PP (S-QS1

TITLC ' " ~ ------------------------------------------------- ------------ -----KAY ELECTRIC COMHHt, PIMC BROOK. Ni, QlM. M|NTCO IN USA

IMPEDANCE OR ADMITTANCE COORDINATES

OW*. NO
OATl

CONFIDENTIAL

UTM
UNIVERSITI TEKNOLOGI MALAYSIA

FINAL EXAMINATION SEMESTER II


SESSION 2010/2011

COURSE CODE

SEI4143

COURSE NAME

COMPUTER PROCESS CONTROL

LECTURERS

DR. NORHALIZA BT ABDUL WAHAB

PROGRAMME

SEI

SECTION

01

TIME

3 HOURS

DATE

28 APRIL 2011

INSTRUCTION TO CANDIDATE

YOU ARE REQUIRED TO ANSWER ANY FOUR


(4) QUESTIONS (25 MARKS EACH). YOU ARE
REQUIRED TO ATTACH THIS EXAM
QUESTION TO YOUR ANSWER BOOKLET.

THIS EXAMINATION BOOKLET CONSISTS OF 15 PAGES INCLUDING THE FRONT COVER

-2-

SEI4143

Ql.
a.

In process control design, setting the control objectives requires an engineer to


have a clear understanding of the process. Describe four major categories of
control objectives.

[4 marks]

b.

Figure Ql(b) shows a liquid level system in a tank. Your goal is to determine the
relationship between the input flow rate, q, and the output level of the liquid in
the tank, h. The differential equation of the level of liquid in the tank, h is derived
based on the component material balance equation and is given by:
dh

(1.1)

dt A t
where equation (1.1) is nonlinear and the constant parameters are (A, - crosssectional area of the tank, A or - cross-sectional area of an orifice, g - gravity (g =

!))
i.

Prove that equation (1.1) can be linearized using Taylor series expansion and
is written as follows:
dh' q'i Aor^ h,
__

(1.2)

dt A, 2A,Jh,

where the deviation variables given as h' = h-h sand q] = q t -q ls


Note: The subscript s denotes steady state values
[8 marks]

-3SEI4143
ii.

Find the transfer function which relate the output level of the liquid in the
tank, h and the input flow rate, q, such as G ( s ) -
Q'\s)

[5 marks]

c.

Name two modelling methods to develop the mathematical model for a chemical
process. Give one advantage and one disadvantage of each method.
[4 marks]

d.

State whether the following statements are either TRUE or FALSE:


i.

Usually, the models used in process reaction curve methods take the form of
high-order transfer functions with dead time which is capable of predicting
most aspects of the process performance. (TRUE/FALSE).

ii.

Empirical modeling is mostly effective when there are a lot of process input
and output data available and formulating the fundamental model is difficult.
(TRUE/FALSE).

iii.

The statistical process estimation approach requires extensive computation


and only works for step inputs. (TRUE/FALSE).

iv.

When conducting an experiment using process reaction curve approach, the


focus will always be between one specific input and one specific output
variables of a process plant only. (TRUE/FALSE).
[4 marks]

-4SEI4143

Q2.
a.

Briefly describe two situations where you should use the following control
strategies:
i.
ii.
iii.

Feedforward control
Cascade control
Inferential control
[6 marks]

b.

Consider the stirred-tank reactor with cooling jacket as shown in Figure Q2(b).
The control objective is to keep the temperature, T at a desired value. Possible
disturbances to the reactor are the feed temperature, T and the coolant
temperature, Tc. The manipulate variable is the coolant flow rate, Fc.

Observation '. T will respond much faster to changes in 7} than to changes in Tc

i.

Based

on

the

above

observation,

briefly

comment

on

the

control

performance in term of disturbance rejection (r, and Tc) if only single-loop


control is implemented in the system shown in Figure Q2(b).
[3 marks]

ii.

Design a suitable control strategy using two different measurements, T and

Te, but sharing a common manipulated variable, Fc. Sketch the required
changes of your design in Figure Q2(b).
[4 marks]

iii.

Draw a block diagram for the control strategy given in part ii and label the
input-output relationship completely.
[3 marks]

c.

The approximate model for the system shown in Figure Q2(b) is obtained using
empirical method and is given as:
, 0.039e~55s

i.

Determine the tuning constant for a feedback PID controller for a


disturbance response (step in T,:) using the Ciancone tuning correlation
procedures.

-5SEI4143

Note: Use the Ciancone correlation graph in the attachment to determine the
dimensionless tuning constant based on the calculated fraction dead time.

[9 marks]

T,

Figure Q2(b)

Digital control has many advantages compared to analog control. Briefly describe
two main advantages of digital systems.

[3 marks]

Consider a flash separation system given in Figure Q3(b) with flow and level
control using discrete PID controller. The performance of the system under
discrete PID controller depends on the selection of execution period. Given the
process transfer functions for the two controllers as below:

i.

If the digital execution from discrete PID controller is assumed not


significantly affect the control performance, determine the maximum
execution period for each controller (for flow, F and level, L).

[3 marks]

ii.

Design two (2) digital PID controllers for disturbance response of each
variable (flow and level) using the execution period defined in part b) i.

[6 marks]

iii. Considering the effect of an execution period in digital control, design two
(2) digital PID controllers for disturbance response of each variable (flow and
level) when the execution period is set to 4 minutes.

[6 marks]

Briefly discuss the effect of the performance of a discrete PID controller with
respect to the length of the execution period implemented in part part b) ii and iii.

[3 marks]

-7-

SEI4143
d.

The final step in the controller tuning procedure is fine-tuning the initial
controller tuning constant until acceptable performance is obtained. Based on
the dynamic response of the control system shown in Figure Q3(d), diagnose
the performance given by plant A, B and C and suggest one suitable corrective
action for plant C.

[4 marks]

_ Vapor

Feed

Steam

Figure Q3(b)

Figure Q3 (d)

Liquid

Time

SEI4143

Q4.
Figure Q4(a) shows the three series tank of mixing process. The outlet concentration
is to be maintained close to its set point using proportional-only feedback algorithm
(P-only controller). All tanks are considered well-mixed and the dynamics of the
valve and sensor are negligible. Assuming that FB^> FA, the linearized models have
been derived. The process transfer function, Gp(s) and disturbance transfer function,

Gd(s) are given as:


0039

a.

<5)

(5s + l)3

*(

(5j + 1)3

Determine the stability of the three-tank mixing process for a disturbance change
if no controller is applied (Kc is set to zero).

[5 marks]
b.

The system shown in Figure Q4(a) was simulated with a PI controller using
Ziegler-Nichols tuning method. Two different disturbance dynamics have been
tested to the system as follows: (Case 1: Gdi and Case 2: Gdi)

1
(5s + l) 3
1
(5s + l)
Based on the simulation result of the above two cases, the dynamic response of
the system shows better performance in Case 1. Explain by giving at least one
reason for this observation.
[3 marks]

-9SEI4143
B

XAO

Figure Q4(a)
c.

Consider the block diagram of the distillation process shown in Figure Q4(c). The
reflux ratio is the manipulated variable, m and the control objective is to maintain
the overhead product composition, y. The feed composition, d is the unmeasured
disturbance. Since the feed and overhead composition are considered unmeasured,
the inferential control configuration can be used to control an unmeasured
controlled output, y in the presence of unmeasured disturbances, d.
i.

Given the input-output relationship of the system as follows:

y = Gpl(s)m + Gdl(s)d
z = G p 2 (s ) m + G d 2 ( s ) d
where y and z are unmeasured variable and measured variable, respectively.
Based on the above equations, prove the following estimator which relates the
unmeasured controlled output, y and the measured variables ( m and z) :

, . Gj, (s)

y-

gp1(s)-^\4g

/>2(*) m +

Gdi (s')

[5 marks]
ii. Redraw the block diagram given in Figure Q4(c) including the estimator
proven in part (i) to show the inferential control system structure.
[6 marks]

-10SEI4143
iii. For a given transfer function as follows:

T]s
GAs)=

Kn,

V s+ 1

Gdl(s)- td,s + \

TpS + 1

Vr + 1

show that in order to maintain zero steady-state deviation in y in response to a


disturbance D ( s ) = , the relationship between the gains of the inferred and the

K
actual plants transfer function must be such that = .
KH

*,2

[6 marks]

Feed

Figure Q4(c)

-11SEI4143

Q5.
a.

Process interaction is one of major problem in multivariable control. Briefly


discuss the definition of interaction.

[3 marks]
b.

The model derived by the empirical procedure for a distillation tower are as
follows with time in minutes:
" 12.8e-1

XD(f)
XB{s)_

16.75 + 1
6.6e7'

.10.95 + 1

-18.9e31"
215 + 1
-19.4e~3'5 LWJ
14.45 + 1 .

i. Determine whether the input-output combination is controllable

[4 marks]

ii. Determine if either loop pairing can be eliminated based on the sign of the
relative gains (Ay > 0 ) .

[5 marks]
iii.Determine

the initial tuning for PI controllers for all allowable loop

pairings.

[10 marks]
c.

List down three approaches that typically used for tuning multiloop systems

[3 marks]

-12-

SEI4143

Attachment

Table of Commonly Used Equations


Component
Material Balance
Energy Balance

{Accumulation of component mass}


= {component mass in}-{component mass out}
+ {generation of component mass}
{Accumulation of U + PE + KE}={U+PE+KE in due to convection}
-{U+PE+KE out due to convection}

+o -w
Taylor Expansion

d2F

rv \ cv \ d F / \ 1
/ \2
F(x)-F(x)+ (x-xj + , (x xj +...
K s J 2! d x
s dx
X

xs

General 2nd order


system
Peak
Time
relationship

G ( s ) = --------^----------- j
s + 2Qa>ns + con
-T _ *

Overshoot
relationships

%OS = e vx j oo

Continuous form
PID algorithm

MV(t) = kc

Continuous
PID
(in
Laplace
Domain)
Discrete
PID
controller

V
Xs

Uy 1

E{t)

y\E{t')dt'-Td^^- +/
*/ o at j

f E ( s ) 'I
MV(s) = Kc E(s) + -^--TdsCV(s)
I Tis J

MV(t) = Ke

At

lj

T
^+^t^-t7(ckw-cfw.1) +/
,=1 Ar J

-13SEI4143
Ciancone correlations for Dimensionless tuning constants, PID algorithm.

Disturbance (a,b,c)

Set point change (d,e,f)

fraction dead umc (g^;)

1.0
.90 JO
.70
X -60
30
-/
JO * J
,20 .
40

LG
.90
,30

JV
/^SV
r* c\?

+ -60
S -50

20 '

I.,,..I..L.......J-----0 .10 .20 JO ACJSO0 ,70 .*0 .90 1.0

AO
m ... ..... *..1....i--- 3--------1--- 1--0 .10 .20 JO .*V .50 .60 .70 .#0 .90 1,0
r.'m tion tlewi lime

I'racuon ad time

(c)

>
,40

.40
30

% .20

p/5 \

.10

-- J-1-U --- i

,30 +
5 ,20
S*
IQ
m

\
\
ft*,,,

0 .10 ,20 JO AO .50 M .10 JO .90 1.0

0 .10 JO JO .40 JO .> ,70 JO . \J0

Fraction *k*d time

frastkmcteidHme (^)

(/)

-14SEI4143
Ciancone correlations for Dimensionless tuning constants, PI algorithm.
Disturbance (a,b)

Set point (c,d)

Fraction dead time (g+

Fraction dead time

()

(c)

0 0.1 0.2 0.3 0.4 0.5 0.6 0.7 0-8 0.9 1.0
Fraction dead time

Fraction dead time


(J)

-15SEI4143
Relationship between single-loop (SL) and multi-loop (ML) PI controller tuning
when both loops have similar dynamics.

(a)

CONFIDENTIAL

UTM

UNIVERSITI TEKNOLOGI MALAYSIA

FINAL EXAMINATION SEMESTER II


SESSION 2010/2011
COURSE CODE

SEL 4263

COURSE NAME

COMPUTER SYSTEMS & MULTIMEDIA

LECTURERS

ASSOC. PROF. DR. SULAIMAN BIN MOHD NOR


DR. USMAN ULLAH SHEIKH
MR. ZURAIMI BIN YAHYA

PROGRAMME

SEC / SEE / SEM / SEP / SET / SEW

SECTION

01-03

TIME

2 HOURS 30 MINUTES

DATE

10 MAY 2011

INSTRUCTION TO CANDIDATE

PART A : ANSWER ALL THREE QUESTIONS.


PART B : ANSWER ONE (1) QUESTION ONLY.
FOR QUESTION 1 AND 3 ANSWER IN THE
RESPECTIVE BLANK SPACES ALLOCATED TO
THE QUESTIONS.

THIS EXAMINATION BOOKLET CONSISTS OF 18 PAGES INCLUDING THE FRONT COVER

SEL 4263
PART A

Question 1 (One mark for every answer)


(Write your answers in the respective blank space given in the Question)
a. Given the initial condition
int n = 4 , k = 2;
what will the value of n be, if the following statement is executed. The statements are
unrelated from each other during the execution process.

n.

k=n++;//

ANS: n =

, k:

k=++n;//

ANS: n =

,k=
(4 marks)

b. Assuming n = 4, k = 0, determine if the result of the following C++conditional


statement is TRUE or FALSE?
i.
ii.
iii.
iv.
V.

vi.
vii.
viii.
ix.
X.

xi.

IF
IF
IF
IF
IF
IF
IF
IF
IF
IF
IF

(n = 4)
(n == 4)
(n > 3)
(n < 4)
(k = 0)
(n == k)
(n > k)
(n && 4)
(n 1 1 k)
( !n)
( ! k)

//
//
//
//
//
//
//
//
//
//
//

ANS
ANS
ANS
ANS
ANS
ANS
ANS
ANS
ANS
ANS
ANS

(11 marks)
c. What is the output of the following program?
#include <iostream>
using namespace std;
int main()

{
enum color_type {red, orange, yellow, green, blue,violet};
color_type shirt, pants;
shirt = red;
pants = blue;
cout << "Code for shirt" << " " << pants << endl;
return 0;

Ans:
(2 marks)

3
SEL 4263
d. Refer to the following program:
Note: the numbers on the left column are line # (line numbers).
1.
2.
3.
4.
5.
6.
7.
8.

#include <iostream>
using namespace std;
int main()
{
int nNumber[5];
int *pPointer;
nNumber = 15;
pPointer = SnNumber;
9.
cout<<"Address of nNumber is equal to : "<<&nNumber<<endl;
10.
cout<<nNumber<<endl;
11.
cout<< pPointer<<endl;
12.
*pPointer = 25;
13.
cout<<nNumber<<endl;
14.
*pPointer++;
15.
coutnNumberendl;
16.

When the program is executed, line #9 will output the following display:
Address of nNumber is equal to : 0012FB20

Give the program output display resulted from each of the following lines:
Line #10:
Line #11:
Line #15:
Line #16:
(4 marks)

4
SEL 4263

e. Refer to the following program:


Note: the numbers on the left column are line # (ime numbers) which is not part of the
source file.
1.
2.
3.
4.

#include <iostream>
#include <string>
using namespace std;
int main()

5-

6.
7.
8.
9.
10.
11.
12.
13.
14.
15.
16.
17.
18.
19.
20.
21.
22.

struct DataElement
{
Int iVal;
Int hasData;
DataElement()
{
iVal=1234;
hasData=5679;
}
}RealData;
cout << "First element: " << RealData.iVal << endl;
cout << "Second element: " RealData.hasData << endl;
RealData.iVal = 1111;
RealData.hasData = 2222; // True
cout << "First element: " << RealData.iVal << endl;
cout << "Second element: " << RealData.hasData << endl;
}

i.

Name the structure object created in the program. Ans:

ii.

What does the class constructor created in the program.


Ans:

iii.

When the program is executed, give the display resulted from:


Line #16:
Line #21:
(4 marks)

SEL 4263
Question 2.
(Write your answers for this Question 2 in the answer book.)
a. The process of creating a windows program consists of the following steps;
Step 1: Register the Window Class
Step 2: Create the Window
Step 3: Create The Message Loop
Step 4: Create the Window Procedure
Describe briefly the purpose of each step.
(8 marks)

b. The diagram in Figure Q2(b) shows the components involved in handling events
(such as mouse click) and how these events are processed by windows OS and follow
up action by the user application. Explain the components as shown in the diagram
and the processes taking place to manage the events.

(8 marks)

6
SEL 4263

c. The code in Figure Q2(c) shows statement in a windows procedure. Explain the workings of
the code when the user clicks the application window Close button represented by the icon

El located on the upper right of a window.


LRESULT CALLBACK WndProcedure(HWND hWnd, UINT Msg, WPARAM wParam, LPARAM
IParam)
{
switch(Msg)
{
case WMCLOSE:
DestroyWindow(hwnd); // The system sends the WM_DESTROY
Message//
break;
case WMJDESTROY:
PostQuitMessage(WMQUIT); // PostQuitMessage function posts a
//WM_QUIT message to the message queue
break;
default:
return DefWindowProc(hWnd, Msg, wParam, IParam);
}
return 0;
Figure Q2(c).

(9 marks)

7
SEL 4263
Question 3
Given the following program, resource file and header file:
Note: The numbers on the left column are line # (line numbers) which is not part of the
source file.
Content of file Q3.cpp
1.
2.
3.
4.
5.

#include <windows.h>
#include "resource.h"
const char g_szClassName[] = "myWindowClass";
#define IDC_MAIN_EDIT 101
BOOL LoadTextFileToEdit(HWND hEdit, LPCTSTR pszFileName)

6.

7.
8.
9.
10.

HANDLE hFile;
BOOL bSuccess = FALSE;
hFile = CreateFile(pszFileName, GENERIC_READ, FILE_SHARE_READ, NULL,
OPEN_EXISTING, 0, NULL);
if(hFile != INVALID_HANDLE_VALOE)

11.

12.

DWORD dwFileSize;

13.
14.
15.
16.

dwFileSize = GetFileSize(hFile, NULL);


if(dwFileSize != OxFFFFFFFF)
{
LPSTR pszFileText;

17.
18.
19.
20.
21.
22.
23.
24.
25.
26.
27.
28.
29.
30.
31.
32.
33.
34.
35.
36.
37.
38.
39.
40.
41.
42.
43.
44.
45.
46.
47.
48.
49.
50.
51.
52.
53.

pszFileText = (LPSTR)GlobalAlloc(GPTR, dwFileSize + 1);


if(pszFileText != NULL)
(
DWORD dwRead;
if(ReadFile(hFile, pszFileText, dwFileSize, &dwRead, NULL))
{
pszFileText[dwFileSize] = 0; // Add null terminator
if(SetWindowText(hEdit, pszFileText))
bSuccess = TRUE; // It worked!
}
GlobalFree(pszFileText) ;
}
}
CloseHandle(hFile);
}
return bSuccess;
)
BOOL SaveTextFileFromEdit(HWND hEdit, LPCTSTR pszFileName)
{
HANDLE hFile;
BOOL bSuccess = FALSE;
hFile = CreateFile(pszFileName, GENERIC_WRITE, 0, NULL,
CREATE_ALWAYS, FILE_ATTRIBUTE_NORMAL, NULL);
if(hFile != INVALID_HANDLE_VALUE)
{
DWORD dwTextLength;
dwTextLength = GetWindowTextLength(hEdit);
// No need to bother if there's no text.
if(dwTextLength > 0)
{
LPSTR pszText;
DWORD dwBufferSize = dwTextLength + 1;
pszText = (LPSTR)GlobalAlloc(GPTR, dwBufferSize);
if(pszText != NULL)
{
if(GetWindowText(hEdit, pszText, dwBufferSize))
{
DWORD dwWritten;

SEL 4263
54.
55.
56.
57.
58.
59.
60.
61.
62.
63.
64.
65.
66.
67.

if(WriteFile(hFile, pszText, dwTextLength, SdwWritten, NULL))


bSuccess = TRUE;
}
GlobalFree(pszText);
}
}
CloseHandle(hFile);
}
return bSuccess;
}
void Functionl(HWND hwnd)
{
OPENFILENAME ofn;
char szFileName[MAX PATH] =

68.
69.
70.
71.
72.
73.
74.
75.

ZeroMemory(&ofn, sizeof(ofn));
ofn.IStructSize = sizeof(OPENFILENAME);
ofn.hwndOwner = hwnd;
ofn.IpstrFilter = "Text Files (*.txt)\0*.txt\0All Files (*.*)\0*.*\0";
ofn.IpstrFile = szFileName;
ofn.nMaxFile = MAX PATH;
ofn.Flags = OFN_EXPLORER | OFN_FILEMUSTEXIST | OFN_HIDEREADONLY;
ofn.IpstrDefExt = "txt";

76.
77 .
78.
79.
80.
81.
82.
83.
84.
85.
86.
87.
88.
89.
90.
91.
92.
93.
94.
95.
96.
97.
98.
99.
100.
101.
102 .
103.
104 .
105.
106.
107.
108.

if(GetOpenFileName(sofn))
(
HWND hEdit = GetDlgItem(hwnd, IDC_MAIN_EDIT);
LoadTextFileToEdit(hEdit, szFileName);
}

109.
110.
111.
112.
113.
114 .
115.
116.
117.
118.
119.
120.
121.
122 .

}
void Function2(HWND hwnd)
{
OPENFILENAME ofn;
char szFileName[MAX PATH] =
ZeroMemory(sofn, sizeof(ofn));
ofn.IStructSize = sizeof(OPENFILENAME);
ofn.hwndOwner = hwnd;
ofn.IpstrFilter = "Text Files (*.txt)\0*.txt\0All Files (*.*)\0*.*\0";
ofn.IpstrFile = szFileName;
Ofn.nMaxFile = MAX PATH;
ofn.IpstrDefExt = "txt";
ofn.Flags = OFN EXPLORER|OFN PATHMUSTEXIST|OFN HIDEREADONLYIOFN OVERWRITEPROMPT;
if(GetSaveFileName(Sofn))
{
HWND hEdit = GetDlgltem(hwnd, IDC_MAIN_EDIT);
SaveTextFileFromEdit(hEdit, szFileName);
}
}
LRESULT CALLBACK WndProc(HWND hwnd, UINT msg, WPARAM wParam, LPARAM IParam)
<
switch(msg)
{
case WM CREATE:
{
HFONT hfDefault;
HWND hEdit;
hEdit = CreateWindowEx(WS EX CLIENTEDGE, "EDIT",
WS_CHILD j WS_VISIBLE | WS_VSCROLL | WS_HSCROLL | ES_MULTILINE | ES_ADTOVSCROLL | ES_AUTOHSCROLL,
0, 0, 100, 100, hwnd, (HMENU)IDC_MAIN_EDIT,
GetModuleHandle(NULL), NULL);
if(hEdit == NULL)
MessageBox(hwnd, "Could not create edit box.",
"Error", MB_OK|MB ICONERROR);
hfDefault = (HFONT)GetStockObject(DEFAULT_GUI FONT);
SendMessage(hEdit, WM SETFONT, (WPARAM)hfDefault, MAKELPARAM(FALSE, 0) ) ;
}
break;
case WM SIZE:
{
HWND hEdit;
RECT rcClient;
GetClientRect(hwnd, &rcClient);
hEdit = GetDlgltem(hwnd, IDC MAIN EDIT);
SetWindowPos(hEdit, NULL, 0 , 0 , rcClient.right,
rcClient.bottom, SWP NOZORDER);
}

9
SEL 4263
123.
124.
125.
126.
127.
128.
129.
130.
131.
132.
133.
134.
135.
13 6.
137.
138.
139.
140.
141.
142.
143.
144.
145.
146.
147.
148.
149.
150.
151.
152.
153.
154.
155.
156.
157.
158.
159.
160.
161.
162.
163.
164.
165.
166.
167.
168.
169.
170.
171.
172.
173.
174.
175.
176.

break;
case WM_CLOSE:
DestroyWindow(hwnd);
break;
case WM_DESTROY:
PostQuitMessage(0);
break;
case WM_COMMAND:
switch(LOWORD(wParam))
{
case ID_FILE_EXIT:
PostMessage(hwnd, WM_CLOSE, 0, 0);
break;
case ID_FILE_NEW:
SetDlgltemText(hwnd, IDC_MAIN_EDIT, "");
break;
case ID_FILE_OPEN:
Functionl(hwnd);
break;
easelD_FILE_SAVEAS:
Function2(hwnd);
break;
}
break;
default:
return DefWindowProc(hwnd, msg, wParam, IParam);
)
return 0;
)
int WINAPI WinMain(HINSTANCE hlnstance, HINSTANCE hPrevInstance,
LPSTR lpCmdLine, int nCmdShow)
(
WNDCLASSEX wc;
HWND hwnd;
MSG Msg;
wc.cbSize
= sizeof(WNDCLASSEX);
wc.style
= 0;
wc.lpfnWndProc = WndProc;
wc.cbClsExtra = 0;
wc.cbWndExtra = 0;
wc.hlnstance = hlnstance;
wc.hlcon = Loadlcon(NULL, IDI_APPLICATION);
wc.hCursor= LoadCursor(NULL, IDC_ARROW);
wc.hbrBackground = (HBRUSH)(COLOR_WINDOW+l);
wc.IpszMenuName = MAKEINTRESOURCE(IDR_MAINMENU);
wc.IpszClassName = g_szClassName;
wc.hlconSm= Loadlcon(NULL, IDI_APPLICATION);
if(!RegisterClassEx(iwc))
(
MessageBox(NULL, "Result of RegisterClassEx ", "Wow",
MB_ICONEXCLAMATION | MB_OK);
return 0;
}
hwnd = CreateWindowEx(0,g_szClassNarae,"SEL 4263 Final Exam",WS_OVERLAPPEDWINDOW,
CW_USEDEFAULT, CW_USEDEFAULT, 480, 320,NULL, NULL, hlnstance, NULL);

177.
178.
179.
180.
181.
182.
183.
184.
185.
18 6.
187.
188.
189.
190.
191.
192.
193.
194.
195.
196. )

if(hwnd == NULL)
{
MessageBox(NULL, " Result of Window Creation", "Caption",
MB_ICONEXCLAMATION | MB_OK);
return 0;
}
else
{
MessageBox(NULL, "Proceed A ", "Wow",
MB_ICONEXCLAMATION | MB_OK);
}
ShowWindow(hwnd, nCmdShow);
UpdateWindow(hwnd);
while(GetMessage(&Msg, NULL, 0, 0) > 0)
{
TranslateMessage(&Msg);
DispatchMessage(&Msg);
}
return Msg.wParam;

10
SEL 4263
Content of file Q3.rc
1.

#include "resource.h"

2.
3.

#include "windows.h"
ttinclude "resource.h"

4 .
5.
6.
7.
8.
9.
10.
11.
12.
13.
14 .

IDR MAINMENU MENU DISCARDABLE


BEGIN
POPUP "&File"
BEGIN
MENUITEM "&New",
MENUITEM "&Open...
MENUITEM "Save &As...
MENUITEM SEPARATOR
MENUITEM "E&xit",
END
END

ID FILE NEW
ID FILE OPEN
ID_FILE_SAVEAS
ID_FILE_EXIT

Content of file resource.h


#define
#define
#define
#define
#define

IDR MAINMENU
ID FILE EXIT
ID FILE OPEN
ID FILE SAVEAS
ID FILE NEW

102
40001
40002
40003
40004

(Write your answers in the respective blank space given in the Question)
Identifiy in the program, which code (give line numbers) that is requred to implement the
following requirement:
a.

Give the string of the caption of the window.


(1 mark)
Ans:

b.

What will be the displayed if the process of registering the class is successful?
(4 marks)

Ans: Draw the display here

11
SEL 4263
Give the declaration which registers the address of the callback function which will be
called in the statement at line #194, DispatchMessage (&Msg) ; when executed.
(1 mark)
Ans: ________________________________________________________
What is the value of the constant that identifies the menu script for the main window
created by the program.
(1 mark)
Ans: ________________________________________________________
Identify the statement which registers the menu script to be created in the main of the
program.
(1 mark)
Ans: _____ __________________________________________________
Identify the statement which passes a message from the message queue.
(1 mark)
Ans:
Which statement when executed generates WM_CREATE to the message queue?
(1 mark)
Ans: ________________________________________________________
Which event will cause statement at line #76 if (GetOpenFileName (&ofn) ) > to
be executed?
(1 mark)
Ans: ________________________________________________________
Which event sends the WM_CLOSE to the message queue?
(1 mark)
Ans:

12
SEL 4263
j. Draw the main user interface created by the program.
(8 marks)

Ans: Draw the user interface in the box below.

k. Referring to the resource file Q3.rc, draw the dropdown menu created in the program.
(5 marks)
Ans: Draw the dropdown menu in the box below.

13
SEL 4263
PART B

Question 4
(Write your answers for this Question 4 in the answer book.)
A callback function using Child Window Control, which on execution from a properly
built WinMain program, will initially create the user interface shown in Figure Q4(a).
*

Area Calculator

Enter radius

Integer1. Decimal

(Area of a Circle

Figure Q4(a).
The specifications of the 6 controls shown in Figure Q4(a) is given in Figure Q4(b)):

Text displayed
on execution

Type

Position
on Client
Area
(X,Y)

Size
(Width,
height)

ID name

Value

Calculate

Button
Control
Edit Control

10, 10

100,

IDC_CALC

101

Enter

105
120, 10

100, 25

IDC_RADIUS

102

120, 65

100, 25

IDC_AREA

103

220, 65

10, 25

IDC_AREAPT

104

230, 65

100, 25

IDC_AREADC

105

10, 120

320, 25

IDC_AREAS t r

106

radius
Integer
. (decimal
point)
Decimal
Area of a
Circle

Static Text
Control
Static Text
Control
Static Text
Control
Static Text
Control

Figure Q4(b).

14
SEL 4263
A user can enter a decimal entry on the Enter radius and overwriting the Enter
radius text string. The program will accept value of RADIUS to be an integer or

decimal point number but will be received as a string number (may be with decimal
point). Assume that only valid decimal number will be entered into the Enter
radius EDIT Control.

If the user presses the Calculate BUTTON control, the integer portion of the
Area of a Circle will be displayed on the Integer Static Text control, and

the remaining decimal portion will be displayed on the Decimal Static Text control.
Figure Q4(c) shows an example where a user enters 21.3 in the Enter radius
EDIT Control and then presses the Calculate BUTTON control, for the answer of
1425.493, the number 1425 and 493 are displayed on the Integer Static Text
control and Decimal Static Text control, respectively, which is defined in Figure
Q4(a).
*

sap

Area Calculator

21.3
Calculate

1425 493

ire a of a Circle

Figure Q4(c).

i-S i

15
SEL 4263
Based on the above requirement and specifications, the following is partial codes of callback
function:
// This code is define
#define IDC_CALC
#define IDC_RADIUS
#define IDC AREA
//More Codes Here

elsewhere in the program


101
102

103

LRESULT CALLBACK WndProc(HWND hwnd, UINT msg, WPARAM wParam, LPARAM IParam)

{
switch(msg)

{
case WM_CREATE:

{
HFONT hfDefault;
HWND hRadius,hCALC, hArea, hAreaPT, hAreaDC hAreaStr;
hAreaStr=CreateWindowEx(WS_EX_CLIENTEDGE|SS_CENTERIMAGE,
"STATIC", "Area of a Circle",
WS_CHILD | WS_VISIBLE |WS_BORDER,
10, 120, 320, 25, hwnd, (HMENU)IDC_AREAStr,
GetModuleHandle(NULL) , NULL);
/ /More Codes Here

}
break;
case WM_CLOSE:
// More Codes Here

break;
case WM_DESTROY:
// More Codes Here

break;
default:
// More Codes Here

case WM_COMMAND:
switch(LOWORD(wParam))

{
case 101:

{
char szRadius [20];
char buffer[65];
int bSuccess = GetDlgltemText(hwnd, IDC_RADIUS, szRadius, 20);
double Radius = atof(szRadius);
double Area=3.142*Radius*Radius;
SetDlgltemlnt(hwnd, IDC_AREA, (int)Area,FALSE) ;
// More Codes Here

}
break;

return 0;

16
SEL 4263
a. Show the definition of the three more ID names.
(3 marks)
b. In which structure should the codes for the following controls be located (written) in the
program?
Calculate

Button
Control

Enter

Edit Control

radius
Integer
(decimal
point)
Decimal
Area of a
Circle

Static Text
Control
Static Text
Control
Static Text
Control
Static Text
Control
(2 marks)

c. Show the codes which will create the following controls:

Calculate

Button
Control

Enter

Edit Control

radius
Integer

Static Text
Control
(12 marks)

d. Show the code(s) which will display the decimal value of calculated Area of Circle.
(5 marks)
e. In which structure and which position should the code(s) that will display the decimal
value of Area of a Circle be located (written) in the program?
(3 marks)

17
SEL 4263

Question 5.
(Write your answers for this Question 5 in the answer book.)

a.

In windows programming, what is Graphics Device Interface (GDI) and what is the
advantage of using GDI to the programmer.
(4 marks)

Referring to the following codes shown in Figure Q5, which are within a callback procedure
are used to display two straight lines representing the x and y axes of a graph. The following
questions are based on the codes.
LRESULT CALLBACK WndProc( HWND hwnd, UINT msg, WPARAM wParam, LPARAM IParam)

HDC hdc;
PAINTSTRUCT ps;
RECT rect;
static int w=250, h=150;
int x, y;
int i;
switch(msg)

case WM_SIZE:
GetClientRect(hwnd, Snect);
w = LOWORD(lParam) + 1;
h = HIWORD(lParam) + 1;
InvalidateRect(hwnd, &rect, TRUE);
break;
case WM_PAINT:
hdc = BeginPaint(hwnd, &ps);
//Draw the y-axis
for (i = h/12; i<(ll*h/12); i++)

y = i; //set the y value between the maximum to th minimum


x = w/12;
SetPixel(hdc, x, y, RGB(0, 0, 0)); //black

//Draw the x-axis


for (i = w/12; i<(ll*w/12); i++)

x = i;
y = h/2;
SetPixel(hdc, x, y, RGB(0, 0, 0)); //black

//Put test
SelectObject (hdc, GetStockObject (SYSTEM_FIXED_FONT)) ;
SetBkMode (hdc, TRANSPARENT) ;
TextOut (hdc,w/24,h/12,y",l);
TextOut (hdc,ll*w/12,h/2,"x",l);
EndPaint(hwnd, &ps);
break;
case WM_DE5TR0Y:
PostQuitMessage(0);
break;

return DefWindowProc(hwnd, msg, wParam, IParam);

}
Figure Q5.
b. What event causes the codes in the case WM_SIZE be executed?
(2 marks)
c. What event causes the codes in the case WM PAINT be executed?

18
SEL 4263
(2 marks)
What is the use of the BeginPaint function?
(2 marks)
What is the use of the SelectObject function?
(2 marks)
Explain all the parameters used in the SetPixel function
(2 marks)
The output of the program is as shown below. Every time the windows is resized using
the mouse, the axes x and y would also be resized accordingly in proportion to the new
size of the window. Explain which codes in the procedure are responsible for this?

Modify (by adding additional codes) so that a straight line is drawn from point (w/12,
3h/4) to point (1 lw/12, h/12). The line drawn must be green in colour. Also the text, y =
mx + c is also shown on the window. The text is output at point (w/2,h/12). The final
output should be as shown below. The value of w and h is as obtained from the original
program. All calculations should be shown clearly.
(3 marks)

(8 marks)

CONFIDENTIAL

UTM
UNIVERSITI TEKNOLOGI MALAYSIA

FINAL EXAMINATION SEMESTER II


SESSION 2010/2011

COURSE CODE

SET 3583

COURSE NAME

DIGITAL COMMUNICATION SYSTEM

LECTURERS

DR. NURUL MUAZZAH BT ABDUL LATIFF

PROGRAMME

SET

SECTION

01

TIME

2 HOURS 30 MINUTES

DATE

06 MAY 2011

INSTRUCTION TO CANDIDATE

ANSWER FOUR (4) QUESTIONS ONLY.

THIS EXAMINATION BOOKLET CONSISTS OF 7 PAGES INCLUDING THE FRONT COVER

-2-

SET 3583
Q.l (a) Nyquist filter is often used to describe the general class of filtering and pulseshaping that satisfies zero 1S1 at sampling points. With the aid of diagram, explain
about ideal Nyquist pulse as a method to control ISI.

(b)

(6 marks)

The receiving filter in digital communication can be implemented with either a


matched filter or a correlator. Show that the correlator can be realized using
matched filter.

(c)

(7 marks)

An analog signal is PCM formatted and transmitted using binary waveforms over
a bandlimited channel of 100 kHz. Assume that 32 quantization levels are used
and the signal is transmitted using 8-ary PAM waveforms. The overall equivalent
transfer function is of raised cosine type with roll-off, r = 0.6.
i.

Find the maximum bit rate that can be used by this system without
introducing ISI.

ii.

(6 marks)

Find the maximum bandwidth of the original analog signal that can be
accommodated with these parameters.

(6 marks)

-3SET 3583
Q.2 (a) What is the advantage of using basis function in matched filters or correlators?
(2 marks)

(b)

Given the maximum likelihood decision rule as follows:


i Vector r lies inside region Z, if
N

|
E k , is maximum for all k = i \

7=1

where E k is the energy of s k (t ).


Based on this decision rule, draw the block diagram maximum likelihood detector
for M decision regions.

(c)

(7 marks)

Figure 1 below displays the waveforms of three signals s \ ( t ) , $2(0 and $3(0i.

Using the Gram-Schmidt Orthogonalization procedure, find a set of


orthonormal basis functions to represent these signals.

(10 marks)

ii. Express each of these signals in terms of the set of basis functions found in
part c (i).

(6 marks)

(0 (ii) (iii)

Figure Q2

-4SET 3583

Q.3 (a) Modulation is the process of converting a baseband signal to a passband


counterpart by varying some characteristics of a carrier in accordance with the
baseband signal. Explain why modulation is needed.

(b)

(3 marks)

i. Sketch the signal space diagram representing orthogonal BFSK signal.


(4 marks)
ii.

Based on the signal space diagram in b(i), prove that probability of bit
error for BFSK is given by,

Pb=Q

(6 marks)

v u y

(c)

As an engineer, you need to choose a modulation scheme for a system which its
main

performance

criterion

is

bit-error

probability.

Given

the

following

modulation schemes, and assuming that a Gray code is used for the MPSK
symbol-to-bit assignment, which of the following will you select for transmission
over an AWGN channel? Show your computations.

coherent 8-ary orthogonal FSK with =8 dB


N0

coherent 8-ary orthogonal PSK with =13 dB


No
(12 marks)

-5SET 3583

Q.4 a) Define the word Hamming distance used in error control coding. (2 marks)

b)

It is required to design a (6,3) systematic linear block code. The three parity check
bits are decided to be p { - w, m 3, p 2 = m ] m 2 @ m i , and p 3 = m l m 2 .
i.

ii.

Write down the generator matrix, G.

Fill up the code words in Table Q4.

Data

(6 marks)

(8 marks)

'able Q4
Codeword

000
001
010
Oil
100
101
110
111

iii.

Suppose the received word is 010101. Determine the estimated transmitted


data bits.

(9 marks)

-6SET 3583

Q.5 (a) Differentiate how multiple access is achieved in DS-CDMA and FH-CDMA.

(6 marks)

(b)

Spread spectrum (SS) techniques can provide impressive error-performance


benefits against interfering signals. One might therefore think that such SS
techniques might provide similar benefits against AWGN. Explain why this is not
possible.

(c)

(4 marks)

Consider a FH/MFSK system. Let the pseudonoise (PN) generator be defined by a


20-stage linear feedback shift register with a maximal length sequence. Each state
of the register dictates the new center frequency within the hopping band. The
minimum step size between centre frequencies (hop to hop) is 200 Hz. The
register clock rate is 2 kHz. Assume that 8-ary FSK modulation is used and the
data rate is 1.2 kbits/s.

i.

What is the hopping bandwidth?

(3 marks)

ii.

What is the chip rate?

(3 marks)

iii.

How many chips are there in each data symbol?

(5 marks)

iv.

What is the processing gain in dB?

(4 marks)

-7SET 3583

Table 1: Values of Q(x) for 0 x s 9


X

0.00
0.05
0.10
0.15
0.20
0.25
0.30
0.35
0.40
0.45
0.50
0.55
0.60
0.65
0.70
0.75
0.80
0.85
0.90
0.95

1.00
1.05
1.10
1.15
1.20
1.25
1.30
1.35
1.40
1.45
1.50
1.55
1.60
1.65
1.70
1.75
1.80
1.85
1.90
1.95
2.00
2.05
2.10
2.15
2.20
2.25

Q(x)
0.5
0.48006
0.46017
0.44038
0.42074
0.40129
0.38209
0.36317
0.34458
0.32636
0.30854
0.29116
0.27425
0.25785
0.24196
0.22663
0.21186
0.19766
0.18406
0.17106
0.15866
0.14686
0.13567
0.12507
0.11507
0.10565
0.0968
0.088508
0.080757
0.073529
0.066807
0.060571
0.054799
0.04947.1
0.044565
0.040059
0.03593
0.032157
0.028717
0.025588
0.02275
0.020182
0.017864
0.015778
0.013903
0.012224

2.30
2.35
2.40
2.45
2.50
2.55
2.60
2.65
2.70
2.75
2.80
2.85
2.90
2.95
3.00
3.05
3.10
3.15
3.20
3.25
3,30
3.35
3.40
3.45
3.50
3.55
3.60
3.65
3.70
3.75
3.80
3.85
3.90
3.95
4.00
4,05
4.30
4,15
4,20
4,25
4.30
4.35
4,40
4.45
4.50

Q(x)
0.01.0724
0.0093867
0.0081975
0.0071428
0.0062097
0.0053861
0.0046612
0.0040246
0.003467
0.0029798
0.0025551
0.002186
0.0018658
0.0015889
0.0013499
0.0011442
0.0009676
0.00081635
0.00068714
0.00057703
0.00048342
0.00040406
0.00033693
0.00028029
0.00023263
0.00019262
0.00015911
0.00013112
0.0001078
8.8417 xlO-5
7.2348 xlO"5
5.9059 xl0~5
4.8096x10-
3.9076 xlO"5
3.1671x10-
2.5609x10"
2.0658x10-
1.6624x10-
1.3346 xlO"5
1.0689x10-
8.5399 xlO"6
6.8069 xlO"6
5.4125xl0-(i
4,2935 xl0~6
3.3977x1.0-

Q(x)

QUO

4.55
4.60
4.65
4.70
4,75
4.80
4.85
4.90
4.95
5.00
5.05
5.10
5.15
5.20
5.25
5.30
5.35
5.40
5.45
5.50
5.55
5.60
5.65
5.70
5.75
5.80
5.85
5.90
5.95
6.00
6.05
6.10
6.15
6.20
6.25
6.30
6.35
6.40
6.45
6.50
6.55
6.60
6.65
6.70
6.75

2.6823x1 O'6
2.1125xl0-6
1.6597x10-
1.3008xl0-6
1.0171xl0-6
7.9333 xlO-7
6.1731 xlO-7
4.7918x10-"
3.7107x10-"
2.8665 xlO-7
2.2091 xlO"7
1.6983xl0-7
1.3024 xlO"7
9.9644 xlO-8
7.605xl0~8
5.7901x10-*
4.3977x10-
3.332 xlO-8
2.5185 xlO"8
1.899xl0-8
1.4283x10-
1.0718x10-
8.0224xl0-9
5.9904 xlO"9
4,4622 xlO"9
3.3157 xlO"9
2.4579x10-
1,81.75 xlO-9
1.3407xl0-9
9.8659X10-10
7.2423x10"10
5.3034xl0-)O
3.8741x10-10
2.8232 x 1.0-10
2.0523x 10-10
1.4882xlO-10
1.0766xl0-10
7.7688X10"11
5.5925 xlO"11
4,016 xlO"11
2.8769X10-11
2.0558x10-
1.4655 x 10-1
1.0421 xlO11
7.3923x10-12

6.80
6.85
6.90
6.95
7.00
7.05
7.10
7.15
7.20
7.25
7.30
7.35
7.40
7.45
7.50
7.55
7.60
7.65
7.70
7.75
7.80
7.85
7.90
7.95
8.00
8.05
8.10
8.15
8.20
8.25
8.30
8.35
8.40
8.45
8.50
8.55
8.60
8.65
8.70
8.75
8.80
8.85
8.90
8.95
9.00

5,23.1x10-12
3.6925xl0~12
2.6001 xlO"12
1.8264X10"12
1.2798xl0-12
8.9459xl0-13
6.237Sxl0-13
4.3389 xlO"13
3.0106X10"13
2.0839x 10-13
1.4388xl0-13
9.9103X10-14
6.8092xl0->4
4.667xl0"14
3.1909X10"14
2.1763xl014
1.4807xl0"14
1.0049x 10-14
6.8033X10-15
4.5946X 10-15
3.0954X10-1
2.0802X10-1
1.3945 xlO-15
9.3256xl0~16
6.221 xlO-16
4.1397X10"16
2.748 xl.0-16
1.8196xl0"16
1.2019x 10-16
7.9197xl0-17
5.2056x 10-17
3.4131 xlO-17
2.2324X10"17
1,4565x10-17
9.4795xlO-18
6.1544X10"18
3.9858xl0-18
2.575xlO-18
1.6594xl0-18
1.0668 xlO-18
6.8408xl0-19
4.376 xlO'19
2.7923xl0~19
1.7774xl0-19
1.1286X.10-19

CONFIDENTIAL

UTM

UNIVERSITI TEKNOLOGI MALAYSIA

FINAL EXAMINATION SEMESTER II


SESSION 2010/2011

COURSE CODE

SEL 4533

COURSE NAME

MICROCONTROLLERS

LECTURERS

DR. YEONG CHE FAI

PROGRAMME

SEC / SEI / SEL / SEM / SEP

SECTION

01

TIME

2 HOURS 30 MINUTES

DATE

12 MAY 2011

INSTRUCTION TO CANDIDATE

ANSWER ALL QUESTIONS IN SECTION A IN


THIS PAPER IN THE AREA PROVIDED.
ANSWER ANY TWO (2) QUESTIONS FROM
SECTION B.
SPLIT THE QUESTIONS SECTION A AND TIED
THEM TOGETHER WITH ANSWER SHEET
FOR SECTION B.

THIS EXAMINATION BOOKLET CONSISTS OF 19 PAGES INCLUDING THE FRONT COVER

2
-SEL 4533-

SECTION A

1.

50 MARKS

The following program is written to create a continuous square wave output at


PORTB bit 7 (PB7) using a subroutine DELAY. The program first SETS bit PB7 and
calls subroutine DELAY. Then, the program CLEARS bit PB7 and calls subroutine
DELAY. The program will continue looping. Hence, a continuous square wave is
generated. There are two errors in this program. Circle and explain the errors.
[5 MARKS]

ANSWER:

PORTB

LAGI

o
00

2.

LDAA
DECA
BNE
RTS

$1004
$B600
#PORTB
0,Y $70
DELAY
o'

DELAY
AGAIN

EQU
ORG
LDY
BSET
JSR
BCLR
JSR
BSR

DELAY
LAGI
#100
AGAIN

The following subroutine is written for M68HC11 microcontroller using an 8 MHz


oscillator. Calculate the value of Count that can generate a delay of 0.4 milliseconds.
Calculate the total bytes used by this subroutine.
[5 MARKS]

DELAY
AGAIN

ANSWER:

N
Total bytes

LDAA
DECA
BNE
RTS

# Count
AGAIN

3
-SEL 4533-

3.

The following flow chart is an algorithm to identify the smallest byte stored in
memory between address $5000 and $5020. Write a program to perform this
algorithm. The program will first load accumulator B (ACCB) with decimal 255.
Then it will load one byte from memory content pointed by X (starting from address
$5000) to accumulator A (ACCA). Next, the value in ACCA will be compared to
ACCB. If the value ACCA is less than ACCB, then ACCB will be overwritten with
new lower value from ACCA. If ACCA is larger than ACCB, then nothing changes.
The process continues until the loop reach memory address $5020.
[5 MARKS]
ANSWER:

B = A

X = X+ 1

4. State the values in memory $55 (in hexadecimal) after every instruction is executed.
The initial value at address $55 is $0. All instructions are executed continuously.
[5 MARKS]
ANSWER:

LDAB
STAB
ASR
BCLR

#55
$55
$55
$55 $02

($55) = $ 0
($55) =
($55) =
($55) =

4
-SEL 4533-

5. The following list file is generated by THRSIM compiler. State the content in memory
address $00FC - SOOFF and Program Counter (PC) right after instruction RTS is
executed. Assume all initial values are $FF. Write your answer in the table below in
hexadecimal format.
[5 MARKS]

Address

Machine code

Assembly

B600
B603
B606
B609
B60B
B60C
B60E
B60F

8E 00 FF
CC 02 00
BD B6 0B
20 F8
36
C6 03
3D
39

ORG SB600
LDS #$00FF
LDD #512
JSR PROCESS
BRA AGAIN
PSHA
LDAB #3
MUL
RTS

AGAIN

PROCESS

ANSWER:
Address/Register
Content

SOOFF

$00FE

S00FD

S00FC

PC

6. The circuit below is for an Analog to Digital Converter using M68HC11. Calculate
the contents of ADR1 and ADR2 after the A/D conversion is completed.
[5 MARKS]

10KQ

68HC11

, AAA

PE.O
iokqJ

PE.l

Vrh

---- AAAH +5 V

1KQ

Vrl

1
ANSWER:
ADR1 = S

ADR2 = $

5
-SEL 4533-

7. Explain the main function of the following registers for Analog-to-Digital Converter
in M68HC11.
[6 MARKS]
ANSWER:
Register
ADCTL

Main function

OPTION

ADR1

The following is part of a program for Serial Communication Interface (SCI) for
M68HC11. Explain the function of the instructions which are in bold.
[4 MARKS]
SCCR1
SCCR2

ANSWER:

EQU
$2C
EQU
$2D
ORG SB600
LDY #$1000
BCLR SCCR1, Y $10
BSET SCCR2, Y $C0

6
-SEL 4533-

9. Describe the function of BAUD register ($102B) for Serial Communication Interface
(SCI) M68HC11. Explain why the integrated circuit (IC) MAX232 is often used
during the SCI communication?
[5 MARKS]
ANSWER:

10. For the circuit below, write a program to switch off all the LEDs.
[5 MARKS]
ANSWER:

7
-SEL 4533-

SECTION B

50 MARKS

Ql
+6V
Vrh

M68HC11
Thermometer

FAN

-------

PEO

Va --------

PEI
PE2

Vb %------

PBO

V
Vrl

Figure 1: Intelligent fan system


An intelligent fan system (Figure 1) is created using M68HC11 microcontroller. The
system will switch ON the fan if temperature is above Va, upper limit voltage of 30C
and switch OFF the fan if temperature is below Vb, lower limit voltage reference of
25 C. A thermometer is used to measure the temperature and gives the output in the form
of voltage, which is directly proportional to the temperature degree Celsius. The
relationship between output voltage and temperature is shown in Table Ql:

Table Ql: Relation between voltages measured by thermometer


with its temperature in degree Celsius.______________________
Temperature
Voltage measured
by thermometer (Volt)
(Celsius)
0 Volt
0C
6 Volt
100 C

The thermometer is connected to PEO. The upper limit voltage reference is determined by
a constant voltage, Va connected to PEI and lower limit voltage reference is determined
by constant voltage, Vb connected to PE2. The fan connection is active high.

a. What is the definition of Resolution for Analog-to-Digital Converter in M68HC11?


Calculate the Resolution for the A/D converter.
[2 MARKS]

b. Calculate the required voltage constants for Va, upper limit voltage reference and Vb,
lower limit voltage reference.
[4 MARKS]

c. Part of the program has the following instructions:


LDX
BSET
JSR
LDAA
STAA

#$1000
OPTION, X %11000000
DELAY 100US
#%00110000
ADCTL, X

Explain what are the effects of these instructions on the A/D converter in M68HC11?
[6 MARKS]

d. Write a full program in assembly language for the intelligent fan system as described
above. You can use part of the program in (c) in your answer. Assume DELAY100US
is provided to delay 100/zs and you do not have to create this subroutine.
[13 MARKS]

9
-SEL 4533-

Q2.

A pole climbing robot (Figure Q2) is built using microcontroller M68HC11 as shown in
figure below. The robot has one wheel driven by a DC Motor. The chip L293B (datasheet
in appendix) is used as interface between microcontroller and the DC Motor. The two
wires of the DC motor are connected to the pins PBO and PB1 of the microcontroller
respectively. The robot will climb up if PB0=0 and PB1=1 and climb down if PB0=1 and
PB1=0. The robot will stop when PB0=PB1. Two limit switches are connected to PCO
and PCI to detect if the robot reaches the ceiling (PCO) or floor (PCI).

DC Motor with wheel

Limit
switches

Robot will climb up when the wheel is


turning counter-clockwise while it will
climb down when the wheel is turning
clockwise. This figure shows a robot is
climbing up the pole.

Floor

Figure Q2: Pole climbing robot

10
-SEL 4533-

a. Explain briefly the differences between DC Motor and Stepper motor.


[4 MARKS]

b. Explain why motors cannot be connected directly to M68HC11?


[2 MARKS]

c.

Draw a complete working circuit diagram for this robot using M68HC11. Note the
interfacing circuit between DC motor and limit switches (normally open) to
M68HC11 using bootstrap mode. A motor driver L293B is used for interfacing the
DC motor and connected to pins PBO and PB1 of the M68HC11. Datasheet of the
motor driver L293B and M68HC11 pin out are available as appendix for your
reference. PCO and PCI are connected to the limit switches. Label the circuit
accordingly.
[9 MARKS]

d.

Write a program to control the robot for climbing up and down continuously. The
robot initially will climb up the pole until it reaches the ceiling. Once the limit switch
senses the ceiling, the robot will reverse to climb down the pole. And once it reaches
the floor, the robot will climb up again. The process continues. Write the program
according to the hardware connectivity of Part (c) where PBO and PB1 are to control
the DC Motor and PCO and PCI are connected to the limit switches.
[10 MARKS]

11
-SEL 4533-

Q3.

One prototype decorative light system is being developed (Figure Q3). This prototype is
used to display two selectable blinking patterns for 4 LEDs using microcontroller
M68HC11. A user can use a remote control to select the pattern wirelessly via RF
module.
PORTB:

Remote control

Prototype decorative light system

Figure Q3: Prototype decorative light system


The operation of the system is as follows:
i.
Remote control: When a button is pressed, two bytes are transmitted from the
RF. The first byte is followed by 1 or 2 depending on which button is
selected.
ii.
Prototype decorative light system: An RF module is connected to
Communication Serial Communication (SCI) of microcontroller. 4 active high
LEDs are connected to PORTB. Each LEDs current is limited with a 330
Ohm resistor.
iii.
In the program, two patterns of LED blinking with 100ms delay in between
blinks are designed as shown in Table Q2. Pattern 1 starts with switching ON
all the LEDs. Then, a delay of 100ms is executed before switching OFF all the
LEDs. This process continues. The similar program is written for Pattern 2.
Table Q2: Four LED blinking patterns
PORTB
Pattern 1
Pattern 2

iv.

v.

PB3 PB2 PB1 PBO


1
1
1
1
0
0
0001
0 0 10
0 10 0
10 0 0

The system always reads from the serial communication of the


microcontroller. If data @1 is received, then Pattern 1 is selected to blink the
-4 LEDs. If data @2 is selected, then pattern 2 is selected.
This process continues looping.

12
-SEL 4533-

Assuming an 8Mhz oscillator is being used, please answer the questions below:

a. Draw and complete the missing parts of the output connection circuit at PORTB of the
microcontroller only comprising the LEDs. All the LEDs are active high.
[4 MARKS]

b. State the three initialization steps in programming needed before SCI interface can be
used.
[6 MARKS]

c. Write two subroutines PAT1 and PAT2 for the two different blinking LEDs. Assume
subroutine DELAY is already created to delay 100ms.
[6 MARKS]

d. Write a full program for the system to read data from serial communication and run the
running light patterns according to the users selection. Subroutine READ1 is already
created as follow:
READ1 BRCLR SCSR,X $20 READ1
LDAA SCDR,X
RTS
The program should be able to identity users selection and uses subroutine PAT1, PAT2
and DELAY as described in (c). You do not have to re-write all these subroutines in your
answer.
[9 MARKS]

Source
Form(s)

Boolean Expression

Operation

Addressing
Mode for
Operand

Machine Coding
(Hexadecimal)
Opcode

Operand(s)

w
4>
*
01

I Cycle I

Table 10-1 MC68HC11A8 Instructions, Addressing Modes, and Execution Times


(Sheet 1 of 6)
Cycle
Condition Codes
by
Cycle* 8XHINZVC

ABA

Add Accumulators

A + B -A

INH

1B

2-1

ABX

AddBtoX

IX + 00:B -IX

INH

3A

2-2

ABY

AddBtoY

IY + 00:B - IY

INH

16 3A

2-4

ADCA

Add with Cany to A

A + M +C -+A

AIMM
A DIR
A EXT
A IND.X
A IND.Y

89 ii
99 dd
BS hh 0
AS
18A9

ff
ff

3
2
3

2
3
4
4
5

1-1
4-1
5-2
6-2
7-2

BIMM
B DIR
BEXT
B IND.X
B IND.Y

C9
DS
FS
ES
18 E9

ii
dd

2
2
3
2
3

2
3
4
4
5

5-1
4-1
5-2
6-2
7-2

2
2
3
2

2
3
4
4
5

3-1
4-1
5-2
6-2
7-2

--Mill

2
2
3
2
3

2
3
4
4
5

3-1
4-1
5-2
6-2
7-2

--Mill

3
2
3
2
3

4
5
6
6
7

3-3
4-7
5-10
6-10
7-8

-mi

2
2
3
2

2
3
4
4
5

3-1
4-1
5-2
6-2
7-2

------ 110-

AOCB(opr) Add with Carry to B

8 + M +C -* B

hh a

ff

ff

ADDA (opr) Add Memory to A

A + M - A

AIMM
A DIR
A EXT
A IND.X
A IND.Y

6B
9B
BB
AB
18 AB

ii
dd
hh n

ADDB (opr) Add Memory to B

B + M -B

BIMM
B DIR
BEXT
B IND.X
B IND.Y

CB
DB
FB
E8
18 EB

ii
dd

IMM
DIR
EXT
IND.X
IND.Y

C3
D2
F3
E2
18 E3

A-M -A

AIMM
A OIR
A EXT
A IND.X
A IND.Y

B*M B

BIMM
B DIR
BEXT
B IND.X
B IND.Y

84
94
BA
M
18 A4
04
D4
F4
E4
18 E4

ADDD (opr) Add 16-Bit to D

D + M:M+ 1 -* D

AN DA (opr) AND A with Memory

ANDB (opr) AND B with Memory

ASL (opr)

____

Arithmetic Shift Left

I W I I II! i I l-o
C b7 bO

EXT
IND.X
IND,Y
B INH

ASLD

___

Arithmetic Shift Left Double

ff
ff

hh II

ff
ff

jj kk
dd
hh II

ff

ff
ii
dd
hh II
ff
ff

Min

--i-iin

ii
dd
hh II
ff
ff

2
3
4
4
5

3-1
4-1
5-2
6-2
7-2

------ 110-

2
3
2
3

78 hh II
68 ff
18 68 ff
46
58

3
2
3
1
1

6
6
/
7
2

SB
6-3
7-3
2-1
2-1

----nn

2-2

----tin

3
2
3
1
1

6
6

---nil

7
2

5-8
6-3
7-3
2-1
2-1

8-1

3
3
4

6
7
8

4-10
6-13
7-10

INH

05

EXT
IND.X
IND.Y

77 hh II
67

CKCD - - -LDo
C blS bO
ASR (opr)

___ _

Arithmetic Shift Right

H 1 11 1 11 1 1*11
b? bO C

ASRB
BCC (ref)

B INH
Branch if Carry Clear

?C = 0

BCLR (opr) Clear Bit(s)


(msk)

REL
DIR
JND.X
IND.Y

ff

47
57
24 rr
15 dd mm
1D ff mm
181D ff mm

?C = 1

REL

8-1

Branch if = Zero

?Z= 1

REL

25 rr
27 rr

BEO (rel)

8-1

BGE(rel)

Branch if a Zero

? N V * 0

REL

2C rr

8-1

BGT (rel)

Branch if > Zero

?Z + (NV) = 0

REL

2E rr

8-1

BHI (rel)

Branch if Higher

?c+z=o

REL

22 rr

8-1

BHS (rel)

Branch if Higher or Same

?C =0

REL

24 rr

B-1

BCS (rel)

Branch if Carry Set

no.

*Cyde-by-cyde number provides a reference to Tables 10-2 through 10-8 which detail cycle-by-cycle operation.
Example: Table 10-1 Cyde-by-Cycle column reference number 2-4 equalsTable 10-2 line item 2-4.

MOTOROLA

10-6

CPU, ADDRESSING MODES, AND INSTRUCTION SET

MC68HC11A8
TECHNICAL DATA

Table 10-1 MC68HC11A8 Instructions, Addressing Modes, and Execution Times


(Sheet 2 of 6)
Boolean Expression

Operation

Machine Coding
(Hexadecimal)
Opcode Operand(s)

2
3
4
4
5

3-1
4*1
5-2
6-2
7-2

110-

3-1
4-1
5-2
6-2
7-2

------ 11 0-

8-1

85
95
B5
A5
18A5

ii
dd
hh II
ff
ff

B*M

BIMM
B DIR
BEXT
B IND.X
B IND,Y

C5
D5
F5
E5
18 E5

ii
dd
hh n
ff
ff

2
3
4
4
5

Branch if S Zero

?Z + (NV) = 1

REL

2F rr

Branch if Lower

?C*1

REL

?C+Z=1

REL

25 rr
23 n

Branch If < Zero

?NV31

REL

2D rr

BM1 (rel)
BNE (rel)

Branch if Minus
Branch if Not = Zero

?N=1
?Z = 0

REL
REL

2B rr
26 rr

BPL (rel)

Branch if Plus

?N = 0

REL

BRA (rel)

Branch Always

71=1

REL

? Mmm = 0

OIR
IND.X
IND,Y

BITS (opr)

8LE (re))
BLO (rel)
BLS (rel)
BLT (rel)

Bit(s) Test A with Memory

Bit(s) Test B with Memory

Branch if Lower or Same

BRCLR(opr) Branch if Bit(s) Clear


(msk)
(rel)
BRN (rel)
Branch Never

Cycle
Condition Codes
by
Cycle* SXHINZVC

i.
t
to

AIMM
A DIR
A EXT
A IND.X
A IND.Y

BITA (opr)

A*M

Addressing
Mode for
Operand

Cycle

Source
Form(s)

3
2
3

8-1

8-1
3

8-1

3
3

8-1
8-1

2A rr

8-1

20 rr

8-1

4
4
5

6
U

4-11
6-14
7-11

13 dd mm rr
1F ff mm rr
18 IF ff mm rr

?1=0

REL

8RSET(opr) Branch if Bft(s) Set


(msk)
(rel)

? (M) mm * 0

DIR
IND.X
IND.Y

12 dd mm rr
1E ff mm rr
18 1E ff mm rr

4
4
b

6
/
8

8-1
4-11
6-14 711

BSET(opr)
(msk)

Set Bit(s)

M + mm -+ M

DIR
IND.X
IN D,Y

14 dd mm
1C ff mm
10 1C ff mm

3
3
4

6
7
8

4-10
6-13
7-10

8-2

BSR (rel)

21 rr

Branch to Subroutine

See Special Ops

REL

Branch if Overflow Clear

80 rr
28 rr

8-1

Branch if Overflow Set

?V=0
? V= 1

REL

BVS (rel)
CBA

Compere A to B

A-B

REL
INH

29 n
11

2
1

3
2

8-1
2-1

CLC

Clear Carry Bit

0-C

1NH

OC

cu

Clear Interrupt Mask

0-+I

INH

CLR (opr)

Clear Memory Byte

0 -> M

EXT
IND.X
IND.Y

3
2
3

2
6
6
7

2-1
2-1

BVC (ref)

0E
7F hh II
6F ff
18 6F ff

5-6
6-3
7-3

------ 11 0-

r-tm
..............0
. . . 0 ------------ 0100

Clear Accumulator A

0 ~+A

A INH

4F

2-1

------ 0 100

CLRB

Clear Accumulator B

<o
T
o
0-+V

5F
0A

1
1

2-1

Clear Overflow Rag

B INH
INH

CLV

2-1

-------0 100
........... 0-

2
2
3
2
3

2
3
4
4
5

3-1
4-1
5-2
6-2
7-2

-1111

2
2
3
2
3

2
3
4
4
5

3-1
4-1
5-2
6-2
7-2

----mi

3
2
3

6
6
7

5-8
6-3
7-3

CLRA

CMPA(opr) Compare A to Memory

81
91
B1
A1
18 A1

C1
D1
F1
E1
18 El
73 hh n
63 n
18 63 ff

CMPB (opr) Compare B to Memory

B-M

B IMM
3 DIR
3 EXT
3 IND.X
B IND.Y

COM (opr)

$FF-M -M

EXT
ND,X
IND.Y

n
n
t
>
I
>

Vs Complement Memory Byte

ii
dd
hh n
ff
fr
ii
dd
hh n
ff

AIMM
A DIR
A EXT
A IND.X
A IND.Y

ff

COMA

ts Complement A

2-1

----1101

COMB

1's Complement B

$FF - B - B

B INH

53

2-1

------ 11 0 1

CPD (opr)

Compare D to Memory 16-Bit

D - M :M + 1

IMM
DIR
EXT
ND,X
IND.Y

1A 63 jj kk
1A 93 dd
1AB3 1h 0
1A A3
CD A3

4
3
4
3
3

5
6
7
7
7

3-5
4-9
5-11
6-11
7-8

A INH

43

ff
ff

---1101

--1111

*CycJe-by-cycle number provides a reference to Tables 10-2 through 10-8 which detail cycle-by-cycle operation.
Example: Table 10-1 Cyde-by-Cycle column reference number 2-4 equals Table 10-2 line item 2-4.

MC68HC11A8
TECHNICAL DATA

CPU, ADDRESSING MODES, AND INSTRUCTION SET

MOTOROLA
10-7

Table 10-1 MC68HC11A8 Instructions, Addressing Modes, and Execution Times


(Sheet 3 of 6)
Source
Form(s)

Boolean Expression

Operation

Addressing
Mode for
Operand

Machine Coding
(Hexadecimal)
Opcode

Operand(s)

v>
9> w
% .
a o
3

CPX (opr)

Compare X to Memory 16-Sit

IX -M:M + 1

IMM
DIR
EXT
IND.X
IND.Y

8C
9C
BC
AC
CD AC

ij kk
dd
hh fl
ff
ff

CPY (opr)

Compare Y to Memory
16- Bit

fY-M:M + 1

IMM
DIR
EXT
IND,X
IND.Y

18 8C
18 9C
18 BC
1A AC
16 AC

jj kk
dd
hh II
ff
ff

4
3
4
3
3

19
7A hh n
6A
18 6A

3
2
3

DM

Decimal Adjust A

DEC (opr)

Decrement Memory Byte

OECA
DECS
OES

4
5
6
6
7

3-3
4-7
5-10
6-10
7-8

----nn

3-5
4-9
5-11
6-11
7-8

---mi

2-1

----nn

6
6
7

5-8
6-3
7-3

---ni-

2-1
2-1

7
7
7

Adjust Sum to BCD


M - 1 -* M

INH
EXT
IND,X
IND.Y

Decrement Accumulator A
Decrement Accumulator B

A- 1 -A

A INH

4A

B-1 -* B

B INH

5A

Decrement Stack Pointer

ff
ff

34

Cycle
Condition Codes
by
Cycle* SXHiNZVC

2-3

SP-1 -* SP

INH

DEX

Decrement Index Register X

IX 1 -IX

INH

09

2-2

DEY

Decrement Index Register Y

2-4

Exclusive OR A with Memory

INH
AIMM
ADIR
A EXT
A IND.X
A IND,Y

18 09

EORA(opr)

IY 1 fY
A M - A

ii
dd
hh D
ff

2
2
3
2
3

2
3
4
4
S

3-1
4-1
5-2
6-2
7-2

EORB (opr)

ExtUiske OR B with Memory

BIMM
B DIR
BEXT
B IND.X
B IND.Y

ce ii
06 dd
FG hh n
EG ff
18 E8 ff

2
2
3
2

2
3
4
4
5

4-1
5-2
6-2
7-2

FDIV

Fractional Divide 16 by 16

IDIV
INC (opr)

Integer Divide 16 by 16
Increment Memory Byte

0/IX -IX; r - D
D/IX -IX; r -> D
M + 1 -M

INH
INH
EXT
IND.X
IND.Y

88
98
88
A8
18A8

ff

ff

........ i -........i - no-

no-

41
41

2-17
2-17

......... 101

5-8
6-3
7-3

in-

2
3

6
6
7

03
02
7C hh II
6C
18 6C ff

-in in-

......... in

fNCA

Increment Accumulator A

A+ 1 -* A

A INH

4C

2-1

---ni-

INCB

increment Accumulator B
increment Stack Pointer

B + 1 - B

B INH

5C

2-1

m-

INS

SP 1 - SP

rNH

2-3

3
4

2-2
2-4

INX

Increment Index Register X

IX + 1 -4 IX

INH

31
08

INY

Increment Index Register Y

IY + 1 -IY

INH

18 08

JMP(opr)

Jump

See Special Ops

EXT
IND.X
IND,Y

3
2

3
3
4

5-1
6-1
7-1

JSR (opr)

Jump to Subroutine

See Special Ops

DIR
EXT
IND.X
IND.Y

2
3
2
3

5
6
6
7

4-8
5-12
6-12
7-9

LDAA (opr)

Load Accumulator A

M - A

AIMM
ADIR
A EXT
AIND.X
A IND,Y

7E hh It
6E ff
18 6E ff
9D dd
BD hh n
AD ff
18 AD
86 ii
96 dd
B6 hh n
A6
18 A6 ff

2
2
3
2
3

2
3
4
4
5

3-1
4-1
S2
6-2
7-2

-I10-

LDAB (opr)

Load Accumulator B

M -* B

B IMM
B DIR
B EXT
B IND.X
B IND,Y

3-1
4-1
5-2
6-2
7-2

----110-

Load Double Accumulator D

M - A.M + 1 -> B

IMM
DIR
EXT
IND,X
IND,Y

2
2
3
2
3
3
2
3
2
3

2
3
4
4
5

LDD(opr)

C6 ii
D dd
FG hh II
EG ff
18 E6 ff
CC jj kk
DC dd
FC hh n
EC ff
18 EC ff

3
4
5
5
6

3-2
4-3
5-4
6-6
7-6

-------IIO-

ff
ff

........ i - i..

*Cycle-by-cycle number provides a reference to Tables 10-2 through 10-8 which detail cycle-by-cycle operation.
Example: Table 10-1 Cycle-by-Cycle column reference number 2-4 equals Table 10-2 line item 2-4.

MOTOROLA

10-8

CPU, ADDRESSING MODES, AND INSTRUCTION SET

MC68HC11A8
TECHNICAL DATA

Table 10-1 MC68HC11A8 Instructions, Addressing Modes, and Execution Times


(Sheet 4 of 6)

LDX (opr)

LDY (opr)

LSL (opr)
LSLA

M:M + t -+SP

Load Stack Potnter

Load fndex Register X

load Index Register Y

M M + 1 -IX

M:M + 1 - IY

IMM
DIR
EXT
IND.X
IND.Y

C b? bO

Opcode
8E
9E
BE
AE
18 AE

Operand(s)
0 kk
dd
hh II
ff
ff

3
4
5
5
6

3-2
4-3
5-4
6-6
7-6

------ 11 0-

3-2
4-3
5-4
6-6
7-6

------ 110-

4
5
5
6

3-4
4-5
5-6
6-7
7-6

------ 11 0-

3
2
3
2

IMM
DIR
EXT
IND.X
fND.Y

18 CE
18 DE
18 FE
1AEE
18 EE

4
3
4
3
3

jj kk
dd
hh D
ff

ff
ff

Cycle
Condition Codes
by
Cycle* SXHINZVC

3
3
2

CE jj kk
DE dd
FE hh a
EE ff
CD EE ff

-----I M I I I ! I I I Io

Logical Shift Left Double

Machine Coding
(Hexadecimal)

IMM
DIR
EXT
IND.X
IND.Y

EXT

Logical Shift Left

"

LSLD

Addressing
Mode for
Operand

Cycle

LDS (opr)

Boolean Expression

Bytes

Operation

Source
Form(s)

78 hh II

4
6
6
6

5-8

A INH
B INH

4
58

2
2

3-7
2-1
2-1

INH

05

2-2

-I1I1

EXT

74 hh n
64 ff

6
6

5-8

1111

HH------------- CDo
C b15 bO

LSR (opr)

Logical Shift Right


-----

LSRA
-
LSRB

LSRD

A INH
B INH

44
54

2
2

7-3
2-1
2-1

INH

04

2-2

AxB -* 0

INH

3D

0-M -M

EXT
IND.X
IND.Y

67 60 C

----- -

Logical Shift Right DoubFe

out

o-H I I------- -II II I


bl5 bO C
MUL
NEG (opr)

Multiply 8 by 8
2s Complement Memory Byte

< mi
t t:
< CO
I
o o

70 hh 0
60 ff
18 60 ff
40

NEGA

7s Complement A

NEGB

2s Complement B

NOP

No Operation

No Operation

INH

ORAA(opr)

OR Accumulator A (Inclusive)

A + M-A

A IMM
A DIR
A EXT
A IND.X
A INO.Y

8A
9A
BA
AA
18 AA

fi
dd

BIMM
B DIR
3 EXT
3 IND.X
B fND.Y

CA
DA
FA
EA
18 EA

K
dd

ORAB (opr)

OR Accumulator B (Inclusive)

B + M -B

A INH
B INH

hh n
ff
fr

hh n
ff
ff

36

10

2-13

..............1

6
6
7
2

5-8
6-3
7-3
2-1

-mi

-mi

1
1

2-1

----mi

2-1

2
2
3
2
3

2
3
4
4
5

3-1
4-1
5-2
6-2
7-2

-110-

2
2
3
2
3

2
3
4
4
5

3-1
4-1
5-2
6-2
7-2

-------110-

PSHA

Push A onto Stack

2-6

PSHB

Push B onto Stack

8 -> Stk, SP SP-1

B INH

37

2-6

PSHX

Push X onto Stack (Lo First)

IX - Stk, SP * SP-2

INH

3C

2-7

18 3C

32

5
4

2-9

A - Stk, SP * SP1

A INH

50
01

1
3
2
3
1

PSHY

Push Y onto Stack (Lo First)

IY - Stk, SP SP-2

NH

PULA

Pull A from Stack

SP * SP 1, A-Stk

A INH

PULB

Pull B from Stack

SP=SP + 1, B4-Stk

B INH

2-9

PULX

Pull X from Stack (Hi First)

SP = SP + 2, IX-Stfc

NH

38

2-10

PULY
ROL (opr)

Pull Y from Stack (Hi First)

SP* SP+2, IY-Stk

INH

18 38

3
2

6
6
/

1
1

2-11
5-8
6-3
7-3
2-1
2-1

Rotate Left

ROLA
ROLB

-----IMIHIIIIMI
C b7 bO C

EXT
IND.X
MNH
B INH

33

79 hh II
69 ff
ff
59

2-8

1111

*Cyde-by-cyde number provides a reference to Tables 10-2 through 10-8 which detail cycle-by-cycle operation.
Example: Table 10-1 Cyde-by-Cycle column reference number 2-4 equals Table 10-2 line item 2-4.

MC68HC11A8
TECHNICAL DATA

CPU, ADDRESSING MODES, AND INSTRUCTION SET

MOTOROLA
10-9

Source
Form(s)
ROR (opr)

Operation

Boolean Expression

Rotate Right
----- n-n 1

RORA

ii ri i hi

Addressing
Mode for
Operand
EXT
INO.X
IND.Y
B INH

Machine Coding
(Hexadecimal)
Opcode

Operand(s)

76 hh n
66 ff
1866 ff
46
56

e>

aa
3
2

Cycle j

Table 10-1 MC68HC11A8 Instructions, Addressing Modes, and Execution Times


(SheetS of 6)
Cycle
by
Cyde*

6
6
/

S-8
6-3
7-3
2-1
2-1

Condition Codes
8XHINZVC
----nn

RTI

Return from Interrupt

See Special Ops

INH

3B

12

2-14

RTS

Return from Subroutine

See Special Ops

INH

39

2-12

SBA

Subtract B from A

A-B A

INH

10

2-1

--nn

a-m-c-a

AIMM
ADIR
A EXT
A IND.X
A IND.Y

82
92
B2
A2
18A2

----nn

C2
D2
F2
E2
18 E2

2-1

............... 1
... 1

SBCA (opr) Subtract with Carry from A

ii
dd
hh 0
ff
ff
ii
dd
hh n
ff
ff

2
2
3
2
3

4
4
5

2
2
3
2
3

2
3
4
4
S

3-1
4-1
5-2
6-2
7-2
3-1
4-1
5-2
6-2
7-2

2-1

SBC8 (opt)

Subtract with Carry from B

e-M-c -> B

BIMM
BOIR
BEXT
B IND.X
8 IND.Y
INH

OD

INH

OF

OB

SEC

Set Cany

SEi
SEV

Set Interrupt Mask


Set Overflow Flag

1 ->!
1 -+V

INH

STAA (opr)

Store Accumulator A

A -M

ADIR
A EXT
A IND.X
A IND.Y

STAB (opr)

Store Accumulator B

B - M

STO (opr)

Store Accumulator D

A-M, B-+M + 1

STOP

Stop Internal Clocks

STS (opr)

Store Stack Pointer

SP M:M +1

DIR
EXT
IND.X
IND.Y

STX (opr)

Store Index Register X

IX -M:M +1

STY (opr)

Store Index Register Y

IY + 1

DIR
EXT
IND.X
IND.Y
DIR
EXT
IND.X
IND.Y

SU8A (opr)

Subtract Memory from A

A-M -* A

AIMM
ADIR
A EXT
A IND.X
A IND.Y

SUBB (opr) Subtract Memory from B

B-M -> B

SUBD(opf)

Subtract Memory from D

D - M:M + 1- D

SW1

Software Interrupt

See Special Ops

INH

TAB

Transfer A to 8

A-B

INH

TAP

Transfer A to CC Register

A-CCR

INH

06

TBA

Transfer Bto A

B -A

INH

17

-c

unnn

----nn

2-1

............ 1 -

dd
hh 11
ff
fT

2
3
2
3

3
4
4
5

4-2
5-3
6-5
7-5

--no-

8 DIR
BEXT
8 IND.X
B IND.Y

97
B7
A7
18 A7
D7
F7
E7
18 E7

dd
hh n
ff
ff

2
3
2
3

3
4
4
5

4-2
5-3
6-5
7-5

- - IIo-

DIR
EXT
IND.X
IND.Y

DD
FC
ED
18 ED

dd
hh II
ff
ff

2
3
2
3

4
5
5
6

4-4
5-5
6-8
7-7

--no-

2-1

9F
BF
AF
18 AF

dd
hh n
ff
ff

2
3
2
3

4
5
5
6

4-4
5-5
6-8
7-7

....no-

DF
FF
EF
CD EF

dd
hh H
ff
ff

2
3
2
3

4
5
5
6

4-4
5-5
6-8
7-7

- no-

18 DF
18 FF
1AEF
18 EF

3
4
3
3

5
6
6
6

4-6
5-7
6-9
7-7

no-

80
9C
BG
AO
18 A0

dd
hh II
ff
ff
ii
dd
hh II
ff
ff

2
2
3
2
3

2
3
4
4
5

3-1
4-1
5-2
6-2
7-2

- tin

BIMM
3 DIR
BEXT
6 IND.X
B IND.Y

CO
DO
FC
EC
18 E0

ii
dd
hh II
ff
ff

2
2
3
2
3

2
3
4
4
S

3-1
4-1
5-2
6-2
7-2

----mi

IMM
DIR
EXT
IN0.X
IND.Y

83 ii kk
92 dd
B3 hh II
A3 ff
18 A3 ff

3
2
3
2
3

4
5
6
6
7

3-3
4-7
5-10
6-10
7-8

....nt i

3F

14

2-15

...1

16

2-1

tio-

2-1

2-1

liinnt
-.--no-

INH

CF

*Cycle-by-cycfe number provides a reference to Tables 10-2 through 10-8 which detail cycle-by-cycle operation.
Example: Table 10-1 Cycle-by-Cycle column reference number 2-4 equals Table 10-2 line item 2-4.

MOTOROLA

10-10

CPU, ADDRESSING MODES, AND INSTRUCTION SET

MC68HC11A8
TECHNICAL DATA

Source
Form(s)

Operation

Boolean Expression

Addressing
Mode for
Operand

Machine Coding
(Hexadecimal)
Opcode

Operand(s)

VI
ti

TEST

TEST (Only in Test Modes)

TPA

Transfer CC Register to A

Adcfress Bus Counts


CCR -* A

INH
INH

TST (opr)

Test for Zero or Minus

M0

A-0

EXT
[NO.X
IND.Y
A INH

B-0

BINH

5D

INH

30

INH

18 30

TSTA

Cycle I

Table 10-1 MC68HC11A8 Instructions, Addressing Modes, and Execution Times


(Sheet 6 of 6)

00
07
7D hh It
6C ff
186D ff
40

2
3
2
3

Cycle
Condition Codes
by
Cycle* SXHINZVC
2-20
2-1

6
6

5-9
6-4
7-4

2-1

1100

2-1

sxoo

2-3

TSTB
TSX

Transfer Stack Pointer to X

TSY

Transfer Stack Pointer to Y

SP + 1 -IX
SP + 1 -IY

2-5

TXS

Transfer X to Stack Pointer

IX-1 -SP

INH

35

TYS

Transfer Y to Stack Pointer

IY 1 ->SP

[NH

18 35

2-2
2-4

WAI

Wait for interrupt

Stack Regs & WAIT

INH

3E

XGDX
XGDY

Exchange D with X

IX -> D, D -IX

INH

8F

2-2

Exchange D with Y

IY 0. D - IY

INH

19 8F

2-4

Itoo

2-16

Cyde-by-cycle number provides a reference to Tables 10-2 through 10-8 which detail cycle-by-cycle operation.
Example: Table 10-1 Cycle-by-Cycle column reference number 2-4 equals Table 10-2 line item 2*4.
Infinity or Until Reset Occurs
***12 Cycles are used beginning with the opcode fetch. A wait state is entered which remains in effect for an integer
number of MPU E-clock cycles (n) until an interrupt is recognized. Finally, two additional cycles are used to fetch
the appropriate interrupt vector (14 + n total),
dd = 8-Bit Direct Address ($0000 -SOOFF) (High Byte Assumed to be $00)
ff = 8-Bit Positive Offset $00 (0) to $FF (255) (Is Added to Index)
hh = High Order Byte of 16-Bit Extended Address
ii = One Byte of Immediate Data
jj = High Order Byte of 16-Bit Immediate Data
kk = Low Order Byte of 16-Bit Immediate Data
II = Low Order Byte of 16-Bit Extended Address
mm = 8-Bit Bit Mask (Set Bits to be Affected)
rr = Signed Relative Offset $80 (-128) to $7F (+ 127)
(Offset Relative to the Address Following the Machine Code Offset Byte)

MC68HC11A8
TECHNICAL DATA

CPU, ADDRESSING MODES, AND INSTRUCTION SET

MOTOROLA
10-11

Table 3-1 Register and Control Bit Assignments (Sheet 1 of 2)


Bit 0 | PORTA I/O Port A

$1000

Reserved

$1001

INVB

PIOC

Parallel I/O Control Register

$1003

PORTC

I/O PortC

$1004

PORTB

Output PortB

$1005

PORTCL

Alternate Latched Port C

$1007

DDRC

Data Direction for Port C

$1008

PORTD

I/O Port D

$1002

$1006

$1009

DDRD

Data Direction for PortD

$100A

PORTE

Input Port E

$100B

CFORC

Compare Fores Register

$100C

OC1M

OC1 Action Mask Register

$1000 OC1D7 OC1D6 OC1D5 0C104 OC1D3

OC1D

OC1 Action Data Register

$100E

Bit 15

$1 OOF

Bit 7

$1010

Bit 15

$1011

Bit 7

$1012

Bit15

$1013

Bit 7

$1014

Bit 15

$1015

Bit 7

Bite

BitO

Bite

8it0

Bit 8

BitO

Bite

BitO

$1016

Bit 15

Bite

$1017

Bit 7

BitO

$1018

Bit 15

Bitfi

$1019

Bit 7

BitO

$101A

Bit 15

Bit 8

$101B

Bit 7

BitO

$101C

Bit 15

Bit 8

S101D

Bit 7

BitO

$101E

Bit 15

Bit 8

$101F

Bit 7

BitO

MOTOROLA
3-2

ON-CHIP MEMORY

Timer Counter Register

Input Capture 1 Register

Input Capture 2 Register

Input Capture 3 Register

Output Compare 1 Register

Output Compare 2 Register

Output Compare 3 Register

Output Compare 4 Register

Oulput Compare 5 Register

MC68HC11A8
TECHNICAL DATA

Table 3-1 Register and Control Bit Assignments (Sheet 2 of 2)


$1020

OM2

OL2

OM3

013

OM4

OL4

OM5

OL5

Ttmer Control Register 1

$1022

OCtl

OC2I

EDG1B

EDG1A

EOG2B

EDG2A

EDG3B

EOG3A

Timer Control Register 2

OC3I

OC4I

OC5I

IC1I

IC2I

IC3I

Timer Interrupt Mask Register 1

$1023

OC1F

$1024

TOI

OC2F

OC3F

OC4F

OC5F

IC1F

IC2F

IC3F

Timer Interrupt Flag Register 1

RTII

PAOVI

PAH

PR1

PRO

Timer Interrupt Mask Register 2

PAIF
RTR1

RTRO

$1021

$1025

TOF

RTIF

PAOVF

$1026

DDRA7

PAEN

PAMOD

Timer Interrupt Flag Register 2

PEDGE

Putse Accumulator Control Register

$1027

Bit 7

8ft 0

$1028

SPiE

SPE

DWOM

MSTR

CPOL

CPHA

SPR1

SPRO

$1029

SPIF

WCOL

MODF

$102A

Bit 7

BitO

$102B

TCLR

SCP1

SCPO

RCKB

SCR2

SCR1

SCRO

SCI Baud Rate Control

SBK

SCI Control Register 2

Pulse Accumulator Count Register


SPI Control Register
SPI Status Register
SPI Data Register

SCI Control Register 1

S102C

R8

T8

WAKE

S102D

TIE

TCIE

RIE

ILIE

TE

RE

RWU

$1026

TRDE

TC

RDRF

IDLE

OR

NF

FE

$102F

Bit 7

^SCSRSCI Status Register


BitO

SCI Data {Read RDR, Write TOR)

$1030

CCF

SCAN

MULT

CD

CC

C8

CA

A/D Control Register

$1031

Bit 7

BitO

A/D Result Register 1

$1032

Bit 7

BitO

A/D Result Register 2

$1033

Bit 7

BitO

A/D Result Register 3

$1034

Bit 7

BitO

A/D ResuR Register 4

$1039

ADPU

CSEL

IRQE

DLY

CME

CR1

CRO

System Configuration Options

$103A

Bit 7

BitO

Arm/Reset COP Timer Circuitry

BYTE

ROW

ERASE

EELAT

EEPGM

EEPROM Program Control Register

IRV

PSEL3

PSEL2

PS ELI

PSELO

Highest Priority 1-Bit Int and Misc


RAM and I/O Mapping Register

$1035
thro
$1038

$1038

ODD

EVEN

S103C

RBOOT

SMOD

$1030

RAM3

RAM2

$103E

TILOP

$103F

MC68HC11A8
TECHNICAL DATA

MDA
RAM1

RAMO

REG3

REG2

REG1

REGO

OCCR

CBYP

DISR

FCM

FCOP

TCON

Factory TEST Control Register

NOSEC

NOCOP

ROMON

EEON

COP. ROM, and EEPROM Enables

ON-CHIP MEMORY

MOTOROLA

3-3

17
-SEL 4533-

APPENDIX: M68HC11 PIN OUT

ii
PA7/PAI/0C1 C 1

48 ^ VDD

PA6/OC2/OC1 C 2

47 | PD5/SS

PA5/OC3/OC1 c 3

46 I PD4/SCK

PA4/OC4/OC1 [ 4

45 ] PD3/MOSI

PA3/OC5/OC1 : 5

44 ] PD2/MISO

PA2/IC1 [ 6

43 ] PD1/fxD

PA1/IC2 C 7

42 ] PDO/RxD

PA0/IC3 [ 8

41 ] IRQ

PB7/A15 [ 9

40 ] XIRQ

PB6/A14C 10

39 | RESET

PB5/A13 [ 11

38 ] PC7/A7./D7

PB4/A12 [ 12

37

PB3/A11

| PC6/A6/D6

13

36 ] PC5/A5/D5

PB2/A10C 14

35 ] PC4/A4/D4

PB1/A9 [ 15

34 ] PC3/A3/D3

PB0/A8 C 16

33 ] PC2/A2/D2

PEO/ANO C 17

32 ] PC1/A1/D1

PE1/AN1 C 18

31

PCO/AO/DO

PE2/AN2 C 19

30

XTAL

PE3/AN3 [ 20

29

EXTAL

21

28

STRB/RM

22

27

C 23

26

STRA/AS

MODBA/g-pgy C 24

25

MODA/UR

V RL

V rh
vss

E
C

18
-SEL 4533-

APPENDIX: MOTOR DRIVER L293B

SGS-THOMSON

L293B
L293E

PUSH-PULL FOUR CHANNEL DRIVERS


. OUTPUT CURRENT 1A PER CHANNEL
. PEAK OUTPUT CURRENT 2A PER CHANNEL
(non repetitive)
. INHIBIT FACILITY
. HIGH NOISE IMMUNITY
. SEPARATE LOGIC SUPPLY
. OVERTEMPERATURE PROTECTION

DIP16
ORDERING NUMBER: L293B

DESCRIPTION

The L293B and L293E are quad push-pull drivers


capableof delivering output currents to IA per chan
nel. Each channel is controlled by a TTL-compatible
logic input and each pair of drivers (a full bridge) is
equipped with an inhibit input which turns off all four
transistors. A separate supply input is provided for
the logic so that it may be run off a lower voltage to
reduce dissipation.
Additionally, the L293E has external connection of
sensing resistors, for switchmode control.

POWERDIP (16 * 2-> 2)


ORDERING NUMBER: L293E

The L293Band L293E are package in 16 and 20-pin


plastic DIPs respectively ; both use the four center
pins to conduct heat to the printed circuit board.

PIN CONNECTIONS
DIP16 - L293B
CHIP ENABLE

I| 1

INPUT

1|

OUTPUT

1|

16

15

GNO |

OUTPUT ? |

13

] INPUTt

] CND

V
20 I

19

OUTPUT 1

IB ] OUTPUT 4

SENSE 1

i.

17 | SENSE 4

nitti

16 ] GNO

GNO

15 ] GNO

SENSE 2

U ] SENSE 3

OUTPUT 2

13

INPUT I

1?

j INPUT 3

*4

10

11

jCHIP ENABLE 2

INPUT I

SN0

] INPUT *

11 ] OUTPUT 1

CHIP ENA8U.ll

U ] OUTPUT C

GNO |

INPUT J

POWERDIP (16+2+2) - L293E

10

] INPUT 3
[chip enable 2

] OUTPUT 3

S-&1B9

19
-SEL 4533-

APPENDIX: MOTOR DRIVER L293B (CONT)


DIP16 - L293B

Inputs
Vin n

=H

Vinh =

L = Low

Function
C=H;D=L

Turn Right

C = L; D = H

Turn Left

C=D

Fast Motor Stop

C = X; D = X

Free Running
Motor Stop

H = High

X = Dont Care

CONFIDENTIAL

UTM
UNIVERSITI TEKNOLOGI MALAYSIA

FINAL EXAMINATION SEMESTER II


SESSION 2010/2011
COURSE CODE
COURSE NAME

SEL 4223
DIGITAL SIGNAL PROCESSING I

LECTURERS

ASSOC. PROF. DR. SYED ABDUL RAHMAN BIN


SYED ABU BAKAR

PROGRAMME

SEC / SEM / SEP / SET / SEW

SECTION

01

TIME

2 HOURS 30 MINUTES

DATE

29 APRIL 2011

INSTRUCTION TO CANDIDATE

ANSWER QUESTION ONE (1) IN PART A


ANSWER ANY THREE (3) QUESTIONS IN PART
B.
THIS IS AN OPEN BOOK EXAM.

THIS EXAMINATION BOOKLET CONSISTS OF 11 PAGES INCLUDING THE FRONT COVER

SEL 4223
-2-

Part A

QUESTION 1
a) State 2 reasons why the study of Linear Time-Invariant system is important.
(5 marks)

b) State 2 reasons why z-transform is more useful in system analysis compared to


Fourier transform.

(5 marks)

c) State 2 differences between FIR and IIR systems.

(5 marks)

d) A signal is defined as follows

x(t ) = cos (27r3000t)

i)

< oo

Sketch the frequency representation for 2n < co < 2n if the signal is


sampled at 10 kHz.

ii)

- oo < t

(5 marks)

What is the minimum sampling frequency for this signal? Explain your
reasons.

(5 marks)

SEL 4223
-3-

PART2
QUESTION 2
An impulse response of an LTI system is given as follow:

h[n ] = u[n] + u[n - 2] - 2u[n - 5]

a)

State whether this system is an FIR or an IIR system and justify your
answer.

b)

(3 Marks)

Compute the system function, H(z), of the above system and determine its
Region of Convergence (ROC).

c)

(7 Marks)

If the input, x[n], to this system is given as follow

x[n] = u[n] - u [ n - 1] + u [ n - 3] - u [ n - 4]

i) Sketch the output signal, y[n].

(5 Marks)

ii) Express Y (z ), the z-transform of the output signal in terms of H(z).


(5 Marks)

d)

If the z-transform of the output, Y (z), can be written as follows:

Y(z) = 1 + z-1 + z~ 2 + z~3 2 z~5 2 z-6

State what is the new input, x n e w [n], to this system?

(5 Marks)

SEL 4223
-4-

OUESTION 3

An LTI system has the following pole-zero plot shown in Figure Q3.

Figure Q3

a) Can this system be causal and stable? Justify your answer.

(3 Marks)

b) What should be the ROC if the system is to be stable?

(3 Marks)

c) Give the difference equation for the system shown in Figure Q3. (10 Marks)

d) Properly sketch and label the magnitude spectrum for the above system for
0 < (o < n.
(9 Marks)

SEL 4223
-5-

OUESTION 4
An impulse response, h[n], of an LTI system is shown in Figure Q4a below:-

Figure Q4a
a) State whether the above system is a zero-phase, linear-phase, tninimum-phase, or
reverse-phase system.

(2 Marks)

b) Suppose the input to this system is as shown in Figure Q4b, obtain the output
signal to this system.

(10 Marks)
0

x[n]

T I T

0 1 2 3 4 n

Figure Q4b
c) Obtain the magnitude spectrum |//[&]| using trigonometry terms for the above
system using 10-point DFT.

(10 Marks)

d) State the difference between Discrete-Time Fourier Transform (DTFT) and


Discrete Fourier Transform (DFT).

(3 Marks)

SEL 4223
-6-

OUESTION 5
Consider the following 16-point DFT of x[n] which was sampled at 800 Hz

Figure Q5

a) What is the resolution for each index kl

(2 Marks)

b) Suppose X[6] and X[10], represent noise, what type of filter should be used to
removed this noise?
(3 Marks )
c) Propose a suitable cutoff frequency such that the noise whose frequency
mentioned in b) could be somewhat eliminated.

(2 Marks)

d) Using the 2nd order Butterworth HR prototype given below do the following
H c (s) =

i.

1
s 2 + V2s + 1

Obtain the prewarp critical frequency such that the above normalized
prototype filter gives the desired response.
(3 Marks)
ii. Obtain the analog version of the desired Iowpass filter, H(s). (5 Marks)
iii. Using bilinear transformation technique, obtain the digital filter, H(z ).
(10 Marks)

SEL 4223
-7-

OUESTION 6

Observe the following impulse response of an LTI system and answer the following
questions. Note: n is the time index while k is the frequency index.

h[n]

1
-2 -1 0 1 2 3 4 5 6 7 8

a) Is this system stable? Justify your answer.

(3 Marks)

b) Is this system causal? Justify your answer.

(3 Marks)

c) Determine what is the type of this linear phase filter.

(3 Marks)

d) Without having to compute H[k], determine the value for H[ 0],

(3 Marks)

e) Without having to compute H[k], determine the phase response,

(3 Marks)

f) Express |//[fc].|, the 10-point DFT of the given h[n] in terms of cosine functions.
(10 Marks)

Table 1 Fourier transform properties


Properties

x(t)

Differentiation in time

d n x(t )
dt"

Til
[ x(t)dt

Integration in time

772

X(f) \e j 2 *'dt

-77 2

Time-shift

<t-Td)

Frequency-shift

x{t)e J 2 ^

-772
X (f)e'

X(f + fi)

x(t) + y(t )

Linearity
Product of two signals

X{f) + Y{f)
00

*(0X0

\x{v)Y(f-v)dv
CO

x(t) * y{t)

Convolution of two signals

X<J)Y<J)

x(at )

Time scaling

rA L \
M \a)
Parsevals theorem

00

oo

E x = \[X{f)] 2 df

E x = j[x(t)fdt
00

00

Table 2 DFT properties


Properties
Linearity
Shift in time

x[]
axxx (n ) + a2x2{ri)
x{n - m)

Shift in frequency

.2 7mm

Circular convolution

J N

a l X l (k ) + a 2 X 2 (k )
.

2xkm

e~ J ~ X(k)
X(k-m)

x(n)

N-\

^ x x (A)x2 [( - A) mod N ]

X x (k)X 2 (k)

x=o

Multiplication

x i(n)x

2 (n)

N-\

X, (A)X 2 [(A: - X) mod TV]


A=0

Parsevals theorem
^|x()|2
n=0

EN2
k=0

SEL 4223
-9Table 3 DTFT properties

Sequence

Fourier Transform

x[n\
y[n]

X(e^)
Y(e J 2 4 )
aX{e J 2 ) + bY(e J W )

1. ax[n\+ by[ri\
2. x[n - n d ], (n d an integer)

e-M>*

3. e j 2 ^x(n)

X{e J 2 * ( f ~ f o ) )

4. x[-ri\

X(e~ J 3 )

X^ e J*f)

X*(e j 2 4 ) ifx[]real.
5. nx[ri\

dX(e J 2 n f )
J

6. x[ri\ * y[n\

dlnf

X(e i W )Y(e J W )

7. x[n\y[n\

\ f , ' f 2 X(e J l m )Y(e

i2*u-v)

)dv

Parsevals Theorem
8. V \x(nf = f/l/2 1 X{e J 2 *f df

^ 2 *()/() = C

I? W

Table 4 z-transform properties


Properties
Linearity
Shift in time
Multiplication by n
Multiplication by r"
Convolution

x[n]
a]X[(ji) + a2x2 (n)
x(n +1)
nx(n)

~z~X(z)
dz

r"x(ri)
00

(k)x 2 (n-k)
Initial Value

X(z)
aiX\(z) + a 2 X 2 (z)
z{X{z)-xm

k=Q
x(0)

atA

r
X^X^z)
lim X(z)

z> oo

Final Value

lim x(n )

lim[(z-l)X(z)]
Z-l

SEL 4223
-10-

Table 5 z-transform pairs

x[ n ] for n > 0

X (z )

Radius
convergence

N> R
S(n)

5(n- in)
u (n)

z~ m
z

0
1

z -1
n

z
(z-1)2

n1

z(z + 1)

(z-1)3
an
ncT

z-a
az
(z -a ) 2

(n+l)an

z2

M
W

(z -a ) 2
(n +1 )(n + 2)...( + m)a"
ml

H
(z -ay + I

cos Q 0 n

z (z- cosCl 0 )
z2

-2zcosQ0 +1

sin Cl0n

z sin Q0

z2 -2zcosQ0 +1
z(z-acosQ0)

a" cos Q.0n


z2

-2zacosCl 0 +a 2
za sinQ0

a sin Q 0 n
z2

| a\

- 2zacosQ 0 + a 1

exp [- anT]

exp[-a7]|

z - expf-aT1]
nT

Tz

(z -1)2
nT exp [ - anT]

Tzexpf-ar]

jexp[-a7]|

[z-exp[-a7]]2
cos nca0T

z(z -cosa>0T)

z2 -2zcoscoaT + 1
sin nco0T

zsin coqT
z2 -2zcosco0T + 1

of

SEL 4223
-11-

exp[-a7]cos ncoQT

z(z - exp[-ar] cos co0T)


z2

exp[-ar]sin nco0T

-2zexp[-ar]cosft)or + exp[-2ar]
z(z - exp[-aT]sin a>0T)

z2

exp[-ar]|

-2zexp[-ar]cosft)or + exp[-2ar]

|exp[-ar]|

CONFIDENTIAL

UTM
UNIVERSITI TEKNOLOGI MALAYSIA

FINAL EXAMINATION SEMESTER II


SESSION 2010/2011
COURSE CODE

SEI4123

COURSE NAME

APPLIED ROBUST CONTROL

LECTURERS

ASSOC. PROF. DR. YAHAYA BIN MD. SAM


ASSOC. PROF. DR. MOHAMAD NOH BIN
AHMAD
DR. ZOOL HILMI BIN ISMAIL

PROGRAMME

SEI

SECTION

01

TIME

2 HOURS 30 MINUTES

DATE

25 APRIL 2011

INSTRUCTION TO CANDIDATE

ANSWER ANY FOUR (4) QUESTIONS ONLY.

THIS EXAMINATION BOOKLET CONSISTS OF 8 PAGES INCLUDING THE FRONT COVER

2
(SEI4123)

Question 1
a) Define a robust control system.

[2 marks]
b) Discuss the relationship between sensitivity and the percent overshoot in the robust
control design.
[3 marks]
c) A robust control system is represented by the block diagram as in Figure Ql,

R(s)

K
s(s + 2co)

C(s)

Figure Ql
The dominant poles of the system for K = 10 are located at -1.5 + y'4.21.
i)

Calculates the system sensitivity and percentage overshoot at this location.

[9 marks]
ii)

What happen to the system sensitivity and percentage overshoot if K is increased


to K = 50, where the new dominant poles are located at -1.5 + J6.92 .

[8 marks]
d) Comment the results obtained in (i) and (ii) in terms of sensitivity and percentage
overshoot.
[3 marks]

(SEI4123)

Question 2

Consider a unity feedback control system as illustrated in Figure Q2.

(s + 5)(s - 4)
K

---- ^

(s +10)09+ 2)2

C(s)
----- W

ii.

Figure Q2

a) What is the range of K for a stable system.

[3 marks]
b) Determine the gain of K for a zero steady-state error if the input unit step is applied to the
system.

[5 marks]
c) Obtain the system stability if K is varies 15% from the gain obtained in (b).

[5 marks]
d)

Design ITAE system if the proportional gain K is changed to the PID controller so that
the peak time response to a step input is less than 2.5 seconds with suitable damping
ratio.
[12 marks]

(SEI4123)

Question 3

1
8
a) A closed-loop unity feedback system has G(s) =-----------------where b = 5, nominally. Determine the
s(s + b)
sensitivity of the closed-loop transfer function T(s) to changes in b.
[7 marks]
b) Consider a unity feedback control system as shown in Figure Q3 where the system
characteristic equation is given by
S + CljS 0\S + Q q = 0

and the variation of parameters of the nominal characteristic equation are as in Table Q3.
Table Q3: Variation of parameters
Variables

Minimum

Maximum

ai

ao

Under nominal condition, the controller and the plant transfer function is represented by
G (s)G(s) = -------- --------s(s + 2)(s + 3)

i)

Using the Routh-Hurwitz criterion, determine the stability of the system under nominal
condition.
[3 marks]

ii) Determine the four worst-case polynomials due to the variations in the parameters of the
nominal characteristic equations. [3 marks]

'5

(SEI 4123)

iii) Determine the stability of the system under the presence of the uncertainty in the nominal
characteristic equation.

[12 marks]

Figure Q3

6
(SEI 4123)

Question 4

Antilock Braking Systems (ABS) is operated by regulating the wheel slip, thus this action will
maximize the coefficient of friction between the tire and road for any road surface. The
simplified model of ABS is represented by a plant transfer function G(s) with a system as shown
in Figure Q4 with
G( S ) =^1=--------------- -------U(s) (s + a)(s + b)

where normally a = 1 and b = 4.


a) Using a PID controller, design a robust system where for a step input, the overshoot is
less than 4% and the settling time is 1 second or less. The steady-state error must be less
than 1% for a step input. It is expected that a and b to vary by 50%.

[12 marks]
b) Design a system using an ITAE performance index as given in Table Q4 and determine
the suitable pre-filter to improve specification in part (a).
[10 marks]
c) Estimate the overshoot and the settling time for the design described in (b).

[3 marks]

D(s)

Figure Q4

(SEI4123)

Question 5

Figure Q5 shows a control system with internal mode control scheme. The system is controlled
by a PID controller given as follows
G c (s) =

K 3 s 2 + K 2 s + K,

Figure Q5

a) Show that y(t) will track r f t j asymptotically in the steady-state mode.

[5 marks]
b) Design a state variable feedback and a PID compensator, to track a step input with zero
steady-state error. The system response due to unit step input must achieve a settling
time (to within 2% of the final value) in less than 1 second and a deadbeat response.
Assume that the two poles of the plant can change by 50%.
[20 marks]

(SEI4123)

Table Q4: ITAE Criterion for a Step Input


S + U)

+ 1.4ojv +

&I

r + 1.75ft;,r + 2,15 + a ^
+ Ha,,?' + 3.4<a',r + 2.7a>is + m*
5
.r + 2.8u> n s 4 4- 5.0<^? + SSw^s 2 + 3Acots + <4
a' + 3.25 <u.v + 6.60 + 8.60a>;^r + 7.45 + 3.95 + <tf

v4

Table Q5: Coefficients and Response Measures of a Deadbeat System


System

Order

Coefficients

e PO

PU

Tr90

0.10%

0.00%

3.47

6.58 4.82

1.65%

1.36%

3.48

4.32 4.04

0.89%

0.95%

4.16

5.29 4.81

Tr

Ts

2nd

1.82

3rd

1.90

2.20

4th

2.20

3.50

2.80

5 th

2.70

4.90

5.40

3.40

1.29%

0.37%

4.84

5.73

6th

3.15

6.50

8.70

7.55

4.05 1.63%

0.94%

5.49

6.31 6.04

5.43

CONFIDENTIAL

UTM
UNIVERSITI TEKNOLOGI MALAYSIA

FINAL EXAMINATION SEMESTER II


SESSION 2010/2011

COURSE CODE

SEP 4253

COURSE NAME

MEDICAL TOMOGRAPHY

LECTURERS

ASSOC. PROF. IR. DR. ING. EKO SUPRIYANTO

PROGRAMME

SEP

SECTION

01

TIME

2 HOURS 30 MINUTES

DATE

27 APRIL 2011

INSTRUCTION TO CANDIDATE

THIS QUESTION PAPER CONSISTS OF FOUR


(4) QUESTIONS. ANSWER ALL QUESTIONS IN
THE ANSWER BOOKLET PROVIDED. BEGIN
EACH ANSWER ON A NEW PAGE.
CANDIDATES ARE PROHIBITED TO BRING
ANY MATERIAL INTO THE EXAMINATION
HALL UNLESS PERMITTED TO DO SO BY
CHIEF INVTLIG AT OR.
DO NOT TURN THIS PAGE UNTIL YOU ARE
TOLD TO DO SO.

THIS EXAMINATION BOOKLET CONSISTS OF 7 PAGES INCLUDING THE FRONT COVER

SEP 4253

Question 1
Single element ultrasound transducer is used to measure the red blood cell velocity in the
depth of 2 mm from transducer as shown in figure 1. The blood cell has form of a ball
with diameter of 0.05 mm. Measurement result shows that the red blood cell velocity is
10 cm/s. Doppler angle, 0 is 60.

Figure Question 1: Blood flow measurement

The acoustic properties of related tissue are listed in Table 1


Table 1 Tissue acoustic property

Sound Velocity (c)

Acoustic Impedance (Z)

(m/s)

(MRayls)

Skin / Muscle

1580

1.7

1.2

Water

1540

1.5

0.0022

Blood

1570

1.6

0.18

Tissue

Attenuation (dB/cm)

3
SEP 4253
A piezoceramic (PZT) with thickness of 0.35 mm is used to send and receive ultrasound waves.
Physical properties of this material are listed in Table 2. The transducer is a tube with diameter
of 5 mm.
Table 2 Physical property of PZT

Parameter
Density
Young Modulus (Stiffness)
Acoustic Impedance

Value
7 x 103 kg.m'3
8.55 x 1010 Nm2
10 MRayls

Quality factor

22

Electromechanical coupling coefficient

0.4

(a) Determine the resonance frequency (fr) of the transducer.


(2 Marks)
(b) If the electric power applied to the transducer is 1 W, calculate the output power of
transducer for the reflection due to the difference acoustic impedance between skin and
blood vessel.
(7 Marks)
(c) If a continues wave is applied to the transducer with frequency fr, calculate the output
frequency of transducer due to the blood flow with velocity of 10 mm/s.
(3 Marks)
(d) If a pulse wave with pulse width l/fr is applied to the transducer to measure the red blood
cell velocity in the depth of 2 mm from the transducer, find the maximum Pulse Repetition
Frequency (PRF) to measure the blood cell velocity.
(4 Marks)

4
SEP 4253

Question 2
Figure 2, 3,4 and 5 are pictures of a 4T MRI machine and its important parts.

Figure 2: MRI Machine

Figure 4: Gradient Coil

Figure 3: Permanent Magnet

Figure 5: Radio Frequency Coil

5
SEP 4253

(a) Sketch the block diagram of the MRI machine

(4 Marks)
(b) Explain the function of Magnet, Gradient Coil and RF Coil

(6 Marks)

(c) Explain the work principle of MRI machine starting from the generation of high field
magnet up to display the image.

(4 Marks)

SEP 4253

Question 3

(a) Explain the working principle of Positron Emission Tomography (PET).


(4 Marks)

(b) State the advantages of PET scan compared to CT Scan.


(2 Marks)

(c) Mention two sensors for PET detector.


(2 Marks)

7
SEP 4253

Question 4
a) In order to protect people from ionizing radiation, or to calculate the risk/benefit ratio for
exposing a patient to possibly necessary radiation, it is obviously necessary to measure
radiation to which the subject is exposed. The unit of exposure used to be the roentgen
(R) that was defined as The quantity of radiation that will release an electrical charge of
...(a)... coulombs in one kilogram of dry air. This is equivalent to about ...(b)..............................
electrons. A more useful measure is derived from the concept of radiation dose, which
describes the dose of radiation absorbed by human tissue. The unit of dose is the gray
(Gy). A dose of 1 Gy means that...............(c)........eV of energy have been absorbed in 1 kg
of tissue. The unit of dose equivalent is that dose that gives the same risk of damage or
detriment to health caused by any type of radiation. This unit is called Sievert (Sv). The
International Commission on Radiological Protection (ICRP) recommends maximum
annual dose equivalent for radiation workers as ....(d)...................mSv, with a 5-year average
less than ...(e).... mSv per year. Larger doses are allowed to specific body parts. For
members of the public, the recommended whole-body dose is.................. (f).......mSv averaged
over 5 years.
(6 Marks)
(B) List and describe three biological effects of ionizing radiation accordingly
(6 Marks)

CONFIDENTIAL

UTM
UNIVERSITI TEKNOLOGI MALAYSIA

FINAL EXAMINATION SEMESTER II


SESSION 2010/2011

COURSE CODE

SEL 4233

COURSE NAME

MICROELECTRONICS I

LECTURERS

PROF. DR. RAZALI BIN ISMAIL


DR. NIHAD K. ALI AL-OBAIDI

PROGRAMME

SEL / SEW

SECTION

01-02

TIME

2 HOURS 30 MINUTES

DATE

28 APRIL 2011

INSTRUCTION TO CANDIDATE

ANSWER FOUR (4) QUESTIONS ONLY.

THIS EXAMINATION BOOKLET CONSISTS OF 9 PAGES INCLUDING THE FRONT COVER

SEL 4233
2
Ql. (a) Draw the energy band diagram of a compensated semiconductor
showing clearly the followings :
(i)

thermal electrons and holes,

(ii)

ionized and unionized donors,

(iii)

ionized and unionized acceptors,

(iv)

donor electrons and acceptor holes,

(v)

donor electrons recombine with acceptor holes.

(10 marks)

(b)

Sketch the plot of the position of Fermi level as a function of:


(i)

donor concentration (n-type) and acceptor concentration (ptype),

(ii)

temperature for both n-type and p-type.

In both of the above cases, give some explanation of the plot.

(5 marks)

(c)

A Si sample at T = 450 K is doped with 1.5 x 1015 cm'3 boron and with
arsenic at a concentration of 8 x 1014 cm'3.
(i)

Is the material n or p type ?

(ii)

Determine the electron and hole concentrations.

(iii)

Calculate the total ionized impurity concentration.

(10 marks)

SEL 4233
3
Q2.

(a)

What

do

you

understand

by

mobility?

Give

mechanisms

which

affect mobility value. Explain clearly how these mechanisms influence


mobility.

(10 marks)

(b)

The resistivity of n-type material is typically smaller than the


resistivity of comparably doped p-type material. Explain why.

(5 marks)

(c)

Consider silicon at T = 300 K. A Hall effect device (Fig. Q2(c)) is


fabricated with the following geometry : d = 5 x 1 O'3 cm, W = 5 x 1 O'2
cm, and L = 0.50 cm. The electrical parameters measured are Ix = 0.50
mA, Vx = 1.25 V, and Bz = 650 gauss = 6.5 x 10'2 tesla. The Hall field
is Eh= -16.5 mV/cm. Determine
(i)

The Hall voltage,

(ii)

The conductivity type,

(iii)

The majority carrier concentration,

(iv)

The majority carrier mobility.

(10 marks)

Fig. Q2(c)

SEL 4233
4
Q3. (a)

An n-type silicon sample contains a donor concentration of Nd = 1016


cm' . The minority carrier hole lifetime is found to be xpo = 20 (is.
(i)

What is the lifetime of the majority carrier electrons ?

(ii)

Determine the thermal equilibrium generation rate for electrons


and holes in this material.

(iii)

Determine the thermal equilibrium recombination rate for


electron and holes in this material.

(8 marks)

(b)

Impurity concentrations of Nd = 1015 cm'3 and N a = 6 xlO15 cm'3 are


added to silicon at T = 300 K. Excess carriers are generated in the material
such that the steady-state concentrations are 8n = 8P = 2 x 1014 cm'3.
(i)

Find the thermal equilibrium Fermi level with respect to Epi.

(ii)

Calculate E F and E f p with respect to E F l .

(6 marks)

(c)

In a silicon semiconductor material at T = 300 K, the doping


concentrations are N d

1015 cm"3 and N a

0. The equilibrium

recombination rate is R p q = 1011 cm'3-s'. A uniform generation rate


produces an excess-carrier concentration of 8n = Sp = 1014 cm'3.
(i)

By what factor does the total recombination rate increase ?

(ii)

Find the excess-carrier lifetime.


(5 marks)

(d)

Describe the concept of excess generation and recombination.

(6 marks)

SEL 4233
5
Q4. (a) A silicon abrupt junction in thermal equilibrium at T = 300 K is doped
such that; E c - Ep= 0.2 eV in the n region and Ep - E v = 0.18 eV in the
p region.

(i)

Draw the energy band diagram of the pn junction.


(ii)

(iii)

Determine the impurity doping concentrations in each region.

Determine V^.

(12 marks)

(b)

Draw the basic structure of the pn junction showing the space charge
region. Drawing should be labelled completely with major notations (space
charge region, electric field, and the force acting on the charged carriers).

(6 marks)

(c)

Describe why and how the space charge region is formed in pn


junction.

(7 marks)

SEL 4233
6

Q5. (a) Draw the energy band diagram of a zero-biased, forward-biased and
reverse-biased pn junctions. Drawing should be labelled completely
with major notations.

(9 marks)

(b)
An n + p silicon diode junction with a cross-sectional area of 104 cm2
has the following properties at 7=300 K :

N<r 10l8cm'J
T-no T'po- 10 S
D = 25 cm2/s

N a = 10lb cnr3
D p = 10 cm2/s

Determine the diode current for:


(i)

A forward-bias voltage of 0.2 V.

(ii)

A reverse-bias voltage of 0.2 V.

(8 marks)

(c)

Explain briefly the two physical mechanisms by which the reverse-bias


current increases rapidly in a pn junction.

(8 marks)

SEL 4233
7
EQUATION SHEET

h = 6.626 x 1 O'34 J - s
h = 1.055 xl0~34 J-s
q = 1.602 x 10',9C
eV = 1.602 x 1 O'19 J
m0 = 9.11 x 10'31 kg
k = 8.617 x 10'5 eV/K
*2k2
E = + V
2m*

3/2
*1
kT"T^

Nc = 2

2nh 7
f

*1kT
t-^3/2

mp

N =2

27i h 2

(im*m
* V/2
n p j exp

np = 4

np = nj

K 2%h,

n-Nd+ -p + N =0

I * * V 3/4e X
KmJ p/ P

n, =2
fE-Ef
1 + exp
v kT~ ~ j
ni

f(E):

f(E)=-

exp

= VNCNv

exp
v 2kT

2kT

E-E f
f

kT

c(E) =

m
E _ Ec + E 3
+ kT In p
2 4
vmv

2*3

71 h

E.-E)
g.(E) =

n = Jgc(E)f(E)f(E)dE
1/2

p = |gv(E)(l-f(E))dE

Tt2/?.3
n =n ; exp

_ dV 1 dE

E f ~E,
kT

dx q dx
f\

Ef = Ec + kT In

n = N c exp

(E -E ^

kT

vN c y
f\

E f = E v +kTln

p = n f exp

_P

vN v y
p = N v exp

E f = E c +kT

kT

_1__n_
In
+
vNcy V8 N c

E v -E f
kT

+ nE f =E v +kT

In
+
V8 Nv
vNvy

SEL 4233
8
r N -N
iNA 1N D

D _ kT

n q
a = nq|in + pqu

dn
J

=q^nF + qDn

ex.

dx

nr
dp

J P =q^ p nF + qD p
dn
15J
= -^ + (0-1*
at q d x

dx

5Ap n 5 2 Ap n Ap n

^r-D'^_Tr+G
,

d An 5 2 An n An n
P - D------- ^-------- - + G
" a.,2
dt
dx"

L P= a /D

PTP

kT. " n d N a x
V b l = In
q

w= /2eV>'

"na+Ndn
v NaNd ,

n, = 2.5x10

19

* *
m n mP

vmo mo

Nc =

2.5xl019

| (T

N =2.5x10

f T? \

exp
v 2kT y

V300y

{*

\3/2 , \ 3/2

cm

v300 y

Vmo J

19

\ V4 r j ^3/2

x3/2

Vmo /

3/2

cm -3

JL"
V300

cm -3

+ n

SEL 4233
9

Table of properties of selected semiconductors (at 300K)


Property
(cm' J )

Nc
N v (cm' 3 )
nj (cm' J )
E g (eV)

mn7m0
mD7m0
r
7. (eV)

Si

Ge
10 iy

2.78 x
9.84 x 10 1S
10 IU
1.12
1.18
0.81
11.8
4.05

GaAs
10 iy

1.04 x
6.0 x 10 1!5
2.3 x 10 1J
0.66
0.55
0.36
16
4.0

4.45 x 10 r/
7.72 x 10 18
1.8 x 10 b
1.42
0.067
0.52
13.1
4.07

CONFIDENTIAL

UTM
UNIVERSITI TEKNOLOGI MALAYSIA

FINAL EXAMINATION SEMESTER II


SESSION 2010/2011

COURSE CODE

SEI 4153

COURSE NAME

TRANDUCER & APPLICATIONS

LECTURERS

DR. HERLINA BT ABD RAHIM

PROGRAMME

SEI/SEP

SECTION

01

TIME

2 HOURS 30 MINUTES

DATE

05 MAY 2011

INSTRUCTION TO CANDIDATE

ANSWER FOUR (4) QUESTIONS ONLY.

THIS EXAMINATION BOOKLET CONSISTS OF 6 PAGES INCLUDING THE FRONT COVER

SEI4153

Question 1
A inear variable differential transformer (LVDT) is normally used to measure small linear
displacement of object.
a. Please give an example and explain how this transducer is used in a paper industry
application and ballistocardiography measurement.

(6 M)

b. A cross section of a balanced LVDT is shown in Fig. 1.1 and the input voltage is
shown in Fig 1.2. The maximum voltage output is 80% of the input voltage. Vout
i.

Sketch Vout when the core is fully moved to the left

(2 M)

ii.

Sketch V0ut when the core is fully moved to the right

(2 M)

iii.

The core is moving on the x plane slowly for 20 seconds. Plot the position of
the core in x plane versus time when the Vout is as shown in Fig. 1.3
(4 M)

c.

Differential variable reluctance transducer (DVRT) is another inductive transducer


used for micro range linear displacement measurement. Explain what is the major
difference between DVRT and LVDT?

(7 M)

d. State the advantages and disadvantages of an LVDT used for linear displacement
measurement.
(4 M)

3
SEI4153
Question 2
a. In a motor drive experiment, an optical rotary transducer is used to measure the rotation
speed of the shaft movement. The motor shaft will move continuously in one direction.

i.

What optical transducer can be used in this experiment? (1 M)

ii.

Explain how this transducer can be used to measure the rotational speed of the

iii.

shaft by giving an example?

(6 M)

What is Gray Code and why is it important?

(3 M)

b. A Wheatstones Bridge is used to measure the change of resistance. A rotary


potentiometer is connected to a Wheatstone bridge as shown in Figure 2.1.

i.

Find Rpot if the bridge is balanced, using current and voltage analysis

(4 M)
ii.

What is the Rpot if V j n = 10 V, Ra= 25 kQ, Rb= 15 kQ and Rc= 35 kQ

(2M)
c. A resolver is used for rotary displacement measurement.

i.
ii.

Draw and label the schematic diagram of a resolver .

(3 M)

Explain the construction and operation of the resolver to be used for rotary
displacement measurement.

(6 M)

4
SEI4153
Question 3
a. A load cell or pressure cell is normally used for measuring weight of heavy loads.

i.

Explain how a load cell is used to measure the weight of heavy loads.

(6 M)
ii.

Define Stress, Strain and Gauge Factor?

(6 M)

b. Strain gauge can be used to continuously monitor the surface deflection of bridges.
However, due to outdoor environmental issue, the reading of a strain gauge is affected.

i.

What is the disadvantages of this resistive sensor for outdoor strain


measurement?

(2M)
ii.

What is the solution to obtain an accurate reading for strain or surface

(5M)

deflection of the bridge?


iii.

A gauge made of a material having a resistance temperature coefficient of 12 x


10'4 / C, has a resistance of 150 Q. and a gauge factor of 2. It is connected to a
Wheatstones Bridge having resistance of 150 Q each. The bridge is balanced
at ambient temperature. If the temperature changes by 20 C, find the
equivalent
temperature.

strain

in

ambient

temperature

represented

by

the

change

(6M)

in

5
SEI4153
Question 4
a.

You are the Process Engineer of KIKO fabric factory. You are assigned to prepare 20
liters of color dye per hour and send to the printing room for the newly designed fabric.
There are a few steps to follow in order to ensure that the dye quantity and quality is
following the requirements and the output is the same for every batch. The recipes and
procedures of preparing 20 L dye are:
1. 500 kg of each Red and Blue powder are mixed in a tank
2.

The tank is then added with precisely 18 L of warm water at 60 C and mixed gently
for precisely 10 minutes.

3.

Then 1 L of acid solution is added slowly to the mixture and mixed for another 5
minutes.

4. Then the heating element gradually heated up the dye to 80 C.


5.

Once the dye solution reaches the desired temperature, the heating element will be
turned off and the tank will be transferred to the printing room via a conveyor belt.

i.

According to the procedures mentioned above, what are the measurement


parameters and your suggested type of transducers in order to prepare this dye?

(6 M)
ii.

The dye mixture is highly stained and is very difficult to clean unless using strong
acid. Suggest and explain the operational principle of a temperature transducer that
you would use in the production line. Why?

b.

(5 M)

Selecting suitable transducers for your production line is very important to promise good
production throughout the manufacturing period.

i.

What are the criterions need to be considered when selecting suitable transducer
for a production line?

ii.

(8 M)

Define and give an example of transducer type self generator, modifier and
modulator.

(6 M)

SEI4153
Question 5

Your company is managing a few cooling districts in Malaysia. One of your company liquid
cooling districts that provide daily cooling services to MENARA KL and TNB tower is
located 15 km away from the city centre. The clear chilled water (-10 C with 6.67 |j.S/cm) is
pumped and distributed to the whole MENARA KL and TNB tower through underground
pipelines. Flow transducers are used in order to monitor the flow rate of cool water and also to
ease identification of fault along the pipeline.

i.

Suggest two (2) possible flow transducers that can be used in the pipeline to measure
the amount of chilled water usage of the MENARA KL and TNB tower. (4 M)

ii.

Choose one of the most suitable flow transducer and explain the operational principle
of the flow transducer in general flow measurement and the pros and cons of this
transducer.

iii.

(5M)

Explain why you choose the flow transducer mentioned in Q5(ii), how it can be
installed and used to measure the flow rate as well as monitoring the fault in the
pipelines at the same time.

iv.

(4 M)

The chilled water is stored in pressurized tanks at the cooling district, the level of each
storage tank of the chilled water have to be monitored at all time to ensure no leakage
or faulty occured between the distributor and clients. You are assigned to find the most
suitable level transducer and installed it to 10 storage tanks in the storage hall. The
level of these 10 tanks can be monitored from the control room near the entrance of
the storage hall. Your manager would like to see a brief report and comparison of
three (3) different types of level transducers that are possible to use in this case and
which one is the best among the others.

(12 M)

CONFIDENTIAL

UTM
UNIVERSITI TEKNOLOGI MALAYSIA

FINAL EXAMINATION SEMESTER II


SESSION 2010/2011

COURSE CODE

SEE 4133

COURSE NAME

INDUSTRIAL CONTROL TECHNOLOGY

LECTURERS

DR. SOPHAN WAHYUDI BIN NAWAWI


MR. MOHAMAD SHUKRI BIN ABDUL MANAF

PROGRAMME

SEE / SEI

SECTION

01 02

TIME

2 HOURS 30 MINUTES

DATE

06 MAY 2011

INSTRUCTION TO CANDIDATE

ANSWER FOUR (4) QUESTIONS ONLY.

THIS EXAMINATION BOOKLET CONSISTS OF 7 PAGES INCLUDING THE FRONT COVER

2
SEE4133
Question 1
a) Discuss four basic principles of valve which is normally used in process control.
(4 Marks)
b) The liquid from tank Po is transferred to a heat exchanger by using a single pump and
past throw the valve with a rate of 1 OOgpm as shown in Figure Ql. The change in
pressure inside the heat exchanger is 30 psi. Plot the flow rate versus position (x) for the
linear and equal percentage control valve (a=50). The operation for both valves at fx
(fraction of the total flow area) is equal to 0.5. Assume the total pressure drop for whole
system is constant. The change in pressure rate at the valve is 40 psi
(6 Mark)

c) Discussed briefly about the Relay.


(4 Marks)

d) A lighting control system is to be developed. The system will be controlled by four


switches, SWITCH1, SWITCH2, SWITCH3, and SWITCH4. These switches will control
the lighting in a room based on the following criteria:
i.

Any of three of the switches SWITCH1, SWITCH2, and SWITCH3, if turned ON


can turn the lighting on, but all three switches must be OFF before the lighting
will turn OFF.

ii.

The fourth switch SWITCH4 is a Master Control Switch. If this switch is in the
ON position, the lights will be OFF and none of the other three switches have any
control.

iii.

Design the wiring diagram for the controller connections, assign the inputs and
outputs and develop the ladder diagram which will accomplish the task.
(11 Marks)

3
SEE4133
Question 2
a) A programmable logic controller (PLC) or programmable controller is a digital
computer which is used for automation. Briefly discussed:
i. The purpose of a Programmable Logic Controller
ii. The features of a Programmable Logic Controller
iii. Two steps that PLC must perform during operation
iv. The procedure for solving a rung of logic
(10 marks)
b) The PLC program is executed as part of a repetitive process. By using an appropriate
figure, explain:
i. 11
Scan Cycle ?
ii. If a PLC program is 7.5K long and the scan rate of the machine is 7.5
msec/K, what will the length of time between I/O updates be?
(10 marks)
c) By referring to the ladder diagram in figure Q2 , list down all the conditions to
energize CR1.
(5 marks)

CR7 CR6

CR1
()

CR2 CR5
CR1 CR4 CR3

Figure Q2: Illustration of allowed current flow in a ladder rung

4
SEE4133
Question 3
Process

Regulated

Voltage

Actuator
Feedback Loop

Error
Signal

Correction
Signal

Processor

1
JVfewofy i
Controller

Variable
Amplitude

Power to
^/Actuator

Power
Driver

Power

Set Point

Figure Q3
a) Figure Q3 shows the block diagram of the elements that make up the feedback path
on a process control loop, give the definition of the blocks listed below:
i.
Feedback loop
ii.
Set point
iii.
Sensors
iv.
Actuator
(6 Marks)
b) There are situations in which a PLC must make a decision based on the results of the
majority of inputs. Let us assume that a PLC is monitoring five tanks of liquid and
must give a warning light to the operator when;
i.

Three of them are empty. It doesnt matter which tanks are empty, only that
any three of the five are empty.
ii. Four or five empty tanks but as we will see, those cases will be automatically
included when we design the system for three empty tanks above.
iii. If the empty tank is four or more, it will give a blinking light together with a
warning light.
iv. You can label the tank with A, B, C, D and E.

Based on this situation:


i.
ii.
iii.

Draw a state diagram.


Write down a Boolean expression.
Draw the ladder diagram.
(19 Marks)

5
SEE4133
Question 4
a) Draw schematic diagrams and explain in brief the control systems below together with
one real application:
i.
ii.
iii.

Feedback and feedfoward control


Ratio control
Bypass control

(3 Marks)
b) List the types of memory address table in PLC. Briefly explain how we can write number
in memory address table of a PLC.
(6 Marks)
c) Draw a complete block diagram for general architecture of an embedded control system.
(8 Marks)
d) You are required to change the Data Memory table in a PLC to control the duration of
green light at four junction traffic light based on the following condition;
i.

Every junction has installed 3 sensors to detect Long, Medium and Less number
of cars.
ii. The timing diagram of duration is shown in the following table;
Junction 1, Sensor (SI, S2, S3)
Situation
Duration of green light
=
=
Long Queue
100 Second
SlA l, S2A 1, S3A=1
Medium Queue
70 Second
SlA = l, S2A = 1, S3A = 0
Less Queue
30 Second
SlA = l, S2A = 0, S3A=0
Junction 2, Sensor (SI, S2, S3)

S]B=1, S2B = 0, S3B=0

Situation
Long Queue
Medium Queue
Less Queue

Duration of green light


90 Second
60 Second
40 Second

Junction 3, Sensor (SI, S2, S3)


SlC = l, S2C=1, S3C=1
SlC = l, S2C=1, S3C=0
SlC=L S2C=0, S3C=0

Situation
Long Queue
Medium Queue
Less Queue

Duration of green light


80 Second
50 Second
20 Second

Junction 3, Sensor (SI, S2, S3)

Situation
Long Queue
Medium Queue
Less Queue

Duration of green light


95 Second
65 Second
35 Second

SlB = l,

S2B = 1,

S3B=1

S)B = 1, S2B = 1, S3B=0

SlD=l,

S2D=1,

S3D=1
Sid=1, S2D=1, S3D=0
Sid=1, S2D=0, S3D=0

iii.

Select appropriate address and draw a ladder diagram to write the above condition
in the memory address table of a PLC. You do not have to write a program to
control the sequence of red, green and yellow light of the Traffic light.
(8 Marks)

SEE4133
Question 5
a) Since SCADA systems are designed for reliability, availability and data integrity, extra
consideration must be given to confidentiality and authentication. Write down some
consideration to choose a SCADA for a process control.
(6 Marks)
b) A typical control system consists of one or more remote terminal units (RTU) connected
to a variety of sensors and actuators, and relaying information to a master station. A PLC
can be used as RTU. Discuss the connection between the PLC to the sensors and HMI for
the process in Figure Q5(a).
(7 Marks)

/PMh
^PIC>
(

,1.
FE

PT
^234j

rPSK'
<222j

..221.
50-HL-22

-L
PopcS
wed
Figure Q5(a)

Figure Q5(b)

7
SEE4133

You are required to control the following continuous process shown in Figure Q5(b).
Explain briefly the connection using PLC.
i.
ii.
iii.

As a PID Controller to control level and temperature.


The method to tune PID controller parameter suitable for the process
As RTU to control and interface for monitoring using SCADA for the process.
(12 Marks)

CONFIDENTIAL

UTM

UNIVERSITI TEKNOLOGI MALAYSIA

FINAL EXAMINATION SEMESTER II


SESSION 2010/2011

COURSE CODE

SEE 1223

COURSE NAME

DIGITAL ELECTRONICS

LECTURERS

DR. AZLI BIN YAHYA


DR. SHARIF AH HAFIZAH BT SYED ARIFFIN
DR. EILEEN SU LEE MING
MR. AMERUDDIN BIN BAHAROM
MR. MUHAMMAD ARIF BIN ABD RAHIM
MR. JAMEEL ABDULLA AHMED MUKRED

PROGRAMME

SEC / SEE / SEI / SEL / SEM / SEP / SET / SEW /


SWB

SECTION

01-05 /10

TIME

2 HOURS 30 MINUTES

DATE

07 MAY 2011

INSTRUCTION TO CANDIDATE : ANSWER ALL QUESTIONS IN SECTION A AND


THREE QUESTIONS FROM SECTION B. ALL
WORKING MUST BE SHOWN CLEARLY IN
THE ANSWER SHEET.

THIS EXAMINATION BOOKLET CONSISTS OF 12 PAGES INCLUDING THE FRONT COVER

-2SEE 1223

SECTION A [40 marks]

1. Convert

(10110)oray code tO

(i)

B CD.
[4 marks]

(ii)

5742g to hexadecimal.
[4 marks]

2.

The pin diagram of 7400 chip (quad 2-input NAND gates) is shown in Figure A(i).
Copy the diagram on your answer script and identify pin 1 and pin 14 as well as the
DC supply Vcc and GND pins on the diagram.
[4 marks]

3.

By using Boolean algebra,

(i)

simplify A.B + A.(B + C) + B.(B + C).


[3 marks]

(ii)

prove that A + B = A + A.B .


[3 marks]

4.

F(A,B,C,D) = n(l, 3, 5, 9, 13, 15) and don't care = D(2, 4, 7, 12, 14).

(i)

Obtain the minimized POS expression using the K-map.


[4 marks]
(ii)

Implement the minimized function in NOR-NOR configuration.


[3 marks]

-3 SEE 1223

5.

A medium scale integrated circuit (MSI circuit) accepts four inputs and produces two
outputs as shown in the Table A(i). X indicates dont care conditions. Give a suitable
name for this circuit.
[3 marks]

Table A(i)

6.

I3

I2

I.

Io

Oi

Oo

Figure A(ii) shows a 4-1 multiplexer with a D flip-flop. Copy the diagram on your
answer script and show how these two devices can be connected to become a J-K flipflop. The J and K inputs are connected to the select bits Si and So respectively. You
can't use any additional gates.
[4 marks]

O
O'

>
Qo

<?

Figure A(ii)

7.

Figure A(iii) is a logic symbol of a 4-bit parallel adder. If X [X3..Xo]= 1111 and
Y[Y3..Yo] = 0101, show how this chip can be used as a 4-bit subtractor X - Y. Copy
the diagram on your answer script and label the diagram with the input and output
values. [Hint: 2's complement = l's complement + 1]
[4 marks]

-5 SEE 1223

SECTION B [60 marks]

Question 1
(a)

A student is required to simplify a Karnaugh map. He tries to form groups of 1 s as


shown in Figure Ql(a)
CD
AB

\
00

,00, 01 11 10
0

11 J

01

[ 1

11

i i

10

x!

1 i |
1

! xj

0
1 j
1 j
0

Figure Ql(a)

(i)

Explain the mistakes done by the student.


[4 marks]

(ii)

Obtain the correct Boolean equation from the map.


[3 marks]

(b) The block diagram and the truth-table of a combinational logic circuit are shown in
Figure Ql(b) and Table Ql respectively.

Figure Ql(b)

-6SEE 1223

Table Q1

(i)

Obtain the minimized SOP expression for output P2 using Karnaugh Map.
[4 marks]

(ii)

Implement P2 in NAND-NAND configuration.


[3 marks]

(iii)

Implement Pi using one 4-to-l multiplexer only without any additional gates.
[4 marks]

(iv)

Suggest a suitable name for the combinational logic circuit operation.


[2 marks]

-7SEE 1223

Question 2

(a)

A 2-to-4 line decoder (active-HIGH output) and an OR gate can be used to realize the
function F(A,B) = EIM (1,3). By using your inspired creative and innovative minds,
show how this same function can be implemented using a 3-to-8 line decoder (activeHIGH output) shown in Figure Q2(a) with 2-input OR gates only. A and B should be
connected to the select bits S2 and Si of the decoder respectively.
[6 marks]

S2

Oo
0,
02

Si

03
04

So

05

o6
O7

Figure Q2(a)

(b) Consider a circuit shown in Figure Q2(b).

Figure Q2(b)

(i)

Identify the type of circuit, draw the logic symbol and write down the truth
table for this particular circuit.
[3 marks]

(ii)

Using the logic symbol drawn in part (i), construct the simplest positive edgetriggered J-K flip-flop. You may add additional gates, if necessary. Show also
the positive edge-triggering circuit and draw the logic symbol.
[3 marks]

(iii)

By using the logic symbol of J-K flip-flop in part(ii), show how this flip-flop
can be modified to make a D and a T flip-flops.
[2 marks]

Sketch the output waveforms Qi and Q2 for four clock pulses for the circuit shown in
Figure Q2(c). Qi and Q2 are initially 0.
[6 marks]

Q.

>

Q2

>
K
CLK

Figure Q2(c)

-9SEE 1223

Question 3

The state transition diagram of a counter is shown in Figure Q3(a)

(a)

Give two advantages of designing the counter using D flip-flops instead of J-K flipflops.
[4 marks]

CLK
Figure Q3(b)

- 10SEE 1223

(b)

The block diagram of the counter for the given state transition diagram is shown in
Figure Q3(b).

(i)

What is the purpose of the combinational logic circuit?


[2 marks]
(ii)

Design the combinational logic circuit using only 4-to-l multiplexers.


[12 marks]

(iii)

What is the advantage of using multiplexers instead of basic gates for the
combinational logic circuit?
[2 marks]

-11 -

SEE 1223

Question 4

(a) (i)

Show how a NOT function is implemented using each of the following gates;
2-input NOR gate, 3-input NAND gate and 2-input XOR gate.
[6 marks]

(ii)

Implement a 3-input OR function using minimum number of 2-input


NAND gates only.
[4 marks]

(b)

A ROM circuit to implement various logic operations is shown in Figure Q4.

(i)

What is the ROM size?


[2 marks]

(ii)

Explain how the ROM can be configured so that it can function as a 2-input
NAND gate and also 2-input XOR gate.
[4 marks]
Address
decoder

SEL-

S2

A-

Si

B-

So

ROM
matrix
On

1 - 0 - 0

o,

0 - 1 - 0

Ot
0,

OH i Ho
r
0 - 0
l

Oi

1 - 1 - 0

0,

iTTo>
^
rTHojz ^
oTT

0,
07

F2 Fi Fo

Figure Q4

(iii)

The ROM also stored the data for a 1-bit full-adder. Explain how you can
retrieve the data for the full-adder.
[4 marks]

-12SEE 1223

APPENDIX

BOOLEAN ALGEBRA

A+A=1

AA=0

Theorem 3

A+A=A

A.A = A

Theorem 4

A+l=l
ii

o
ll
o

<

IIC

Theorem 5

ii

Theorem 2

>

A+0=A

>

Theorem 1

>

Boolean's Theorem

Theorem 6

A+B=B+A

A.B = B.A

Theorem 7

A + (B + C) = (A + B) + C

A.(B.C) = (A.B).C

Theorem 8

A.(B + C) = A.B + A.C

A + B.C = (A + B).(A + C)

Theorem 9

A + A.B = A

A.(A+B)=A

Theorem 10

A + A.B = A + B

A.(A + B) = A.B

DeMorgan's Theorem
(i)
{ (ii) A + B = A.B

A.B = A + B

CONFIDENTIAL

UTM
UNIVERSITI TEKNOLOGI MALAYSIA

FINAL EXAMINATION SEMESTER II


SESSION 2010/2011

COURSE CODE

SEE 2113

COURSE NAME

CONTROL : MODELLING & SIMULATION

LECTURERS

ASSOC. PROF. ZAMANI BIN MD ZAIN


DR. ABDUL RASHID BIN HUSAIN
DR. ZUWAIRIE BIN IBRAHIM
DR. AHMAD ATHIF BIN MOHD FAUDZI
DR. MOHD RIDZUAN BIN AHMAD
DR. ZOOL HILMI BIN ISMAIL
DR. SHAFISHUHAZA BT SAHLAN

PROGRAMME

SEC / SEE / SEI / SEL / SEM / SEP / SET / SEW /


SWB

SECTION

01-06

TIME

2 HOURS 30 MINUTES

DATE

9 MAY 2011

INSTRUCTION TO CANDIDATE

ANSWER QUESTION 1 AND ANY OTHER


THREE (3) QUESTIONS.

THIS EXAMINATION BOOKLET CONSISTS OF 14 PAGES INCLUDING THE FRONT COVER

-2-

SEE 2113

Question 1
(a)

Figure Qla shows the block diagram of a control system. Name the block diagram
components W, X, J, M, L and Z.

L
M

Figure Qla

(3 marks)

-3SEE 2113
(b)

Find the transfer function 6i(s)/E a (s), for the electromechanical system shown in Figure
Qlb. The following values are given:
Jl = 600 kg m2
N\ = 10
Dl = 900 Nms/rad

(10 marks)

Tm

Figure Qlb

-4SEE 2113
(c)

A modified azimuth position control system to give different output performance is


as shown in Figure Qlc.
ivuhui anu luau

Power amplifier

VP(s) +
---------

EM

100

0.2083

e 0 (s)

w
(5 + 1.71)

(5 + 100)

Figure Qlc
(i)

Find the transfer function, 6 0 (s)/V p (s).

(2 marks)
(ii)

Under certain operating condition, we need to adjust the system so that it will
give a damping ratio of 0.707. Find the value of K and natural frequency
(con), needed to get the required damping ratio.

(4 marks)

(iii)

For the values obtained in part (c)(ii) above, calculate the corresponding
settling time, peak time and percentage overshoot.

(4 marks)
(iv)

Sketch and name the output response in this case.

(2 marks)

-5SEE 2113
Question 2
(a)

Define control system. Give ONE example of a feedback control system of your
choice.

(3 marks)
(b)

Give FOUR (4) benefits of control systems. Support your answer by giving one
example (real application) for each benefit.

(6 marks)
(c)

Figure Q2c shows THREE (3) unit step responses. Response B is the
desired/reference performance. Discuss the step responses A and C in terms of their
transient response, steady-state response and stability. Comparisons with the
desired response should also be made.

(6 marks)

-6-

SEE 2113
(d)

Diabetes mellitus, or more commonly referred to as diabetes, is a group of


metabolic diseases in which a person has high blood sugar.
This is due to the body does not produce enough insulin (Type 1 diabetes), or
because cells do not respond to the insulin that is produced (Type 2 diabetes).
For the Type 1 diabetes, which results from human bodys inability
insulin, it requires the patient to inject the insulin into the body. Current
developing miniaturized sensors have meant that it is now possible to
automated insulin delivery system. This system monitors blood sugar
delivers an appropriate dose of insulin when required.

to produce
advances in
develop an
levels and

A software-controlled insulin delivery system works by using a micro-sensor


embedded in the patient to measure some blood parameter that is proportional to the
sugar level. This signal is then sent to the pump controller. This controller computes
the sugar level and the amount of insulin that is sufficient for the body needs. It then
sends signals to an amplifier and driver circuit to drive a miniaturized pump to
deliver the insulin via a permanently attached needle.
Draw the functional block diagram of this automated insulin delivery system.

(10 marks)

-7SEE 2113
Question 3
(a)

(i) What is the definition of transfer function and what assumption is made
concerning the initial conditions of the system when dealing with transfer
functions?

(2 marks)
(ii)

Figure Q3a
A simple passive-element testing rig is shown in Figure Q3a. A constant 1-A
input current is supplied to three passive elements, A, B and C producing
three corresponding output voltages:
Element A - a constant ramp voltage
Element B - 0 V
Element C - a constant voltage of 3 V
From the voltage reading, determine the passive elements A, B and C. Also,
find the discrete value of element C.
(4 marks)

-8-

SEE 2113

^ Determine the transfer function, ^ , of the system shown in Figure Q3b.

(9 marks)
Vo

(c)

6 (s)
Given the rotational system in Figure Q3c, find the transfer function, G(s) =
06O)

(10 marks)

-9SEE 2113
Question 4
(a)

(i) Name THREE (3) basic forms for interconnecting subsystems.

(3 marks)
(ii)

The Mason gain formula is given as

C(s) _ X k T k A k
R(s) A
If a forward path, 7*, touches all closed loops, what would be the value of
Ak7

(2 marks)
(b)

Obtain the transfer function of the block diagram shown in Figure Q4b by using the
block diagram reduction technique.

(10 marks)

Figure Q4b

- 10SEE 2113
(c)

(i) Draw the corresponding signal flow graph for the block diagram shown in
Figure Q4c(i).

(3 marks)

Figure Q4c(i)

(ii)

Find the transfer function C(s)/R(s ) using Mason Rule for the signal flow
graph shown in Figure Q4c(ii) below.

(7 marks)

Figure Q4c(ii)

-11 SEE 2113


Question 5
(a)

Explain the significance of the following terms:


(i) Time constant, r, in the transfer function of a first-order system
(ii) Damping ratio, , in the transfer function of a second-order system
(iii) Undamped natural frequency, a) n , in the transfer function of a second-order
system

(5 marks)
(b)

For each of the transfer functions listed below, draw an s-plane diagram indicating
the location of system poles. Then, in ONE graph, sketch the unit step responses
for all the transfer functions. Label the response for each transfer function clearly.
(i)

C(s)
10
2
R(s)~ s -1

(ii)

C(s)
10
2
R(s) s + 3s + 2

(iii)

C(s)
10
R(s) s 2 + 2s + 2

(6 marks)
(c)

An electrical network in Figure Q5c has a transfer function of


K(s)

R,

V,(s) (R 2 + R, )CLs + (CR 2 R ] + L)s + R x

KliMAVi
+
1KQ

C vc(t)

Figure Q5c
Find the values of Ri and C to yield a 20 % overshoot and 20 ms settling time for
Vc(t).

-12SEE 2113

(8 marks)
(d)

Figure Q5d shows the unity feedback system, where

(s + 2) (s+3)

R(s) + E ( s )

G(s)

C(s)

Figure Q5d
(i)

Find the system type.

(2 marks)
(ii)

What error can be expected for an input signal of r(t) = 10u(i)7

(2 marks)
(iii)

What error can be expected for an input of r(t) = 10tu(t)l

(2 marks)

- 13SEE 2113

TABLE OF LAPLACE TRANSFORMS


?, L ** ~ 'r * & i V'' <
P * 3*-.
\ jpy u 1.'' u)' t
i - (S) 1 \ f i
f(t)
1
Unit impulse, 6(0
A
A
r

e"*

s
1

te' a<

s+a
1

0 + a?
1

tn

s2
n\
s n+1
CO
2
S +2CO
s
22
S +00
(0
2
2
S -CO
s

sin a)t
cos cot
sinh cot
cosh cot

s 2 -co 2
sF(s)-f(0)

dm
dt
d"f{t )

s n F(s) - s'" 7(0) - s n - 2 f (l) ( 0)... - f nA) (0)

df
F(s )
]f{t)dt
0

s
,d M F(s )

tf{t),n = 0,1,2,-..
1

ds M

e- al sinwt

e- at koswt

(s - a f - w 2
s-a
(s-aj - w2

-14SEE 2113

LAPLACE TRANSFORMS THEOREMS

Item no. Theorem

Name
r a>

1.

Wit)) = F(s) = f(t)e- st dt

Definition

JO-

2.

mm] = kF(s)

Linearity theorem

3.

% [ f i ( 0 + M t ) ) = F\ ( s ) + F 2 (s)

Linearity theorem

4.

<"/(/)] = F(s + a)

5.

c Af{t

6.

%[f(at)] = 1 - f I~)

7.

~ T)} = e~ sT F(s)

dt

Time shift theorem


Scaling theorem

= sF(s) /(0-)

Differentiation theorem

= s 2 F(s) - 5/(0-) -/(0)

Differentiation theorem

1 1

8.

Frequency shift theorem

1 1

...............

9.

= s n F(s) - s n ~ k f*~\ 0-)


k= 1

Differentiation theorem

'1

10.

[ A t ) dr
Jo-

5*1
s

Integration theorem

11.

/(-)

= lim sF(s)
s-* 0

Final value theorem1

12.

A o+)

= lim sF(s)

Initial value theorem2

1 For this theorem to yield correct finite results, all roots of the denominator of F ( s ) must have
negative real parts and no more than one can be at the origin.
2 For this theorem to be valid, f ( t ) must be continuous or have a step discontinuity at t = 0 (i.e.,
no impulses or their derivatives at / = 0).

CONFIDENTIAL

UTM
UNIVERSITI TEKNOLOGI MALAYSIA

FINAL EXAMINATION SEMESTER II


SESSION 2010/2011

COURSE CODE

SET 3573

COURSE NAME

MICROWAVE ENGINEERING

LECTURERS

DR. NOR HISHAM BIN KHAMIS

PROGRAMME

SET

SECTION

01

TIME

2 HOURS 30 MINUTES

DATE

13 MAY 2011

INSTRUCTION TO CANDIDATE

ANSWER FOUR (4) QUESTIONS ONLY.

THIS EXAMINATION BOOKLET CONSISTS OF 5 PAGES INCLUDING THE FRONT COVER

- S E T 3573 -2-

Q1 a. The general input impedance formula for an L-C transmission line is given by:
7 _ 7 _ _n _ 7 Zl +
1

tan

J0

Z 0 + jZ L tan m

Find an expression for Z when:


i.

the load is a short,

(3 marks)

ii.

the load is an open,

(3 marks)

iii.

the line is a half wavelength long line terminated by Z L , and (4 marks)

iv.

the line is a quarter wavelength long line terminated by Z L . (4 marks)

b.

An ideal transmission line is terminated by a resistance equal to half the

characteristics impedance of the transmission line, i.e., Z L = y. To eliminate


reflections back to the generator, what matching impedance must you use
i.

if you place the matching element in series, one quarter of a wavelength


from the load (towards the generator)?

ii.

iii.

(5 marks)

if you place the matching element in parallel, one quarter of a


wavelength from the load (towards the generator)?

(5 marks)

Comment on your the solution in (i).

(1 marks)

Q2 Using a Smith chart, find the lengths of the two stubs (in term of wavelengths) in
Figure Q2, to achieve matching. There are two possible solutions, give both of them.
All steps and calculations must be written in the answer books and all points must be
shown and marked clearly on the Smith chart.

(25 marks)

< 25S < '2^


ZL=(20 + j30)n

Figure Q2

- SET 3573-3-

Q3 Figure Q3a below shows a piece of transmission line with a leakage loss represented
by Z0.
a) Determine the scattering parameters of the transmission line. (12 marks)
b) Find the return loss and insertion loss of the transmission line. (4 marks)
c) If the transmission line is assumed to be lossless by ignoring the leakage loss,
i.e. leakage loss Zo is taken out, replaced and terminated with a complex load
impedance of Zl = (20 - jl5)Q (as shown in Figure Q3b), and the intrinsic
impedance of the line is 75Q, find
i. input impedance to the line,

(3 marks)

ii. the reflection coefficient at the input of the line,

(2 marks)

iii. the SWR at the input of the line, and

(2 marks)

iv. return loss at the input of the transmission line.

(2 marks)

Figure Q3a.

Zt=(20-jl5}Q

Figure Q3b.

- SET 3573-4-

A rectangular waveguide has a cross-section of 1.5 cm x 0.8 cm, cr= 0, // = fo,


and s s 0 . The magnetic field component is given as:
H = 2 sin () cos sin( n x 10"t - J3z) A/m
a
b2
Determine:
i.

the modes of operation,

(2 marks)

ii.

the cut-off frequency, fc

(3 marks)

iii.

the phase constant /?,

(4 marks)

iv.

the propagation constant y, and

(2 marks)

v.

the intrinsic impedance 77 for all modes in part i).

(6 marks)

If the waveguide is now filled with a dielectric of s = 4s 0 , determine the new


cut-off frequency.

(4 marks)

Give comment from part b).

(2 marks)

Two modes which have the same cut-off frequency are called what kind of
mode?

(2 marks)

i)

Define VSWR.

(2 marks)

ii)

Give an application of circular waveguide.

(2 marks)

ii)

State the coupling parameters of a directional coupler. (4 marks)

iii)

What are the desirable properties of the slow wave structure to be used
in TWT amplifier?

i)

(4 marks)

Name the device in Figure Q5 below and explain how it works.


(10 marks)

ii)

Describe the applications for such device.

Figure Q5

(3 marks)

- SET 3573 -5-

List of formulas:
y cc

+ jp -yj (R + jojV)(G + jo)C)


R + ja)L

Z0 =

G + jo)C

V = VfeJZ + Vre>*
* _ Xe^Xe^

I = Le^ + Le

Zn
yt

Vfe~^ Vfe

ZL cos /%? + jZ0 sin pi


Z0 cos pi + jZL sin pi

7=7

+ jZ0tan pi
+ jZLtan pi

/ \2
>2
11
1,1
+
UJ

2^1
vp

= Zf

1-

\=

A,
Q.

Q.
(OS

The Smith Chart

-^gjScTToN COEFFICIENT IN

CONFIDENTIAL

UTM
UNIVERSITI TEKNOLOGI MALAYSIA

FINAL EXAMINATION SEMESTER II


SESSION 2010/2011

COURSE CODE

SET 4593

COURSE NAME

ACOUSTICS ENGINEERING

LECTURERS

ASSOC. PROF. DR. MOHAMAD NGASRI BIN


DIMON

PROGRAMME

SET

SECTION

01

TIME

2 HOURS 30 MINUTES

DATE

10 MAY 2011

INSTRUCTION TO CANDIDATE

ANSWER ALL QUESTIONS.

THIS EXAMINATION BOOKLET CONSISTS OF 4 PAGES INCLUDING THE FRONT COVER

2
SET 4593

Ql. (a) Through proper mathematical equation, discuss properly the sound absorption
coefficient for an open window.
(4 marks)
(b)

Mathematicaly, show that the perforated wall panel able to act as sound absorber with
proper illustration, discuss the general trend of sound absorption performance of
porous material and perforated wall panel.
(4 marks)

(c)

Through proper illustration, discuss properly four (4) applications of sound absorber
in realising or contributing good acoustics primarily in an enclosed room.
(8 marks)

(d)

Mathematically, show and discuss the measurement of random incidence sound


absorption coefficient of an acoustics material.
(4 marks)

Q2. (a) As an acoustics engineer, you are required to suggest acoustic criteria for a secondary
classroom with 30 students. This class room is adjacent to the school principal office.
Based on the above.
(i)

Suggest 3 acoustics criteria requires attention to have good an functioning


classroom.
(5 marks)

(ii)

Suggest the Sound Transmission Coefficient (STC) for the dividing, wall
adjacent to the school Principal Office. Explain your suggested STC.
(4 marks)

(b)

The 700m3 empty untreated classroom RT60 is 1.5 second. The classroom is able to
accommodate 60 students in a wooden seat. Based on above
(i)

Discuss the empty RT60 quality being measured.

(2 marks)

(ii)

Suggest RT60 with 60 students suitable to enable the classroom to function


effectively.
(2 marks)

(iii)

Determine the additional amount of sound absorber in Sabine (A) required to


achieve optimum RT60 with 60 students.
(2 marks)

(iv)

Discuss the technique and requirement to determine the classrooms area


needed acoustics treatment.
(4 marks)

Discuss the followings

(i)
(ii)
(iii)
(iv)

SPL = 80 dB(A)
RT60 = 1.0 second
STI = 0.65
NRC = 0.60
(4 marks)

There is a sound source in the middle location and middle height of the square height
of 4 meter width and 4 meter height. The length of the tunnel is 50 meter. Based on
the above
(i)

Derive the Sound Pressure Level (SPL) due to the sound source at a distance
of 4 meter from the sound source.
(4 marks)

(ii)

If the sound pressure measured at a distance of 2 meter from the sound source
is 0.01 Pa, calculate the SPL at 4 meter. Discuss any assumption used in the
calculation.
(4 marks)

Seat X in a Istana Budaya concert hall is 15 meter from the, singer Yuna playing her
guitar. Singer Yuna strikes a single, mighty note on her guitar string. The Sound
Pressure Level (SPL) of the direct sound of her guitar note at seat X is SPL 80 dB.
The 1st reflection from the nearest side wall arrived in seat X in 98 milisecond after
Yuna strikes her guitar string.
(i)

How far does the reflected sound level to reach seat X.


(2 marks)

(ii)

Calculate the SPL of the reflected sound at seat X. Discuss any assumption
in this SPL calculation.
(2 marks)

(iii)

How long the sound reflection delayed after the arrival of direct sound at seat
X.

(iv)

Draw its corresponding Energy Time Curve (ETC) and discuss the behaviour
of this ETC.
(2 marks)

Through proper illustration, discuss thoroughly the followings


(i)

The acoustics design for the library.


(4 marks)

(ii)

The acoustics design for the open plan office.


(4 marks)

With proper illustration, discuss properly the effect and consequences of the untreated
dome shape surface in an enclosed hall.
(4 marks)

4
SET 4593

c)

5.

There are 5 split units aircond in a lecture theatre. The SPL generated at 1kHz for
each split unit is 70 dB, 72 dB, 68 dB, 75 dB and 71 dB. Calculate the SPL when all
the split unit aircond is being ON simultaneously.
(4 marks)

(a) A siren positioned vertically at a height of 14 meter of on observation tower. It


produces sound pressure of 6.44 Pa (rms) at radius of 1 meter. The SPL measured on
the ground level at 19.6 meter horizontally is 87 dB. Based on the above, calculate
the following
(i)

The directivity angle, 8

(1 marks)

(ii)

The directivity index, at an angle 0

(4 marks)

(iii)

The siren is moved to 1 meter above the ground level, calculate the SPL at
the same distance, 19.6 meter and at the same angle, 9.
(3 marks)
(b)
(i) Discuss the 5 components needed for a sound system design in a
multipurpose hall.
(5 marks)

(ii)

(c)

Discuss properly and thoroughly 2 sound systems design strategy that can be
used in a multipurpose hall.
(4 marks)

Discuss the loudspeaker requirement for a lecture theatre and musical theatre hall.
(3 marks)
Constant

c = 340 m/s
12
2
L
Io = 1 x 10' W/m
5
P0 = 2 x 1 O' Pa
a (1 person) = 0.35
p = 1.2 kg /m3
W0 = 1 x 1012 Watt

CONFIDENTIAL

UTM
UNIVERSITI TEKNOLOGI MALAYSIA

FINAL EXAMINATION SEMESTER II


SESSION 2010/2011
COURSE CODE

SEU 2033

COURSE NAME

CIRCUIT THEORY

LECTURERS

ASSOC. PROF. DR. ZOLKAFLE BIN BUNT AT

PROGRAMME

SPE / SPR

SECTION

01

TIME

2 HOURS 30 MINUTES

DATE

30 APRIL 2011

INSTRUCTION TO CANDIDATE

ANSWER FOUR (4) QUESTIONS ONLY.


ALL WORKING NEED TO BE SHOWN
CLEARLY.
DRAW NEAT DIAGRAMS WHEREVER
NECESSARY.

THIS EXAMINATION BOOKLET CONSISTS OF 7 PAGES INCLUDING THE FRONT COVER

2
SEU 2033

Ql. (a) Explain, in your own words, the steps taken to implement Superposition
principle to do circuit analysis. What are the limitations of Superposition
principle.

[ 5 marks ]

(b) Apply the superposition principle to find vo in the DC circuit of


Figure Ql(b).

[ 12 marks ]
6Q

Figure Ql(b)

(c) Given the DC circuit in Figure Ql(c), obtain the Thevenin equivalent at
terminals a-b and use the result to get v x .
[ 8 marks ]

40 V

Figure Ql(c)

3
SEU 2033

Q2. (a) In an AC circuits, the nodal and mesh analysis are performed in the same
manner as DC circuit analysis except it involves complex number. Explain
clearly, steps to be carry out for the analysis of AC circuit using Nodal
analysis.
[ 5 marks ]
(b)

Determine the current Io in the circuit of Figure Q2(b) using mesh


analysis.
[ 12 marks ]
4fi

120 Z90 V

Figure Q2(b)

(c)

Obtain Io in Figure Q2(c) using nodal analysis.

[ 8 marks ]
10 Q

------- m------------Io
20 sinlOOOt A

0 <

>20 Q

Figure Q2(c)

50 |ixF

10 mH

4
SEU 2033

Q3. (a) Using a suitable and simple diagram, explain Nortons Theorem.
[ 3 marks ]
(b)

In an AC circuit as shown in Figure Q3(b), compute io using Norton's


theorem.
[ 10 marks ]

Figure Q3(b)

(c)

For the circuit of Figure Q3(c), find:


(i) the real power dissipated by each element,
(ii) the total apparent power supplied by the circuit,

[ 12 marks ]

20 Q

50 Q

Figure Q3(c)

5
SEU 2033

Q4. (a) Draw the equivalent circuit when the Z-parameter network is said to be
reciprocal. What are the characteristic of the symmetrical and reciprocal for
Z-parameter two-port networks.
[ 5 marks ]
(b)

Calculate I] and h in the two port network of Figure Q4(b). Determine


whether the network is symmetrical or reciprocal?
[ 8 marks ]
12

2Q
AW

Zn = 6 Q
2Z30 V

v,

Z \2= j4 O
Z21 ~ j4 O

V2

Z22 = 8 Q

Figure Q4(b)

(c)

For a series two port network as in the circuit of Figure Q4(c);


(i)

Find the parameters for the overall networks

(ii)

Evaluate V2/Vs in the circuit in Figure Q4(c).

[ 12 marks

Figure Q4(c)

SEU 2033

Q5. (a) Briefly define the quality factor, Q of a series resonant circuit.
[ 3 marks ]
(b)

Prove that at the resonant angular frequency, the circuit shown in


Figure Q5(a) and Figure Q5(b) has the same impedance as seen from
terminal a-b if:
R-i +
Lp

cd1!}

toL

[ 9 marks ]
o

Figure Q5(a)

(c)

Figure Q5(b)

For a series RLC resonant circuit in Figure Q5(c).


i)
ii)

Find, I, Vr, Vl and Vc at resonance


What is the quality factor, Qs of the circuit
iii)
If the resonant frequency is 5000 Hz, find the bandwidth, BW
iv)
What is the power dissipated in the circuit at the half-power
frequencies.

R=4Q X l = 15Q
+
240 V

Xc= 15D

Figure Q5(c)

7
SEU 2033

Conversion of two-port parameters

y
Z22

yn yn

AA

Z2

y 2 i y 22

-Z21

h
z12

y 22 - y !2
Ay

hn

z i2

Ay

yn yn

Az

Z12

Z22

Z22

Z21

Zli

yn yn

il

y 2) y 2 i
Ay

Z21

-yn

y2i y2i

Ap=PllP22-Pl2P2

Al
Z21
Z22

Z21 Z21

Nota: Parameter T = transmission Parameter

"A
C

B"
D

t1n

T121

T21

T22 _

"^12 T

^11 At

^12

h 22 h 22

hn h 12

^21

T21

^22

^21 T21
^12 AT

T22T1 22

h 2 j h 22

Z22 Z22

-y 22 -i
T

Ah

zl III

hji

h 22 h 22

Ay Ay

T,2 T12

-h 2] 1
Z21 Z22

-yn

T22 - At

^21 AA

Z,[

zn

-y 2 i yn

1 -h 12
hn hn

y
A

" Ah
^21 ^21
^22

h 21 h 21

__ ^ -1
21

1T22 T
1 22

T
A nT12
Tl 2lT22

CONFIDENTIAL

UTM
UNIVERSITI TEKNOLOGI MALAYSIA

FINAL EXAMINATION SEMESTER II


SESSION 2010/2011

COURSE CODE

SEU 2052

COURSE NAME

ELECTRICAL TECHNOLOGY FOR


MANAGEMENT

LECTURERS

MR. MOHD FADLI BIN RAHMAT


MR. MOHD ZAKI BIN DAUD

PROGRAMME

SHD

SECTION

01-02

TIME

2 HOURS

DATE

30 APRIL 2011

INSTRUCTION TO CANDIDATE

PART A:
THIS PART CONSISTS OF THREE QUESTIONS.
YOU ARE REQUIRED TO ANSWER ANY TWO
(2).
PART B:
THIS PART CONSISTS OF TWO QUESTIONS.
YOU ARE REQUIRED TO ANSWER EITHER
ONE.

THIS EXAMINATION BOOKLET CONSISTS OF 4 PAGES INCLUDING THE FRONT COVER

2
SEU 2052

PART A
THIS PART CONSISTS OF THREE QUESTIONS (QI, Q2 & Q3). YOU ARE
REQUIRED TO ANSWER ANY TWO.

QI. (a) Explain the following rules with the aid of diagrams.
(i)
Voltage Divider Rule
(ii)
Current Divider Rule
(b) For the circuit in Figure Ql(b), determine:
(i)
Rtotal
(ii)
Is
(iii)
Ia
(iv)
Ib
(v)
Power dissipated in 4 Q resistor
(vi)
Power dissipated in 9 Q resistor

[5 marks]
[5 marks]

[3 marks]
[2 marks]
[2 marks]
[2 marks]
[3 marks]
[3 marks]

60.

Figure Ql(b).

Q2. (a) Explain briefly, advantages of AC system compare to DC system. [5 Marks]


(b) The recorded voltage and current in time domain equations of the air-conditioning
system are as below:
v(t)= 339.42 sin(314t + 30)Volt
i(t) = 14.14 sin(314t 15 ) Ampere
(i)
(ii)

For both v(t) and i(t), find peak-to-peak, average and root-mean-square
values.
[3 Marks]
Calculate the system frequency.

[1 Mark]

3
SEU 2052
(iii)

Rewrite the above equations into phase domain and draw the phasor
diagrams.

[3 Marks]

(iv)

Is v(t) leading or lagging i(t)? State the phase angle.

[1 Mark]

(v)

Calculate the apparent power, real power and reactive power for this airconditioning system.
[3 Marks]

(c)
A workshop which is a three-phase balanced load with wye-connection received
electric supply from Tenaga Nasional Berhad (TNB) with a three-phase balanced voltage
with wye-configuration, where line voltage, V|jne = 415 Volt, 50 Hz.
(i)

Draw the configuration between load and supply with neutral connection of
the above mentioned system.
[3 Marks]

(ii)

How do we know the loads are balanced?

(iii)

Calculate the current consumed by the workshop if it used 23 kW real


power with 0.85 lagging power factor.
[1 Mark]

[1 Mark]

(iv)

Find the apparent power and the reactive power used by the workshop.

(v)

[2 Marks]
Describe briefly how the power factor of the workshop can be improved.
[2 Marks]

Q3. (a) Below are the list of devices which convert electrical
energy. Explain briefly how these devices function.
(i) Resistor

energy to another form of


[1 Mark]

(ii) Diode

[1 Mark]

(iii) Inductor

[1 Mark]

(iv) Capacitor

[1 Mark]

(v) Transistor

[1 Mark]

(b) Describe briefly how does the relay work and state its advantages.

[5 Marks]

(c) Building D06 in Management Faculty UTM have 8 lecture rooms. Each room are
installed with 2 air-conditioning units, 48 florescent lamps, 1 desktop computer and 1
projector. If all lecture rooms are fully utilized, calculate how much UTMs electric
consumption in 30 days if all load operate in 12 hours everyday with 0.95 power
factor. Refer to the Table Q3 c(i) and Table Q3 c(ii) for electrical devices power
rating and TNB electric tariff.

4
SEU 2052
Electrical Devices

Apparent Power, S
(kVA) .
Air-conditioning unit 3.2
Florescent lamp
0.2
Desktop
0.5
Projector
1.2
Table C>3 c(i)
Tariff A
Kadar/Rate
(kWh)
(cent/unit)
1-200
21.8
201-1000
28.9
1001 and above 31.2
Table Q3 c(ii)
[15 Marks]

PART B
THIS PART CONSISTS OF TWO QUESTIONS. (Q4 & Q5). YOU ARE REQUIRED
TO ANSWER EITHER ONE.
Q4. (a) What is the main function of a transformer?
(b) Transformer is based on two principles. What are they?
(c) What are the relationships between primary and secondary
voltage, current and power in an ideal transformer?
(d) What types of losses that affect the efficiency of a transformer?
(e) Write down the equation for transformer efficiency.
(f) A single phase transformer 200 kVA, 6600/400 V, 50 Hz has 80 turns on the
secondary side. Find:
(i)
The values of primary and secondary currents.
(ii)
The number of turns on primary side.

[2 marks]
[4 marks]
[6 marks]
[6 marks]
[1 mark]

[4 marks]
[2 marks]

Q5. A linear power supply converts 240 V AC to 25 V DC.


(a). Describe the steps required to change the 240 V AC to 25 V DC. State the supply
type (AC or DC) with the help of diagrams showing the supply waveform
transformations.
[14 marks]
(b). Explain the differences between boost, buck, and buck-boost converters.
[6 marks]
(c). Compare the operation of DC motor and AC motor.
[5 marks]

CONFIDENTIAL

UTM
UNIVERSITI TEKNOLOGI MALAYSIA

FINAL EXAMINATION SEMESTER II


SESSION 2010/2011

COURSE CODE

SEL 4373

COURSE NAME

IC TESTING TECHNIQUES

LECTURERS

DR. OOI CHIA YEE

PROGRAMME

SEC / SEL / SET / SEW

SECTION

01

TIME

2 HOURS 30 MINUTES

DATE

04 MAY 2011

INSTRUCTION TO CANDIDATE

PART A : ANSWER ALL QUESTIONS.


PART B : ANSWER TWO QUESTIONS ONLY.

THIS EXAMINATION BOOKLET CONSISTS OF 11 PAGES INCLUDING THE FRONT COVER

-2-

SEL 4373
PART A: ANSWER ALL QUESTIONS
QI. (20 MARKS)
a. Answer the following questions referring Figure 1.
i.
List all faults in Circuit 1 given that those faults are singly introduced. How many
faults are in total?
[3 marks]
ii.
List all checkpoints in Circuit 1. How many checkpoints are in total?
[3 marks]

Figure 1. Circuit 1.
b. Answer the following questions based on the information given below.

The process uses 8-inch wafers.


The cost of processing a wafer is $150.
Each wafer has 600 chips.
a=0.5
d=0.25 defects/cm2
Chip area without DFT, A=8mmx8mm=0.64cm2

i.
ii.
iii.

What is the defect level in parts per million (PPM) if the fault coverage of the
available test patterns is 94%?
[5 marks]
Suggest two ways to improve the PPM.
[4 marks]
The measures in (ii) improve the fault coverage to 99% but increase the area by
15%. Calculate the new PPM.
[5 marks]

-3SEL 4373
Q2. (30 MARKS)
a. Given that a characteristic polynomial is x 5 +x 2 +x+1. Design an external and an internal
XOR LFSRs.
[6 marks]
b. Does the polynomial x2+x+l divides x9+x6+x+l without any remainder? Show your work.
[4 marks]
c. Answer the following questions by referring to Figure 2.

Figure 2. Circuit 2.

i.

Full scan technique is used to improve the testability of the circuit. How many

inputs and outputs are required for the purpose of test generation? [4 marks]
ii.

752 test patterns are generated to achieve 100% fault coverage. What is the total

number of clock cycles for test application time?

[4 marks]

-4SEL 4373

Figure 3. S-graph.
d. Figure 3 shows the S-graph of a sequential circuit. Answer the following questions.
i.

How many cycles are there in this circuit?

[3 marks]

ii. What is the smallest number of flip-flops we need to include in the scan path to
break all the cycles such that the resulting sequential depth is the smallest?
Identify the flip-flops.

[9 marks]

-5SEL 4373
PART B: ANSWER TWO QUESTIONS ONLY
Q3. (25 MARKS)
A_
B~
C

E----

Figure 4.
a. Answer the following questions based on Figure 4.
i.
Derive a test pattern for J stuck-at 1.
[7 marks]
ii.
List all faults detected when C=l, D=1 and E=1. You can use deductive fault
simulation by filling in the following table.
[12 marks]
Line
A
B
C
D
E
F
G
H
J
K
L
M
N
P
Q
Signal
Line
fault
list

b. Draw the time frame expansion model for the sequential circuit in Figure 5 for two time
frames.
[6 marks]

Figure 5.

-6SEL 4373
Q4. (25 MARKS)

Figure 6.
a. Figure 6 shows the sub-circuit of a big circuit. Complete the following table for SCOAP
measures of the sub-circuit.
A

cco

10

50

27

10

CC1

35

44

b.

[10 marks]
G

Let us assume that PODEM sets an objective 0 at line Q. Which path (e.g. QNFB ) does
PODEM first choose to back-trace to a primary input in Figure 6?

[3 marks]

c. Define drivability. What is the use of drivability?


d.

[2 marks]

Find drivability for the lines involved when X is stuck-at 0 in Figure 7. Based on the
drivability measures, which is the best fault propagation path? Find the test sequence to
propagate the fault through the best fault propagation path.

[10 marks]

t
...
>F1

P~

DQ

t> F2

Q
CLK

Figure 7.

-7SEL 4373
Q5. (25 MARKS)

Figure 8.
a. Answer the following questions using Figure 8.
i.
Derive the test pattern for the stuck-at 1 in the circuit.
[6 marks]
ii.
Using dynamic compaction, derive the test pattern for the faults in Part a.i. and k
stuck-at 1.
[5 marks]

Figure 9.
b. Answer the following questions using Figure 9.
Identify THREE of the redundant faults in the circuit.
ii.
Show the simplified hardware after redundancy removal.

[9 marks]
[5 marks]

-8-

SEL 4373

Testability Measures

CCO (a)
CC1 (a)
a

z CCO (z) = min (CCO (a), CCO (b)) + 1


CC1 (z) = CC1 (a) + CC1 (b) + 1
z CCO (z) = CCO (a) + CCO (b) + 1
CC1 (z) = min (CC1 (a), CC1 (b)) + 1
Z

a
b

CCO (z) = min (CCO (a)+ CCO (b), CC1 (a) + CC1 (b)) + 1
CC1 (z) = min (CC1 (a)+ CCO (b), CCO (a) + CC1 (b)) + 1

CCO (z) = CC1 (a) + CC1 (b) + 1


CC1 (z) = min (CCO (a), CCO (b)) + 1

z CCO (z) = min (CC1 (a), CC1 (b)) + 1


CC1 (Z) = CCO (a) + CCO (b) + 1
z CCO (z) = min (CC1 (a)+ CCO (b), CCO (a) + CC1 (b)) + 1
CC1 (z) = min (CCO (a)+ CCO (b), CC1 (a) + CC1 (b)) + 1
z cco (z) = cci (a) + 1
CC1 (z) = cco (a) + 1

-9SEL 4373

CO (a)
CC1 (a)
CCO (a)

CO (a) CO (z) +CC1 (b) + 1


CO (b) CO (z) +CC1 (a) + 1

CO (z)
z

CO (b)
CC1 (b)
CCO (b

CO (a)-= CO(z) + CCO (b) + 1

a.

CO (b) = CO(z) + CCO (a) + 1

h-

CO (a)--CO(z)
+ min (CCO (b),
CC1 (b)) + 1 a=
CO (b)-= co(z) + min (CCO (a),
CC1 (a)) +1 hCO (a)---CO(z) + CC1 (b) + 1

a.

= co(z) + CC1 (a) + 1


bCO (a) ==co(z) + CCO (b) + 1
a
CO (b)-= co(z) + CCO (a) + 1
b
CO (a) = CO(z) + min (CCO (b),
CC1 (b)) + 1a
CO (b) =
- -CO(z) + min (CCO (a),
CC1 (a)) + 1 b
CO (b)

CO (a) == co(z)+1

CO (a) =
= min(CO (z1), CO (z2), CO (zn))

a
.zn

Important formulas for yield and defect level estimation:


i)
Probability of a chip passing the test, Y(T)=[l+TA/7p]"
ii)

Defect level, DL(T)=1-

Y(T)

Where T = fault coverage


A = chip area
f= fault density
P = clustering parameter

-10-

SEL 4373
Table 1. Fault list propagation in a deductive fault simulation

Gate type

Inputs
a
0
0
1
1

b
0
1
0
1

Output
c
0
0
0
1

OR

0
0
1
1

0
1
0
1

0
1
1
1

[Lau Lb]u ci
[Lan Lb]u Co
[Lan Lb]u Co
[Laf~N' Lb]U Co

NOT

0
1

1
0

Lau Co
La^j Ci

NAND

0
0
1
1

0
1
0
1

1
1
1
0

[Lan Lb]u Co
[Lan Lb]'* Co
[Lan Lb]u Co
Lb]^1 Ci

NOR

0
0
1
1

0
1
0
1

1
0
0
0

[Lau Lb]u c0
[Lan Lb]u ci
[LaO Lb]u Ci
[Lan Lb]u ci

AND

Output fault
list Lc
[Lan Lb]u ci
[Lain Lb]u ci
[Lan Lb]u ci
[La^1 Lb]LJ C0

-11-

SEL 4373

Fault Equivalence Rules:

F1
s-a-1

s-a-1
-x
F2 dominates FI.

CONFIDENTIAL

UTM

UNIVERSITI TEKNOLOGI MALAYSIA

FINAL EXAMINATION SEMESTER II


SESSION 2010/2011

COURSE CODE

SEE 4463

COURSE NAME

HIGH VOLTAGE TECHNOLOGY

LECTURERS

PROF. DR. HUSSEIN BIN AHMAD


ASSOC. PROF. DR. MUHRIDZA BIN YAACOB
ASSOC. PROF. DR. MOHAMED AFENDI BIN
MOHAMED PIAH

PROGRAMME

SEE

SECTION

01-02

TIME

2 HOURS 30 MINUTES

DATE

03 MAY 2011

INSTRUCTION TO CANDIDATE

ANSWER FOUR (4) QUESTIONS ONLY.

THIS EXAMINATION BOOKLET CONSISTS OF 9 PAGES INCLUDING THE FRONT COVER

-2SEE4463

1.

(a) Briefly describe any two (2) of the followings:


(i)
(ii)

Field enhancement factor;


Medium high voltage (MHV), High voltage (HV), Extra high voltage
(EHV) and Ultra high voltage (UHV);

(iii)

Three applications of high voltages excluding those in the generation,


transmission and distribution of electrical energy.

(6 marks)

(b)

There are several properties of a dielectric which are of practical importance for
an engineer. Name five (5) most important properties of a dielectric and briefly
describe each of them.

(10 marks)

(c)

A 50kV AC voltage is applied to a square-shaped structure made from stainless


steel, except its base plate which is insulated from the rest of the structure and is
grounded. Taking a cross section of the structure and using a grid with sixteen
equal squares (giving nine points with unknown voltage), determine the voltages
at all nine points after one iteration.

(9 marks)

2.

(a) The build-up of high currents in a breakdown is due to the process of ionization
in which electrons and ions are created from neutral atoms or molecules. Explain
how the ionization process occurs prior to gas breakdown phenomena.

(5 marks)

(b)

The ionization coefficient alp as a function of field strength E and gas pressure p
is given by the following threshold equation;

-= f{
p u,

-3 SEE4463

By using the Townsends breakdown criterion, show that the breakdown voltage
for uniform field gaps is a function of gap length (d) and gas pressure (p).
(8 marks)

(c)

Fig. Q2 shows the experimental set-up for studying the Townsend discharge. The
experiment is conducted by measuring the current / at the different gap distance,
d. Table Q2 gives the set of observation obtained when studying the conduction

and breakdown in a gas.

i)

Determine the initial current, I 0 .

ii)

Calculate the values of the Townsends primary and secondary ionization


coefficients.

Table Q2 Townsend experimental data


Gap
distance,
d (mm)
Current
/(pA)

10

12

14

16

19

21

26

32

40

45

80

106

152

255

430

Ultra-violet light
Cathode

Anode

ir}'
7

d
R

Current limiting
resistor

Adjustable
h.v. source
-I || |-^{a)~
Vs

Fig. Q2 Townsend experimental set-up

(12 marks)

-4SEE4463

Fig. Q3(a) shows a schematic diagram of a tilted transmission line tower and an
impulse current waveshape, i(t). Consider the tower top is struck by the lightning
current i(t) and voltage rises to u(t).
Current CkA')

ifrt

Time ('microsec')

Also Z g is the surge impedance of the ground wire


Z t is the surge impedance of the tower
u(t) is the impulse surge function
i(t) is the current wave function
T t is time of surge propagation from tower top to the tower footing.

Rt is tower footing resistance


UiT(t) is the potential distribution on the top of tower,
a is the coefficient of reflection on the tower bottom side
(3 is the coefficient of reflection on the tower top side

(i)

Show that
u(t) = Z g Z,/ (Z g + 2Zt). i(t)

(3 marks)
(ii)

Determine whether the following equation is right or wrong (write the


detailed derivation in order to prove it)

Ujrit) = u(t) + a T (\ + P)[u(t -2T t ) + (a r P)u(t -4T r ) + (a r fi) 2 u(t - 6T r ) + .................]

(7 marks)

-5SEE4463

A lightning current surge with the wave shape as shown in Fig. Q3(b), strikes a
tower, which has a single ground wire in both directions. The characteristics are
as follows:
Surge impedance of lightning channel, Zi

= infinity

Surge impedance of tower, Zt

= 150 Q

Surge impedance of ground wire, Zg

= 340 Q

Velocity of wave propagation on lines

= 298 m/pis

Velocity of wave propagation on tower

= 240 m/|is

Height of tower

= 30m

Effective tower footing resistance

= 40 Q

Lightning current peak magnitude

= 40 kA

20fis ---------------------- 1 jLts -------

Fig. Q3(b) The simplified lightning current wave shape

Based on Fig. 3(a), determine the maximum tower top potential for a duration 5
times the time of surge propagation from the tower top to the tower base after the
lightning strike the tower.

(15 marks)

-6SEE4463

(a)

Fig. Q4(a) shows an insulation coordination practice using gaps/arcing horns.


Briefly describe what you understand from the diagram.

Fig. Q4(a) Coordination using gaps/arcing horns

(5 marks)

(b)

Fig. Q4(b) shows an evaluation of risk factor in an insulation coordination


practice using the statistical technique. Briefly describe what you understand
from the diagram as f0(V) and P(V) move to either direction (left and right).

Fig. Q4(b) Evaluation of risk factor

(5 marks)

SEE4463

A 500 kV steep fronted wave (rate of rise 1000 kV/(a.s) reaches a transformer of
surge impedance 1500 Q through a line surge impedance 500 Q and protected by
a lightning arrester with a protective level of 700 kV, 60 m from the transformer.
Sketch the voltage waveforms at the arrester location. Determine the time at
which the arrester operates. Assume all waves travel at 3.0 x 108 m/s.

(15 marks)

Explain the purpose of insulation diagnostic tests on electrical power equipment.


What are the parameters or properties normally measured when investigating the
insulation performance?

(5 marks)

The circuit diagram for a Schering bridge is shown in Fig. Q5. Both ends of the
sample and the standard capacitor are connected to the high voltage side of the
bridge. The standard capacitor used in the circuit has losses and can be
represented as a capacitance (C2 ) and resistance (7 *2 ) in series.
Show that at balanced condition, the capacitance and the resistance of the sample
are;

standard
capacitor
with losses

Fig. Q5 Schering bridge

(12 marks)

-8SEE4463

(c)

Show that for a solid insulating material of relative permittivity s r , containing a


cylindrical air-filled cavity of depth t, which is small in relation to the thickness T
of the dielectric, the voltage across the sample (V a) is given by the expression;

V' = V'+ 'V,


M

where, V c is the voltage across the cavity.

From the above equation, explain why the partial discharge can occur in the
cavity even though only the normal service voltage is applied across the
insulating material.
(8 marks)

6.

(a) Describe the types of tests conducted on high voltage equipment.


(4 marks)

(b)

With the aid of suitable diagrams discuss the generation of high voltage direct
current (HVDC) using the full wave rectifier circuit.
(5 marks)

(c)

Lightning impulse voltage is simulated in the laboratory using an impulse


generator and is used to conduct lightning impulse tests on high voltage
equipment based on standard test procedures. An impulse generator (unmodified
Marx) has five (5) stages. The following circuit elements are available: 100 kVrated 0.2 p.F capacitors, a 300 2 front resistor, and a 2500 Q tail resistor. The
load capacitor is given as 1000 pF.

i)

Determine the output impulse waveshape of the generator and give


comments on the waveshape as compared to the standard test procedure.
(8 marks)

-9SEE4463

ii)

What is the maximum output voltage of the generator if the charging


capacitor is charged up to the maximum rated voltage?
Hint:
Vout max

PC L R f

c +c
Rf ct cL

/3~

(5 marks)

iii) Sketch the five-stage impulse generator (unmodified Marx).

(3 marks)

CONFIDENTIAL

UTM
UNIVERSITI TEKNOLOGI MALAYSIA

FINAL EXAMINATION SEMESTER II


SESSION 2010/2011

COURSE CODE

SEU 2012

COURSE NAME

ELECTRONICS

LECTURERS

ASSOC. PROF. DR. JEGANNATHAN


SRINIVASAN
DR. YOU KOK YEOW
DR. KUMERESAN AIL A. DANAPALASINGAM
DR. KHURRAM KAMAL
DR. AHMAD SAUDI SAMOSIR

PROGRAMME

SMB / SMI / SMK / SMM / SMP /SMT / SMV

SECTION

01-08

TIME

2 HOURS

DATE

30 APRIL 2011

INSTRUCTION TO CANDIDATE

PART A : ANSWER ALL QUESTIONS.


PART B : ANSWER ALL QUESTIONS.
PART C : ANSWER THREE (3) QUESTIONS
ONLY.

THIS EXAMINATION BOOKLET CONSISTS OF 13 PAGES INCLUDING THE FRONT COVER

2
SEU2012
A. Multiple Choices. (Answer ALL questions). Choose the best answers.

When sufficient energy is added to a valence electron, it will jump from


A. valence band to the conduction band.
B. conduction band to the valence band.
C. energy band to the conduction band.
D. valence band to the energy band.

At room temperature, the number of free electron in intrinsic semiconductor is_______


A.

more than the number of hole

B.

equal to the number of hole

C.

less than the number of hole

D.

none of the above.

When the source voltage increases in a zener regulator, which of these currents remains
approximately constant?
A. Series Current B. Load Current C. Total Current D . Zener Current

Output voltage, V0 for the circuit in figure A4


-------- o v0
Si

Figure A4
Vz=10V

A.

10V

B.

10.7V

C.

9.3V

D.

15V

The power dissipated by a transistor approximately equals the collector currents multiply with
A.

base-emitter voltage

B.

supply voltage

C.

base supply voltage

D.

collector emitter voltage

3
SEU2012
For circuit in Figure A5 , when value of Rb increases,
QVCC
Rb

Rc
Figure A5

A.

Ic decreases

B.

ft decreases

C.

Vce decreases

D.

Ic increases

E.
When an NPN BJT transistor is operating in saturation region, what is the biasing condition of
the base emitter junction
A.

Either reversed or forward biased

B.

Forward biased

C.

Reversed biased

D.

Cannot be determined

DC analysis is performed on an amplifier to


A.

ensure the transistor can operate as an amplifier

B.

find operating point for the amplifier circuit

C.

determine amplifier gain

D.

avoid transistor to work in saturated or cut off condition

For a JFET, the threshold voltage, Vos(th) is the


A.

maximum allowable gate source voltage before breakdown

B.

gate source voltage that produces a leveling off of Id

C.

minimum gate-source voltage that makes drain current flow

D.

none of the above

Valence electrons are


A.

in the closest orbit to the nucleus

B.

in the most distant arbit from the nucleus

C.

in various arbits around the nucleus

D.

not associated with a particular atom.

4
SEU2012
11. The process of adding an impurity to a semiconductor is called
A.

recombination

B.

doping

C.

purification

D.

ionization

12. The purpose of a pentavalent impurity is to


A.

reduce the conductivity of silicon

B.

increase the number of holes

C.

increase the number of free electrons

D.

create minority carriers

13. For operation in the active-region, the base of an npn transistor must be
A.

positive with respect to the emitter

B.

negative with respect to the emitter

C.

positive with respect to the collector

D.

at zero volt.

14. A certain common emitter amplifier has a voltage gain of 100. If the emitter
bypass capacitor is removed,
A.

the circuit will become unstable

B.

the voltage gain will decrease

C.

the voltage gain will increase

D.

the Q-point will shift.

15. For an op-amp with a negative feedback, the output is


A.

feed back to the noninverting input

B.

increased

C.

feed back to the inverting input

D.

none of the above

5
SEU2012
PART B. (Answer ALL questions).

1. (a) When sufficient energy is added to a__________

electron, it will jump to

energy band.
(b)

In N-type extrinsic semiconductor, electrons are the................... carriers and


holes are the....................carriers.

(c)

[1 mark]

[2 marks]

Depletion zone of a P-N junction occurs due to the..................... process.


[1 mark]

(d)

The following figure is a block diagram of power supply system. Name the
blank block.

[1 mark]

AC in

2. (a) Match between two boxes using arrow

[3 marks]
(b)

How must the base-emitter and base-collector junctions be biased for proper
transistor operation?

(c)

[2 marks]

Figure B2 is a diagram of a typical single-crystal silicon NPN small-signal


transistor connected to a basic biasing circuitry in a common-emitter configuration,
(i)

Label the terminals of transistor with C for collector, B for base


and E for emitter.

[2 marks]

(ii)

SEU2012
Label the base to emitter voltage as VBe and collector to emitter voltage
as VCe* Mark the polarity of each voltage with correct plus or minus
signs.

[2 marks]

Figure B2

3. (a) Give TWO major advantages of a Field-Effect Transistors (FET) over a


Bipolar Junction Transistors (BJT).

[2 marks]

[2 marks]

4. (a) List two ideal characteristic of an op-amp.

(b)

State TWO important assumptions made when analyzing circuits with ideal
op-amps.

(c)

[2 marks]

State the amplifier function of the op-amp for A v = 1 +

R,

5. Sketch Vo of the following figure when Vref=0 and Vi = Vm sin cot V.


+V,
'cc

V.

[1 marks]

R
[4 marks]

7
SEU2012
PART C: ANSWER (3) QUESTION ONLY

QI. Rectifier

(a) Draw the circuit diagram of a full-wave bridge rectifier (including transformer and filter).
[4 marks]

(b) If the input to the transformer primary is a 60 Hz sine wave at 120 V, and if the
transformer has a tums-ratio of 10:1, determine the peak value of the secondary voltage.
[4 marks]

(c) Assuming that Rl is 220Q, determine the peak value of the unfiltered full-wave rectified
voltage.

[5 marks]

(d) Assuming that the filter capacitor is lOOOpF, determine the peak-to-peak ripple voltage.
[4 marks]

(e) Determine the percentage ripple factor.

[3 marks]

SEU2012

Q2. When Rl = 200 Q, determine Voltage and Current across each component in the
circuit in Figure Q2, with the following step:

(a)

Determine the state of zener diode in the circuit.

[2 marks]

(b)

Draw the equivalent circuit base on the state of zener diode.

[2 marks]

(c)

Determine Voltage and Current across resistor Rl

[2 marks]

(d)

Determine Voltage and Current across resistor Rs

[2 marks]

(e)

Determine Voltage and Current across zener diode

[2 marks]

Repeat the above question when Rl = 50 Q.

(f)

Determine the state of zener diode in the circuit.

[2 marks]

(g)

Draw the equivalent circuit base on the state of zener diode.

[2 marks]

(h)

Determine Voltage and Current across resistor Rl

[2 marks]

(i)

Determine Voltage and Current across resistor Rs

[2 marks]

O')

Determine Voltage and Current across zener diode

[2 marks]

R<

Rl

Figure Q2

9
SEU2012
Q3. Refer to a small signal amplifier circuit in Figure Q3.
The transistors parameter are: Pdc = Pac = 100, Vbe= 0.7V, Vt = 26mV

(a)

Draw the DC equivalent circuit, then calculate:

[1 marks]

(i) the base and collector current, Ib and Ic-

[4 marks]

(ii) the collector to emitter voltage, Vce-

[3 marks]

(b)

Calculate new Q-point (Ibq and Icq) if R2 is halved

[4 marks]

(c)

Draw the A.C. equivalent circuit of the transistor circuit.

[4 marks]

(d)

If the voltage gain of the amplifier is A vs = V</V s = lOdB, and the input signal is a
sinusoidal waveform of Vs = 3.75sinl OOOt V, what would be the amplitude (peak
value) of the output Vo?

[4 marks]

VCC
+12V

Figure Q3

10
SEU2012

Q4.

Refer to Figure Q4. The JFET has IDss=5.4mA and VP=-6V.

(a)

Calculate the DC Operating Point VGsq , for IDQ= 2.7mA

[3 marks]

(b)

Calculate the drain to source voltage Vos-

[3 marks]

(c)

Draw the A.C equivalent circuit of the FET circuit.

[2 marks]

(d)

Calculate parameter gm

[2 marks]

(e)

Calculate the input impedance, Zi, and the output impedance Zo.

[3 marks]

(f)

Calculate the amplifier voltage gain, A v = Vg/Vj.

[4 marks]

(g)

Calculate the input voltage, vi; if the AC signal of the output voltage is
v0=5.4 sin 2ut volts. Draw the input signal, v*.

[3 marks]

V DD

[jfo
Rd

Ri
5MQ

2JC D.

-O
A

+
O

^DS
V,o

R,
15

Rs

JOOuF

soon
O

-G

Figure Q4

11

SEU2012
Q5. Refer to an Operational Amplifier circuit in Figure Q5(a).

(a)

Name three function/application of operational amplifier ?

[3 Marks]

(b)

The common standard |iA-741 operational amplifier is shown in Figure Q5(a), state
the name for each terminal of the operational amplifier.

[4 marks]

HA-741

Figure Q 5 (a)

(c)

In Figure Q5(b), the operational amplifier has infinite gain, infinite input impedance
and negligible output impedance. Derive an expression for the circuits transfer
function V ou /V in .

[4 marks]
r2

out

Figure Q5(b)

12

SEU2012

(d)

Calculate Ri from the given circuit as shown in Figure Q5(c) ?

[4 Marks]

i?2 = 200ka

Figure Q5(c)

(e)

Determine the output voltage of the following Summing Amplifier circuit as shown in
Figure Q5(d).

[5 Marks]

10 kQ

Figure Q5(d)

13
SEU2012

Appendix - List of Equations

PIV = 2V p(mit) + 0.7F


y

8m

/3Vr

1 CQ

vr

nns

V,p
21 DSS

JT

V,cs
V.
!= (T

jj-, j is mO

r /!

V
-M

V DC 2V 3jr l c

yr ( p - p )

]/
\!P/

2V3
y
r

'

a=
is nr

CO

v DC 4J 3 me
y
r(p-p)

F r(rms)

2-S

VT

Vp

v =

r(p-p)

/ D

Rl C fR L C

DSS

a
1 -a

ft

(T = <7

6m o?0

I E= 1C +B/

I c od

AI,

+ l CBO
JT

&m

[/
V

p/

AV,os

CONFIDENTIAL

UTM
UNIVERSITI TEKNOLOGI MALAYSIA

FINAL EXAMINATION SEMESTER II


SESSION 2010/2011
COURSE CODE

SEE 4433

COURSE NAME

POWER ELECTRONICS AND DRIVES

LECTURERS

: ASSOC. PROF. DR. NAZIHA BT AHMAD AZLI

PROGRAMME

SEI / SEE / SEL / SEM

SECTION

01

TIME

2 HOURS 30 MINUTES

DATE

09 MAY 2011

INSTRUCTION TO CANDIDATE

ANSWER FOUR (4) QUESTIONS ONLY.

THIS EXAMINATION BOOKLET CONSISTS OF 7 PAGES INCLUDING THE FRONT COVER

SEE4433
Question 1
(a) Discuss the need of driver circuits in a power electronics converter system. Include a
suitable diagram to support the discussion and an example of a driver circuit for any
suitable power device.
[7 marks]
(b) Figure Ql(b) shows the output voltage waveform of an unknown power converter. Given
that the input of the particular converter is a 240 VrmS, 50 Hz supply. Based on Figure
Ql(b), answer the following questions.

Figure Ql(b)
(i) Name the type of power converter used to produce the above output voltage
waveform. Identify the load used.
[2 marks]
(ii) Sketch the output current, i 0 (t) waveform.
[2 marks]
(iii)If a Free Wheeling Diode (FWD) is connected in parallel with the load. Sketch the
output voltage, v' 0 (t) waveform and the output current, i' 0 (t) waveform.
[3 marks]
(iv)Based on the scenario in Ql(b)(iii), derive the expression for the average output
voltage.
[3 marks]
(v) In the case if the above converter is using a controllable power switch (without
FWD), a delay angle of a would occur. Sketch the new output waveform and prove
that the controllable average output voltage is expressed by
Vo

i m m g e ) =^(cosoc-cosP)

under discontinuous current mode, where 1 6 is the extinction angle.

[4 marks]
(vi) Based on the observation of the controllable output waveform, it can be identified that
the output current of the power converter start to rise at t = 2.2 ms and decay to zero at
t = 12.5 ms during the first cycle. Calculate the average output voltage.
[4 marks]

3
SEE4433
Question 2
(a) Using relevant diagram, sketch and label a block diagram of a mains-fed switched mode
power supply (SMPS). Discuss briefly the function of each element in the diagram.
[8 marks]

(b) The waveform of inductor current, i L, of a boost converter in steady-state is shown in


Figure Q2(b). The output voltage of the boost converter was found to be \V 0 \ = 48 V and
its peak-to-peak ripples to be Av0 = 0.25 V.
i. Determine the switching frequency and the duty cycle of the circuit.

[3 marks]
ii. Determine the input voltage, Vs, and the load resistance, R.

[4 marks]
iii. Draw the inductor voltage waveform. Label the key quantities.

[2 marks]
iv. Determine the values of the inductor and capacitor.

[4 marks]

(c) For a Buck converter shown in Figure Q2(c), determine the minimum design rating of
voltage and current for both power switch {v sw , i sw ) and power diode (V x , i x ) to ensure the
converter works well.
[4 marks]
+ K -

Figure Q2(c)

4
SEE4433
Question 3
(a) Consider a single-phase full-bridge voltage source inverter. Suggest and then describe
briefly a control strategy for the inverter that can eliminate a single harmonic in the
inverter output voltage to prevent interference with other signaling system utilizing the
same frequency. Provide diagrams if necessary to support your suggestion.
[7 marks]
(b) Figure 3(b) shows the output voltage across the R load of an inverter.
V,. ..

-K,r 4

Figure Q3(b)
(i) Show that the Fourier series of each voltage term is
r 4V,
V. = __ dc_ cos na
\nn j
Hint: cos n(n -a) = cos inn') cos (na)

[3 marks]
(ii) Given that the inverter output frequency is 50 Hz, R = 100 Q, L = 20 mH and the Vdc
= 100V, determine the value of a to produce 110.3 V fundamental-frequency voltage.
[3 marks]
(iii)By using the same a calculated in (ii), determine the THD of the inverter load current
up to 7th harmonic order.
[8 marks]
(iv)Under the condition in b(iii), plot the frequency spectrum of the inverter load current.
Label the important parameters clearly.
[2 marks]
(v) Suggest and then describe a switching strategy for the inverter power devices that can
improve the quality of the inverter load current.
[2 marks]

5
SEE4433
Question 4
(a) DC motor drive system employed power electronic converters to obtain variable speed for
certain application. Explain briefly two types of power electronic converter normally
used. Your explanation must include their typical schematic diagram and performances.
[8 marks]

(b) A 200 V, 1000 rpm and 150 A separately excited DC motor has an armature resistance of
0.03 Q. The motor is fed from a chopper circuit, which is able to operate in both motoring
and braking operations. The source has a voltage of 200 VDC. Assume that the system
operates in continuous conduction.
1. Sketch the schematic diagram of the power converter used in DC motor-drive system
as mentioned above.
[3 marks]
ii. Calculate the duty cycle, D, of the chopper circuit for motoring mode at rated torque
and speed of 500 rpm.
[4 marks]
iii. Calculate the duty cycle of the chopper for braking at rated torque and speed of 500
rpm.
[3 marks]
iv. If the chopper maximum operating duty cycle is set to 0.9 and the motor current is at
twice its rated value, calculate the maximum permissible motor speed during braking
mode.
[4 marks]
v. With the condition of Q4(b)(iii), determine the power fed back to source. Assume no
losses in the power electronics converter.
[3 marks]

6
SEE4433
Question 5
(a) Based on a centrifugal pump, explain how and adjustable speed drive can save energy
compared to a constant speed drive.
[7 marks]

(b) Sketch the characteristic of a typical torque versus speed which depicts the constant
power and constant torque regions in an electric drive system. Then, briefly explain how
the speed is controlled based on the T-co characteristic.
[5 marks]

(c) Figure Q5(c) shows the torque-speed characteristics of a three-phase induction motor that
supplied by a three-phase voltage source inverter for its variable speed-drive operation
using a constant V/f control scheme. The induction motor has the following parameters:
Rated voltage : 415 V (peak)
Pole pair (P) : 2
Torque

Figure Q5(c)
i.

Determine the supply frequency of characteristic under rated condition.

[1 marks]
ii.

Determine the supply frequency of characteristic A.

iii.

Calculate the slip if the motor is rotating at 24 Hz at rated supply frequency.

[1 marks]
[2 marks]
Question 5(c)iv-vi continues on the next page.

7
SEE4433
Question 5(c)...continued
If the inverter gives 415 V (peak) fundamental sinusoidal voltage with modulation
index of 0 .8 , calculate the required modulation index if the motor need to be operated
at rotor mechanical speed of 18 Hz under constant Volt-Hertz ratio at full load
condition.

[3 marks]
Under variable voltage variable frequency operation, calculate the required voltage to
maintain the constant V//ratio for the inverter frequency under
1.
Rated condition
2.
Characteristic A
3.
Characteristic B.
[3 marks]
Under the rated condition, calculate the speed of the motor (in rpm) at 80% of the full
load torque.
[3 marks]

CONFIDENTIAL

UTM
UNIVERSITI TEKNOLOGI MALAYSIA

FINAL EXAMINATION SEMESTER II


SESSION 2010/2011
COURSE CODE

SEI3133

COURSE NAME

ANALOG AND DIGITAL INSTRUMENTATION

LECTURERS

ASSOC. PROF. DR. SALLEHUDDIN BIN


IBRAHIM

PROGRAMME

SEI

SECTION

01

TIME

2 HOURS 30 MINUTES

DATE

30 APRIL 2011

INSTRUCTION TO CANDIDATE

ANSWER FOUR (4) QUESTIONS ONLY.

THIS EXAMINATION BOOKLET CONSISTS OF 9 PAGES INCLUDING THE FRONT COVER

SEI3133

Question 1
(a) Calculate the sensitivity of a 200|iA meter movement which is to be used as a dc voltmeter.
(2 marks)

(b) A basic DArsonval movement with a full scale deflection of 50 jiA and internal resistance of
500 Cl is used as a voltmeter. Determine the value of the multiplier resistance needed to measure
a voltage range of 0 - 10 V.
(3 marks)
(c) Convert a basic DArsonval movement with an internal resistance of 50Q and a full-scale
deflection current of 2mA into a multirange dc voltmeter with voltage ranges of 0 - 10V, 0 50V, 0 - 100V and 0 - 250V as in Figure 1.1 by determining the values of Ri, R2 , R3 and R4 at
Vi, V2 , V3 , and V4 respectively.

Figure 1.1
(10 marks)

3
SEI3133

(d) Design an Ayrton shunt as in Figure 1.2 by determining the values of Rj, R2 , R3 and R4 in
order to construct an ammeter with a current range of 0 - 1mA, 10mA, 50mA and 100mA. A
DArsonval movement with an internal resistance of 100Q and full scale current deflection of 50
|jA is utilized.
(10 marks)

1 mA

D'Arsonval

Figure 1.2

4
SEI3133
Question 2
(a) (i) Assuming ideal diode, calculate the d.c. output voltage for the network shown in Figure
2.1.
(b) Repeat part (a) if the ideal diode is replaced by a silicon diode, having a cut in voltage of
0.7V. Ignore diode forward resistance.

R l * 1 kQ

Figure 2.1
(5 marks)
(c) Calculate the peak input and output voltages in Figure 2.2.

(7 marks)

5
SEI3133
(d) The circuit of a shunt ohmmeter used for the measurement of resistance is shown in Figure
2.3. The calibration resistance R is adjusted for the full scale deflection when X > oo. For the
meter having 0.5 mA full scale deflection current and resistance of 50Q, and supply voltage of
3V, determine
(i) Value of Rand
(ii) Value of X causing half scale deflection.

Figure 2.3
(8 marks)
(e) State four advantages and one disadvantage of PMMC instruments.
(5 marks)

SEI3133
Question 3
(a) State four advantages of digital instruments over analog instruments.
(4 marks)

(b) A 41 digit voltmeter is used for voltage measurements.


2

(i) Find its resolution.


(ii) How would 12.98V be displayed on 10V range?
(iii) How would 0.6973 be displayed on IV range?
(iv) How would 0.6973V be displayed on 10V range?
(5 marks)

(c) A 31 digit voltmeter has an accuracy specification of 0.5% of reading 1 digit.


(i) What is the possible error in volt, when the instrument is reading 5.00V on the 10V range?
(ii) What is the possible error in volt, when reading 0.10V on the 10V range?
(iii) What percentage of the reading is the possible error in the case of (c) (ii) ?
(iv) Is the result in (c) (iii) acceptable?
(8 marks)

(d) Describe how inductive and capacitive impedances are converted into voltage components in
digital LCR meters. Your answer must include the basic circuits for performing both operations.
(8 marks)

7
SEI3133
Question 4
(a) State two differences between noise and interference?
(4 marks)
(b) The circuit shown in Figure 4.1 generates noise in the resistor from two sources: thermal
noise in the resistor and shot noise in the diode. Calculate the total noise voltage in the resistor at
a bandwidth of 1 MHz from these sources. Assume the temperature of the resistor is 27C. e =
electronic charge, 1.60 x 10'19C, k = Boltzmanns constant, 1,38 x 10'23 J/K, and the diodes
forward voltage drop = 0.7V.
0+5V

R
220kQ

Figure 4.1
(5 marks)
(c) Can both ends of a shield be connected to grounds? What would happen if both ends are
connected to grounds?
(4 marks)
(d) A TLE2027 op amp with a noise specification of 2.5nV/VHz is used over an audio frequency
range of 20 Hz to 20 kHz, with a gain of 40dB. The output voltage is 0 dBV (IV). Compute its
signal-to-noise ratio.
(6 marks)

SEI3133

(e) A pn junction carries a current of 1mA at room temperature. Determine its rms shot noise
voltage over an audio frequency range of 20 Hz to 20 kHz. (e = electronic change, 1.60 x 1019C,
k = Boltzmanns constant, 1,38 x 10'23 J/K).
(3 marks)
(f) Describe burst noise.
(3 marks)

9
SEI3133
Question 5
(a) An 8-bit Analog-to-Digital Converter (ADC) outputs all ls when its input voltage is 5.1V.
Find its (i) Resolution and (ii) Digital output when its input voltage is 1.28V.
(6 marks)
(b) Given a 3-bit Digital-to-Analog Converter (DAC) with a 1 V full scale voltage and accuracy
of 0.2%,
(i) find its resolution in terms of voltage,
(ii) find its accuracy in terms of voltage, and
(iii) sketch a graph of analog output voltage (as the vertical axis) versus digital input word (as the
horizontal axis) for the 3-bit DAC and show the resolution and the accuracy on the graph.
(6 marks)
(c) The RS232 signal in Figure 5.1 is sent with 8 data bits and odd parity.

;i
Logic 1

Logic 0

____

8[9

10

i
i
i

12 ;

i
i
i
i

t
i
i
i
i
i
t
i

:V

i
i
i

One bit time


Figure 5.1
(i) What is the binary value being transmitted in this signal?
(ii) Is the parity bit correct?
(iii)Would the answer to question 25 change if the RS232 signal was set for 7 data bits?
(8 marks)
(d) Distinguish between parallel and serial communication.
(5 marks)

CONFIDENTIAL

UTM

UNIVERSITI TEKNOLOGI MALAYSIA

FINAL EXAMINATION SEMESTER II


SESSION 2010/2011
COURSE CODE

SEE 3433

COURSE NAME

ELECTRICAL MACHINES

LECTURERS

DR. AWANG BIN JUSOH


MR. NIK DIN BIN MUHAMAD

PROGRAMME

SEE

SECTION

01-02

TIME

2 HOURS 30 MINUTES

DATE

30 APRIL 2011

INSTRUCTION TO CANDIDATE

ANSWER FOUR (4) QUESTIONS ONLY.


ALL WORKING NEED TO BE SHOWN
CLEARLY.

THIS EXAMINATION BOOKLET CONSISTS OF 9 PAGES INCLUDING THE FRONT COVER

SEE 3433
-2-

Question 1
(a)

By considering linear electromechanical energy systems and with the help of equations,
explain briefly the production of torque in a singly excited and a doubly excited
electromechanical energy system.

(b)

[6 marks]

In the" electromagnetic relay system, the flux linkage A and current i relationship is given
by:
i = kA 2 e 2x

Evaluate the electromechanical energy force, f m when 1 = 2 V.s, x = 1 m and k = 6


A/(V. s)2
[6 marks]
(c)

An elementary two-pole cylindrical rotating machine with a uniform air gap is shown in
Figure Ql(c): The mutual inductance between the rotor and the stator is given by
L u -O.5cos0 H

A DC current source z, = 4 A is applied to the rotor and an AC current source


i 2 = 8sin(10/) A is applied to the stator.

(i)

Write the general equation of the torque for the machine in Figure Ql(c) as
functions of i\, i 2 and L\ 2 .

(ii)

[2 marks]

Derive the developed torque when the rotor is locked at 6= 60. Also, determine
the average torque produced by the machine.

(iii)

[5 marks]

Derive the developed torque when the rotor can freely rotate. Let Q = 60 at t = 0
and a> m = 10 rad/s. Also, determine the average torque developed by the machine.
Rotor
axis

[6 marks]

comt + 5

Stator
axis

Figure Ql(c)

SEE 3433
-3Question 2
(a)

Show typical V t - I a characteristics of differentially compounded and cumulatively


compounded DC generators. Explain briefly their characteristics.

[6 marks]

(b) A six pole DC machine has a wave winding of 300 turns. The flux per pole is 0.025 Wb. The
DC machine rotates at 1000 rpm.
(i)

Determine the generated voltage, E a .

(ii)

Determine the kW rating of the machine if the rated current through the turn is 25 A.
[5 marks]

(c)

A 10 kW, 100 V, 800 rpm, R a = 0.1 Q DC machine operated as shunt self-excited generator
has a magnetization characteristic at 800 rpm as shown in Figure 2(c) in Attachment Q2(c).
The shunt field winding resistance R f w - 100 Q and the number of turns N f = 1200 turns per
pole. The rated field current If = 0.8 A. The machine is provided with a series winding with
R$r = 0.04 Q so that it can operate as a compound DC machine as well.

(i)

Draw an equivalent circuit of the compound DC machine. Label all key quantities.
[3 marks]

(ii)

The machine is operated as a shunt generator at 800 rpm and the no load terminal
voltage is adjusted to 100 V. Determine the full load terminal voltage. Assume the
effect of armature reaction at full-load is /f(AR) = 0.05 A.

(iii)

[4 marks]

The machine is operated as a compound DC machine at 800 rpm so that the terminal
voltage of 100 V can be achieved at no load as well as at full load (i.e., zero voltage
regulation). How many series turns per pole, N sr are required to obtain this zero
voltage regulation. Assume the effect of armature reaction at full-load is If(AR) - 0.05
A.

[7 marks]

You must submit Attachment Q2(c) with your answer booklet

SEE 3433
-4Question 3
(a)

Explain briefly the terminal voltage control for controlling the speed of a separately excited
DC motor.

(b)

[4 marks]

Explain briefly the effect of armature reaction on the operation of a DC motor and a DC
generator.

(c)

[4 marks]

A 10 kW, 100 V, 800 rpm, R a - 0.1 Q DC machine operated as shunt self-excited generator
has a magnetization characteristic at 800 rpm as shown in Figure 3(c) in Attachment 03(c).
The shunt field winding resistance R f w = 100 Q and the number of turns N f = 1200 turns per
pole. The rated field current I f = 0.8 A. The machine is provided with a series winding with
the number of turns N s r and negligible R s r so that it can operate as a compound DC motor as
well. The machine is connected to a 100 V DC supply and is operated as a shunt DC motor.
At no load conditions the motor runs at 800 rpm and the armature takes 5 amperes.

(i)

Find back emf E- d , field current If, and field resistance, Rf at no load conditions.
[3 marks]

(ii)

Find the speed of the motor when the rated current flows in the armature. Neglect the
armature reaction effect.

(iii)

[4 marks]

Find the speed of the motor when the rated current flows in the armature. Consider
that the effect of armature reaction at full load is 5% reduction in the air gap flux (i.e.
^fl = 0.95^nl)-

(iv)

[5 marks]

The machine is operated as a cumulatively compound DC motor so that the speed of


700 rpm can be achieved at full load. How many series turns per pole, N sr are
required? Assume the armature reaction effect at full load is /f(AR) = 0.15 A.
[5 marks]

You must submit Attachment Q3(c) with your answer booklet

SEF3433
-5Question 4

(a)

Why is terminal voltage speed control for induction machine limited in operating range?
[3 marks]

(b)

How is torque developed in induction motor? Why is it impossible for an induction motor
to operate at synchronous speed?

(c)

[4 marks]

The parameters for a 6-pole, 3-phase, star-connected, 415 V, 50 Hz, 950 rpm wound rotor
induction machine are as follows:

R\ = 0.25 Q

Ri 0.20 Q

Zi = 0.65 Q

X 2 '= 0.60 O

X m = 60Q.

The motor is connected to a 3-phase 415 V, 50Hz supply. The rotational losses are
assumed constant at 1200 watts whenever the motor rotates. With the rotor terminal short
circuited, find:

(i)

Line current, power factor, and input power at full load.

[6 marks]

(ii)

Current drawn by the motor and torque at starting.

[6 marks]

(iii)

Motor efficiency at which maximum torque is developed.

[6 marks]

SEE 3433
-6-

Question 5

(a)

Draw the power-angle characteristics and torque-speed characteristics of a synchronous


machine. Label key quantities.

(b)

[4 marks]

With the help of a suitable phasor diagram, explain briefly how the power factor of the
synchronous motor connected to infinite bus can be controlled by the field current at a
constant output power.

(c)

[6 marks]

A star-connected 3-phase, 20 kVA, 415 V, 50 Hz, 4-pole synchronous machine has a


synchronous reactance X s = 10 Q. The armature resistance can be neglected. The machine
is connected to an infinite bus bar of 415 V, 50 Hz.

(i)

The mechanical power and field current excitation are adjusted such that the machine
is delivering 10 kW at a power factor of 0.8 lagging. Determine the excitation
voltage Ef and the power angle S. Draw the phasor diagram for this condition.
[5 marks]

(ii)

If the field excitation current is now increased by 15 percent without changing the
prime mover power, find reactive kVAr supplied by the machine. Draw the phasor
diagram for this condition.

(iii)

[5 marks]

Determine the maximum power the synchronous machine can deliver for the
excitation current as in (i). Draw the phasor diagram for this condition.
[5 marks]

SEE 3433
-7-

Attachment Q2(c)

Name:

Seksyen:

01/02

Magnetization Curve at 800 rpm

Field Current

Figure Q2(c)

SEE 3433
-8-

Attachment Q3(c)

Name:

Seksyen:

01/02

Figure 03fc)

SEE 3433
-9Potentially Useful Formulae

H 0 - x 10 7 H/m

Rf=Rfiv+Rfc

W,+/,

B = = jjH
A
I
R=
M
HI = Ni

h(eff) ~

f If(AR)

K. =

Np

V t = J?,/,

/(/<)

&> =

~ i ~ i ~ R
X = N(j> = Li

(KJ) 2 KJ
Ea

dX

e=

= KJ a <O m
T = KIsr* a2

v,

G)_ ~

K,

120/

W' f = jxdi
0

Ef=V,Z0IJXs

N
= /. +^L-J

X _ N(j> _ N 2

dt
i
W f = fidX
0

=
V,=EaIaRa
Ea

s=

= \HdB
0
H

N. -N.

3EfV,

P=^ sin

^r = a-jR

wy=

0
W'/ = Ji-IW

Pg=I22 ^_

P m ec h =ll~ { l - s ) = P ag (\-s)

(/I, X)
fm =

dx
cWf (i, x)

fm =

r=

dx

i2

A2

T =

ag
<5J
syn

T max

Wn+(X,t+X,?}
X.

V
r T h=

L=2^UX)

fm

sin(x)cos(jF) =

A=constant

[R?+(Xl+Xm)2}
d

2L(x) 2 dx

L{x)

d L12
} _ i 2 dL 1L+ 1.2 dL^ + . . --------2 1 2 2 dd dd

7:
Th

j X m ( R ] + j X ,)
Ri+j(Xl+XJ

r=

1
a)syn(Rrh+R2/S

P = E a I.=Ta> m

KT h
)2+(X

R,
Th+X2

)2

T = KJI a

CONFIDENTIAL

UTM
UNIVERSITI TEKNOLOGI MALAYSIA

FINAL EXAMINATION SEMESTER II


SESSION 2010/2011
COURSE CODE

SET 4533

COURSE NAME

WIRELESS COMMUNICATION SYSTEM

LECTURERS

ASSOC. PROF. DR. JAFRI BIN DIN


ASSOC. PROF. DR. RAZALI BIN NGAH

PROGRAMME

SET

SECTION

01-02

TIME

2 HOURS 30 MINUTES

DATE

30 APRIL 2011

INSTRUCTION TO CANDIDATE

SECTION A: ANSWER ALL QUESTIONS.


SECTION B: ANSWER TWO (2) QUESTIONS
ONLY.

THIS EXAMINATION BOOKLET CONSISTS OF 11 PAGES INCLUDING THE FRONT COVER

2
SET4533

SECTION A (ANSWER ALL QUESTIONS)

Ql.(a) Describe the mobile telephone standards of second generation Global System Mobile (2G
GSM) and third generation (3G) in terms of operating frequency, channel BW, data rate,
and modulation type.
(8 marks)

(b)

A cellular radio system has radio base station (RBS) antenna gain of 5 dB at a height
30 m above the ground and mobile station (MS) has a unity antenna gain at a height of 2
m above the ground. The RBS transmitted power is 5 W and operates at 2 GHz. MS has
bandwidth of 200 kHz and 5 dB noise figure. The distance between RBS and MS is 5 km.
(i)

Calculate carrier-to-noise ratio (C/N) at the MS assuming plane earth path loss
propagation exists and the losses due to environment is 10 dB.
(3 marks)

(ii)

Using the Okumura model predicts the C/N at MS for an open area. Compare the
result obtained from (i).

(iii)

From (ii), calculate how many second per hour the signal will be below the MS
sensitivity level by assuming Rayleigh fading occur at MS.

(iv)

(7 marks)

(3 marks)

From (iii), design a new RBS so that the loss in communication is 20 second per
hour. The same antenna gain of 5 dB will be used.

(4 marks)

3
SET4533

. You are supposed to design a communication link through a geostationary satellite to meet
a Carrier-to-Noise ratio, C/N and link margin specification as follows:
Satellite
- Geostationary at 730 W longitude.
24 C-band transponders, 28 Ku-band transponders 3.2kW RF power output.
Antenna gain, on axis, C band and Ku band (transmit and receive)
=31 dB
- Receive system noise temperature (C band and Ku band)
= 500K
Transponder saturated output power (C band)
= 40W
- Transponder bandwidth (C band)
=36 MHz
- Transponder saturated output power (Ku band)
= 80W
- Transponder bandwidth (Ku band)
= 54 MHz
Signal
FM-TV analog signal to be received in a bandwidth of 27 MHz.
- Multiplexed digital TV signals transmitted as QPSK with symbol rate 27 Msps
using half rate FEC with coding gain 5.5 dB.
Minimum permitted C/N overall of 9.5 dB.
(a) Determine the transmitting power for the earth station to provide a clear air C/N of 26 dB
in a C-band transponder at a frequency of 6.285 GHz. Given the uplink antenna diameter
is 9 m with efficiency of 68%. The uplink station is located on the 2 dB contour of the
satellite footprint. Assume 0.5 dB for clear air atmosphere attenuation and other losses.
(11 marks)
(b) What could be your transmitting power if a Ku band transponder is to be design
compared to your answer in Q2 (a). (No calculations involved).
(3 marks)
(c) Design a Ku-band receiving earth station (Determine the diameter of the receiving
antenna) to provide an overall clear air C/N of 17 dB in a 27 MHz IF noise bandwidth at
a carrier frequency of 11.45 GHz. The antenna noise temperature is 30 K and the LNA
noise temperature is 110 K. The receiving terminal is located on the 3 dB contour of the
satellite footprint, and clear air attenuation on the path and other losses total 0.8 dB.
(9 marks)
(d) Under conditions of heavy rain, the Ku-band path to the satellite station suffers an
attenuation of 6 dB. Calculate the overall C/N at the earth station in a bandwidth of 27
MHz under these conditions.
(2 marks)

4
SET4533

SECTION B (ANSWER TWO (2) QUESTIONS')


Q3.(a) Assuming n th propagation power law exists between radio base station (RBS) and mobile
station (MS) in cellular system, show that the carrier to interference ratio (C/7) is given
by.
(D V
C ! I = --1

U
where D is the co-channel separation and R is the cell radius.

(3 marks)

(b)

State two assumptions to be made for the above expression.

(c)

From part (a), if seven cells per cluster are employed and free space condition exists
between RBS and MS, calculate the carrier to interference ratio (C/7) in dB. Discuss your
result in term of MS reception.
(5 marks)

(d)

Analyze (C/7) if the number of cell per cluster is now to be reduced to four. (3 marks)

(e)

Figure Q3 shows the discrete power delay profile for multipath environment obtained
from the field measurement in Kuala Lumpur by a cellular telephone company.
(i)

(2 marks)

Calculate, (a) mean excess delay, (b) mean square delay, (c) rms delay spread, and
(d) coherence bandwidth for frequency correlation function above 90%.
(7 marks)
(ii) Analyze your results in Q3 (e)(i)in term of type of fading for this channel.
(3 marks)
(iii) Suggest the requirement (if any) needed for designing a receiver to be used in this
environment.
(2 marks)
Pr(x) dB

Figure Q3

5
SET4533

Q4.(a) State three advantages of the sectorized cell in cellular telephone network. (3 marks)
(b)

With the aid of suitable diagram, explain how sectorized cell can reduce the co-channel
interference compared to omni cell.
(4 marks)

(c)

Describe two types of interference occurence in cellular radio system and the methods to
overcome or minimize these interferences.
(4 marks)

(d)

As an engineer you have been assigned to plan radio base stations to provide cellular
telephone coverage for an area of 3000 square km and operating at 900 MHz having a
total bandwidth allocation of 19.2 MHz with simplex channel of 30 kHz. The area of each
cell is 10 square km using 4 cells per cluster. Assume the grade of service (GOS) of 2 %
for Erlang B system and each user make average 3 calls per hour at average call duration
of 2 minutes. Calculate,
(i)
(ii)
(iii)
(iv)
(v)

the traffic intensity of each user


(2 marks)
the number of cells and cluster in service area
(2 marks)
the number of channel per cell
(2 marks)
the maximum carried traffic for the service area
(2 marks)
the total number of users that can be served for 2% GOS
(2 marks)
(vi) the theoretical maximum number of users could be served at one time. (2 marks)
(vii) Discuss the results in (v) and (vi) in terms of number of users.
(2 marks)

6
SET4533

Q5.(a) Satellites(spacecraft) which orbit the earth follow the same laws that govern the motion of
the planets around sun. Johannes Kepler derived three(3) laws describing planetary
motion.

(i)

What are Keplers three laws of planetary motion?


(6 marks)

(ii)

What do the terms perigee and apogee mean when used to describe the orbit of a
satellite orbiting the earth?
(4 marks)

(iii) A satellite in an elliptical orbit around the earth has an apogee of 39,152 km and a
perigee of 500 km. What is the orbital period of this satellite? (The mathematical
formulation of the third law is T 2 = (47t2a3)/jj., where T is the orbital period, a is the
semimajor axis of the orbital ellipse, and u is Keplers constant = 3.986 x 105 km2/s2.
Assume the average earth radius of the earth is 6378.137 km)
(5 marks)
(b) Satellite orbits the earth follow the same laws that govern the motion of the planets
around the sun.
(i)

Use Keplers law to compute the radius of a circular satellite orbit for which the period
is 1 day.
(6 marks)

(ii)

State one example of satellite system that may employ this orbit. Explain the
operations of such services which require this orbital feature.

(4 marks)

BASIC MEDIAN ATTENUATION Arau (f,d) (dB)

SET4533

300 500 TOO K>00 20003000


FREQUENCY KMHz)

8
SET4533

100

200 300 500 700 1000 2000 3000


Frequency f (MHz)

9
SET4533

PERCENT PROBABILITY THAT NORMALISED SIGNAL POWER < ABSCISSA

Signal power normalized to the median (dB) CDF of the received signal power plotted on Rayleigh paper

0.05

Probability of Blocking

0.02

0.01

0.005

0.002

0.001
1.0 10.0

Traffic Intensity in Erlangs

100.0

11
SET4533
List of formulas
Boltzmann Constant = 1.38 xlO J/K

1 W/m>
And And 77

P - P A -01,4 JIG^
' d e 120n e A80n 2

Ad } d 2
P=P,G,G r

yd 2

A(d x + d 2)

for d yjh'h.

Z50 (dB) = L f + A mu ( f . d ) - G(h,) - G(h r ) - G a


G(h :) = 20 log(h, / 200),30m <h t < 1000m

f10 log(h r /3),h r <3m


[20 log(h r / 3),3m < h r < 30m

r_

C = MkN = MS

and (r) =

D ~ \li 2 +U + J' 2
A u = A H , A = U A u , A c =^

R es = 23.188 xlO6 ^3.381-cos/, cos/= cos<9 cos((Z^-(Z^)


[C/N] = [EIRPJ + [Gr] - [Losses] - [k] - [Ts] - [BN]

<JT = yjr2 - ( t ) 2 where t 2 =

~(XT

^ to

vcosd
L=

_______ 1

G(h r ) =

CONFIDENTIAL

UTM
UNIVERSITI TEKNOLOGI MALAYSIA

FINAL EXAMINATION SEMESTER II


SESSION 2010/2011
COURSE CODE

SEE 4012

COURSE NAME

PROFESSIONAL ENGINEERING PRACTICE

LECTURERS

PROF. IR. DR. ABDUL HALIM BIN MOHAMED


YATIM
ASSOC. PROF. DR. MOHAMAD NOH BIN
AHMAD
ASSOC. PROF. IR. DR. SHARUL KAMAL BIN
ABDUL RAHIM
DR. MOKHTAR BIN HARUN

PROGRAMME

SEB / SEC / SEI / SEL / SEM / SEP / SET

SECTION

01-03/10

TIME

2 HOURS

DATE

13 MAY 2011

INSTRUCTION TO CANDIDATE

PART A : ANSWER FOUR (4) QUESTIONS


ONLY.
PART B : ANSWER ALL QUESTIONS.

THIS EXAMINATION BOOKLET CONSISTS OF 5 PAGES INCLUDING THE FRONT COVER

-2-

SEE4012

PART A (60%) - ANSWER ANY 4 QUESTIONS.

Ql. Professional basketball players have been playing basketball all their life, and in addition
that is what they do for living. Define the term professional, and discuss why basketball
which has been played professionally is not termed as professional.

(15 marks)
Q2. Define what BEM Code of Professional Conduct is. With example discuss why the code,
even though not totally comprehensive, provides a framework for ethical judgment for an
engineering professional.

(15 marks)
Q3. Answer any three of the followings:
a. List all objectives of Registration of Engineers Act 1967 (Act 138)
b. List all functions of Board of Engineers Malaysia (BEM).
c. List the eight elements necessary for a contract to be enforceable by law.
d. List rights and responsibilities of an engineer

(15 marks)
Q4. What are the ethical issues? With the aid of an example, discuss how each ethical issue is
evaluated in ethical judgment.

(15 marks)
Q5. List and define three types of accident? With aid of an example, discuss how an
engineering design is deemed safe before it is mass produced for public use.
(15 marks)

-3SEE4012

PART B (40%) - ANSWER ALL QUESTIONS

Answers to Q6 to Q9 are based on the following case, and Code of Professional Conduct
(Registration of Engineer Act - Act 138) on page 5.
Marvin Johnson is Environmental Engineer for Wolfog Manufacturing, one of several local
plants whose water discharges flow into a lake in a flourishing tourist area. Included in Marvin's
responsibilities is the monitoring of water and air discharges at his plant and the periodic
preparation of reports to be submitted to the Department of Natural Resources.
Marvin has just prepared a report that indicates the level of pollution in the plant's water
discharges slightly exceeds the legal limitations. However, there is little reason to believe that
this excessive amount poses any danger to people in the area; at worst, it will endanger a small
number of fish. On the other hand, solving the problem will cost the plant more than $200,000.
Marvin's supervisor, Plant Manager Edgar Owens, says the excess should be regarded as a mere
"technicality," and he asks Marvin to "adjust" the data so that the plant appears to be in
compliance. He explains: "We can't afford the $200,000. It might even cost a few jobs. No doubt
it would set us behind our competitors. Besides the bad publicity we'd get, it might scare off
some of tourist industry, making it worse for everybody."
[This hypothetical case is an adaptation from Roger Ricklefs, "Executives Apply Stiffer Standards Than Public to Ethical
Dilemmas," The Wall Street Journal, November 3, 1983.]

Questions:
Q6. List 3 (three) relevant codes in BEM Code of Professional Conduct that might be violated
should Marvin goes ahead with his supervisors suggestion. Briefly explain one of your
answers.

(5 marks)

Q7. What do you understand about the word adjust, the action Marvin was asked to do on
the data so that the plant appears to be in compliance. Justify your answer

-4SEE4012

(10 marks)

Q8. Lets say that you were in the team that was responsible for gathering and analyzing the
pollution emission data. How would you justify to Edgar Owens, who says the pollution
excess should be regarded as a mere "technicality", that his statement is not true.
(15 marks)

Q9. From the whole scenario of the case given, would you suggest to Marvin to do
whistleblowing? Justify your answer.

(10 marks)

-5 SEE4012

CODE OF PROFESSIONAL CONDUCT


(REGULATION NO 23 - 33 REGISTRATION OF ENGINEER ACT)
23. Every Registered Engineer shall at all times uphold the dignity, high standing and reputation of his
profession.
24. A Registered Engineer in his responsibility to his employer, client or the profession shall have full
regard to the public interest.
25. (1) A Registered Engineer shall discharge his duties to his employer or client as the case may be with
complete fidelity.
(2)
Except with the permission of his employer, a Registered Engineer shall not accept any
remuneration for services rendered other than from his employer.
26. A registered Engineer shall not maliciously injure or attempt to maliciously injure whether directly or
indirectly, the professional reputation, prospects or business of another Registered Engineer.
27. A registered Engineer shall not;
(a) canvass or solicit professional employment
(b) offer to make by way of commission or any other payment for the introduction of his professional
employment; or
(c) except as permitted by the Board, advertise in any manner or form in connection with his
profession.
28. A Professional Engineer in private practice shall not practice engineering with any person whose
registration has been cancelled.
29. A Professional Engineer in private practice shall not be a medium of payment made on his clients
behalf unless he is so requested by his client nor shall he, in connection with work on which he is
employed, place contracts or orders except with the authority of and on behalf of his client.
30. A Professional Engineer in private practice shall not without the approval of the Board enter into
professional partnership with any person other than a Professional Engineer in private practice, a
Registered Architect, a Registered Quantity Surveyor or a Licensed Land Surveyor.
31. A Professional Engineer in private practice shall not directly or indirectly(a) supplant or attempt to supplant another Professional Engineer in private practice;
(b) intervene or attempt to intervene in or in connection with engineering work of any kind which to
his knowledge has already been entrusted to another Professional Engineer in private practice; or
(c) take over any work of that other Professional Engineer in private practice
acting for the same client unless he has(i) obtained the consent of that other Engineer; or
(ii) been formally notified by the client that the service of that other Engineer have been
terminated in accordance with the provisions of any contract entered into between that other
Engineer and the client.
32. (1) Except with the prior approval of the board, a Professional Engineer in private practice shall not be
a director or executive of or substantial shareholder in or agent for any contracting or manufacturing
company or firm or business related to building or engineering.
(2)
If such approval is given, such Professional Engineer shall not undertake any contract work
wherein he is engaged as a consulting engineer in such project unless it is in respect of a design and
build project.
33. Every Registered Engineer while acting in his professional capacity shall disclose in writing to his
client of the fact if he is a director or member of a substantial shareholder in or agent for any contracting
or manufacturing company or firm or business or has any financial interest in any such company or firm
or business, with which he deals on behalf of his client.

CONFIDENTIAL

UTM
UNIVERSITI TEKNOLOGI MALAYSIA

FINAL EXAMINATION SEMESTER II


SESSION 2010/2011
COURSE CODE

SEE 2043

COURSE NAME

SIGNAL AND NETWORKS

LECTURER

DR. YUSRI BIN MD. YUNOS

PROGRAMME

SEC / SEL / SEM / SET / SWB

SECTION

01

TIME

2 HOURS 30 MINUTES

DATE

12 MAY 2011

INSTRUCTION TO CANDIDATE

ANSWER FOUR (4) QUESTIONS ONLY.


EACH CANDIDATE WILL BE GIVEN THE
FOLLOWING ITEM:i) A BOOK OF FORMULA
ii) SEMILOG GRAPH

THIS EXAMINATION BOOKLET CONSISTS OF 16 PAGES INCLUDING THE FRONT COVER

SEE2043
Given the following signals /j (t), f 2 (?) and f 3 (t ). Answer the following
questions.

f 2 (0 = 4[w(7 - 2) - (/ - 4)]
f3{t) = -2(t-6)[u(t-A)-u(t-6)]

(i)

Sketch the signals of f x (t ), f 2 ( t ) and /3 (?).

(6 Marks)

(ii)

If /4 (0 = /, (0 + f 2 ( t ) + f 3 (0 Sketch /4 ( t )

(3 Marks)

(iii)

If f 5 (0 = - f 3 (20 Sketch /5 (0

(4 Marks)

A fullwave rectified signal, V : ( t ) of Figure Ql(a), acts as an input to a


simple RLC circuit of figure Ql(b)

Vi(t)

Figure Ql(a) Full wave rectified signal


L=0.2H

Figure Ql(b) RLC Circuit

3
SEE2043
(i)

Find the trigonometry fourier series of V i (7) up to 2nd harmonics?


(3 Marks)

(ii)

Find the circuit current, I ( t )

(iii)

Sketch the amplitude frequency spectrum of the current, I ( t ) up to


2nd Harmonics.

(6 Marks)

(3 Marks)

4
SEE2043
Q2.

(a)

Briefly

discuss

ONE

similarity

and

ONE

difference

between

Fourier

transform and Laplace Transform in the application of signal analysis and


signal processing.
(4 marks)

(b) (i) Write the mathematical expression of signal x(t ) in


TRIANGULAR function and then find the Fourier Transform of
the signal.
(2 marks)
x(t)

Figure Q2(b)(i)

(ii)

The signal x { t ) in (b)(i) is now shifted to the right by 2 units and is


represented by signal y(t) below in Figure Q2(b)(ii). Find the
Fourier Transform of the signal in (b)(ii).
(2 marks)

y(t)

FigurebQ2(b)(ii)

5
SEE2043
(iii)

Signal x(t) in (b)(i) and signal y(t) in (b)(ii) are combined to


form

signal

z(t)

below.

By

using

the

property

of

Fourier

Transform, find the Fourier Transform for signal z(t ).


(2 marks)

Consider the system. h{t) is given as h(t) = 2 sin c (201) + sin c (10/).

x(t) -------

(i)

h(t)

+y(t)

By using the duality and linearity properties, find the Fourier


Transform for signal h ( t ) .
(4 marks)

(ii)

The Fourier Transform of signal x(t) is given as X(a >) below.


Draw the spectrum of Y(a>).

X(a>)
A

-25 -15 -5

co

5 15 25

Figure Q2(c)(ii)
(2 marks)

(iii)

Based on your answer in (c)(ii), write the expression for Y( g >) .

(2 marks)

SEE2043

(d)

Consider a system shown in Figure Q2(d)(i). An input signal, x(t) is


multiplied by a carrier signal cos(<y/)' The resulting signal is then passed
through an ideal filter h(t ). The signal is then multiplied by carrier signal
cos(co 2 t). The spectrum of X(co ) and H(a>) are as shown in Figure
Q2(d)(ii) and (iii).

x(t)

cos (co 2 t)

cos (co x t)
Figure Q2(d)(i)

mco)

0 250 750 co
Figure Q2(d)(ii)

i)

Figure Q2(d)(iii)

If a> y is set at 500rad/s, draw the spectrums of A(co) and B(a >).
(3 marks)

ii)

Determine the value of co 2 and draw the spectrum of filter H 2 (co)


if the desired output signal Y(co) is as shown below in Figure
Q2(e). Show your calculations by drawing the spectrum of C(a >).

7
SEE2043
Y(a>)

Figure Q2(e)

(4 marks)

SEE2043
Q3 (a) The Laplace Transform has properties similar to the properties of the
Continuous Time Fourier Transform. Using the linearity, time shifting,
time scaling and the differential equation properties;
Find the Laplace transform of

(i)

x(t) = u(t) - u(t - 3)

(2 marks)

(ii)

y(t) = x(21)

(3 marks)

(iii)

~ (z(0) + 7 ^ (*(0) +12 z(t) = 0


at
dt
for times t > 0, subject to the initial conditions z(0) = 2 and

0(0),=o =~4
dt

(4 marks)
(b) Consider the waveform x(t ) given in Figure Q3(a);

xft)

-> t

(i)

Write a mathematical expression for x(t)

(3 marks)

(ii)

Find the Laplace Transform, X(^)

(3 marks)

9
SEE2043
An electrical circuit shown in Figure Q3(b) consist of two resistors, an
inductor, a capacitor and a voltage source. Initially, at t = Os switch S is at
position^ and at t = Is the switch is at position B and remain there.

Figure Q3(b)

(i)

Sketch the Laplace transform equivalent circuit for 0 < t < Is


Assume . v c (0) = 0 and /(0) = 2
(2 marks)

(ii)

Find the current i(t ) and vc (t) provided by the power supply
immediately after the switch S is closed at position A.
(5 marks)

(iii)

Find the voltage across the capacitor for t > Is

(3 marks)

SEE2043
Using the semilog graph, plot the magnitude and phase bode plot for

88 ^ +10)
(j + 8)(j + 11)

(15 marks)

If Y ( s ) = X ( s ) H ( s ) , where X ( s ) = s + 3000 , sketch the magnitude and


phase bode plot for Y ( s ) .

(10 marks)

11
SEE2043
Figure Q5 shows different response of passive low pass filters.

(a)

(b)

0 0.2 0.4 0.6 0.8 1

(d)

(c)
Figure Q5
(i)

What is the type of response shown in Figure Q5(a)?


(2 marks)

(ii)

A typical response of a Chebyshev filter is shown in Figure Q5(b).


On the other hand, a slightly different response of a Chebyshev is
shown in Figure Q5(c). What is the different?
(2 marks)

(iii)

The response in Figure Q5(d) shows a steeper roll-off (small


transition band) compared to Figure Q5(b) and Figure Q5(c).
Based on your observation, how a steeper roll-off can be obtained?
(2 marks)

You are asked to design a second order Chebyshev filter.

12

SEE2043
(i)

Calculate the maximum allowable magnitude of ripple, A max, of


the filter. Given c o s =1.844r a d I s and a s = 45d B .
(5 marks)

(ii)

Find the normalized transfer function, H n ( s ) , of the Chebyshev


filter.

(iii)

(5 marks)

What is the actual transfer function, H ( s ), of the Chebyshev filter


if the c o p = 20rad / s.
(2 marks)

By using the same system order, co s, and a s , calculate the following:.


(i)

The normalized transfer function, H n ( s ) , of a Butterworth filter.


(2 marks)

(ii)

The value of a p

(5 marks)

13
SEE2043

LAMPIRAN (APPENDIX)
F: MEREKABENTUK PENAPIS ANALOG
(Analog Filter Design)
CHEBYSHEV
f \
1

4nax = 20 log ^ =-20 lOg

( a, 'N

/ f A ^max

O
0
1

cosh -i

max

10

-1

I
o
O

n=

= 101og(l + -2) , =\10

/
/ vJ

V/
o) c = co p cosh cosh 1
n
\sjj
Hn(s) =

K.

K,

T n (s) s n + a n _ l s n 1 + ....+ a^s + a0

; n ganjil(odd)
an

a,

+s

10

; n genap(everi)

20

BUTTERWORTH
mjcoi =
f

i+

CO

\ in

> Hn(s) =

B n (s) s" + a n _ l s n +....+ a x s + 1

KmoJ

/\

1+

a = 201og10 \H(ja>)\ = -lOlog

In ~

CO

V CCO

In '
a. = -lOlog

1+

P
CO

\c

\ 2n

= 10p/' -1,

In '

, a s = -10 log

1+

C \In
CO,

vcy

= 10"'/,0-l ,

oj .

\apj

\2n

10J1 -1
10p/1 -1

14
SEE2043
__ logl(lOg'/10 - l)/lQ^/10 - 1

n =

coc =

10

10/

1/2 n

atau =

[lO"*/10 - if"

Jadual F.l: Pengiraan frekuensi sudut jalur henti temormal, a> sn .


(Calculations of normalized stopband angle frequency )

Jenis penapis
(Types of filters)
Penapis lulus rendah, LPF
(Low Pass Filter)

Frekuensi sudut jalur henti ternormal, 0)m


(Normalized stopband angle frequency)

<P

Vp

Penapis lulus tinggi, HPF


(High Pass Filter)
Penapis lulus jalur, BPF
(Band Pass Filter)

G)nlcoD2 - col co2s2 - CODlCODl


----- 1---------------- 1 atau(or)------- ?--------------- s, pilih yang lebih kecil
,iK2-^ij rfK2-%lj
(choose smaller value)

ct)'Acon-, - o) n , ) coAa>nl - O) ,)

Penapis jalur henti, BSF


(Band Stop Filter)

----------------- atau(or) ----------------------- , pilih yang lebih kecil


^pl^pl ~ s2 ~ (Qpl^pl
(choose smaller value)

15
SEE2043
Jadual F.4: Pekali penapis Chebyshev untuk Amax 0.5, 1, 2 dan 3 dB.
('Coefficients of Chebychev filter for A max 0.5, 1, 2 dan 3 dB)

n
1
2
3
4
5
6
7

a0
2.8627
1.5162
0.7156
0.3790
0.1789
0.0947
0.044

n
1
2
3
4
5
6
7

a0
1.9652
1.1025
0.4913
0.2756
0.1228
0.0689
0.0307

n
1
2
3
4
5
6
7

a0
1.3075
0.8230
0.3268
0.2057
0.0817
0.0514
0.0204

n
1
2
3
4
5
6
7

a0
1.0023
0.7079
0.2505
0.1769
0.0626
0.0442
0.0156

ai

a2

a3

a4

1.4256
1.5348
1.0254
0.7525
0.4323
0.2820

1.2529
1.7168
1.3095
1.1718
0.7556

1.1973
1.9373
1.589
1.6479

1.172
2.1718
1.869

ai

a2

a3

a4

1.0977
1.2384
0.7426
0.5805
0.3070
0.2136

0.9883
1.4539
0.9743
0.9393
0.5486

0.9528
1.6888
1.2021
1.3575

0.9368
1.9308
1.4287

ai

a2

a3

a4

0.8038
1.0221
0.5167
0.4593
0.2102
0.1660

0.7378
1.2564
0.6934
0.7714
0.3825

0.7162
1.4995
0.8670
1.1444

0.7064
1.7458
1.0392

ai

a2

a3

a4

0.6448
0.9283
0.4047
0.4079
0.1634
0.1461

0.5972
1.1691
0.5488
69909
0.3000

0.5815
1.4149
69060
1.0518

0.5744
1.6628
0.8314

as
0.5 db ripple

1.159
2.4126
as
1 db ripple

0.9282
2.1760

a5
2 db ripple

0.7012
1.9935
a5
3 db ripple

0.5706
1.9115

a6

1.1512
a6

0.9231

a6

0.6978
a6

0.5684

16
SEE2043
Jadual F.5: Transformasi frekuensi penapis
(.Frequency Transformations of filter)
Jenis penapis
Transformasi
(Types of filters)
(Transformation)
ss
Penapis lulus rendah, LPF
, (, bagi Butterworth)
(Low Pass Filter)
(Op a c
Penapis lulus tinggi, HPF
(High Pass Filter)

, (, bagi Butterworth)
5s

Penapis lulus jalur, BPF


(Band Pass Filter)

s 1 +a>piO>p2

Penapis jalur henti, BSF


(Band Stop Filter)

(o) p2 ~ CO p i)s
S 2 + P 1 P 2

Jadual F.6(a): Pekali-pekali Polinomial Butterworth Bn(s) = sn+an-isI1'1+................ +ais+l


n
2
3
4
5
6
7
8
9
10

ai
1.4142
2.0000
2.6131
3.2360
3.8637
4.4939
6.1258
6.7587
6.3924

a2

a3

a4

a5

a6

a7

a8

a9

2.0000
3.4142
6.2360
7.4641
10.0978
13.1370
16.5817
20.4317

2.6131
6.2360
9.1416
14.5917
21.8461
31.1634
42/8020

3.2360
7.4641
14.5917
26.6883
41.9863
64.8823

3.8637
10.0978
21.8461
41.9863
74.2334

4.4939
13.1370
31.1634
64.8823

6.1258
16.5817
42.8020

6.7587
20.4317

6.3924

Jadual F.6(b): Polinomial Butterworth dalam bentuk pemfaktoran Butterworth


_____________ (.Butterworth Polynomials in Factorized Form) ___________________
n Bn(s)_____________________________________________________________________________
1
2
3
4
5
6
7
8
9
10

s+1
s2+ 1.41421356s + 1
(s + 1) (s2+ s + 1)
(s2 + 0.76536686s + 1) (s2 + 1.84775907s + 1
(s+ 1) (s2 + 0.61803399s + 1) (s2 + 1.931803399s + 1)
(s2 + 0.51763809s + 1) (s2 + 1.41421356s + 1) (s2 + 1.93185165s + 1)
(s + 1) (s2 + 0.44504187s + 1) (s2 + 1.24697960s + 1) (s2 + 1.80193774s + 1)
(s2 + 0.39018064s + 1) (s2 + 1.11114047s + 1) (s2 + 1.66293922s + 1) (s2 + 1.96157056s + 1)
(s + l)(s2 +0.34729636s + 1) (s2 + s + 1) (s2 + 1.53208889s + 1) (s2 + 1.87938524s + 1)
(s2 + 0.31286893s + 1) (s2 + 0.90798100s + 1) (s2 + 1.41421356s + 1) (s2 + 1.78201305s + 1)
(s2 + 1.97537668s + 1)

CONFIDENTIAL

UTM
UNIVERSITI TEKNOLOGI MALAYSIA

FINAL EXAMINATION SEMESTER II


SESSION 2010/2011
COURSE CODE

SEP 4123

COURSE NAME

BIOMEDICAL INSTRUMENTATION AND


MEASUREMENT

LECTURERS

MR. AB. RAHIM BIN AB. RAHMAN

PROGRAMME

SEP

SECTION

01

TIME

2 HOURS 30 MINUTES

DATE

13 MAY 2011

INSTRUCTION TO CANDIDATE

ANSWER FIVE (5) QUESTIONS ONLY.

THIS EXAMINATION BOOKLET CONSISTS OF 19 PAGES INCLUDING THE FRONT COVER

SEP4123

ANSWER FIVE ( 5 ) QUESTIONS ONLY

QI. Refer to Figure QI. Assume Op Amps are ideal,


(a)

Prove that

(0

[6 marks]
f f T? ^
i+5l

1+*L
R,
Vo

R,

^ ^ T? ^

Rz \ + v
i + 5i V ^ - G j

-^V p+ os
1+5*.
R,

R,

V2

Rji-Ri,

(iii)

Rp=0

R,

i+5i

vv ^4 J

(ii)

R|

i +
R<

+
V^G J

1 +
V

R0

^G j

VV ^4 J

[3 marks]

R,2-R3+R4,

[1 mark]

Vpc

r2
AAA

V1
Rii
VEE
A/W
R5

Vcc

Rg:

Vr

Re

+-T

Vcc

Rc
AVv

v2

+ Vcc

V Ee

AAV

Vcc

Ro

V2

R3

r7

AAV- _ ,

Rx

R4 Vos

V,EE

VEE
Rs

-Vee
Figure QI

(b)

By separating Rx into RX7 above the potentiometers wiper and Rxg below the
potentiometers wiper respectively, please express Vos in term of Rx7, Rxg, R7, Rg, +Vcc
and -Vee-

[5 marks]

3
SEP4123

With Vos being adjusted to OV, the offset voltage of IV appears at the
amplifier output, explain the methods you use to remove this offset voltage.

[5 marks]

4
SEP4123

Q2. (a) Refer to Figure Q2(a). Calculate the common mode gain for instrumentation amplifier
OPA37A, OPA11 IB and OPA128LM. Which amplifier do you think amplifies the least
of common-mode signal?

[8 marks]

trill the best low noise, offset, {end temperKture drift performance.

At $oufce
wpedifKcs above aiboul 10kQ, tile bias current noise of the OPA37 reacting
wtit] dk inpii tnpcdfiBce begins fo domMile tfve nojse performance. Pw these
*pphcaiiokut}n^ iiie OPA1} I or DehI OPA211I FET input

op-amp wil
pafsjftMrtcfc. Forlower coal, use (be OPA12J pl^tk. To

ae elKttomaef, use the OPM28,

CMRR
{dB) Max I
100
128
4GnA
100
110
Ip A
LOG
118
75fA

Own

AjfAz
OPA37A
QPAtlli
0PA12SLM

R,<0)
50-5
202
202

<V/V)

2J*
Iflk
10k

Building three op-amp precision instrumentation


amplifiers with NPN and FET Inputs* (Courtesy of
Burr-Brown under copyright 1993 Burr-Brown
Corporation)
Figure Q2(a)

Noise at 1kHz
fnV/yfE)

4
20
38

5
SEP4123

(b)

Refer to Figure Q2(b). Discuss the purpose of this circuit and how it performs its task.
[6 marks]

1 MQ

Auto-zero amplifier. (Courtesy of Burr-Brown under


copyright 1BBB Burr-Brown Corporation)
Figure Q2(b)

(c)

Refer to Figure Q2(c) and Figure Q2(d). Discuss how the shield is effectively reduced the
effect of distributed capacitances along the long input cables to the amplifier.
What happen if there is no shield installed?

[6 marks]

SEP4123

Guarded input circuit.

Figure Q2(c)

Transducer or
analog sign&l

Amplificaiiori of transformer-coupled signal with shield driver. (Courtesy of Burr-Brown under

copyright 1993 Burr-Brown Corporation)

Figure Q2(d)

7
SEP4123

Refer to Figure Q3(a), Q3(b) and Q3(c).

A Medical ECG Monitor Circuit

Figure Q3(a)

The letter G at the amplifier blocks of the diagram denotes the gain or amplification of
that particular amplifier block.
(a)

What is the overall gain of the ECG monitor circuit?

[3 marks]

(b)

Calculate the value of (V3-V2) = (V+-V~) if IVtput= IV.

[4 marks]

(c)

What is the output voltage of the op amp AD705J if the common-mode voltage
[(l>8 + V\) / 2] = 10sinl007it mV?

[4 marks]

SEP4123

The circuit for the output amplifier is shown in Figure Q3(b).

Rf

Figure Q3(b)

(d)

If the gain of this amplifier is 143 and Ri = lkH, what is the required
value of Rf?

[4 marks]

9
SEP4123

The circuit for the high pass filter is shown below in Figure Q3(c).

c
o v: out

Figure Q3(c)

(e)

Briefly explain the function of the circuit and state where probably this circuit is
used?

(f)

[2 marks]

If the cut off frequency of the filter, fc = 0.03Hz and C = l|iF, calculate the value
of R?

[3 marks]

10
SEP4123

Q4. Refer to Figure Q4.

4-Vctaiip

DSffAMfW

4 Mat All Inputs

(Model

Buffers

i ira

R*fd Gtitw = 5oftG


LeadI

H >^y^r *

Ut

LL

Leicllll

014 RLMve

LeriaVL
UnUVF
Analog CoffliiMt o=^ 47 pH

1010

-o Switch *ni In
WCT

Witen Mat ICft AwRA.UUX= <RA+U+LL)ft

WlisociQi^Osrt
ftccoHBtfiOet

Figure Q4

(a)

Briefly explain the function of the circuit in Figure Q4.

[3 marks]

(b)

What can be achieved by incorporating the Wilson network in this circuit? [4 marks]

(c)

By using RA, LA and LL as input voltages to the amplifier system, calculate the voltages
at Wilson Central Out, Lead I, Lead II, Lead III, Lead aVR, Lead aVL, Lead aVF and
Precordial 1 Out if the input voltages at RA, LA, LL and Precordial 1 In are all equal to
sin lOOut volt?

(d)

[5 marks]

What is the function of 47 pF capacitor in the feedback loop of RL amplifier? What


happen if this capacitor is omitted? What is the cut-off frequency of this circuit?
[4 marks]

11
SEP4123

(e)

Draw the practical op amp circuits for +RA, +LA, +LL, +Prec, Lead I, Lead II, Lead III,
Lead aVR, Lead aVL, Lead aVF and Precordial 1 Out amplifiers.

[4 marks]

12

SEP4123

Q5. Blood pressure measurement is very important procedure in assessing ones health.
The following questions could clarify something regarding pressure:
(a)

Calculate the force required to produce a pressure of 100 mmHg on the area of 1 mm2.
[3 marks]

(b)

A barometer measures an atmospheric pressure of 500 mmHg. Convert this pressure


to atmosphere, psi, pascals and newtons per square meter.

(c)

Explain how you classify the blood pressure into systolic and diastolic pressures.
Give one example of how these two values are recorded.

(d)

[5 marks]

[4 marks]

Explain clearly with the aid of suitable diagrams (for example the diagram in Figure
Q5(a), Q5(b) and other diagram of yours) of how the indirect oscillometric blood
pressure measurement method is accomplished? State two advantages of this
[8 marks]

Vi {Volts )>

method.

linefsECtndBj

Figure Q5(a)

13
SEP4123

ID

13

21

Figure Q5(b)

25

35

14
SEP4123

Q6. Refer to the diagrams in Figure Q6(a) and Q6(b). These two diagrams are related to the
measurement of Cardiac Output (CO) by thermodilution method.

WHOm

mmm.

Mow TMicaiatipr J

Figure Q6(a)

15
SEP4123

*!& COMPUTID'
p^MCMWiAX

MI

brpok in zQwrmm *a m
ICIISIIIftTlOii OF INPICflTOK

mw to

Pi;CItCL Oil Of ION


CCHIVI

(SVMISID
OILUTrOU

M&RMKNf m

INiPICWT'61
INJICHON

csijgii in mm mud- A***

TUMI
fcaiffiMfflftjr PII1EPT5 if SUIT
m THIS MDMIMf

mm fMJtf* SHIFT

Figure Q6(b)

(a)

Explain clearly with the aid of the diagrams in Figure Q(a) and Figure Q(b) of how the
the cardiac output measurement is performed.

(b)

[8 marks]

The cardiac output is calculated based on the area under the curve of the density indicator
versus time. Could you further elaborate this method and what are the other parameters
needed to complete the calculation.

[12 marks]

16
SEP4123

Q7. (a) Refer to Figure Q7.


(i) Define the lung volumes of TV, IRV, RV and ERV.
(ii) Define the lung capacities of IC, VC and TLC.

[4 marks]
[3 marks]

6.000 rof

S.OOOnV
(WSPlRXTOfiY

RESERVE
VOLUME

4,000 mi 3100 ml-

S.DOOfN
__ , SXMWOflY
2.DOQfrt

RESEHVS
VOLUME

UKXJmJ

i,odOml

RESKKIAL .<
.. VOLUME

I-;;: 1,200 mi "<

Figure Q7

(b) (i) Calculate the partial pressure of oxygen, nitrogen and carbon dioxide if their
percentage volume are 20.96%, 79% and 0.04% respectively. Assume the
atmospheric pressure is equal to 760 mmHg.

[6 marks]

(ii)

Define respiration.

[2 marks]

(iii)

Explain how the exchange of oxygen and carbon dioxide occur between the
capillary and the body cell in internal respiration.

[5 marks]

17
SEP4123

Q8. Figure Q8(a to d) may be used to explain EEG system.


(a) Draw a typical instrumentation amplifier circuit used in this EEG system.

[4 marks]

(b) Calculate the CMRR of the EEG amplifier in dB if the EEG amplifier output is
1000 mV to a 100 p.V differential EEG input and the noise output is 1 mV to a 100 jaV
common-mode noise input.

[4 marks]

(c) The amplifiers used in EEG system are very high gain (xlOOO) amplifiers. Give at least one
reason why such a high gain amplifiers are needed?

[2 marks]

(d) What are the most critical design specifications for the Low Voltage Power Supply circuit for
the EEG system if the EEG input signal peak to peak to the system is less than 5 (J.V.
[2marks]
(e) Let assume the cranial generator voltage, e to be 100 f-iVp.p, the cranial impedance,
r to be 10 kQ, the equivalent electrode-scalp resistances, Rl and R2 to be 10 kQ each, and
the amplifier input impedance, Rjn to be 10 MQ. Let also assume the existent of common
mode noise, en0iSe to be 1 Vp-p coupled to the electrode-scalp interfaces through stray
capacitances Cl and C2 for each electrode.
(i)

Draw the EEG input circuit to the instrumentation amplifier.

[4 marks]

(ii)

Calculate the input voltage at the differential input of the instrumentation amplifier.
[2 marks]

(iii)

If EEG instrumentation amplifier gain is xlOOO and one allows only 0.1 (J.VP_P
of the common-mode signal appears at the output of the EEG amplifier,
what should be the CMRR of the EEG amplifier.

[2 marks]

18
SEP4123

> 10% A
TASION,

20%

20/c

10%

'0
-INION'

Figure Q8(a)

interface

Figure Q8(b)

19
SEP4123

Scalp
Figure Q8(c)

Figure 8(d)

CONFIDENTIAL

UTM
UNIVERSITI TEKNOLOGI MALAYSIA

FINAL EXAMINATION SEMESTER II


SESSION 2010/2011
COURSE CODE

SEL 4743

COURSE NAME

BASIC DIGITAL VLSI

LECTURERS

PROF. DR. MOHAMED KHALIL BIN


MOHD HANI
DR. SHAIKH NASIR BIN SHAIKH HUSIN
MR. ZULKIFLI BIN MD YUSOF

PROGRAMME

SEC / SEL / SEW / SET

SECTION

01-02

TIME

2 HOURS 30 MINUTES

DATE

14 MAY 2011

INSTRUCTION TO CANDIDATE

ANSWER FOUR (4) QUESTIONS ONLY.

THIS EXAMINATION BOOKLET CONSISTS OF 6 PAGES INCLUDING THE FRONT COVER

SEL 4743
2
FORMULAE & PARAMETERS:
Supply voltage: Vdd = 2.5 V
Drain current:

'

T _|Kj-|Frl)Fmi,-5FL,](i+UV

Vmin =

DS |

min( |Vgs| - |Vt|, |Vds| |Vdsat| )

V t = V to + y

Standard transistor parameters:


Vto (V) Y Cv )
0.43
NMOS
0.4
PMOS
-0.4
-0.4

<Pf(V) Vdsat (V)


-0.3
0.63
0.3
-1

k' (|iA/V")
115
-30

MV'1)
0.06
-0.1

Question 1
(a)

Calculate the current for a pMOS transistor for the cases in Table 1.

(i)
(ii)
(iii)
(iv)

(b)

W/L
1.35 nm/0.3 |im
1.35 fim/0.3 n.m
1.35 nm/0.3
4.5 pm/1.5 fim

Table 1
VGS (V)
-1
-1
-1.5
-1.6

VDS (V)
-0.5
-0.8
-1.4
-1.5

[12 marks]

VBS (V)
0
1
0
0

Both transistors in Fig. 1 have identical size. Determine the voltage for Vx. Ignore
body effect in your calculation.
[13 marks]
VDD = 2.5 V

Fig. 1

SEL 4743
3
Question 2
(a)

Refer to the circuit in Fig. 2.


(i) Write the Boolean equation for output Y.

[5 marks]

(ii) Minimize the Boolean equation such that it can be implemented using eight
transistors only.
[4 marks]
(iii) Draw a new schematic circuit based on your minimized Boolean equation.
[6 marks]

A HI

hc[ hcJ

HI
HI

b hiz

a nr

Fig. 2

(b)

(i) Draw a pseudo-nMOS circuit for 2-input NAND gate.

[3 marks]

(ii) Choose the size of the nMOS transistors for your circuit to ensure the output low
value Vql is below 0.2 V. Given (W/L)p = 0.9 nm/0.6 ^m.
[7 marks]

SEL 4743
4

Question 3
(a)

Refer to the circuit in Fig. 3(a).


(i) The size of Ml transistor is W/L = 2.7 fim/0.3 (im. Size the other seven transistors
such that the tpHL and tpLH of the gate is equivalent to an inverter with (W/L)p = 0.9
Hm/0.3 urn and (W/L)n = 0.45 |j.m/0.3 urn.
[8 marks]
(ii) Draw a stick diagram corresponding to the circuit. Use the convention shown in
Fig. 3(b).You should use a single active strip for both PMOS and NMOS
transistors, and you must minimize the output capacitance.
[7 marks]

x
Fig. 3(a)

(b)

active
poly
metal 1
contact cut

Fig. 3(b)

Fig. 3(c) shows a layout for the pull-down network of a CMOS circuit.
(i) Draw the corresponding transistor schematic for the layout.

[6 marks]

(ii) State the size (W and L) of all transistors in unit of L In Fig. 3(c), 1 grid = 1 L
You can label the size next to each transistor in your schematic for part (b)(i)
above.
[4 marks]

SEL 4743
5

ndiff

pdiff

poly

metal 1
I active
I contact

vss

bar

bar

AC

bar

Fig. 3(c)

Question 4
A chain of four inverters is shown in Fig. 4. The second and fourth inverters drive an external
load, as indicated. Assume each inverter has a symmetrical VTC, and C int = C g (Y = 1).
Equivalent input capacitance of unit-sized inverter is C.
In

Out

s, = l

s2 = ? Jj4

C S', = 4

X
T

C = 16 C

Fig. 4

(a)

Obtain the optimal sizing factors S2 and S4 for minimum propagation delay.
[10 marks]

(b)

Determine the minimum delay (in terms of t p o ) for the inverter chain.

[5 marks]

(c)

If the 4C load does not exist, and only the output load remains, determine the lowest
possible delay attainable, using unlimited number of inverters.
[10 marks]

SEL 4743
6
Question 5
(a)

A logic gate network is shown in Fig. 5(a). The input inverter size is (W/L)p = 4 7J2 X
and (W/L)n = 3 )J2 X. The input capacitance Cin = 6 fF, while the output capacitance
Cl = 256 fF. Use logical effort approach to optimize the sizes (in X unit) of inverter sO
and NAND gate si to minimize delay for the logic network. Assume C int = C g(y = 1).
The logical effort g for the NAND gate is 4/3.
[10 marks]

Fig. 5(a)

(b)

A similar logic gate network is shown in Fig. 5(b). The only difference is the input
inverter gO now drives three similarly sized inverters sOa - sOc. Determine the new
sizing for the inverters and NAND gate to achieve minimum delay.
[15 marks]

Fig. 5(b)

You might also like